You are on page 1of 664

Deeper Understanding, Faster Calculation

--Exam FM Insights & Shortcuts



Part I: Theories of Interest

12
th
Edition



by Yufeng Guo


Fall 2010




http://actuary88.com









This electronic book is intended for individual buyer use for the sole purpose of
preparing for Exam FM. This book may NOT be resold or otherwise redistributed to
others. No part of this publication may be reproduced for resale or multiple copy
distribution without the express written permission of the author.


2010, 2011 By Yufeng Guo

1
Contents

Part One: Theories of Interest

Chapt e r 1 Exam-t aking and s t udy s t rat e gy ............................ 5
A tale of two Exam FM takers, Mr. Busy and Mr. Lazy................................................ 5
Truths about Exam FM................................................................................................... 9
How to study hard yet fail the exam miserably ............................................................ 11
Recommended study method........................................................................................ 13
How to build a 3 minute solution script........................................................................ 14
How to eliminate errors ................................................................................................ 30
Chapt e r 2 Ge t t ing s t art e d ............................................................. 48
Chapt e r 3 FM Fundament al.......................................................... 49
Time value of money .................................................................................................... 49
Principal ........................................................................................................................ 49
Interest rate.................................................................................................................... 49
Simple interest rate ....................................................................................................... 50
Compound interest rate................................................................................................. 50
Force of interest ............................................................................................................ 52
Nominal interest rate..................................................................................................... 57
APR............................................................................................................................... 58
Annual effective interest rate........................................................................................ 59
Continuous compounding............................................................................................. 60
Effective annual rate of discount .................................................................................. 60
Simple annual rate of discount...................................................................................... 63
Nominal annual rate of discount ................................................................................... 63
Relationship between i , , d ,
( ) m
i ,
( ) m
d ....................................................................... 64
Future value .................................................................................................................. 75
Present value ................................................................................................................. 75
Convert interest rate to discount rate or vice versa....................................................... 77
PV of a stream of cash flows ........................................................................................ 82
Net Present Value ......................................................................................................... 82
Internal rate of return (IRR).......................................................................................... 82
Asset and its price ......................................................................................................... 84
Convert a cash flow from one point of time to another point of time .......................... 85
Collapse multiple cash flows into a single cash flow................................................... 86
Annuity collapsing n parallel cash flows into a single cash flow.............................. 89
Avoid the common pitfall ............................................................................................. 90
Perpetuity.................................................................................................................... 104
Annuity payable m-thly in advance ......................................................................... 107
Annuity payable m-thly in arrears ........................................................................... 109
Increasing annuity....................................................................................................... 121
Continuously increasing annuity................................................................................. 121
Decreasing annuity...................................................................................................... 122
2
Chapt e r 4 Calc ulat or t ips ............................................................ 137
Best calculators for Exam FM .................................................................................... 137
New features added in BA II Plus Professional.......................................................... 137
How to reset calculators to their best conditions for FM............................................ 137
Compound interest ...................................................................................................... 140
Annuity ....................................................................................................................... 144
Loan/bond amortization.............................................................................................. 153
Compare Cash Flow Worksheet with TVM Worksheet ............................................. 159
Increasing annuity....................................................................................................... 159
Comprehensive calculator exercise............................................................................. 167
Chapt e r 5 Ge ome t ric ally inc re as ing annuit y ..................... 175
Chapt e r 6 Re al vs . nominal int e re s t rat e ............................. 204
Chapt e r 7 Loan repayment and amort izat ion.................... 210
Chapt e r 8 Sinking fund ................................................................ 240
Chapt e r 9 Callable and non-c allable bonds ......................... 247
Chapt e r 1 0 Valuat ion of s t oc ks ................................................... 272
Chapt e r 1 1 Pric e of a bond s old bet we en t wo coupon
payment s 275
Chapt e r 1 2 Time we ight ed re t urn and dollar we ight ed
re t urn 288
Chapt e r 1 3 Inve s t ment year & port folio met hod ............... 303
Chapt e r 1 4 Short Sale s ................................................................... 305
Chapt e r 1 5 Te rm s t ruct ure of int e re s t rat e , s pot rat e,
forward rat e , and arbit rage ............................................................. 316
Chapt e r 1 6 Macaulay durat ion, modified durat ion,
c onvexit y 341
Chapt e r 1 7 Immunizat ion.............................................................. 376
Chapt e r 1 8 Cas h flow mat ching .................................................. 394
Va lu e of t h is PDF s t u dy ma nu a l...................................................... 401
About t he aut hor................................................................................. 402
3
4
http://actuary88.com
Chapt e r 1 Exam-t aking and s t udy
s t rat e gy

Re ad t his c hapt e r be fore ope ning your t e xt books !

Th is ch a pt er r equ ir es s ome kn owledge a bou t t h e t ime va lu e of mon ey,
a n n u it y, a n d loa n a mor t iza t ion . If you don t kn ow t h es e con cept s , don t
wor r y. J u s t s kip t h e det a iled ma t h ca lcu la t ion s a n d focu s on t h e ma in
idea s in t h is ch a pt er . La t er on , a ft er you u n der s t a n d t h e t ime va lu e of
mon ey, a n n u it y, a n d loa n a mor t iza t ion , come ba ck t o t h is ch a pt er a n d go
t h r ou gh t h e ma t h .
It s cr it ica l t h a t you u n der s t a n d t h e es s en ce of t h is ch a pt er befor e you
r u s h t o r ea d t h e t ext books .
A t ale of t wo Exam FM t ake rs , Mr. Bus y and Mr.
Lazy

It wa s t h e bes t of t he t imes , it wa s t h e wor s t of t imes , it wa s t h e a ge of
bein g la zy, it wa s t h e a ge of bein g bu s y, it wa s t h e epoch of pa s s in g
Exa m FM, it wa s t h e epoch of fa ilin g Exa m FM, it wa s t h e h ope of get t in g
ASA, it wa s t h e des pa ir of goin g n owh er e.

Two a ct u a r ia l s t u den t s , Mr . Bu s y a n d Mr . La zy, a r e bot h pr epa r in g for
Exa m FM. Th ey h a ve t h e s a me h eigh t a n d weigh t . Th ey h a ve t h e s a me
level of in t elligen ce. As a ma t t er of fa ct , t h ey a r e s imila r a bou t a lmos t
ever yt h in g except t h a t Mr . Bu s y is ver y bu s y a n d Mr . La zy is ver y la zy.

Mr . Bu s y a n d Mr . La zy bot h wou ld h a ve a wor r y-fr ee life if t h ey don t
n eed t o a mor t ize a loa n wit h geomet r ica lly in cr ea s in g pa ymen t s .

Challe nge -- loan amort izat ion wit h ge ome t ric ally inc re as ing
payme nt s

Mr . Bu s y a n d Mr . La zy bot h love s t a n da r d a n n u it y pr oblems t h a t r equ ir e
t h e u s e of memor ized for mu la s s u ch a s
i n
a a n d
i n
a`` . Th ey bot h h a t e
geomet r ica lly in cr ea s in g a n n u it y a n d loa n a mor t iza t ion pr oblems . Th ey
wou ld gla dly s olve on e h u n dr ed s t a n da r d a n n u it y pr oblems t h a n
a mor t ize a mes s y loa n . Sa dly t h ou gh , SOA loves t o t es t t h e pr oblems t h a t
Mr . Bu s y a n d Mr . La zy h a t e.

5
Fin a lly t h e exa m da y h a s come. Mr . Bu s y a n d Mr . La zy wa lk in t o t h e
exa m r oom. Th e 1
s t
pr oblem in t h e exa m is a bou t in cr ea s in g a n n u it y a n d
loa n a mor t iza t ion .

Proble m 1
Da t e of loa n 1/ 1/ 2005
Amou n t of loa n $150,000
Ter m of loa n 25 yea r s
Pa ymen t s An n u a l pa ymen t s wit h fir s t pa ymen t du e
12/ 31/ 2005. Ea ch s u bs equ en t pa ymen t is 2% la r ger
t h a n t h e pr eviou s pa ymen t .
In t er es t r a t e 8.5% a n n u a l effect ive
Qu es t ion : Wh a t s t h e t ot a l in t er es t pa id du r in g t h e fir s t 18 pa ymen t s ?

Mr. Bus y:

Th ou gh n ot fon d of t h is t ype of pr oblems , Mr . Bu s y wa s n ot a bit wor r ied.
Wh en pr epa r in g for Exa m FM, Mr . Bu s y bou gh t t h e s olu t ion ma n u a l for
Br over ma n s t ext book. He s olved a ll of t h e pr a ct ice pr oblems in t h e
s olu t ion ma n u a l. Some of t h e pr oblems in t h e s olu t ion ma n u a ls a r e
geomet r ic a n n u it y a n d loa n a mor t iza t ion pr oblems .

In a ddit ion , Mr . Bu s y bou gh t a n ot h er ma n u a l wit h t on s of pr a ct ice
pr oblems a n d s olved a ll t h e pr a ct ice pr oblems in t h e ma n u a l.

Mr . Bu s y felt r ea dy t o t a ckle t h is pr oblem. He s olved over a t h ou s a n d
pr a ct ice pr oblems . Su r ely s ome of t h e pr oblems h e s olved wer e a bou t
loa n a mor t iza t ion wh er e pa ymen t s wer e geomet r ica lly in cr ea s in g. He
s t a r t ed t o men t a lly s ea r ch for h ow h e s olved s u ch pr oblems in t h e pa s t ,
h opin g t o r eca ll a qu ick s olu t ion .

To h is dis ma y, Mr . Bu s y cou ldn t r emember a n y qu ick s olu t ion s t o loa n
a mor t iza t ion wit h geomet r ica lly in cr ea s in g pa ymen t s . Th ou gh h e s olved
ma n y pr oblems befor e t h e exa m, Mr . Bu s y wa s a lwa ys in a r u s h t o s olve
t h e n ext pr a ct ice pr oblem. He n ever h a d t h e t ime t o con den s e h is
s olu t ion s t o qu ickly r eca lla ble s olu t ion s r ea dy t o be u s ed in t h e exa m. He
didn t even h a ve t h e t ime t o t h or ou gh ly u n der s t a n d t h e ba s ic con cept
beh in d loa n a mor t iza t ion a n d beh in d t h e pr es en t va lu e ca lcu la t ion of a
geomet r ica lly in cr ea s in g a n n u it y. He wa s a lwa ys in a big h u r r y t o s olve
mor e pr a ct ice pr oblems .

Time s eemed t o go mu ch fa s t er in t h e exa m r oom. An d t h e pr es s u r e wa s
keen . 5 min u t es pa s s ed. Mr . Bu s y wa s goin g n owh er e. Relu ct a n t ly, Mr .
Bu s y a ba n don ed t h is pr oblem a n d moved t o t h e n ext on e.
6
Mr. Lazy:

Un like Mr . Bu s y, Mr . La zy h a s a la zy a ppr oa ch t o loa n a mor t iza t ion .

Mr . La zy r ea lized t h a t loa n a mor t iza t ion a n d geomet r ic a n n u it y pr oblems
wer e r epea t edly t es t ed in t h e pa s t . Yea r a ft er yea r , SOA a s ks ca n dida t es
t o a mor t ize a loa n . Somet imes t h e loa n t o be a mor t ized h a s level
pa ymen t s ; ot h er t imes t h e pa ymen t s a r e a r it h met ica lly or geomet r ica lly
in cr ea s in g or decr ea s in g. Loa n a mor t iza t ion a n d geomet r ic a n n u it y
pr oblems a r e s o pr edict a ble t h a t Mr . La zy s u s pect ed t h a t SOA wou ld t es t
it a ga in t h is yea r wh en h e t a kes t h e exa m.

Mr. Lazy s t art s t o s t rat e gize :

1. SOA loves t o t es t loa n a mor t iza t ion . Su ch a pr oblem is doomed t o
occu r wh en I t a ke t he exa m.

2. Loa n a mor t iza t ion is n a s t y, es pecia lly wh en t h e pa ymen t s a r e
geomet r ica lly in cr ea s in g or decr ea s in g. It s h a r d for me t o figu r e it
ou t fr om s cr a t ch in t h e h ea t of t h e exa m.

3. Im la zy. I wa n t t o pas s Exa m FM wit h lea s t effor t .

Mr. Lazys c onc lus i on:

1. Befor e t h e exa m, Ill des ign a s t a n da r d cookie-cu t t er s olu t ion t o
loa n a mor t iza t ion wit h geomet r ica lly in cr ea s in g pa ymen t s . Th is
wa y, I don t h a ve t o in ven t a s olu t ion fr om s cr a t ch in t h e exa m.

2. Ill ma ke my s olu t ion les s t h a n 3 min u t e lon g; 3 min u t es is pr et t y
mu ch a ll t h e t ime I h a ve per qu es t ion in t h e exa m.

3. Ill wa lk in t o t h e exam r oom wit h t h e 3 min u t e s olu t ion s cr ipt
r ea dy in my h ea d. Ill u s e t h is s cr ipt t o s olve a n a s t y loa n
a mor t iza t ion pr oblem 100% r igh t in 3 min u t es u n der pr es s u r e.

Re s ult :

1. Mr. Bus y got a 5 in t he e xam. Th ou gh h e s olved h u n dr eds of
pr a ct ice befor e t h e exa m, h e n ever t r ied t o bu ild a n y r eu s a ble
s olu t ion s cr ipt s t o a n y of t h e common ly t es t ed pr oblems in Exa m
FM. As a r es u lt , ever y r epea t a ble pr oblem t es t ed in t h e exa m
beca me a br a n d n ew pr oblem, wh ich h e mu s t s olve fr om s cr a t ch .
Th is , in t u r n , ma kes h is s olu t ion lon g a n d pr on e t o er r or s . Sor r y,
Mr . Bu s y. Good lu ck t o you r 2
n d
t r y for Exa m FM.
7
2. Mr. Lazy got a 6 in t he e xam. He didn t bot h er t o s olve a n y
pr a ct ice pr oblems in a n y t ext books . Nor did h e bu y old SOA
pr oblems du g u p fr om t h e gr a veya r d. He ju s t down loa ded t h e
Sa mple FM Qu es t ion s a n d Ma y a n d November 2005 FM exa m
fr om SOA webs it e. Th en for ever y pr oblem t es t ed in t h e Sa mple
FM Exa m a n d 2005 FM exa m, h e bu ilt a r eu s a ble 3 min u t es
s olu t ion s cr ipt . Th en wh en h e wa s t a kin g FM exa m, h e s olved a ll
of t h e r epea t a ble pr oblems u s in g h is s cr ipt s . Of cou r s e, SOA t h r ew
in s ome n ew pr oblems , t o wh ich Mr . La zy s imply gu es s ed t h e
a n s wer s . Nice job, Mr . La zy. See you in Exa m M.

Le s s ons t o be le arne d from Mr. Bus y and Mr. Lazy:

Good e xam t ake rs s olve proble ms . Gre at e xam t ake rs build 3 minut e
s olut ion proc e s s e s (i. e . s c ript s ). For exa mple, good ca n dida t es ca n
s olve ma n y in t egr a t ion pr oblems u s in g in t egr a t ion -by-pa r t s :

2 2 2 2
3 2 4
, , , , ...
x x x
x
x e dx x e dx x e dx x e dx

} } } }
However , beca u s e t h e in t egr a t ion -by-pa r t s is a complex a n d er r or -pr on e
pr oces s , t h os e ca n dida t es wh o u s e t h is met h od oft en flu s t er in t h e h ea t
of t h e exa m.

Gr ea t exa m t a ker s , on t h e ot h er h a n d, focu s on bu ildin g a fla wles s
s olu t ion pr oces s . Pr ior t o t h e exa m, t h ey bu ilt t h e followin g gen er ic
s olu t ion :

u u
u u
u
+

| |
= + +
|
\ .
}
2 / 2 2 /
1
[( ) ]
x a
a
x e dx a e
Aft er get t in g t h is gen er ic pr oces s r igh t , gr ea t exa m t a ker s s imply a pply
t h is gen er ic s olu t ion t o ever y in t egr a t ion pr oblem
u
u
+

| |
|
\ .
}
2 /
1
x
a
x e dx a n d
a r e a ble t o s olve s imila r pr oblems 100% r igh t in a h u r r y.

Why building a proc e s s is s upe rior t o s olving proble ms

Wit h a gen er ic pr oces s , if you h a ve s olved on e pr oblem, you h a ve s olved
t h is t ype of t h e pr oblems on ce a n d for a ll. In con t r a s t , if you s olve 99
in dividu a l pr oblems wit h ou t a bu ildin g gen er ic pr oces s , you r e n ever s u r e
t h a t you ca n s olve t h e 100
t h
pr oblem cor r ect ly.

8
Bu ildin g pr oces s , n ot s olvin g pr oblems , is t h e mos t efficien t wa y t o pa s s
SOA exa ms , es pecia lly wh en you a r e s h or t of s t u dy t ime. Follow t h is
s t u dy met h od a n d r igor ou s ly bu ild a fla wles s s olu t ion pr oces s for ea ch of
t h e pr eviou s ly t es t ed FM pr oblems . Next , t es t you r s olu t ion pr oces s in
t h e exa m con dit ion a n d s olve a ll t h e pr eviou s ly t es t ed FM pr oblems
100% r igh t . Us e t h is s t u dy met h od for Exa m M a n d C. You ll zip t h r ou gh
t ou gh exa ms wit h a fr a ct ion of s t u dy t ime wh ile ot h er bu s y folks get
s t u ck in on e exa m for yea r s .

It ma y t a ke you a lit t le wh ile t o get u s ed t o t h is pr oces s -or ien t ed s t u dy
met h od. Th ou gh you ma y feel in s ecu r e wh en ot h er ca n dida t es boa s t of
h a vin g s olved 1,000 pr a ct ice pr oblems , plea s e be a s s u r ed t h a t t h is
a ppr oa ch is fa r s u per ior .

Th is ma n u a l is wr it t en t o t ea ch you h ow t o bu ild a gen er ic pr oces s t o
s olve SOA FM pr oblems . It s n ot a book t o give you 500 pr oblems for you
t o s olve.

Trut hs about Exam FM

1. To pa s s Exa m FM, you n eed t o lea r n h ow t o s olve pr oblems in a
h u r r y u n der pr es s u r e. In ma n y pr ofes s ion s , t h e differ en ce bet ween
a n exper t a n d a n a ma t eu r is t h a t a n exper t ca n s olve a r ou t in e
pr oblem fla wles s ly in a h u r r y u n der pr es s u r e, wh ile a n a ma t eu r
ca n s olve a pr oblem r igh t on ly u n der n o t ime con s t r a in t s . For
exa mple, a n exper t ca r mech a n ic ca n ch a n ge a fla t t ir e fla wles s ly
in les s t h a n 15 min u t es . In compa r is on , a ma t eu r s like me ca n
ch a n ge a fla t t ir e on ly a ft er s ever a l h ou r s . Simila r ly, t h os e wh o
pa s s FM ca n gen er a lly s olve a complex pr oblem in 3 min u t es u n der
t h e exa m pr es s u r e, wh ile t h os e wh o fa il migh t be a ble t o s olve a
complex pr oblem per h a ps in t wen t y min u t es .

2. Un der s t a n d wh a t s goin g in t h e r ea l wor ld. On e common mis t a ke
in pr epa r in g for SOA exa ms a t a ll levels is t o t r ea t bu s in es s
pr oblems a s pu r e ma t h pr oblems . For exa mple, wh en s t u dyin g
s h or t s a les , ma n y ca n dida t es s imply s olve on e s h or t s a le pr oblem
a ft er a n ot h er wit h ou t r ea lly u n der s t a n din g wh a t s goin g on in a
s h or t s a le. If you don t u n der s t a n d t h e bu s in es s es s en ce in a s h or t
s a le, s olvin g pr oblems is ga r ba ge in , ga r ba ge ou t a n d you don t
lea r n mu ch . Wh en lea r n in g a bu s in es s con cept s u ch a s s h or t
s a les , pr icin g of a bon d bet ween t wo cou pon da t es , immu n iza t ion ,
ca s h flow ma t ch in g, t r y t o u n der s t a n d wh a t s goin g on t h e r ea l
wor ld. Th in k t h r ou gh t h e bu s in es s mea n in g. Th is wa y, you ll fin d
t h a t difficu lt for mu la s begin t o ma ke s en s e. You ll be a ble t o s olve
pr oblems mu s t fa s t er .
9
3. Alwa ys s olve pr oblems s ys t ema t ica lly. Res ea r ch in dica t es t h a t
exper t s become exper t s beca u s e t h ey a lwa ys u s e s ys t ema t ic
a ppr oa ch es t o pr oblem s olvin g. Th ey n ever s olve pr oblem
h a ph a za r dly.

4. Wh en pr epa r in g for Exa m FM, focu s on bu ildin g a fla wles s s olu t ion
pr oces s t o a ll of t h e pr eviou s ly t es t ed pr oblems , n ot on a imles s ly
s olvin g on e pr oblem a ft er a n ot h er wit h a s h a ky pr oces s . If you
s olve a gr ea t n u mber of pr oblems (in clu din g SOA pr oblems ) wit h
s h a ky pr oces s , you ll ma ke t h e s a me mis s t a t e over a n d over . In
con t r a s t , if you h a ve a cor r ect pr oces s , you ll fin d t h e r igh t a n s wer
wit h ou t t h e n eed t o s olve ma n y pr oblems .

5. Simplify fa n cy ja r gon a n d complex for mu la s in t o s imple on es .
Wh ile t a lkin g fa n cy a n d t h in kin g fa n cy ma y impr es s lot of
a ma t eu r s , t a lkin g s imple a n d t h in kin g s imple a r e t h e key t o
s olvin g t h or n y pr oblems 100% r igh t in a h u r r y u n der pr es s u r e.
Common s en s e con cept s a n d s imple s olu t ion s a r e a lwa ys t h e
ea s ies t s olu t ion s t o r emember a n d u s e in t h e h ea t of t h e exa m.
Complex a n d u n in t u it ive con cept s a n d for mu la s a r e pr on e t o
er r or s . For exa mple, ma n y ca n dida t es wa s t e t h eir t ime memor izin g
t h e fa n cy ph r a s e a n n u it ies pa ya ble mor e (or les s ) fr equ en t ly t h a n
t h e in t er es t is con ver t ible a n d t h e r ela t ed complex for mu la s . Wh a t
t h ey s h ou ld h a ve don e is t o s implify complex a n n u it ies in t o s imple
a n n u it ies a n d t h r ow a wa y t h e fa n cy ph r a s e a n d complex for mu la s
on ce for a ll.

Exa mple. Th e in t er es t r a t e is
( ) 12
12% i = , bu t t h e a n n u it y pa ymen t s
of $1 a r e ma de qu a r t er ly in a r r ea r s for 2 yea r s . If you n eed t o
ca lcu la t e t h e pr es en t va lu e of t h is a n n u it y, u s e t h e pa ymen t
fr equ en cy (qu a r t er ly) a s t h e in t er es t compou n din g per iod a n d
ca lcu la t e t h e qu a r t er ly effect ive in t er es t r a t e:

( )
3
3
12
12%
1 1 1 1 3.03%
12 12
i
(
(
+ = + =
(
(



Aft er u s in g qu a r t er ly a s t h e compou n din g per iod, t h e or igin a l
a n n u it y becomes a s t a n da r d immedia t e a n n u it y wit h 8 qu a r t er ly
pa ymen t s . Th e pr es en t va lu e of t h is a n n u it y is
3.03% 8
a .
Th is a ppr oa ch is fa r bet t er t h a n u s in g t h e followin g complex
for mu la :

10
( )
( )
1
n i
n
m
m
v
i
a

=
6. Rely on r eca llin g a pr e-bu ilt 3 min u t e s olu t ion s cr ipt t o s olve
r epea t a ble pr oblems in FM. Th r ee min u t es is like t h e blin k of a n
eye in t h e h ea t of t h e exa m. In t h r ee min u t es , mos t people ca n , a t
bes t , on ly r egu r git a t e s olu t ion s t o fa milia r pr oblems . Mos t likely,
t h ey ca n n ot in ven t a fr es h s olu t ion t o a pr eviou s ly u n s een t ype of
pr oblem. In ven t in g a s olu t ion r equ ir es t oo mu ch t h in kin g a n d t oo
mu ch t ime. In fa ct , if you fin d you r s elf h a vin g t o t h in k t oo mu ch in
t h e exa m, pr epa r e t o t a ke Exa m FM a ga in .

How t o s t udy hard ye t fail t he e xam mis e rably

1. Wa lk in t o t h e exa m r oom wit h ou t a men t a l 3 min u t e s olu t ion
s cr ipt , h opin g t o in ven t s olu t ion s on t h e s pot . Th is is by t h e fa r t h e
mos t common mis t a ke. Let s look a t a few common myt h s :

Myt h Re alit y Che c k
#1 I don t n eed a s cr ipt .
I t h in k fa s t on my feet .
Wh y pu t t in g you r s elf on t h e s pot wh en
you ca n ea s ily come u p wit h a s cr ipt
a h ea d of t ime? Never t a ke a n y
u n n eces s a r y r is ks .
#2 I don t n eed a s cr ipt .
Ill ou t per for m mys elf in
t h e exa m.
Th er es n o s u ch t h in g a s ou t per for min g
you r s elf in t h e exa m. You a lwa ys u n der -
per for m a n d s cor e les s t h a n wh a t you r
kn owledge a n d a bilit y des er ve. Th is is
la r gely du e t o t h e t r emen dou s a mou n t of
pr es s u r e you in evit a bly feel in t h e exa m. If
you don t h a ve a s cr ipt r ea dy for a n exa m
pr oblem, don t cou n t on s olvin g t h e
pr oblem on t h e s pu r of t h e momen t .
#3 I don t n eed a s cr ipt .
If I s olve h u n dr eds of
pr a ct ice pr oblems befor e
t h e exa m, a s olu t ion will
a u t oma t ica lly come t o
me wh en Im t a kin g t h e
exa m.
A s olu t ion ma y come t o you
a u t oma t ica lly, bu t s u ch a s olu t ion is oft en
cr u de, complex a n d pr on e t o er r or s . Even
if you h a ve s olved a gr ea t n u mber of
pr a ct ice pr oblems , you s t ill n eed t o r edu ce
you r s olu t ion s t o a ea s ily r epea t a ble 3
min u t e pr oces s .
2. Wa lk in t o t h e exa m r oom wit h ou t ma s t er in g SOA pr oblems . A
Ch ief Execu t ive Officer wa s a bou t t o r et ir e. He a s ked for t h e t h r ee
mos t pr omis in g ca n dida t es t o come t o h is office for a qu iz. Aft er
t h e t h r ee ca n dida t es a r r ived, t h e CEO a s ked t h e fir s t ca n dida t e,
How mu ch is on e plu s on e? On e plu s on e is t wo, s ir , r eplied t h e
11
fir s t ca n dida t e. Th e CEO s h ook h is h ea d dis a ppoin t edly. He t u r n ed
t o t h e s econ d ca n dida t e a n d a s ked t h e s a me qu es t ion . Th e s econ d
ca n dida t e r eplied, On e plu s on e is t h r ee, s ir . On ce a ga in , t h e
CEO s h ook h is h ea d. Fin a lly, h e t u r n ed t o t h e t h ir d ca n dida t e a n d
a s ked a ga in , How mu ch is on e plu s on e? Th e t h ir d ca n dida t e
r eplied, How mu ch do you wa n t it t o be, s ir ? Th e CEO s miled a n d
a ppoin t ed t h e t h ir d ca n dida t e a s t h e n ext CEO.

Key poin t s
To pa s s Exa m FM, you n eed t o t ell SOA wh a t it wa n t s t o h ea r .
SOA does n ot wa n t you t o be cr ea t ive. SOA wa n t s you t o
demon s t r a t e u n der s t a n din g of cor e con cept s t h r ou gh a
s t a n da r d a n d met h odica l s olu t ion in keepin g wit h SOA for ma t .

If you ma s t er SOA pr oblems , you pa s s FM; if you do n ot , you
fa il.

Wh ile pr a ct ice pr oblems in t ext books , s t u dy ma n u a ls , or
s emin a r s a r e u s efu l, a lwa ys ma s t er SOA pr oblems befor e
ma s t er in g a n y ot h er pr oblems .

3. Solve h u n dr eds of pr a ct ice pr oblems (even SOA pr oblems ) wit h ou t
gen er a t in g r eu s a ble s olu t ion s . In ever y exa m s it t in g, t h er e a r e
a lwa ys bu s y ca n dida t es wh o t a ke gr ea t pr ide in s olvin g h u n dr eds
of SOA pr oblems a dmin is t er ed ma n y yea r s ba ck. Wh ile s olvin g
pr oblems a r e n eces s a r y for pa s s in g Exa m FM, s olvin g t oo ma n y
pr oblems a dds lit t le va lu e. Her e is wh y:

Solvin g t oo ma n y pr oblems en cou r a ges ga r ba ge in , ga r ba ge
ou t . Wh en a ca n dida t e is bu s y s olvin g a gr ea t n u mber of
pr a ct ice pr oblems , oft en t h e focu s is on s ea r ch in g for a n y
s olu t ion t h a t ma gica lly pr odu ces t h e cor r ect a n s wer pr ovided
in t h e book (oft en t h e book mer ely pr ovides a n a n s wer wit h
lit t le expla n a t ion ). Th is en cou r a ges pr oblem-s olvin g wit h ou t
fu lly u n der s t a n din g t h e n u a n ces of t h e pr oblem. If you r goa l
is t o s olve 800 pr a ct ice pr oblems in 3 mon t h s , you mis s t h e
poin t of fu lly u n der s t a n din g cor e con cept s a n d pr oblems .

Solvin g t oo ma n y pr a ct ice pr oblems exa gger a t es you r a bilit y.
Wh en a ca n dida t e focu s es on s olvin g h u n dr eds of pr a ct ice
pr oblems , h e oft en does n ot pu t h ims elf u n der exa m-like
con dit ion s . Th is a lmos t a lwa ys lea ds t o in efficien t s olu t ion s ,
s olu t ion s t h a t look good on pa per bu t fa ll a pa r t in t h e h ea t of
t h e exa m.

12
Re c omme nde d s t udy me t hod

1. Sen s e befor e s t u dy. Befor e open in g a n y t ext books , ca r efu lly look a t
Sa mple FM a n d get a feel for t h e exa m s t yle. Th is pr even t s you
fr om wa s t in g t ime t r yin g t o ma s t er t h e wr on g t h in g.

2. Qu ickly go over t h e t ext book a n d s t u dy t h e fu n da men t a ls (t h e cor e
con cept s a n d for mu la s ). Do n ot a t t empt t o ma s t er t h e complex
pr oblems in t h e t ext books . Solve s ome ba s ic pr oblems t o en h a n ce
you r u n der s t a n din g of t h e cor e con cept s .

3. Pu t you r s elf u n der exa m con dit ion s a n d pr a ct ice t h e pr eviou s
Cou r s e 2 exa ms (if t h e pr oblems a r e s t ill on t h e s ylla bu s ), t h e
Sa mple FM Exa m, a n d November 2005 FM pr oblems .
(1) Pu t you r s elf u n der t h e s t r ict exa m con dit ion .
(2) Pr a ct ice on e exa m a t a t ime.
(3) Aft er t a kin g a pr a ct ice exa m, t a ke s ever a l da ys t o a n a lyze
wh a t you did r igh t a n d wh a t you did wr on g (don t do t h is in
a h u r r y).
(4) For ea ch pr oblem in t h e pr a ct ice exa m, bu ild a r eu s a ble 3
min u t e s olu t ion s cr ipt .
(5) Ta ke t h e s a me pr a ct ice exa m t h e 2
n d
t ime, u s in g you r 3
min u t e s olu t ion s cr ipt . Th is pu t s you r 3 min u t e s olu t ion
s cr ipt t o t es t . Fin d wh ich s cr ipt wor ks a n d wh ich does n t .
Impr ove you r 3 min u t e s olu t ion s cr ipt s .
(6) Ta ke t h e n ext SOA pr a ct ice exa m, r epea t in g St ep (1) t o St ep
(5) lis t ed a bove. You will h a ve mor e a n d mor e 3 min u t e
s olu t ion s cr ipt s . You will con t in u e r efin e you r 3 min u t e
s olu t ion s cr ipt s .

4. Wor k a n d r ewor k Sa mple FM Exa m a n d a n y r elea s ed FM exa ms
u n t il you ca n get t h em r igh t 100%. Con t in u e r efin in g you r 3
min u t e s olu t ion s cr ipt s . Ma s t er in g Sa mple FM Exa m a n d n ewly
r elea s ed FM exa ms is t h e fou n da t ion for pa s s in g FM.

5. Don t wor r y a bou t s olvin g t h e s a me FM pr oblems over a n d over . No
ca n dida t es , h owever in t elligen t , ca n over -s olve SOA FM exa ms .
Bes ides , if you r ea lly pu t you r s elf u n der t h e exa m con dit ion , it s
h igh ly u n likely t h a t you ll memor ize t h e a n s wer t o a pr eviou s ly
s olved pr oblem.

6. Never , n ever wa lk in t o t h e exa m r oom wit h ou t bein g a ble t o s olve
Sa mple FM pr oblems a n d n ewly r elea s ed FM exa ms 100% r igh t .
7. Set u p a s t u dy s ch edu le a n d follow it t h r ou gh .
13
8. In t h e t wo weeks pr ior t o t h e exa m da t e, dr y r u n Sa mple FM Exa m
a n d Ma y 2005 FM exa m, even t h ou gh t h is ma y be you r 4
t h
or 5
t h
dr y r u n . You ca n n ever pr a ct ice SOA FM pr oblems t oo mu ch .

How t o bui ld a 3 mi nut e s olut ion s c ript

Bu ildin g a 3 min u t e s olu t ion s cr ipt is cr it ica l t o pa s s in g Exa m FM. Let s
look a t a few exa mples . Th ey a r e n ot n eces s a r ily t h e bes t , bu t t h ey r ea lly
wor k in t h e h ea t of t h e exa m. An d feel fr ee t o cr ea t e you r own s olu t ion
s cr ipt s .

3 minut e s olut ion s c ript e xample #1 -- loan amort izat ion whe re
payme nt s are le ve l

Tr a dit ion a l met h od (3 min u t e s olu t ion s cr ipt )
A loa n is bor r owed a t t ime zer o. It is r epaid by n level pa ymen t s of X , t h e
1
s t
pa ymen t occu r r in g a t 1 t = . In ot h er wor ds , a loa n is r epa id t h r ou gh
a n n yea r a n n u it y immedia t e wit h level pa ymen t s of X .
Qu es t ion h ow t o s plit ea ch level pa ymen t X in t o a pr in cipa l por t ion a n d
t h e in t er es t por t ion ?
Time t 0 1 2 k n
Cash flow $X $X $X $X
Principal
n
Xv
1 n
Xv

...
1 n k
Xv
+
Xv
Interest
n
X Xv
1 n
X Xv

...
1 n k
X Xv
+
X Xv
Plea s e r efer t o t h e t ext book t o u n der s t a n d wh y t h e pr in cipa l por t ion is
in deed
n
Xv ,
1 n
Xv

,
1 n k
Xv
+
, Xv a t 1, 2, , t k n = r es pect ively.

You s h ou ld wa lk in t o t h e exa m r oom wit h t h is r u le memor ized in you r
h ea d. Th en if a n exa m pr oblem a s ks you t o s plit a level pa ymen t in t o
pr in cipa l a n d in t er es t , you don t n eed t o ca lcu la t e t h e a n s wer fr om
s cr a t ch . You s imply a pply t h is memor ized s cr ipt a n d qu ickly fin d t h e
a n s wer .

Impr oved 3 min u t e s olu t ion s cr ipt --- Ima gin a r y ca s h flow met h od
Th e a bove s cr ipt h a s a t r ou ble s pot . You h a ve t o memor ize t h a t t h e
pr in cipa l is
1 n k
Xv
+
a t t ime k . However , memor izin g t h e dis cou n t fa ct or
1 n k
v
+
is a pa in . In t h e h eat of t h e exa m, you migh t u s e a wr on g dis cou n t
14
fa ct or s u ch a s
n k
v

or
k
v . How ca n you r emember t h e cor r ect dis cou n t
fa ct or ? Th is lea ds t o t h e ima gin a r y ca s h flow met h od.

To fin d t h e pr in ciple por t ion of ea ch pa ymen t , we a dd a n ima gin a r y ca s h
flow on e s t ep a ft er t h e fin a l pa ymen t (i.e. we a dd a ca s h flow of $X a t
1 n + ).
Time t 0 1 2 k n 1 n +
Cash flow X X ... X X
1 n k
X v
+


Imaginary
Cash flow
$X
Principal
n
Xv
1 n
Xv

...
1 n k
Xv
+
Xv
Interest
n
X Xv
1 n
X Xv

...
1 n k
X Xv
+
X Xv
To fin d t h e pr in cipa l por t ion of t h e pa ymen t X occu r r in g a t t k = wh er e
k is a pos it ive in t eger a n d 1 k n s s , we s imply dis cou n t ou r ima gin a r y
ca s h flow $X a t 1 n + t o t k = . Th e dis cou n t ed ca s h flow
1 n k
X v
+
is t h e
pr in cipa l por t ion of t h e pa ymen t .
Th e in t er es t por t ion of t h e pa ymen t X occu r r in g a t t k = is
1 n k
X X v
+
.
Do we n eed t o come u p wit h a n in t u it ive expla n a t ion for t h is s cr ipt ? We
don t h a ve t o. If t h is met h od gen er a t es t h e cor r ect a n s wer , well u s e it a s
ou r s cr ipt , even t h ou gh we ma y n ot h a ve a n in t u it ive expla n a t ion for it .

Now a s s u me t h a t you wa lk in t o t h e exa m r oom wit h t h is s cr ipt in you r
h ea d. An d you s ee t h e followin g pr oblem:

15
Proble m 1

Da t e of loa n 1/ 1/ 2005
Ter m of loa n 30 yea r s
Pa ymen t s An n u a l pa ymen t s of $2,000 a t t h e en d of ea ch yea r
1
s t
pa ymen t 12/ 31/ 2005
Loa n in t er es t 6% a n n u a l effect ive
Que s t ion -- What s t he pre s e nt value of t he int e re s t payme nt s at
1 / 1 / 2 0 0 5 ove r t he life of t he loan at a 1 0 % annual e ffe c t ive int e re s t
rat e ?

Th is is NOT a s imple pr oblem. If you h a ve t o figu r e ou t t h e s olu t ion fr om
s cr a t ch u s in g t h e pr os pect ive or r et r os pect ive met h od, you ma y h a ve t o
s pen d five t o t en min u t es on it . Wh a t s mor e, you a r e likely t o ma ke a n
er r or h er e a n d t h er e if you s olve a pr oblem fr om s cr a t ch .

Let s u s e ou r ima gin a r y ca s h flow s cr ipt t o s olve t h is pr oblem.

Time t 0 1 2 k 30 31
Cash flow 2 2 2 2
31
2
k
v



Imaginary
cash flow
2
Principal
30
2v
29
2v ...
31
2
k
v

... 2v
Interest
30
2 2v
29
2 2v
31
2 2
k
v

... 2 2v
Fir s t , t o s imply ou r ca lcu la t ion , well u s e $1,000 a s on e u n it of mon ey.
So $2,000 a n n u a l pa ymen t is 2 u n it s of mon ey.

Next , well a dd a n ima gin a r y ca s h flow of 2 a t 31 t = . Th en we fin d t h e
pr in cipa l por t ion of ea ch level pa ymen t by dis cou n t in g t h is ima gin a r y
ca s h flow t o 1, 2,...., 30 t = .
Th e pr es en t va lu e a t 1/ 1/ 2005 of t h e in t er es t pa ymen t s over t h e life of
t h e loa n a t a 10% a n n u a l effect ive in t er es t r a t e:

( ) ( ) ( ) ( )
30 29 2 28 3 30
2 2 2 2 2 2 ... 2 2 PV v V v V v V v V = + + + +
16
In t h e a bove expr es s ion ,
1
1 6%
v =
+
a n d
1
1 10%
V =
+
.
( ) ( )
2 3 30 30 29 2 28 3 30
2 ... 2 ... PV V V V V v V v V v V vV = + + + + + + +
2 3 30
30 10%
... 9.42691447 V V V V a + + + + = =
30 31 30 1 31
30 29 2 28 3 30
1
1
1.06 1.1 1.1
... 2.92003944
1.1
1
1
1.06
v V V
v V v V v V vV
V
v


+ + + = = =


( ) ( ) 13.01375 $13, 013.75 2 9.42691447 2 2.92003944 PV ~ = =
Plea s e n ot e t h a t t h e ima gin a r y ca s h flow met h od a ls o wor ks for a loa n
r epa id by a n a n n u it y du e. Ill let you pr ove it .

A loa n is bor r owed a t t ime zer o. It is r epaid by n level pa ymen t s of X , t h e
1
s t
pa ymen t occu r r in g a t 0 t = . In ot h er wor ds , a loa n is r epa id t h r ou gh
a n n yea r a n n u it y du e wit h level pa ymen t s of X .
Qu es t ion h ow t o s plit ea ch level pa ymen t X in t o a pr in cipa l por t ion a n d
t h e in t er es t por t ion ?
Time t 0 1 2 k 1 n n
Cash flow $X $X $X $X $X
n k
Xv



Imaginary
cash flow
$X
Principal
n
Xv
1 n
Xv

n k
Xv

Xv
Interest
n
X Xv
1 n
X Xv


n k
X Xv

X Xv
17
3 minut e s olut ion s c ript e xample #2 -- loan amort izat ion whe re
payme nt s are NOT le ve l

Beca u s e SOA ca n a s k u s t o s plit n on -level pa ymen t s in t o pr in cipa l a n d
in t er es t , we n eed t o bu ild a 3 min u t e s olu t ion s cr ipt for t h is .

A loa n is bor r owed a t t ime zer o. It is r epaid by n pa ymen t s of
1
X ,
2
X , ,
n
X a t 1, 2,..., t n = r es pect ively. Of t h e t ot a l pa ymen t
1 2
...
k
X X X + + + ma de
du r in g t h e fir s t k pa ymen t s , h ow mu ch is t h e pr in cipa l pa ymen t ? How
mu ch is t h e in t er es t pa ymen t ?
Time t 0 1 2 k n
Cash flow
1
$X
2
$X $
k
X $
n
X
3 minut e s olut ion s c ript :

St ep 1 Ca lcu la t e
0
P , t h e ou t s t a n din g ba la n ce of t h e loa n a t 0 t = .
St ep 2 Ca lcu la t e
k
P , t h e ou t s t a n din g ba la n ce of t h e loa n a t t k =
immedia t ely a ft er t h e k th pa ymen t is ma de.
St ep 3 Ca lcu la t e
0 k
P P , t h e r edu ct ion of t h e ou t s t a n din g ba la n ce
bet ween 0 t = a n d t k = .
0 k
P P s h ou ld be t h e t ot a l pr in cipa l
r epa id du r in g t h e fir s t k pa ymen t s .
St ep 4 Ca lcu la t e ( ) ( )
1 2 0
...
k k
X X X P P + + + . Th is s h ou ld t h e t ot a l
in t er es t pa id du r in g t h e fir s t k pa ymen t s
Th e cor e logic beh in d t h is s cr ipt :
1 2 0
Total payments made Principal reduction
during the first
payments
... + Interest Payment
k k
k
X X X P P + + + =
_ _

18
Proble m 2

Da t e of loa n 1/ 1/ 2005
Amou n t of loa n $150,000
Ter m of loa n 25 yea r s
Pa ymen t s An n u a l pa ymen t s wit h fir s t pa ymen t du e
12/ 31/ 2005. Ea ch s u bs equ en t pa ymen t is 2% la r ger
t h a n t h e pr eviou s pa ymen t .
In t er es t r a t e 8.5% a n n u a l effect ive
Wh a t s t h e t ot a l in t er es t pa id du r in g t h e fir s t 18 pa ymen t s ?

Solut ion

Time t 0 1 2 3 18 19 25

Cash flow X 1.02X
2
1.02 X
17
1.02 X
18
1.02 X
24
1.02 X
St e p 1 ca lcu la t e t h e ou t s t a n din g ba la n ce a t 0 t = .
0
150, 000 P =
St e p 2 ca lcu la t e t h e ou t s t a n din g ba la n ce a t 18 t = immedia t ely a ft er t h e
18
t h
pa ymen t is ma de (we dis cou n t fu t u r e ca s h flows t o 18 t = ):

( )
18 19 2 24 7 18 19 2 24 7
18
1.02 1.02 ... 1.02 1.02 1.02 ... 1.02 P Xv Xv Xv X v v v = + + + = + + +
So we n eed t o ca lcu la t e X . Beca u s e t h e pr es en t va lu e of t h e loa n a t 0 t =
is 150,000, we h a ve:

2 2 3 24 25
1.02 1.02 ... 1.02 150, 000 Xv Xv Xv Xv + + + =
( )
2 2 3 24 25
1.02 1.02 ... 1.02 150, 000 X v v v v + + + =
( )
25 26 1 25 26
2 2 3 24 25
1
1.02 1.085 1.02 1.085
1.02 1.02 ... 1.02 12.10103628
1 1.02 1 1.02 1.085
v v
v v v v
v


+ + + = = =

150, 000
12, 395.6326
12.10103628
X = =
Th e ou t s t a n din g ba la n ce a t 18 t = immedia t ely a ft er t h e 18
t h
pa ymen t is :

( )
18 19 2 24 7
18
1.02 1.02 ... 1.02 P X v v v = + + +
19
( )
18 25 8 18 1 25 8
18 19 2 24 7
1
1.02 1.02 1.02 1.085 1.02 1.085
1.02 1.02 ... 1.02 7.714226
1 1.02 1 1.02 1.085
v v
v v v
v


+ + + = = =

( )
18
12, 395.6326 7.714226 95, 622.7113 P = =
St e p 3 Ca lcu la t e t h e r edu ct ion of t h e ou t s t a n din g ba la n ce bet ween
0 t = a n d 18 t = .
0 18
150, 000 95, 622.7113 54, 377.2887 P P = =
Th is is t h e t ot a l pr in cipa l pa ymen t du r in g t h e fir s t 18 pa ymen t s .

St e p 4 - Ca lcu la t e ( ) ( )
1 2 18 0 18
... X X X P P + + + . Th is is t h e t ot a l in t er es t
pa id du r in g t h e fir s t 18 pa ymen t s .
( )
2 17
1 2 18
18 2%
... 1 1.02 1.02 ... 1.02 X X X X Xs + + + = + + + + =
18 2%
12, 395.6326 265, 419.1574 s = =
( ) ( )
1 2 18 0 18
... 265, 419.1574 54, 377.2887 211, 041.8687 X X X P P + + + = =
Ext e nd our s c ript Wh a t if we wa n t t o ca lcu la t e t h e t ot a l pr in cipa l a n d
in t er es t du r in g t h e 3
r d
, 4
t h
, a n d 5
t h
pa ymen t s ? In ot h er wor ds , h ow ca n
we s plit
3 4 5
X X X + + in t o pr in cipa l a n d in t er es t ?
St e p 1 Ca lcu la t e
2
P , t h e ou t s t a n din g ba la n ce of t h e loa n a t 2 t =
immedia t ely a ft er t h e 2
n d
pa ymen t is made.
St e p 2 Ca lcu la t e
5
P , t h e ou t s t a n din g ba la n ce of t h e loa n a t 5 t =
immedia t ely a ft er t h e 5
t h
pa ymen t is ma de.
St e p 3 Ca lcu la t e
2 5
P P , t h e r edu ct ion of t h e ou t s t a n din g ba la n ce
bet ween 2 t = a n d 5 t = . Th is is t h e t ot a l pr in cipa l r epa id du r in g t h e 3
r d
,
4
t h
, a n d 5
t h
pa ymen t s .
St e p 4 Ca lcu la t e ( ) ( )
3 4 5 2 5
X X X P P + + . Th is s h ou ld t h e t ot a l in t er es t
pa id du r in g t h e 3
r d
, 4
t h
, a n d 5
t h
pa ymen t s .
Th e cor e logic beh in d t h is s cr ipt :
3 4 5 2 5
Total payments made Principal reduction
during 3rd, 4th, and
5th payments
+ Interest payment X X X P P + + =
_ _

20
Simila r ly, if you a r e a s ked t o s plit t h e 3
r d
pa ymen t in t o pr in cipa l a n d
in t er es t , you ca n cr ea t e a s cr ipt u s in g t h e followin g logic:

3 2 3
3rd payment
Principal reduction
+ Interest payment X P P =
_

3 minut e s olut ion s c ript e xample #3 pre s e nt value of a
ge ome t ric ally inc re as ing annuit y

Let s look a t ou r s olu t ion t o Pr oblem 2, wh ich in volves a geomet r ica lly
in cr ea s in g a n n u it y. In ou r s olu t ion , we ca lcu la t ed t h e pr es en t va lu e of
s ever a l geomet r ica lly in cr ea s in g a n n u it ies fr om s cr a t ch . Ea ch t ime, we
u s e t h e s u m r u le of a power s er ies :

2 1
...
1
n
n
a aq
a aq aq aq
q


+ + + + =

wh er e 1 q =
We a s k ou r s elves , Wh y n ot cr ea t e a s cr ipt for a geomet r ica lly in cr ea s in g
a n n u it y t o a void ca lcu la t in g it s pr es en t va lu e fr om s cr a t ch ? Lu ckily, we
fin d a s cr ipt :

( ) ( ) ( )
2 1
payments
1 1 1 ...... 1
n
n
k k k

+ + +

1
1
1 j
i k
n
k
k
a
=

+
+
( )
1
1
j
i k
n
k
a
=

+
``
For a geomet r ica lly in cr ea s in g a n n u it y wh er e
(1) n geomet r ica lly in cr ea s in g pa ymen t s a r e ma de a t a r egu la r in t er va l;
(2) t h e 1
s t
pa ymen t is $1;
(3) t h e n ext pa ymen t is a lwa ys ( ) 1 k + t imes t h e pr eviou s pa ymen t .
Th en
21
(1) Th e pr es en t va lu e on e s t ep befor e t h e 1
s t
pa ymen t is
1
1
1 j
i k
n
k
k
a
=

+
+
. Th is
va lu e h a s a fa ct or of
1
1 k +
beca u s e t h e geomet r ic pa ymen t pa t t er n a t on e
in t er va l pr ior t o t h e 1
s t
pa ymen t is
1
1 k +
. Th is va lu e a ls o h a s a n a n n u it y
fa ct or of
1
j
i k
n
k
a
=

+
, wh er e j is t h e a dju s t ed in t er es t r a t e.
(2) Th e pr es en t va lu e a t t h e 1
s t
pa ymen t t ime is ( )
1
1
j
i k
n
k
a
=

+
`` . Th e pr es en t
va lu e h a s a fa ct or of 1 beca u s e t h e 1
s t
pa ymen t is 1. Th is va lu e h a s a n
a n n u it y fa ct or of
1
j
i k
n
k
a
=

+
`` , wh er e j is t h e a dju s t ed in t er es t r a t e.
To fin d t h e pr oof of t h is s cr ipt , s ee t h e ch a pt er on geomet r ica lly
in cr ea s in g a n n u it y. For n ow, let s focu s on h ow t o u s e t h is s cr ipt .

Fr om t h is s cr ipt , we s ee t h a t t h e pr es en t va lu e of a geomet r ica lly
in cr ea s in g a n n u it y is a lwa ys t h e pr odu ct of a pa ymen t fa ct or a n d a n
a n n u it y fa ct or :

PV of geomet r ic a n n u it y = Pa ymen t Fa ct or An n u it y Fa ct or @
1
j
i k
k
=

+
Th e a bove s cr ipt is s imple yet power fu l. Let s r edo Pr oblem 2 u s in g t h e
geomet r ic a n n u it y s cr ipt .

Proble m 2

Da t e of loa n 1/ 1/ 2005
Amou n t of loa n $150,000
Ter m of loa n 25 yea r s
Pa ymen t s An n u a l pa ymen t s wit h fir s t pa ymen t du e
12/ 31/ 2005. Ea ch s u bs equ en t pa ymen t is 2% la r ger
t h a n t h e pr eviou s pa ymen t .
In t er es t r a t e 8.5% a n n u a l effect ive
Q: Wh a t s t h e t ot a l in t er es t pa id du r in g t h e fir s t 18 pa ymen t s ?

Solut ion

22
Time t 0 1 2 3 18 19 25

Cash flow X 1.02X
2
1.02 X
17
1.02 X
18
1.02 X
24
1.02 X
St e p 1 ca lcu la t e t h e ou t s t a n din g ba la n ce a t 0 t = .
0
150, 000 P =
St e p 2 ca lcu la t e t h e ou t s t a n din g ba la n ce a t 18 t = immedia t ely a ft er t h e
18
t h
pa ymen t is ma de.

( )
( ) 18
1
@ 18 @ j
i k
k
P payment factor t annuity factor =

+
= =
@ 18 payment factor t = is
17
1.02 X ;
1
@ j
i k
k
annuity factor =

+
is :
7
8.5% 2%
7 %
25 18
1.02 1
6.372549
j j
i k
k
j
a a a
= =

+
=
= =
17
18
7 % 6.372549
1.02
j
P Xa
=
=
Next , well ca lcu la t e X . Beca u s e t h e pr es en t va lu e of t h e loa n a t 0 t = is
150,000, we h a ve:

( )
( )
1
150, 000 of geometric annuity= @ 0 @ j
i k
k
PV payment factor t annuity factor =

+
= =

@ 0 payment factor t = is
1.02
X
if we ext en d t h e pa ymen t pa t t er n t o t ime zer o, well h a ve a pa ymen t of
1.02
X
a t t ime zer o

1
@ j
i k
k
annuity factor =

+
is :
25 % 6.372549
25
1
j
i k j
k
a a
=
=
+
=
25 % 6.372549
150, 000
1.02
j
X
a
=
=
Us in g BA II Plu s / BA II Plu s Pr ofes s ion a l, we qu ickly fin d t h a t

25 % 6.372549
12.34305703
j
a
=
=
23
( )
25 % 6.372549
150, 000 1.02
12, 395.6326
j
X
a
=
= =
( ) ( )
17 17
18
7 % 6.372549
1.02 1.02 12, 395.6326 5.50921141 95, 622.7113
j
P Xa
=
= = =
St e p 3 Ca lcu la t e t h e r edu ct ion of t h e ou t s t a n din g ba la n ce bet ween
0 t = a n d 18 t = .
0 18
150, 000 95, 622.7113 54, 377.2887 P P = =
Th is is t h e t ot a l pr in cipa l pa ymen t du r in g t h e fir s t 18 pa ymen t s .

St e p 4 - Ca lcu la t e ( ) ( )
1 2 18 0 18
... X X X P P + + + . Th is is t h e t ot a l in t er es t
pa id du r in g t h e fir s t 18 pa ymen t s .

( )
2 17
1 2 18
18 2%
... 1 1.02 1.02 ... 1.02 X X X X Xs + + + = + + + + =
18 2%
12, 395.6326 265, 419.1574 s = =
( ) ( )
1 2 18 0 18
... 265, 419.1574 54, 377.2887 211, 041.8687 X X X P P + + + = =
You s ee t h a t ou r ca lcu la t ion is mu ch fa s t er if we u s e t h e geomet r ic
a n n u it y s cr ipt .

3 minut e s olut ion s c ript e xample #4 pre s e nt value of an
arit hme t ic ally inc re as ing/ de c re as ing annuit y

Amon g a ll t h e a n n u it ies , a n a r it h met ica lly in cr ea s in g or decr ea s in g
a n n u it y is mos t pr on e t o er r or s . Er r or s a s s ocia t ed wit h a n a r it h met ica lly
in cr ea s in g or decr ea s in g a n n u it y oft en come fr om t wo s ou r ces :

In cor r ect ly iden t ify t h e ca s h flow pa t t er n . In a n in cr ea s in g or
decr ea s in g a n n u it y, we ca n ea s ily fin d t h e pr ecis e ca s h flow a t
ea ch t ime poin t . However , t o ca lcu la t e t h e pr es en t va lu e of a n
in cr ea s in g or decr ea s in g a n n u it y, kn owin g t h e pr ecis e ca s h flow a t
ea ch t ime is n ot en ou gh . Th e pr es en t va lu e for mu la for ces u s t o
fin d t h e pa t t er n by wh ich t h e ca s h flows ch a n ge over t ime.
Iden t ifyin g t h e ca s h flow pa t t er n is mu ch h a r der t h a n iden t ifyin g
ca s h flows .

24
In cor r ect ly a pply t h e pr es en t va lu e for mu la . Even if we fin d t h e
ca s h flow pa t t er n , we s t ill h a ve t o u s e t h e complex a n d a wkwa r d
for mu la t o ca lcu la t e t h e pr es en t va lu e. An y mis t a ke in u s in g t h e
for mu la will des t r oy ou r pr eviou s wor k.

If ca lcu la t in g t h e pr es en t va lu e of a n in cr ea s in g or decr ea s in g a n n u it y
s ou n ds complex, ima gin e s olvin g s u ch a pr oblem in t h e exa m con dit ion
wh er e you r h ea t bea t s fa s t a n d you r h a n ds t r emble. How ca n we fin d a
ma gic s cr ipt t h a t will en a ble u s t o ca lcu la t e t h e pr es en t va lu e of a n
a r it h met ica lly in cr ea s in g or decr ea s in g a n n u it y 100% in a h u r r y u n der
pr es s u r e?

Th e ba d n ews is t h a t we ca n t a lwa ys fin d a s cr ipt . Th e good n ews is t h a t
we ca n fin d a s cr ipt t h a t wor ks mos t of t ime.

3 minut e s olut ion s c ript -- PV of inc re as ing/ de c re as ing annuit y

If t h e # of ca s h flows is NOT en or mou s (fewer t h a n 50 for exa mple),
s imply en t er t h e ca s h flows in t o BA II Plu s / BA II Plu s Pr ofes s ion a l Ca s h
Flow Wor ks h eet .
Proble m 3

Yea r 1 $50,000
Yea r 2 t o Yea r 11 An n u a lly in cr ea s in g
by $5,000.
Yea r 11 t o Yea r 15 Level
22 en d-of-t h e yea r
depos it s t o a fu n d
Yea r 15 t o Yea r 22 An n u a lly decr ea s in g
by $6,000
In t er es t r a t e 8% a n n u a l effect ive
Ca lcu la t e t h e pr es en t va lu e of t h e fu n d.

Solut ion

We don t wa n t t o s pen d lot of t ime a n a lyzin g t h e ca s h flow pa t t er n or
u s in g t h e complex for mu la . We ju s t iden t ify a ll of t h e ca s h flows :

Time t 0 1 2 3 4 5 6 7 8 9 10 11
Cas h flow $0 $50 $55 $60 $65 $70 $75 $80 $85 $90 $95 $100
Time t 11 12 13 14 15 16 17 18 19 20 21 22
Cas h flow $100 $100 $100 $100 $100 $94 $88 $82 $76 $70 $64 $58
25
In t h e a bove t a ble, we u s e $1,000 a s on e u n it of mon ey t o s implify ou r
ca lcu la t ion .

Next , we en t er t h e a bove ca s h flows in t o Ca s h Flow Wor ks h eet :

CF0 C01 C02 C03 C04 C05 C06 C07 C08 C09 C10 C11
Cas h flow
$0 $50 $55 $60 $65 $70 $75 $80 $85 $90 $95 $100
F01 F02 F03 F04 F05 F06 F07 F08 F09 F10 F11
Fr equ en cy
1 1 1 1 1 1 1 1 1 1 6
C12 C13 C14 C15 C16 C17 C18
Cas h flow
$94 $88 $82 $76 $70 $64 $58
F12 F13 F14 F15 F16 F17 F18
Fr equ en cy
1 1 1 1 1 1 1
Set I=8 (i.e. t h e in t eres t r a t e is 8%).

You s h ou ld get : NPV = 797.80666333 (u n it s )= $797,806.66333

Proble m 4

An n u it y #1 An a mou n t pa ya ble a t t h e en d of ea ch qu a r t er
begin n in g wit h a $2,000 pa ymen t on 3/ 31/ 2005. Ea ch
s u bs equ en t pa ymen t is 1.5% la r ger t h a n t h e pr eviou s
pa ymen t . Th e a n n u it y pa ys for 15 yea r s .
An n u it y #2 An a mou n t pa ya ble a t t h e en d of ea ch yea r begin n in g
wit h a n X pa ymen t on 12/ 31/ 2005. Ea ch s u bs equ en t
pa ymen t is $30 les s t h a n t h e pr eviou s pa ymen t . Th e
a n n u it y pa ys for 20 yea r s .
in t er es t r a t e 9% a n n u a l effect ive
On 1/ 1/ 2007, t h e pr es en t va lu e of t h e r ema in in g An n u it y #1 equ a ls t h e
pr es en t va lu e of t h e r ema in in g a n n u it y #2.

Ca lcu la t e X .
Solut ion

Th is pr oblem is con cept u a lly s imple. However , t r a ckin g down t h e t imin gs
a n d a mou n t s of t h e ca s h flows of ea ch a n n u it y is a n igh t ma r e. Un les s
you u s e a s ys t ema t ic a ppr oa ch , you ll ma ke er r or s h er e a n d t h er e.

26
Fir s t , we dr a w a ca s h flow dia gr a m:

Annuit y #1

Time t 0 1 2 3 ... 15
(Year)
1
4
2
4
3
4
1
1
4
2
4
3
4
1
1
4
2
4
3
4
1
2005 2006 2007
13 4 =52 payments remaining
15 2 13 years remaining
| |
|
|
\ .
=
_
Payment factor =2,000(1.015)
7
Annuity factor =
52 j
a
( )
1
4
1 9% 1 1.5%
1 1.5%
j
(
+
(

=
+
PV of remaining payments =2,000(1.015)
7
52 j
a
Annuit y #2

Time t 0 1 2 3 4 20
(Year)
2005 2006 2007 2008
X 30 X ( ) 2 30 X ( ) 3 30 X ... ( ) 19 30 X
Annuity #2
18 payments remaining
20 2 18 years remaining =
_
27
Well br ea k down t h e r ema in in g pa ymen t s of An n u it y #2 in t o t wo
s t r ea ms :

Time t 0 1 2 3 4 20
(Year)
2005 2006 2007 2008
X X ... X
9 18 %
Xa
Time t 0 1 2 3 4 20
(Year)
2005 2006 2007 2008
( ) 2 30 ( ) 3 30 ... ( ) 19 30
PV of an increasing annuity
PV of the remaining payments =
9 18 %
Xa - PV of a increasing annuity
To pin down t h e PV of t h e in cr ea s in g a n n u it y, well u s e BA II Plu s / BA II
Plu s Pr ofes s ion a l Ca s h Flow Wor ks h eet . En t er t h e followin g ca s h flows :

CF0 C01 C02 C03 C04 C05 C06 C07 C08 C09
Cash flow
$0 $2 $3 $4 $5 $6 $7 $8 $9 $10
F01 F02 F03 F04 F05 F06 F07 F08 F09
Frequency
1 1 1 1 1 1 1 1 1
C10 C11 C12 C13 C14 C15 C16 C17 C18
Cash flow
$11 $12 $13 $14 $15 $16 $17 $18 $19
F10 F11 F12 F13 F14 F15 F16 F17 F18
Frequency
1 1 1 1 1 1 1 1 1
In t h e a bove ca s h flow t a ble, we u s ed $30 a s on e u n it of mon ey t o
s implify ou r da t a en t r y.
28
Next , in Ca s h Flow Wor ks h eet , s et t h e in t er es t r a t e t o 9%. You s h ou ld
get : NPV=72.39722563. Beca u s e on e u n it of mon ey r epr es en t s $30, s o
t h e r ea l NPV is

(30) 72.39722563=2,191.916769

You migh t gr oa n a t s u ch a s olu t ion a s u gly. However , t h is qu ick a n d
dir t y s olu t ion h a s it s bea u t y: it pr ovides a mech a n ic s olu t ion t o a
complex pr oblem. It r ea lly wor ks in t h e h ea t of t h e exa m wh er e you a r e
s t r es s ed.

If you r ea lly wa n t t o u s e t h e PV for mu la , you ca n a dd a ca s h flow of $30
a t 0 t = . Th is gives you a clea n a n d n ice in cr ea s in g a n n u it y.

Time t 0 1 2 18
(Year)
2005 2006 2007 2008
30 ( ) 2 30 ( ) 3 30 ... ( ) 19 30
30
( )
19
30 30 PV Ia = ``
So t h e pr es en t va lu e of t h e decr ea s in g a n n u it y a t t ime zer o is :

( )
19
19
19
19
30 30 30 1 2,171.916769 @ 9%
v
i
d
a
Ia
(
( = = =
(

``
``
Fin a lly, we a r e r ea dy t o s olve for X :
7
9 52 18 %
2,000(1.015) 2,171.916769
j
X a a =
wh er e
( )
1
4
1 9% 1 1.5%
1 1.5%
j
(
+
(

=
+
Solvin g t h e equ a t ion , we h a ve: 11, 354.44396 X =
29
Ke y point s t o re me mbe r about 3 minut e s olut ion s c ript s :

Scr ipt s pr even t u s fr om r ein ven t in g a wh eel a ga in a n d a ga in . Us in g
s cr ipt s , we in ven t a wh eel on ce; n ext t ime, we s imply r eu s e t h e
wh eel in ven t ed befor e.

Scr ipt s cu t t o t h e ch a s e a n d qu ickly get t o t h e cor e of t h e pr oblem
a t h a n d, en a blin g u s t o s olve a complex pr oblem 100% r igh t in a
h u r r y u n der pr es s u r e.

Don t blin dly copy s olu t ion s fr om SOA exa ms , t ext books , s t u dy
ma n u a ls , or s emin a r s . Cu s t omize t h os e s olu t ion s s o t h ey a r e
s impler , fa s t er , a n d les s pr on e t o er r or s . Th e s cr ipt s cr ea t ed by you
a r e t h e bes t s cr ipt s

Keep r efin in g you r s cr ipt s .

How t o e limi nat e e rrors

Of a ll t h e SOA exa ms , Exa m FM r equ ir es u s t o h a ve t h e lea s t a mou n t of
cr ea t ivit y. Th e t ext books h a ve given u s a myr ia d of for mu la s for vir t u a lly
ever y con cept fr om level a n n u it y, t o va r yin g a n n u it y, t o loa n
a mor t iza t ion , t o bon d eva lu a t ion , t o du r a t ion , t o con vexit y. We n eed on ly
t o memor ize t h es e for mu la s a n d a pply t h em t o s pecific s it u a t ion s .

Th en wh y ca n t ever yon e pa s s Exa m FM? Wh y didn t ever yon e pa s s
Cou r s e 2?

An s wer : er r or s .

Even if we h a ve memor ized a ll t h e for mu la s , ma s t er ed a ll t h e con cept s ,
a n d cr ea t ed power fu l 3 min u t e s olu t ion s cr ipt s , we ma y s t ill n ot be a ble
t o s olve ever y pr oblem r igh t . Aft er a ll, we a r e fa llible. We a r e es pecia lly
fa llible wh en we h a ve t o s olve a pr oblem in a h u r r y a n d u n der s t r es s .

To con vin ce you r s elf t h a t you a r e fa llible, pu t you r s elf u n der t h e exa m
con dit ion a n d t a ke a Cou r s e 2 exa m a dmin is t er ed in t h e pa s t . See h ow
ma n y in cor r ect a n s wer s a r e ca u s ed by you r s illy mis t a kes .

Trut h #1 : To pa s s Exa m FM, you n eed t o do t wo t a s ks : (1) lea r n n ew
t h in gs a n d expa n d you r kn owledge, (2) r edu ce er r or s in a r ea s you
a lr ea dy kn ow a n d defen d you r kn owledge.

30
Trut h #2 : Mos t ca n dida t es s pen d t oo mu ch t ime lea r n in g n ew t h in gs ;
t h ey s pen d t oo lit t le t ime r edu cin g er r or s .

Trut h #3 : If you a r e a bor der lin e s t u den t kn owin g t h e min imu m
a mou n t kn owledge on FM, you r ch a n ce t o pa s s Exa m FM s olely depen ds
on wh et h er you ca n elimin a t e er r or s . Sa y you s olved on ly 70% of t h e
exa m lea vin g t h e r ema in in g pr oblems bla n k. If you ma de zero er r or in
you r s olu t ion s , you s u r ely pa s s ed FM if t h e pa s s in g s t a n da r d wa s s olvin g
66% of t h e pr oblems r igh t .

How ca n we r edu ce er r or s ?

Let s look a t a n ever yda y s it u a t ion .
Sit u a t ion Gr ocer y s h oppin g
Er r or For get t in g t o bu y cer t a in it ems
Er r or elimin a t ion s t r a t egy Don t t r u s t you r memor y. Rigor ou s ly u s e a
s h oppin g lis t . Ch eck off a n it em fr om you r
lis t on ce you h a ve bou gh t it . Befor e
lea vin g t h e s t or e, ma ke s u r e t h a t you h a ve
ch ecked off a ll t h e it ems in you r lis t .
How t o elimin a t e ca lcu la t ion er r or s Us e a s ys t ema t ic a ppr oa ch

1. Don t t h in k t h a t Ot h er s will ma ke mis t a kes bu t I won t or If I
ju s t s olve h u n dr eds of pr a ct ice pr oblems , I will n ever ma ke
mis t a kes a ga in . Remember t h a t mis t a kes will s u r ely cr eep in like
a t h ief u n les s you u s e a s ys t ema t ic a ppr oa ch t o pr oblem s olvin g.

2. Tr u s t t h a t if you u n der s t a n d t h e con cept a n d if you u s e a
s ys t ema t ic a ppr oa ch , t h e r igh t s olu t ion will s u r ely emer ge.
3. Un der a s ys t ema t ic a ppr oa ch , a lwa ys move fr om gen er a l pr in ciples
t o s pecific a n s wer s . For exa mple, wh en s olvin g a ca s h flow
pr oblem, you fir s t s et u p t h e fu n da men t a l equ a t ion s u ch a s t h e
pr es en t va lu e of t h e ca s h in flows is equ a l t o t h e pr es en t va lu e of
t h e ca s h ou t flows . Th en you ca lcu la t e t h e mis s in g it em (t h e
in t er es t , a s pecific ca s h flow, or t h e n u mber of pa ymen t s , et c) by
s olvin g t h is equ a t ion .

4. Solve a pr oblem s t ep by s t ep; don t ju mp s t eps . For exa mple, if you
h a ve a ch oice of s olvin g a pr oblem in on e gia n t s t ep or s olvin g t h e
pr oblem in t h r ee s ma ll s t eps , u s e t h e t h r ee-s t ep a ppr oa ch t o a void
er r or s .

31
5. Un les s a pr oblem is over ly s imple, a lwa ys dr a w a ca s h flow
dia gr a m. Th is a voids t h e common off-by-on e er r or (for get t in g on e
pa ymen t or a ddin g a pa ymen t t h a t does n ot exis t ).

6. Wh en t a kin g t h e exa m, u s e ca lcu la t or s t o t h eir fu ll power a n d
delega t e ca lcu la t ion s t o you r ca lcu la t or s a s mu ch a s you ca n .
Wh en s olvin g pr a ct ice pr oblems , h owever , s olve t h em in t wo wa ys .
Fir s t , don t u s e a ca lcu la t or s bu ilt in fu n ct ion a lit ies (s u ch a s BA II
Plu s Pr ofes s ion a ls TVM or modified du r a t ion ) a n d s olve t h e
pr oblem ma n u a lly; t h is s h a r pen s you r con cept u a l t h in kin g. Next ,
ma ximize t h e u s e of you r ca lcu la t or a n d min imize you r wor k; t h is
qu icken s you r s olu t ion .

7. Don t do men t a l ma t h ; u s e you r ca lcu la t or for even t h e s imples t
ca lcu la t ion s . In t h e h ea t of t h e exa m, people t en d t o ma ke s illy
mis t a kes for even t h e s imples t ca lcu la t ion s . For exa mple, if you
n eed t o ca lcu la t e 11 - 9.4 (eleven min u s n in e poin t fou r ) in t h e
exa m. Is t h e a n s wer 1.6 or 2.6? If you do men t a l ma t h , you a r e
likely t o mis ca lcu la t e. So don t be cr ea t ive. J u s t let you r ca lcu la t or
do t h e ma t h for you .

8. Wh en t h e pa ymen t fr equ en cy differ s fr om t h e in t er es t
compou n din g fr equ en cy, u s e t h e pa ymen t fr equ en cy a s t h e in t er es t
compou n din g fr equ en cy. Th is gr ea t ly r edu ces t h e n u mber of
for mu la s you h a ve t o memor ize.

9. Never t r a n s fer in t er media t e va lu es bet ween t h e s cr a p pa per a n d
you r ca lcu la t or . Alwa ys s t or e t h e in t er media t e va lu es in you r
ca lcu la t or s memor ies . Us e s ymbols t o keep t r a ck wh ich n u mber is
in wh ich memor y.

Exa mple of u s in g a s ys t ema t ic a ppr oa ch

Proble m 1

J oh n bu ys t h e followin g bon d:
Fa ce a mou n t $1,000
Ter m t o ma t u r it y 20 yea r s
Cou pon s 8% pa ya ble s emi-a n n u a lly
Yield t o ma t u r it y 10%
Wit h 5 yea r s t o ma t u r it y a n d immedia t ely a ft er r eceivin g t h e 30
t h
cou pon ,
J oh n s ells t h e bon d a t a pr ice yieldin g 9% a n n u a l effect ive t o t h e bu yer .

Ca lcu la t e J oh n s a n n u a l effect ive r et u r n on h is in ves t men t in t h e bon d.
32
Solut ion

Th is pr oblem s eems difficu lt a t fir s t gla n ce. However , let s s olve it s t ep by
s t ep u s in g a s ys t ema t ic a ppr oa ch . Fir s t , we dr a w a ca s h flow dia gr a m t o
h elp u s vis u a lize t h e in for ma t ion given t o u s . Beca u s e t h e ca s h flows
occu r ever y 6 mon t h s , we will u s e 6 mon t h s a s t h e in t er es t compou n din g
per iod. Well con ver t a n a n n u a l effect ive in t er es t r a t e t o a 6-mon t h
effect ive r a t e.

Unit time = 6 months
Time t 0 1 2 3 ... 30 31 32 39 40
Cash flow $40 $40 $40 $40 $40 $40 $40 $40
$1,000
John owns these John sold these
30 cash flows 10 cash flows @9%
John bought the bond @10%
John got out of the bond
We a r e s t ill n ot s u r e h ow t o ca lcu la t e J oh n s r et u r n for in ves t in g in t h e
bon d; we don t h a ve a for mu la r ea dy. Wh a t s h ou ld we do?

In a s ys t ema t ic a pproa ch , we move fr om gen er a l pr in ciples t o s pecifics .
How do we ca lcu la t e a n in ves t men t r et u r n in gen er a l?

Ge ne ral princ iple
We depos it ( ) $ 0 X a t t ime zer o in a fu n d. At t ime t t h e fu n d gr ows t o
( ) $X t . Ou r r et u r n for in ves t in g in t h e fu n d du r in g t h e t ime h or izon [ ] 0, t
is r . We ca n s olve for r a s follows :
( )( ) ( )
( )
( )
1
0 1 1
0
t
t
X t
X r X t r
X
(
+ = =
(


33
Let s a pply t h is gen er a l pr in ciple t o J oh n s in ves t men t . To fin d J oh n s
r et u r n , we n eed t h r ee da t a :

( ) 0 X - J oh n s in it ia l ou t -of-pocket expen s e t o bu y t h e in ves t men t
t - J oh n s in ves t men t h or izon
( ) X t - J oh n s t ot a l wea lt h a t t h e en d of t h e in ves t men t h or izon
J oh n s cos t for bu yin g t h e bon d is t h e bon ds ma r ket pr ice. We kn ow t h a t
a bon ds fa ir ma r ket pr ice is t h e pr es en t va lu e of t h e bon ds ca s h flows
dis cou n t ed a t YTM (yield t o ma t u r it y).

( ) ( )
1
40
2
40
0 40 1, 000 @ 1 10% 1 X v i a = + = +
How lon g did J oh n in ves t men t h is mon ey in t h e bon d? He bou gh t t h e
bon d a t t ime zer o. He got ou t of t h e bon d a t t ime 30. So h is in ves t men t
h or izon is 30 t = . Plea s e n ot e t h a t t h e t ime u n it is s t ill 6 mon t h s .

Fin a lly, we n eed t o fin d ( ) 30 X , J oh n s t ot a l wea lt h a t t ime 30. If J oh n
cou n t s h is mon ey a t t ime 30, h ow mu ch does h e h a ve? J oh n s t ot a l
wea lt h a t t ime 30 comes fr om t wo s ou r ces : (1) r ein ves t in g 30 cou pon s , (2)
s ellin g 10 ca s h flows a t 9%.

( ) ( ) ( )
1 1
10
2 2
30 10
The accumulated value at 30 John's price for selling 10 cash flows
of John reinvesting 30 coupons at 30 @9% per
@10% per year
30 40 @ 1 10% 1 40 1, 000 @ 1 9% 1
t
t
X s i v j a
=
=
| | | |
= = + + + = +
| |
\ . \ .
_
year
_
In t h e a bove equ a t ion , we u s e 10% a s J oh n s a n n u a l effect ive r ein ves t
r a t e. Th ou gh t h e r ein ves t men t r a t e ca n differ fr om t h e YTM, t h is pr oblem
does n t s pecifica lly give u s J oh n s r ein ves t men t r a t e. As a r es u lt , we
a s s u me t h a t J oh n r ein ves t s h is 30 cou pon s a t YTM of 10%.

Fin a lly, we a r e r ea dy t o ca lcu la t e J oh n s r et u r n r . However , we n eed t o
pr oceed ca u t iou s ly. Well per for m ma n y mes s y ca lcu la t ion s t o get r . If we
ma ke a s illy mis t a ke in on e ca lcu la t ion , a ll t h e good wor k we h a ve don e
s o fa r is r u in ed.

To fla wles s per for m mes s y ca lcu la t ion s , we will follow 3 r u les :

34
Ru le #1 In t h e exam, delega t e wor k t o ou r ca lcu la t or a s mu ch
a s we ca n . Ou r ca lcu la t or is ou r obedien t s er va n t , wh o a lwa ys
execu t es ou r or der s pr ecis ely a n d fla wles s ly. As lon g a s ou r
or der s a r e clea r a n d u n a mbigu ou s , we ca n r es t a s s u r ed t h a t
ou r ca lcu la t or will get t h e job don e 100% r igh t .

Ru le #2 In doin g pra ct ice pr oblems , s olve t h e pr oblems t wice.
Fir s t t ime we s olve t h e pr oblem ma n u a lly u s in g for mu la s .
Secon d t ime we let t h e ca lcu la t or do mos t of t h e wor k (ju s t a s
we do in t h e exa m).

Ru le #3 Don t t r a n s fer in t er media t e va lu es bet ween ou r
ca lcu la t or a n d t h e s cr a p pa per . St or e a ll t h e in t er media t e
va lu es in ou r ca lcu la t or s memor ies a n d s ys t ema t ica lly t r a ck
wh ich memor y s t or es wh ich va lu e.

Let s a pply t h es e r u les . Well s olve t h e pr oblem wit h t wo met h ods . Well
s ys t ema t ica lly t r a ck down ou r in t er media t e va lu es in ea ch met h od.

The c alc ulat ion proc e dure for Me t hod #1 - us e BA II Plus / BA II Plus
Profe s s ional TVM Works he e t

St e p Formula St ore in
me mory
Trac k down
value s s t ore d
in me mory
#1 Con ver t
a n n u a l
10% in t o
6-mon t h
r a t e
( )
1
2
1 10% 1 i = +
0.04880885 i =
100 4.88088482 i =
Th er es a s ligh t differ en ce bet ween
i a n d 100i . BA II Plu s / BA II Plu s
Pr ofes s ion a l dis pla ys a n u mber u p
t o 8 decima l pla ces . If a n u mber
h a s mor e t h a n 8 decima l pla ces ,
t h e ca lcu la t or s r ou n ds t h e
n u mber t o 8 decima l pla ces in t h e
dis pla y. However , in t er n a lly BA II
Plu s / BA II Plu s Pr ofes s ion a l s t or es
a n u mber in 13 decima l pla ces ,
even t h ou gh it dis pla ys u p t o 8
decima l pla ces . Plea s e n ot e t h a t
t h e dis pla y does NOT a ffect h ow a
n u mber is s t or ed in t er n a lly in t h e
ca lcu la t or .
0 M 100 0 i M =
35
#2
Ca lcu la t e
( ) 0 X u s in g
TVM.
PMT=40, N=30, FV=1,000
I/ Y=M0 (t o r eca ll M0, pr es s RCL
0)

( )
40
40
0 40 1, 000 @ X v i a = +
( ) 0 846.3502152 X =
1 M ( ) 0 1 X M =
#3 Con ver t
a n n u a l 9%
in t o 6-
mon t h r a t e
( )
1
2
1 9% 1 4.40306509% j = + =
100 4.40306509 j =
2 M
100 2 j M =
#4
Ca lcu la t e
30
40
i
s
u s in g TVM
PMT=40, N=30, I/ Y=M0 (t o r eca ll
M0, pr es s RCL 0)

30
40 2, 603.829665
i
s =
3 M
30
40 3
i
s M =
#5
Ca lcu la t e
10
10
40 1, 000 @ v a +
u s in g TVM
PMT=40, I/ Y=M2, N=10, FV=1,000

10
10
40 1, 000 @ 967.9540424 v j a + =
4 M
10
10
40 1, 000v a +
@ 4 j M =
#6
Ca lcu la t e
( ) 30 X
( )
10
30 10
40 40 1, 000 @
i
s v j a + +
3 4 3, 571.783707 M M = + =
(Pr es s RCL 3 a n d RCL 4 t o
r eca ll M3 a n d M4 r es pect ively)
5 M ( ) 30 5 X M =
#7
Ca lcu la t e
r , J oh n s
r et u r n in a
6-mon t h
per iod
( )
( )
1
30
30
1
0
X
r
X
(
=
(
(


1
30
5
1 4.91667086%
1
M
r
M
(
= =
(


(Pr es s RCL 5 a n d RCL 1 t o
r eca ll M5 a n d M1 r es pect ively)
6 M 6 r M =
#8 Con ver t
r in t o a n
a n n u a l
r a t e R
( ) ( )
2 2
1 1 1 6 1 R r M = + = +
10.07507825% R =
(Pr es s RCL 6 t o r eca ll M6)
7 M 7 R M =
So J oh n s a n n u a l effect ive r et u r n for in ves t in g in t h e bon d is 10.08%.

36
The c alc ulat ion proc e dure for Me t hod #2 (formula drive n approac h)
- us e ge ne ral func t ions of BA II Plus / BA II Plus Profe s s ional

St e p Formula St ore in
me mory
Trac k down
value s s t ore d in
me mory
#1 Con ver t a n n u a l
10% in t o 6-mon t h
r a t e
( )
1
2
1 10% 1 i = +
4.88088482% i =
0 M 0 i M =
#2 Ca lcu la t e
40 i
a
40
40
1
i
v
i
a

=
( )
40
40
1 1 0
0
i
M
M
a

+
=
40
17.44266468
i
a =
1 M
40
1
i
M a =
#3 Ca lcu la t e ( ) 0 X
( )
40
40
0 40 1, 000 @ X v i a = +
( ) ( )
40
0 40 1 1, 000 1 0 X M M

= + +
( ) 0 846.3502152 X =
2 M ( ) 0 2 X M =
#4 Con ver t a n n u a l
9% in t o 6-mon t h
r a t e
( )
1
2
1 9% 1 4.40306509% j = + =
3 M
3 j M =
#5 Ca lcu la t e
30 i
s
( ) ( )
30 30
30
1 1 1 0 1
0
i
i M
s
i M
+ +
= =
30
65.09574162
i
s =
4 M
30
4
i
s M =
#6 Ca lcu la t e
10 j
a
( )
10
10
10
1 1 3
1
3
j
M
v
j M
a

= =
10
7.95056640
j
a =
5 M
10
5
j
M a =
#7 Ca lcu la t e
10
10
40 1, 000 @ v j a +
10
10
40 1, 000 @ v j a +
( )
10
40 5 1, 000 1 3 M M

= + +
967.9540424 =
6 M
10
10
40 1, 000 @ 6 a v j M + =

#8 Ca lcu la t e ( ) 30 X
( )
10
30 10
40 40 1, 000 @
i
s v j a + +
40 4 6 3, 571.783707 M M = + =
7 M ( ) 30 7 X M =
#9 Ca lcu la t e r ,
J oh n s r et u r n in a
6-mon t h per iod
( )
( )
1
30
30
1
0
X
r
X
(
=
(
(


1
30
7
1 4.91667086%
2
M
r
M
(
= =
(


8 M 8 r M =
#10 Con ver t r in t o
a n a n n u a l r a t e R
( ) ( )
2 2
1 1 1 8 1 R r M = + = +
10.07507825% R =
9 M 9 R M =
37
In t h e t wo met h ods a bove, we pa in s t a kin gly t r a ck down wh ich n u mber is
s t or ed in wh ich memor y. Th is t r a ckin g s ys t em h a s t h e followin g
a dva n t a ges :

Elimin a t e t h e n eed t o t r a n s fer n u mber s ba ck a n d for t h bet ween a
ca lcu la t or a n d a s cr a p pa per .

Elimin a t e t h e er r or s ca u s ed by t r a n s fer r in g n u mber s bet ween a
ca lcu la t or s a n d a s cr a p pa per .

Elimin a t e t h e los s of pr ecis ion s ca u s ed by t r a n s fer r in g a fr a ct ion
n u mber s bet ween a ca lcu la t or s a n d a s cr a p pa per . For exa mple, if
we h a ve t o t r a n s fer 4.88088482% i = ba ck a n d for t h bet ween a
ca lcu la t or a n d a s cr a p pa per , we feel compelled t o r ou n d i t o
4.88% i = . However , if we s t or e 4.88088482% i = in a ca lcu la t or s
memor y a n d r eca ll it wh en ever we u s e it , BA II Plu s / BA II Plu s
Pr ofes s ion a l will s t or e i in 13 decima l pla ces in it s in t er n a l
ca lcu la t ion s , yieldin g r es u lt s wit h good pr ecis ion .

Lea ve a n a u dit t r a il, is ola t in g good ca lcu la t ion s fr om ba d. For
exa mple, a ft er a r r ivin g t h e fin a l a n s wer of R , we r ea lized t h a t ou r
St ep #1 ca lcu la t ion wa s wr on g. To fix t h e er r or , we s imply r edo
St ep 1 ca lcu la t ion a n d r eloa d t h e n ewly ca lcu la t ed i t o 0 M . Next ,
we r edo a ll t h e ca lcu la t ion s t h a t u s e 0 M , lea vin g t h e ca lcu la t ion s
t h a t do n ot in volve 0 M in t a ct . In con t r a s t , if we don t h a ve s u ch a
good t r a ckin g s ys t em in pla ce, on e s in gle er r or will blow u p a ll t h e
ca lcu la t ion s , for cin g u s t o r eca lcu la t e ever yt h in g fr om s cr a t ch .

Proble m 2 (EA-1 #1 2 0 0 3 )

Over a 3-yea r per iod, a s er ies of depos it s a r e ma de t o a s a vin gs a ccou n t .
All depos it s wit h in a given yea r a r e equ a l in s ize a d a r e ma de a t t h e
begin n in g of ea ch r eleva n t per iod. Depos it s for ea ch yea r a r e t ot a l
$1,200. Th e followin g ch a r t s h ows t h e fr equ en cy of depos it s a n d t h e
in t er es t r a t e cr edit ed for ea ch yea r .

Yea r Fr equ en cy of depos it s In t er es t r a t e cr edit ed
du r in g t h e yea r
1 Semi-a n n u a lly
( ) 12
6% d =
2 Qu a r t er ly
( ) 3
8% i =
3 Ever y 2 mon t h s 7% o =
Ca lcu la t e t h e va lu e of t h e a ccou n t a t t h e en d of t h e 3
r d
yea r .

38
Solut ion

Th is pr oblem is t r icky. To s olve it r igh t , we n eed t o s ys t ema t ica lly t r a ck
down t h e t imin g a n d t h e a mou n t of ea ch depos it .

To s implify ou r ca lcu la t ion , well s et 1 u n it of mon ey = $100. So ea ch
yea r , t h e t ot a l a mou n t of depos it s ma de ea ch yea r is 12.

Time
(yea r )
0 0.5 1 1.
25
1.5 1.7
5
2 2
2
12
4
2
12
6
2
12
8
2
12
10
2
12
3
Mode Semi-
a n n u a l
Qu a r t er ly On ce ever y 2 mon t h s
Depos it $6 6 3 3 3 3 2 2 2 2 2 2
In t er es t
r a t e
( ) 12
6% d =
( ) 3
8% i =
7% o =
Get t in g t h is t a ble r igh t is h a lf t h e ba t t le. If you ca n s et u p a t a ble like
t h is , you a r e on t h e r igh t t r a ck.

Next , we n eed t o a ccu mu la t e depos it s yea r by yea r ; t h e in t er es t r a t es
ea r n ed a r e differ en t yea r by yea r .

Find FV @ 1 t = of t he 1
s t
ye ar de pos it s :

Du r in g Yea r 1, t h e mon t h ly dis cou n t in g fa ct or is
( ) 12
1
12
d
(

(

; t h e mon t h ly
a ccu mu la t in g fa ct or is ju s t t h e r ever s e of t h e dis cou n t fa ct or :
( )
1
12
1
12
d

(

(

.
In ot h er wor ds , if you h a ve $1 a t t h e begin n in g of a mon t h , t h en t h is $1
will a ccu mu la t e t o
( )
1
12
1
12
d

(

(

a t t h e en d of t h e mon t h . You s h ou ld lea r n
t h is t ech n iqu e a n d qu ickly con ver t a n omin a l dis cou n t r a t e
( ) n
d in t o a n
a ccu mu la t in g fa ct or .

39
FV @ 1 t = of t h e 1
s t
yea r depos it s :

( ) ( )
12 6
12 12
$6 @ 0 accumulate $6 @ 0.5 accumulate
for 12 months for 6 months
6 1 6 1
12 12
t t
d d

= =
( (
+
( (

_ _
12 6
6% 6%
6 1 6 1
12 12

| | | |
= +
| |
\ . \ .
( )
12 6
6 0.995 0.995 12.55517

= + =
Next , we n eed t o a ccu mu la t e t h is a mou n t t h r ou gh Yea r 2 a n d Yea r 3.
Th e FV @ 3 t = of Yea r 1 depos it s :

( )
3
3
3
7%
accumulate 1
year in Year 3
accumulate 4
quarters in Year 2
8%
12.55517 1 12.55517 1 14.57175
3 3
i
e e
o
(
(
+ = + =
(
(


_
_

Find FV @ 2 t = of t he 2
nd
ye ar de pos it s :

Compou n din g per iod = 0.25 (qu a r t er ly)
# of compou n din g per iods = 4
Th e qu a r t er ly in t er es t r a t e j .
To fin d j , let s a ccu mu la t e $1 a t t h e begin n in g of Yea r 2 t o t h e en d of
Yea r 2. If we u s e t h e qu a r t er ly in t er es t r a t e j , t h e a ccu mu la t ed va lu e @
t h e en d of Yea r 2 is ( )
4
1 j + . On t h e ot h er h a n d, if we a ccu mu la t e $1
u s in g
( ) 3
i , t h e a ccu mu la t e va lu e is :

( )
3
3
3
8%
1 1
3 3
i
(
(
+ = +
(
(



( )
3
3
4
4 8% 8%
1 1 , 1 1 1.993406%
3 3
j j
( (
+ = + = + =
( (


FV @ 2 t = of t h e 2
n d
yea r depos it s :
4 1.993406%
3 12.610062 s = ``
Next , we a ccu mu la t e t h is a mou n t t h r ou gh Yea r 3

40
Th e FV @ 3 t = of Yea r 2 depos it s :

7%
accumulate 1
year in Year 3
12.610062 12.82162 13.52439 e e
o
= =
_
Find FV @ 3 t = of t he 3
rd
ye ar de pos it s :

Compou n din g per iod = t wo mon t h s
# of compou n din g per iods = 6
In t er es t r a t e per per iod:
2 2
7%
12 12
1 1 1.1734988% e e
o
| | | |
| |
\ . \ .
= =
FV @ 3 t = of t h e 3
r d
yea r depos it s :
6 1.1734988%
2 12.50262 s = ``
Fin a lly, we s u m u p t h e FV @ 3 t = of ea ch yea r s depos it s :

14.57175 13.52439 12.50262 40.59876 + + =
Remember t h a t ea r ly on we s et on e u n it of mon ey=$100. So t h e a ccou n t
va lu e @ t h e en d of Yea r 3 is $4,059.88.

Proble m 3

Bon d fa ce a mou n t $100
Ter m t o ma t u r it y 30 yea r s
Cou pon s 10% s emia n n u a l
Redempt ion Pa r
Yield t o ma t u r it y 12.36% a n n u a l effect ive
You a r e a ls o given t h e followin g for mu la t o ca lcu la t e a bon ds Ma ca u la y
du r a t ion :
( )
( )
1
1
n
t
t
MAC n
t
t
t CF t v
CF t v
D
=
=
=

In t h e a bove for mu la:



( ) CF t = t h e ca s h flow a t t ime t (mea s u r ed in # of yea r s )

1
1
v
i
=
+
, wh er e i is t h e a n n u a l effect ive yield of t h e bon d.

41
Ca lcu la t e
MAC
D u s in g a s ys t ema t ic a ppr oa ch .

Solut ion

Fir s t , we dr a w a ca s h flow dia gr a m. Beca u s e t h e cou pon s a r e pa id
s emia n n u a lly bu t t h e in t er es t given is a n a n n u a l r a t e, well u s e t h e
cou pon pa ymen t fr equ en cy a s t h e u n it t ime.

Unit time = 6 months

Time t 0 1 2 59 60

Cash flow $5 $5 $5 $5 $5
$100
Th e in t er es t r a t e per u n it t ime is :
1 12.36% 1 6% i = + = ,
1 1
1 1.06
v
i
= =
+
Next , we expa n d, s t ep by s t ep, t h e for mu la
( )
( )
1
1
n
t
t
MAC n
t
t
t CF t v
CF t v
D
=
=
=

.
( )
( )
( )
( )
60 60
1 1
60 60
1 1
1
1 2
2
t t
t t
MAC
t t
t t
t CF t v t CF t v
CF t v CF t v
D
= =
= =
| |
|
\ .
= =



We a dd a fa ct or of
1
2
in t h e r igh t h a n d s ide beca u s e 6 mon t h s =
1
2
yea r .

( ) ( ) ( ) ( ) ( ) ( )
2 3 60 60
60
1
1 5 2 5 3 5 ... 60 5 60 100
t
t
t CF t v v v v v v
=
= + + + + +

( ) ( )
60
60
5 60 100
i
v Ia = +
( )
60
60
1
60
5 100
t
t
CF t v v a
=
= +

( ) ( )
60
60
60
60
5 60 100
1
2 5 100
i
MAC
v
v
D
Ia
a
(
+
( =
+
(

Well u s e BA II Plu s / BA II Plu s Pr ofes s ion a l t o s olve t h is pr oblem.


42
To a void t r a n s fer r in g in t er media t e va lu es ba ck a n d for t h bet ween a
ca lcu la t or a n d t h e s cr a p pa per , well t r a ck down wh a t va lu e is s t or ed in
wh ich memor y.

1 1
0
1 1.06
v M
i
= = =
+
(s o we s t or e v in M0 )

( )
60
60
60
60
6%
i
v a
Ia

=
``
( )
60
60
60
1 0
1 1
1
1 1 0
n
M
v v
M
d v M
a


= = = =

``
( )
( )
60
60
60
60
60
1 60 0
2
6% 6%
i
v
M M
M
a
Ia


= = =
``
( )
60
60
60
1 0
1 1
3
6% 6%
n
M
v v
M
i
a


= = = =
( ) ( )
( ) ( ) ( )
( ) ( )
60 60
60 60
60
60
5 60 100
5 2 60 100 0
1 1
4
2 5 100 2
5 3 100 0
i
MAC
v
M M
M
v
M M
D
Ia
a
(
+ (
+
( = = = (
+
( +
(

Aft er doin g t h e ca lcu la t ion , we get 8.69472773


MAC
D =
Proble m 4

Bon d fa ce a mou n t $100
Ter m t o ma t u r it y 30 yea r s
Cou pon s 10% s emia n n u a l
Redempt ion Pa r
Yield t o ma t u r it y 12.36% a n n u a l effect ive
Ca lcu la t e t h e bon ds du r a t ion .

Solut ion

Wh en t a kin g t h e exa m, we wa n t t o delega t e ca lcu la t ion s t o ou r
ca lcu la t or s a s mu ch a s we ca n . Th e s olu t ion t o Pr oblem 2 is complex. We
wa n t t o a void t h is h a r d cor e ca lcu la t ion in t h e exa m if we ca n .

43
Befor e t a kin g Exa m FM, we h a ve t h or ou gh ly r es ea r ch ed SOA a ppr oved
ca lcu la t or s . We kn ow t h a t BA II Plu s Pr ofes s ion a l h a s a bon d wor ks h eet ,
wh ich ca n ca lcu la t e t h e modified du r a t ion of a bon d. So well let BA II
Plu s Pr ofes s ion a l Bon d Wor ks h eet fin d t h e modified du r a t ion of t h e
bon d. Th en we con ver t t h e modified du r a t ion in t o Ma ca u la y du r a t ion .

Ple as e not e t hat Bond Works he e t in BA II Plus doe s NOT have
func t ionalit y for dire c t ly c alc ulat ing a bonds modifie d durat ion.

To ca lcu la t e t h e modified du r a t ion of t h e bon d, we en t er t h e followin g in
Bon d Wor ks h eet .

Key s t r okes in BA II Plu s Pr ofes s ion a l (du r a t ion fu n ct ion a lit y n ot
a va ila ble in BA II Plu s Bon d Wor ks h eet ):

2
n d
Bon d Th is a ct iva t es Bon d Wor ks h eet .
En t er
SDT=1.0100

Th is s et s SDT=1-
01-2000

SDT = s et t lemen t da t e (i.e. pu r ch a s e da t e of t h e
bon d)

We a r bit r a r ily s et t h e pu r ch a s e da t e of t h e bon d is t o
1/ 1/ 2000. You ca n s et t h e pu r ch a s e da t e t o a n ot h er
da t e s u ch a s 6/ 1/ 1988. However , 1/ 1/ 2000 is a n
ea s y n u mber .
En t er CPN=10 Set cou pon is 10% of pa r .
En t er
RDT=1.0130

Th is s et s RDT=1-
01-2030
RDT = r edempt ion da t e (or bon ds ma t u r it y)

We s et RDT=1-01-2030 (t h e bon d h a s a 30 yea r
ma t u r it y).
En t er RV=100 Redempt ion va lu e. Beca u s e t h e bon d is r edeemed a t
pa r a n d t h e pa r =100, we s et RV=100.
Da y cou n t in g
met h od
Us e 360 cou n t in g met h od (i.e. a s s u me ever y yea r h a s
360 da ys a n d ever y mon t h h a s 30 da ys ). Don t u s e
t h e a ct u a l cou n t in g met h od.
Cou pon fr equ en cy 2/ Y (i.e.. t wice a yea r )
En t er
YLD=2(6)=12
Set bon ds yield t o ma t u r it y t o 12%.

We n eed t o be ca r efu l h er e. Th e yield t o ma t u r it y in
Bon d Wor ks h eet mu s t be a n omin a l in t er es t r a t e
compou n din g a t t h e s a me fr equ en cy a s t h e cou pon s
a r e pa id. Beca u s e we s et cou pon fr equ en cy a s
s emia n n u a l (r emember we s et by 2/ Y a s cou pon
fr equ en cy), we n eed t o en t er a n omin a l yield
44
compou n din g s emia n n u a lly. So we dou ble t h e 6%
s emia n n u a l effect ive in t er es t r a t e t o get t h e n omin a l
r a t e compou n din g s emia n n u a lly. Don t en t er t h e 6-
mon t h effect ive in t er es t r a t e of 6% a s t h e yield t o
ma t u r it y .You n eed t o r emember t h is in or der t o u s e
Bon d Wor ks h eet .
CPT PRI (compu t e
pr ice of t h e bon d)
We get PRI=83.83857229. Th is is t h e PV of t h e bon d
ca s h flows dis cou n t ed a t 12.36% a n n u a l effect ive (or
6% per 6 mon t h s ).
AI=0 Accu mu la t ed In t er es t ( AI ) is zer o if bon d is s old
exa ct ly a t a cou pon da t e.

Wh en a bon d is s old bet ween t wo cou pon pa ymen t s ,
t h e bon d bu yer mu s t pa y t h e or igin a l bon d h older a
por t ion of t h e cou pon . AI is ca lcu la t ed a s follows :

Days between settlement and last coupon payment
AI=Coupon
Total days between two coupon payments

Th is is a min or fu n ct ion of Bon d Wor ks h eet . Don t


wor r y a bou t t h is .
DUR DUR=8.20257333

Remember t h is is t h e modified du r a t ion ca lcu la t ed
u s in g t h e Wa ll St r eet met h od. Th is is differ en t fr om
t h e t ext book defin it ion .
8.20257333
MOD
Wall Street
D =
Plea s e n ot e t h a t t h e bon d wor ks h eet in BA II Plu s / BA II Plu s Pr ofes s ion a l
u s es t h e Wa ll St r eet con ven t ion in qu ot in g a bon d. In Wa ll St reet , a
bon ds yield t o ma t u r it y is qu ot ed a s a n omin a l yield compou n din g a s
fr equ en t ly a s cou pon s a r e pa ya ble per yea r . In con t r a s t , in Exa m FM, t h e
yield t o ma t u r it y of a bon d is a lmos t a lwa ys qu ot ed a s a n a n n u a l effect ive
in t er es t r a t e. As a r es u lt , if you u s e Bon d Wor ks h eet t o do a n y
ca lcu la t ion s , you ll n eed t o en t er t h e n omin a l yield t o ma t u r it y, n ot t h e
effect ive yield. Th is is a n a n n oyin g det a il t o r emember . So I recommen d
t h a t you u s e Bon d Wor ks h eet on ly t o ca lcu la t e t h e bon ds du r a t ion .
Don t u s e it t o ca lcu la t e t h e bon d pr ice; you ca n ca lcu la t e t h e bon d pr ice
u s in g TVM Wor ks h eet .

45
Plea s e a ls o n ot e t h a t BA II Plu s Pr ofes s ion a l u s es t h e Wa ll St r eet
con ven t ion t o ca lcu la t e t h e bon ds modified du r a t ion :
( )
1
MAC
MOD
m
Wall Street
YLD
m
D
D =
+
(u s in g n omin a l yield)
In t h e a bove equ a t ion , mis t h e cou pon fr equ en cy a n d
( ) m
YLD is t h e
n omin a l yield t o ma t u r it y compou n din g a t t h e cou pon fr equ en cy.
In con t r a s t , t h e t ext book by Br over ma n u s e t h e followin g met h od:
1
MAC
MOD
Textbook
r
D
D
+
= (wh er e r is t h e a n n u a l effect ive yield)
Plea s e n ot e t h a t r is r ea lly t h e oppor t u n it y cos t of ca pit a l (i.e.. t h e
in t er es t r a t e ea r n ed by in ves t or s if t h ey in ves t in ot h er s ecu r it ies ). As a
r es u lt , t h e yield t o ma t u r it y is r ea lly t h e pr eva ilin g ma r ket in t er es t r a t e;
t h e in ves t or s ca n ea r n t h e pr eva ilin g ma r ket in t er es t r a t e if t h ey in ves t in
ot h er s ecu r it ies .
Fin a lly, well con ver t t h e Wa ll St r eet modified du r a t ion in t o Ma ca u la y
du r a t ion .

( )
8.20257333 8.69472773
12%
1 1
2
m
MAC MOD
YLD
D D
m
(
(
= = (
(
(

= + +
If you don t u n der s t a n d t h e Ma ca u la y du r a t ion a n d t h e modified
du r a t ion , t h a t s OK; I will expla in t h em lat er . For n ow, you ju s t n eed t o
kn ow t h a t you ca n ca lcu la t e Ma ca u la y du r a t ion u s in g BA II Plu s
Pr ofes s ion a l Bon d Wor ks h eet .

Beca u s e BA II Plu s does n ot h a ve t h e modified du r a t ion fu n ct ion a lit y,
you migh t wa n t t o bu y a BA II Plu s Pr ofes s ion a l ca lcu la t or .

Plea s e n ot e t h a t t h e modified du r a t ion fu n ct ion in BA II Plu s Pr ofes s ion a l
Bon d Wor ks h eet wor ks on ly for a bon d. A bon d h a s a n ea t ca s h flow
pa t t er n s . If a n exa m pr oblem gives you a s t r ea m of r a n dom ca s h flows
(s u ch a s $100 a t t =1, $104 a t t =2, $200 a t t =3,), t h en you ca n t u s e
Bon d Wor ks h eet t o ca lcu la t e t h e du r a t ion . La t er in t h is book, Ill expla in
h ow t o ca lcu la t e t h e du r a t ion of a s t r ea m of r a n dom ca s h flows .

46
Le s s on le arne d from Proble m 5

Ca lcu la t or s ca n s a ve you t ime. However , you n eed t o kn ow h ow t o
pr oper ly u s e a ca lcu la t or . In t h is pr oblem, if you don t kn ow t h a t you
n eed en t er a n omin a l yield, if you don t kn ow t h a t t h e du r a t ion gen er a t ed
by Bon d Wor ks h eet is a Wa ll St r eet ver s ion of t h e modified, you ll get a
wr on g r es u lt .

Th er e is ext r a complexit y in u s in g BA II Plu s Pr ofes s ion a l Bon d
Wor ks h eet t o ca lcu la t e t h e du r a t ion of bon d wh en a bon d is n ot
r edeemed a t pa r . Don t wor r y a bou t it n ow. Well pick u p t h is t opic la t er
wh en we s t u dy, in mor e dept h s , t h e con cept of du r a t ion a n d con vexit y.

47
http://actuary88.com
Chapt e r 2 Ge t t ing s t art e d

To bes t u s e t h is s t u dy ma n u a l, plea s e h a ve t h e followin g it ems r ea dy:

1. Text books r ecommen ded by SOA. On ly on e t ext book of t h e fou r
books is r equ ir ed.
Ma thema tics of Inves tment a nd Cred it
Ma thema tica l interes t theory
Theory of interes t
Fina ncia l Ma thema tics A Pra ctica l Guid e for Actua ries a nd
other Bus ines s Profes s iona ls
2. Ha ve a BA II Plu s or BA II Plu s Pr ofes s ion a l ca lcu la t or . BA II Plu s
Pr ofes s ion a l is pr efer a ble beca u s e it h a s s ever a l n ew fea t u r es (s u ch
a s ca lcu la t in g modified du r a t ion ) n ot fou n d in BA II Plu s

3. Down loa d Sa mple FM Qu es t ion s , Sa mple qu es t ion s for Der iva t ives
Ma r ket s , Ma y 2005, a n d November 2005 FM Exa m, a ll fr om t h e
SOA webs it e.
4. Review on lin e dis cu s s ion for u ms a bou t FM. Th er e a r e t wo ma jor
dis cu s s ion for u ms : www.a ct u a r ia lou t pos t .com a n d
h t t p:/ / www.a ct u a r y.com. Her e a r e s ome dis cu s s ion t h r ea ds :

h t t p:/ / www.a ct u a r ia lou t pos t .com/ a ct u a r ia l_dis cu s s ion _for u m/ s h
owt h r ea d.ph p?t =153162
h t t p:/ / www.a ct u a r ia lou t pos t .com/ a ct u a r ia l_dis cu s s ion _for u m/ s h
owt h r ea d.ph p?t =140223
h t t p:/ / www.a ct u a r y.com/ a ct u a r ia l-dis cu s s ion -
for u m/ s h owt h r ea d.ph p?t =10394
h t t p:/ / www.a ct u a r y.com/ a ct u a r ia l-dis cu s s ion -
for u m/ s h owt h r ea d.ph p?t =8575
48
http://actuary88.com
Chapt e r 3 FM Fundame nt al

Time value of mone y
youtube video:
http://www.youtube.com/watch?v=BXm5mZqMp6Y
http://www.youtube.com/watch?v=ks33lMoxst0
http://www.youtube.com/watch?v=4LSktB7Pk
http://www.youtube.com/watch?v=3SgVUlEcOBU
http://www.youtube.com/watch?v=6WCfVjUTTEY
Th e va lu e of mon ey depen ds on n ot on ly t h e a mou n t , bu t a ls o
wh en we r eceive it .

We a ll in t u it ively u n der s t a n d t h e t ime va lu e of mon ey. If s omeon e
owes u s mon ey, we wa n t t h e mon ey n ow; if we h a ve t o pa y bills , we
wa n t t o pu t t h em off a s la t e a s we ca n .

$100 t oda y is wor t h y mor e t h a n $100 t omor r ow. If we h a ve $100
t oda y, we ca n s pen d it for plea s u r e t oda y. Alt er n a t ively, we ca n
len d $100 ou t t oda y a n d ea r n in t er es t on it .

Princ i pal

Th e pr in cipa l is t h e in it ia l a mou n t bor r owed a n d yet t o be r epa id.

Int e re s t rat e
http://www.youtube.com/watch?v=GtaoP0skPWc
http://www.youtube.com/watch?v=t4zfiBw0hwM
An in t er es t r a t e is t h e r en t a l pr ice of mon ey. Wh en you bor r ow
s omeon es mon ey, you pa y a r en t for u s in g t h e mon ey for a per iod
of t ime.

An in t er es t r a t e is t h e r en t pa id by t h e bor r ower t o t h e len der per
u n it of mon ey per u n it of t ime. An a n n u a l in t er es t r a t e of 5%
mea n s t h a t for ever y $1 bor r owed for a yea r , t h e bor r ower pa ys a
$0.05 a n n u a l r en t t o t h e len der .

49
http://actuary88.com
Simple int e re s t rat e
http://www.youtube.com/watch?v=B3IdfBcXrLA&feature=related
http://www.youtube.com/watch?v=QeRe0WCOujw
In t er es t is pa id on ly on t h e or igin a l a mou n t bor r owed (ca lled
pr in cipa l) for t h e len gt h of t ime t h e bor r ower h a s t h e u s e of t h e
mon ey.

You depos it $100 in t o a ba n k a ccou n t a n d ea r n 5% s imple in t er es t
per yea r . You a r e a ct u a lly len din g $100 t o you r ba n k. Th e ba n k
pa ys a n a n n u a l r en t equ a l t o 5% of t h e pr in cipa l ($100).

At t h e en d of Yea r 1, you r a ccou n t gr ows t o 100(1+5%)=$105. $100
is you r or igin a l pr in cipa l a n d $5 is t h e in t er es t .

At t h e en d of Yea r 2, you r a ccou n t s gr ows t o 100(1+5%+5%)=$110.
$100 is t h e or igin a l pr in cipa l. You h a ve ea r n ed $5 in t er es t in Yea r
1 a n d $5 in t er es t in Yea r 2. Not ice t h a t t h e $5 in t er es t you h a ve
ea r n ed in Yea r 1 does n ot ea r n a n y a ddit ion a l in t er es t in Yea r 2
(s it t in g idle in t h e ba n k for 1 yea r ).

At t h e en d of Yea r 3, you r a ccou n t s gr ows t o 100(1+5%+5%+5%)
=$115. $100 is t h e or igin a l pr in cipa l. You h a ve ea r n ed $5 in t er es t
in Yea r 1, $5 in t er es t in Yea r 2, a n d $5 in t er es t in Yea r 3. Not ice
t h a t t h e $5 in t er es t you h a ve ea r n ed in Yea r 1 does n t ea r n a n y
in t er es t in Yea r 2 or Yea r 3 (s it t in g idle for 2 yea r s ). Th e $5 in t er es t
you h a ve ea r n ed du r in g Yea r 2 does n ot ea r n a n y in t er es t in Yea r 3
(s it t in g idle for 1 yea r ).

At t h e en d of Yea r n , you r a ccou n t gr ows t o ( ) 100 1 5% n + . $100 is
you r or igin a l pr in cipa l. You h a ve ea r n ed $5 in t er es t per yea r for n
yea r s . All t h e in t er es t you h a ve ea r n ed yea r a ft er yea r is s it t in g
idle.

Compound i nt e re s t rat e
h t t p:/ / www.you t u be.com/ wa t ch ?v=qEB6y4DklNY
h t t p:/ / www.you t u be.com/ wa t ch ?v=dzMvqJ MLy9c
Bot h t h e pr in cipa l a n d t h e a ccr u ed in t er es t ea r n in t er es t .

In t er es t is pa id on t h e or igin a l pr in cipa l plu s a ll in t er es t a ccr u ed t o
t h a t poin t in t ime.
50
http://actuary88.com
You depos it $100 in t o a ba n k a ccou n t a n d ea r n 5% compou n d
in t er es t per yea r . Th is mea n s t h a t for ea ch dolla r in you r a ccou n t
in t h e begin n in g of t h e yea r , you will ea r n $0.05 a t t h e en d of t h e
yea r .

At t h e en d of Yea r 1, you r a ccou n t gr ows t o 100(1+5%)=$105. At
t h e en d of Yea r 2, you r a ccou n t s gr ows t o ( )
2
100 1 5% 110.25 + = . At
t h e en d of Yea r n , you r a ccou n t gr ows t o ( ) 100 1 5%
n
+ . At t h e en d
of ea ch yea r , you ea r n 5% in t er es t on you r a ccou n t ba la n ce a t t h e
begin n in g of t h e yea r .

Comparis on of s imple int e re s t rat e and c ompound int e re s t rat e

Time T Simple int e re s t rat e 5 % Compound i nt e re s t rat e 5 %
0 t = 100 100
1 t =
( ) 100 1 5% 105 + = ( ) 100 1 5% 105 + =
2 t =
( ) 100 1 5% 5% 110 + + = ( ) ( )
2
105 1 5% 100 1 5% 110.25 + = + =

3 t =
( ) 100 1 5% 5% 5% 115 + + + = ( ) ( )
3
110.25 1 5% 100 1 5% 115.7625 + = + =
4 t =
( ) 100 1 5% 5% 5% 5% 120 + + + + = ( ) ( )
4
115.7625 1 5% 100 1 5% 121.550625 + = + =
t n =
( ) 100 1 5% n + ( ) 100 1 5%
n
+
If n =1, s imple in t er es t r a t e a n d compou n din g in t er es t r a t e give you t h e
s a me a mou n t of wea lt h ; if n >1, you a ccu mu la t e mor e mon ey u n der a
compou n din g in t er es t r a t e; if n <1, a s imple in t er es t r a t e gives you mor e
wea lt h .

To s t a y compet it ive in bu s in es s , ma n y ba n ks offer a compou n d in t er es t
r a t e t o t h eir cu s t omer s . To s ee wh y, ima gin e you depos it $100 on
J a n u a r y 1, 2003 in Ba n k A, wh ich offer s you 5% s imple in t er es t r a t e. On
December 31, 2003, you r a ccou n t gr ows t o $105. If you keep you r mon ey
in Ba n k A for a n ot h er yea r a n d do n ot h in g, you r a ccou n t will gr ow t o
$110 on December 31, 2004. Bu t you a r e n ot h a ppy wit h t h e s imple
in t er es t r a t e s ys t em wh er e you a r e for ced t o give u p in t er es t on t h e
in t er es t you ve a lr ea dy ea r n ed. Wh a t ca n you do s o you ca n ea r n in t er es t
on t h e in t er es t ?

Obviou s ly, you cou ld move you r mon ey t o a n ot h er ba n k wh ich offer s you
a compou n d in t er es t r a t e of 5%.

Th e s econ d opt ion is s t a y in Ba n k A bu t open s a n a ddit ion a l a ccou n t .
51
http://actuary88.com
On J a n u a r y 1, 2004, you depos it $100 in Ba n k A, wh ich offer s you a 5%
s imple in t er es t r a t e. On December 31, 2004, you r a ccou n t gr ows t o
$105. If you do n ot hin g, you r a ccou n t will gr ow t o $110 on December 31,
2005.
On December 31, 2004, you open a n ot h er a ccou n t a t Ba n k A a n d you
depos it $5 in t o t h is s econ d a ccou n t . As a r es u lt , you h a ve t wo a ccou n t s
a t Ba n k A--on e a ccou n t h a s $100 a n d t h e ot h er h a s $5. At t h e en d of
December 31, 2005, t h e t ot a l mon ey you h a ve in Ba n k A will be:
( ) ( ) ( )
2
1st account 2nd account
100 1 5% 5(1 5%) 105 1 5% 100 1 5% 110.25 + + + = + = + =


By open in g a n ot h er a ccou n t , you h a ve cr ea t ed a compou n d in t er es t r a t e
in Ba n k A a n d bea t t h e ba n k a t it s own ga me. Beca u s e of t h is , ba n ks
offer you t h e compou n d in t er es t r a t e. If a ba n k does n t offer you a
compou n d in t er es t r a t e, you s imply open mor e a n d mor e a ccou n t s ea ch
yea r (t h is is goin g t o dr ive bot h you a n d t h e ba n k cr a zy).
Forc e of i nt e re s t
Th is is a difficu lt con cept for ma n y. Th e idea , h owever , is r ea lly s imple.
Mon ey does n t s leep; it wor ks on t h e 7 24 ba s is . Con s equ en t ly, mon ey
s h ou ld gen er a t e in t er es t con t in u ou s ly a n d in s t a n t ly. Th e for ce of in t er es t
is a n in s t a n t a n eou s in t er es t r a t e (t h e in t er es t r a t e you ea r n du r in g a n
in s t a n t of t ime).

An ot h er wa y t o s ee h ow you r mon ey ca n ea r n in t er es t in s t a n t ly a n d
con t in u ou s ly, ima gin e t h a t t h e in t er es t compou n din g per iod keeps
s h r in kin g. Now in s t ea d of compou n din g on t h e a n n u a l ba s is , you r mon ey
compou n ds ever y s econ d. As t h e compou n din g fr equ en cy a ppr oa ch es t h e
in fin it y, you r mon ey in deed ea r n s in t er es t in s t a n t ly.

At t ime t , you r a ccou n t h a s ( ) A t dolla r s . At t ime t t + A , you r a ccou n t h a s
( ) A t t + A dolla r s . Wh a t s t h e in t er es t r a t e you ve ea r n ed du r in g t h e
in s t a n t t A ?
( )
( ) ( )
( )
0 0
interest earned during
lim lim
your beginning account
t t
A t t A t
t
t
t A t t
o
A A
+ A
A
= =
A A

In t h e a bove equ a t ion , we divide in t er es t ea r n ed by t h e begin n in g
a ccou n t ( ) A t a n d by t h e len gt h of t ime t A . Th is is beca u s e a n in t er es t
r a t e is $ ea r n ed per u n it of t ime per u n it of mon ey len t ou t .
52
http://actuary88.com
( )
( ) ( )
( )
( ) ( )
( ) ( )
( )
( )
0
0
lim
1
lim ln
t
t
A t t A t
A t t A t dA t
d
t
t A t
A t t A t A t dt dt
o
A
A
+ A
+ A
A
= = = =
A
By in t egr a t in g t h e a bove equ a t ion , we h a ve
( ) ( ) ( )
0
0 exp
t
A t A x dx o
(
=
(

}
Example 1 .
You r $100 depos it gr ows u n der t h e followin g for ce of in t er es t :

( )
2% 0 5
5% 0.3% 5
t
t
t t
o
s s
=

+ >

Ca lcu la t e
(1) You r a ccou n t va lu e a ft er 10 yea r s .
(2) Th e equ iva len t s imple in t er es t you h a ve ea r n ed du r in g t h e 10 yea r s .
(3) Th e equ iva len t compou n d in t er es t you h a ve ea r n ed du r in g t h e 10
yea r s .

Solut ion
Th e a ccou n t va lu e a ft er 10 yea r s :

( ) ( ) ( ) ( ) ( )
10 5 10
0 0 5
10 0 exp 100exp 2% 5% 0.3% A A t dt dt t dt o
( (
= = + +
( (

} } }

( )
10
5
2 46.25%
0
5
1
100exp 2% 5% 0.3% 100 $158.8039
2
t t t e

(
= + + = = (
`
(


)

Fin d t h e equ iva len t s imple in t er es t r a t e:

( ) ( )( )
simple
10 0 1 10 A A i = +
( )
simple
158.8039 100 1 10i = + ,
simple
5.88% i =
Fin d t h e equ iva len t compou n d in t er es t r a t e:

( ) ( )( )
10
compound
10 0 1 A A i = +
( )
10
compound
158.8039 100 1 i = + ,
compound
4.73% i =
53
http://actuary88.com
Example 2 (May 2 0 0 4 SOA EA-1 #2 )
On 1/ 1/ 2004, Smit h pu r ch a s es a n a n n u it y cer t a in t h a t h a s t h r ee s emi-
a n n u a l pa ymen t s of $500 ea ch , wit h t h e fir s t pa ymen t t o be ma de
7/ 1/ 2009. Th e for ce of in t er es t a t t ime t is given by:
( )
1
50 2
t
t
o =
+
wh er e 0 t > ; t is mea s u r ed in yea r s fr om 1/ 1/ 2004

In wh a t r a n ge is t h e pr es en t va lu e of t h e a n n u it y on 1/ 1/ 2004?
[A] Les s t h a n $1,350
[B] $1,350 bu t les s t h a n $1,355
[C] $1,355 bu t les s t h a n $1,360
[D] $1,360 bu t les s t h a n $1,365
[E] $1,365 or mor e

Solut ion
Th e cor r ect a n s wer is A

1/ 1/ 2004 7/ 1/ 2009 1/ 1/ 2010 7/ 1/ 2010

Time 0 5. 5 6 6. 5

Amou n t $500 $500 $500

Gen er a lly, for a given t h e for ce of t h e in t er es t ( ) t o , t h e pr es en t va lu e of a
ca s h flow X occu r r in g a t t ime T is ( )
0
exp
t
PV X s ds o
| |
=
|
\ .
}
.
Time 0 T
( )
0
exp
t
PV X s ds o
| |
=
|
\ .
}
X
Applyin g t h is gen er a l r u le, we fin d t h a t t h e pr es en t va lu e of t h e a n n u it y
is :

( ) ( ) ( )
5.5 6 6.5
0 0 0
500 exp exp exp t dt t dt t dt o o o
( | | | | | |
+ +
( | | |
(
\ . \ . \ .
} } }

54
http://actuary88.com
5.5 6 6.5
0 0 0
1 1 1
500 exp exp exp
50 2 50 2 50 2
dt dt dt
t t t
( | | | | | |
= + +
( | | |
+ + +
(
\ . \ . \ .
} } }

Th e t r icky pa r t is doin g t h e in t egr a t ion .

Let s fin d
0
1
50 2
x
dt
t +
}
. Set 50 2 , 25
2
y
t y t + = = a n d
1
2
dt dy =
50 2
50 0 50
50 2
1 1 1 1 1 50 2 1
ln ln ln 1
50 2 2 2 2 50 2 25
x x x
x x
dt dy y
t y
+ +
+ ( | |
= = = = +
|
(
+
\ .
} }

( )
0 0
1 1
exp exp exp ln 1
50 2 2 25
x x
x
t dt dt
t
o
| | | |
( | |
= = +
| |
| (
+
\ .
\ . \ .
} }

1 1
2 2
exp ln 1 1
25 25
x x
(
| | | |
(
= + = +
| |
(
\ . \ .


Th e pr es en t va lu e is :

1 1 1
2 2 2
5.5 6 6.5
500 1 1 1 1, 347.14274
25 25 25
(
| | | | | |
(
+ + + + + =
| | |
(
\ . \ . \ .


Example 3 (SOA May 2 0 0 5 EA-1 #2 5 )
Loa n a mou n t : $10,000
Pa ymen t Ter ms : Two pa ymen t s :
En d of Yea r 1: X
En d of Yea r 2: 1.1X
For ce of in t er es t : 0.06 0.01t + , for 2 t s
In wh a t r a n ge is X ?
(A) Les s t h a n $5,210
(B) $5,210 bu t les s t h a n $5,280
(C) $5,280 bu t les s t h a n $5,380
(D) $5,380 bu t les s t h a n $5,480
(E) 45,420 or mor e

55
http://actuary88.com
Solut ion
Gen er a lly, for a given t h e for ce of t h e in t er es t ( ) t o , t h e pr es en t va lu e of a
ca s h flow X occu r r in g a t t ime T is ( )
0
exp
t
PV X s ds o
| |
=
|
\ .
}
.
Time 0 T
( )
0
exp
t
PV X s ds o
| |
=
|
\ .
}
X
Th en it follows :
( ) ( )
1 2
0 0
PV of 1.1 at 2 PV of at 1
exp 0.06 0.01 1.1 exp 0.06 0.01 10, 000
X t X t
X t dt X t dt
= =
( (
+ + + =
( (

} }


0.065 0.14
1.1 10, 000 Xe Xe

+ =
0.065 0.14
10, 000
5, 281.61
1.1
X
e e

= =
+
So t h e a n s wer is C.

Example 4
You a r e given t h e followin g for ce of in t er es t :

( )
0.09 0.005 for 6
0.08 for 6
t t
t
t
o
s
=

>

(1) Ca lcu la t e t h e a ccu mu la t ion a t 10 t = of $100 in ves t ed a t 0 t =


(2) Ca lcu la t e t h e pr es en t va lu e a t 0 t = of a con t in u ou s pa ymen t s t r ea m
a t t h e r a t e of
0.2t
e fr om 10 t = t o 15 t = .
Solut ion
(1) Th e a ccu mu la t ion va lu e is :
( ) ( )
10 6 10
0 0 6
100exp 100exp 0.09 0.005 0.08 t dt t dt dt o
( (
= +
( (

} } }

[ ]
6
10
2
6
0
1
100exp 0.09 0.005 0.08
2
t t t

(
= +
`
(


)
( ) 100exp 0.45 0.32 215.98 = + =
56
http://actuary88.com
(2) If we h a ve $1 a t t wh er e t >6, t h en it s PV a t t ime zer o is
( ) ( )
6
0 6 0
0.09 0.005 0.08
t t
s ds ds s ds
e e
o
(
( +
(

} } }
= .
If we h a ve
0.2 t
e a t t wh er e t >6, t h en it s PV a t t ime zer o is
( )
6
0 6
0.09 0.005 0.08
0.2
t
s ds ds
t
e e
(
( +
(

} }

If we h a ve a con t in u ou s ca s h flow of
0.2 t
e fr om t =10 t o t =15, t h en t h e PV
of t h e ca s h flow s t r ea m is

( ) ( )
6
0 6 0
15 15 0.09 0.005 0.08
0.2 0.2
10 10
t t
s ds ds s ds
t t
e e dt e e dt
o
(
( +
(

} } }
=
} }
( )
15 15
0.45 0.08 0.48 0.2 0.2 0.08 0.03
10 10
t t t t
e e dt e e dt
+ +
= =
} }

( )
15 0.03 0.03
15
0.03 0.12 0.12 1.8 1.2
10
10
23.44
0.12 0.12
t t
e e
e e dt e e e ( = = = =
}
Example 5
Th e for ce of in t er es t ( )
2
t a bt o = + wh er e a a n d b a r e con s t a n t s . An
a mou n t of $100 in ves t ed a t 0 t = a ccu mu la t es t o $135 a t 6 t = a n d $200
a t 9 t = . Ca lcu la t e a a n d b .
Solut ion
( )
6 6
2 3
0 0
6 72
1
135 100exp 100exp 100
3
a b
a bt dt at bt e
+
(
(
= + = + =
(
(


}
( )
9 9
2 3
0 0
9 243
1
200 100exp 100exp 100
3
a b
a bt dt at bt e
+
(
(
= + = + =
(
(


}
ln1.35 6 72 a b = + , ln 2 9 243 a b = +
0.0284 a = , 0.0018 b =
Nominal i nt e re s t rat e
h t t p:/ / www.you t u be.com/ wa t ch ?v=wzvpD5ea u n k
Ma n y ba n ks ca lcu la t e a n d pa y in t er es t on t h e qu a r t er ly, mon t h ly, or
even da ily ba s is . Wh en t h e in t er es t is ca lcu la t ed mor e fr equ en t ly t h a n
a n n u a lly, t h e in t er es t r a t e, h owever , is oft en s t ill qu ot ed on t h e a n n u a l
ba s is .

57
http://actuary88.com
For exa mple, you depos it $100 in a ba n k wit h a n omin a l in t er es t r a t e of
6% compou n ded mon t h ly. Her e t h e compou n din g fr equ en t ly is mon t h ly,
bu t t h e in t er es t r a t e is s t ill qu ot ed a s a n a n n u a l r a t e.

To ca lcu la t e you a ccou n t va lu e, you ll n eed t o con ver t t h e a n n u a l in t er es t
r a t e t o t h e mon t h ly in t er es t r a t e. Th e mon t h ly in t er es t r a t e is
6%/ 12=0.5%. You r in t er es t is 100(0.5%)=$0.5 a t t h e en d of Mon t h 1.

At t h e en d of Mon t h 1, you r ba n k a ccou n t gr ows t o
$100.5.

At t h e en d of Mon t h 2, you r ba n k a ccou n t gr ows t o
( )
2
100 1 0.5% $101.0025 + = .
At t h e en d of Mon t h 3, you r ba n k a ccou n t gr ows t o
( )
3
100 1 0.5% $101.5075125 + = .
If t h e in t er es t r a t e is qu ot ed on t h e a n n u a l ba s is bu t is a ct u a lly a pplied
t o s h or t er in t er va ls (s u ch a s qu a r t er ly, mon t h ly, da ily), t h en it is ca lled a
n omin a l in t er es t r a t e. A n omin a l in t er es t r a t e is oft en expr es s ed a s
( )
p
i ,
mea n in g t h a t in t er es t r a t e is compou n ded p -t h ly.

If you depos it $100 in t o a ba n k a ccou n t wit h a n omin a l in t er es t r a t e
( )
2
6% i = , t h en t h e in t er es t you h a ve ea r n ed is ca lcu la t ed t wice a yea r ,
u s in g a 6 mon t h in t er es t r a t e of 6%/ 2=3%. You r ba n k a ccou n t a t t h e en d
of Mon t h 6 is ( ) $100 1 3% $103 + = ; you r ba n k a ccou n t a t t h e en d of Mon t h
12 is ( )
2
$100 1 3% $106.09 + = .
If you depos it $100 in t o a ba n k a ccou n t wit h a n omin a l in t er es t r a t e
( )
4
6% i = , t h en t h e in t er es t you h a ve ea r n ed is ca lcu la t ed fou r t imes a
yea r , u s in g a qu a r t er ly in t er es t r a t e of 6%/ 4=1.5%. You r ba n k a ccou n t a t
t h e en d of Mon t h 3 is ( ) $100 1 1.5% $101.5 + = ; a t t h e en d of Mon t h 6 it is
( )
2
$100 1 1.5% $103.0225 + = ; a t t h e en d of Mon t h 9 it is ( )
3
$100 1 1.5% + ; a t t h e
en d of Mon t h 12 it is ( )
4
$100 1 1.5% + .
APR
APR s t a n ds for a n n u a l per cen t a ge r a t e. APR ca lcu la t ion con s ider s t h e
fa ct t h a t a bor r ower oft en pa ys va r iou s fees in get t in g a loa n a n d pa ys a
h igh er in t er es t r a t e t h a n t h e s t a t ed in t er es t r a t e.

58
http://actuary88.com
For exa mple, you bor r ow $150,000 fr om a ba n k t o fin a n ce you r n ew
h ome. Th e in t er es t r a t e is n omin a l 5% compou n ded mon t h ly. You pla n t o
pa y off you r mor t ga ge in 20 yea r s .

Th e followin gs a r e t h e fees you r ba n k ch a r ges you wh en you get you r
mor t ga ge:

St at e d Princ ipal $ 1 5 0 , 0 0 0
Fe e s c harge d:
Loa n or igin a t ion fees (1% of t h e
pr in cipa l)
1%(150,000)=$1,500
Un der wr it in g fees $400
Cr edit r epor t $50
Pr oces s in g, docu men t a t ion , a n d
mis cella n eou s fees
$500
Tot a l fees 1,500+400+50+500 = $2,450
Ne t amount borrowe d 1 5 0 , 0 0 0 -2 , 4 5 0 = $ 1 4 7 , 5 5 0
To ca lcu la t e t h e APR, we n eed t o fir s t fin d t h e mon t h ly pa ymen t u s in g
t h e s t a t ed pr in cipa l of $150,000. In ot h er wor ds , wh en you s ign t h e
mor t ga ge con t r a ct wit h you r ba n k, you r ba n k ca lcu la t es you r mon t h ly
pa ymen t u s in g t h e s t a t ed pr in cipa l of $150,000. So t h e qu es t ion is t h is :
You bor r ow $150,000 a t a n omin a l in t er es t r a t e of 5%; you a r e pa yin g off
t h e loa n in 20 yea r s . How mu ch do you n eed t o pa y per mon t h (a s s u min g
t h a t you pa y you r mon t h ly mor t ga ge a t t h e en d of ea ch mon t h )?

Righ t n ow, you pr oba bly h a ven t lea r n ed h ow t o ca lcu la t e t h e mon t h ly
pa ymen t . Th a t s OK. Let me give you t h e a n s wer -- $989.93. I ca lcu la t ed
t h is n u mber u s in g BA II Plu s TVM.

Next , you a s k t h e qu es t ion , Im pa yin g $989.93 per mon t h for 20 yea r s .
However , t h e n et a mou n t I bor r owed is r ea lly $147,550. Wh a t s t h e
mon t h ly in t er es t r a t e a m I pa yin g? We do s ome ma t h a n d fin d t h e
mon t h ly in t er es t r a t e is 0.433%. Th en , t h e n omin a l in t er es t r a t e
compou n ded mon t h ly is 0.433%(12)= 5.20%. Th e APR is 5.20%.
Th e s t a t ed in t er es t r a t e is 5%; t h e APR is 5.2%. Th e APR is h igh er t h a n
t h e s t a t ed in t er es t r a t e.

Annual e ffe c t i ve int e re s t rat e
If you depos it $100 in t o a ba n k a ccou n t wit h a n omin a l in t er es t r a t e
( )
2
6% i = , t h en t h e in t er es t you ea r n ed is ca lcu la t ed t wice a yea r , u s in g a
6 mon t h in t er es t r a t e of 6%/ 2=3%. You r ba n k a ccou n t a t t h e en d of
Mon t h 12 is ( )
2
$100 1 3% $106.09 + = . As a r es u lt , t h e a ct u a l in t er es t r a t e on
59
http://actuary88.com
t h e a n n u a l ba s is is ( )
2
1 3% 1 6.09% + = . 6.09% is t h e effect ive a n n u a l
in t er es t r a t e.

If t h e n omin a l in t er es t r a t e is
( )
p
i , t h en t h e a n n u a l effect ive r a t e is

( )
1 1
p
p
p
i
(
(
(

+ .
Cont i nuous c ompoundi ng
You depos it $1 in t o a ba n k a ccou n t . Th e in t er es t is ca lcu la t ed in s t a n t ly
a t a n omin a l r a t e of
( )
i o
+
= . At t h e en d of t h e yea r , $1 will become

( )
1 1
lim
p
p
e
p
i
o
o
+
+

(
| |
= = (
|
+
( \ .

+ +
At t h e en d of t yea r s (t ca n be fr a ct ion a l), $1 will become

1 1
lim lim
p
p p
t
t
p
t
e
p p
o
o
o
o o
+ +


| | | |
= =
` `
| |
\ . \ .

)
)
+ +
Example .
Th e a n n u a l effect ive in t er es t r a t e is 10%. Wh a t s t h e con t in u ou s ly
compou n ded a n n u a l in t er es t r a t e?

Solut ion
If we in ves t $1 a t t ime zer o, t h en well h a ve $1.1 a t t ime on e.

1.1
t
e
o
= wh er e 1 t = ( ) ln 1 ln1.1 9.531% i o = + = =
Effe c t ive annual rat e of dis c ount

In t er es t pa ya ble in adva n ce for bor r owin g $1 for a u n it t ime

Example 1 . 5% effect ive a n n u a l r a t e of dis cou n t mea n s t h a t for ever y
dolla r you bor r ow for on e yea r , you n eed t o pa y $0.05 in t er es t in a dva n ce
(i.e. a t t h e begin n in g of t h e yea r ). Effect ive a n n u a l r a t e of dis cou n t is ju s t
a n ot h er wa y t o ca lcu la t e t h e in t er es t a bor r ower n eeds t o pa y t o t h e
len der .
60
http://actuary88.com
Example 2 . If you bor r ow $100 fr om a ba n k on e yea r . At t h e en d of Yea r
1, you n eed t o r et u r n t h e pr in cipa l t o t h e ba n k. Th e ba n k ch a r ges you a
5% a n n u a l effect ive r a t e of dis cou n t .

Iden t ify you r ca s h flows a n d t h e ba n ks ca s h flows . Ca lcu la t e t h e a ct u a l
in t er es t r a t e t h e ba n k a ct u a lly ch a r ges you .

Solut ion

Time t (yea r ) 0 1
You r ca s h flow
( ) 100 100 5% $95 =
$100
Ba n ks ca s h flow
( ) 100 100 5% $95 + =
$100
At t h e t ime of bor r owin g ( 0 t = ), t h e ba n k gives $100. Simu lt a n eou s ly, t h e
ba n k ch a r ges you 100(5%)=$5 in t er es t . So you r ea lly get $95 n et a t 0 t = .
At 1 t = , you pa y ba ck $100 t o t h e ba n k.

To ca lcu la t e t h e in t er es t r a t e you r loa n is ch a r ged, n ot e t h a t you a ct u a lly
bor r ow $95 a t 0 t = a n d pa ys ba ck $100 a t 1 t = . So t h e bor r owin g r a t e is

100 95
5.263%
95

=
We ca n a ls o ca lcu la t e t h e effect ive in t er es t r a t e fr om t h e ba n ks poin t of
view. Th e ba n k len ds you $95 a t 0 t = bu t get s $100 a t 1 t = . So t h e
ba n ks ea r n in g r a t e is :

100 95
5.263%
95

=
Example 3 . If you bor r ow $1 fr om a ba n k for on e yea r . At t h e en d of Yea r
1, you r et u r n t h e pr in cipa l t o t h e ba n k. Th e a n n u a l effect ive r a t e of
dis cou n t is d . Iden t ify you r ca s h flows . Con ver t t h e effect ive a n n u a l
dis cou n t r a t e t o t h e effect ive a n n u a l in t er es t r a t e.

Solut ion

Time t (yea r ) 0 1
You r ca s h flow
( ) $ 1 d
$1
Ba n ks ca s h flow
( ) $ 1 d
$1
61
http://actuary88.com
Th e ba n k in ves t s ( ) 1 d a t 0 t = bu t get s ba ck $1 a t 1 t = . So t h e
equ iva len t a n n u a l effect ive in t er es t r a t e is :

( ) 1 1 ending fund value - beginning fund value 1
1
beginning fund value 1 1
d
i
d d

= = =


1 1
1 , 1
1 1
i d
d i
+ = =
+
If we s et
1
1
v
i
=
+
, t h en 1 d v =
Example 4 . Der ive t h e followin g equ a t ion a n d expla in t h e mea n in g of t h e
equ a t ion .

1
i
d
i
=
+
Solut ion

1
1
1 1
i
d i v
i i
= = =
+ +

Th e a bove equ a t ion s a ys t h a t if we bor r ow $1 for on e u n it of t ime, t h en
t h e a dva n ce in t er es t d pa id a t t h e t ime of t h e bor r owin g (i.e. 0 t = ) is
s imply t h e pr es en t va lu e of t h e in t er es t i du e a t 1 t = . Th is ma kes
in t u it ive s en s e.

Example 5 . Der ive t h e followin g equ a t ion a n d expla in t h e mea n in g of t h e
equ a t ion .

( )
1
1
1
d
i d d
d

= =

Solut ion

( )
1 1 1
1 1 1
1 1 1
d
i i d d
d d d

+ = = = =


To u n der s t a n d t h e mea n in g of ( )
1
1 i d d

= , plea s e n ot e t h a t ( )
1
1 d

is t h e
a ccu mu la t in g fa ct or for a u n it t ime. If we in ves t $1 a t 0 t = , t h en t h is $1
will gr ow in t o ( )
1
1 d

a t 1 t = .
62
http://actuary88.com
( )
1
1 i d d

= mea n s t h a t if we bor r ow $1 a t 0 t = , t h en t h e in t er es t du e a t
1 t = is s imply t h e a ccu mu la t ed va lu e of t h e a dva n ce in t er es t d pa id a t
0 t = . Th is ma kes lot of s en s e.

Simple annual rat e of dis c ount
J u s t a s in t er es t r a t e ca n be a s imple r a t e or a n a n n u a l effect ive r a t e, a
dis cou n t r a t e ca n be a s imple r a t e or a n a n n u a l effect ive r a t e. It s h a r d t o
t h in k in t u it ively wh a t a s imple dis cou n t r a t e r ea lly mea n s . So don t wor r y
a bou t in t er pr et in g t h e s imple dis cou n t r a t e. J u s t n eed t o r emember t h e
followin g key for mu la :

If a s imple dis cou n t r a t e is d , t h en $1 a t t ime t is wor t h 1 dt a t 0 t = ;
$1 a t t ime 0 t = is wor t h ( )
1
1 dt

a t t ime t .
Nominal annual rat e of dis c ount
Th is is s imila r t o n omin a l a n n u a l r a t e of in t er es t r a t e. Th e dis cou n t r a t e
is qu ot ed on t h e a n n u a l ba s is bu t is a ct u a lly a pplied t o s h or t er per iods
(s u ch a s mon t h ly or qu a r t er ly).

Th e for mu la is :
( )
1 1
m
m
d
d
m
(
=
(

63
http://actuary88.com
Re lat i ons hip be t we e n i , , d o ,
( ) m
i ,
( ) m
d
Example. If
( ) 2
10% d = . Calculate d , i , o ,
( ) 12
i .
Strategy: Dont just memorize a host of formulas. Understand the meaning of each
symbol. Understand how money travels over time. You should do fine.

Lets look at each symbol.

First, i . This symbol means the effective interest rate per unit of time (typically per year).
You are most familiar with this symbol. If i =10%, then if you borrow $1 at 0 t = , youll
need to pay 10%*$1=$0.1 at the end of year.

Next, memorize the following diagram:

0 t = 1 t =
$1 $( ) 1 i +
$
1
1
v
i
=
+
$1
$1 at 0 t = is worth $( ) 1 i + at 1 t = . $1 at 1 t = is worth $
1
1
v
i
=
+
at 0 t = .
Wealth at 1
1
Wealth at 0
t
Wealth Ratio i
t
=
= = +
=
64
http://actuary88.com
d is the interest paid in advance for every $1 you borrow. If you borrow $1 at time zero.
Then immediately you are charged the interest d for each dollar you borrow. If 0.09 d =
and you borrow $1 at time zero, the lender immediately takes away 0.09*$1=$0.09. You
walk away with $0.91 in your pocket. Generally, if you borrow $1 at 0 t = , you walk
away with ( ) 1 d in your pocket; you need to pay $1 at 1 t = to pay off your loan.

Next, memorize the following diagram:

0 t = 1 t =
$( ) 1 d $1
$1 $
1
1 d
$( ) 1 d at 0 t = is worth $1 at 1 t = . $
1
1 d
at 1 t = is worth $1 at 0 t = .
Wealth at 1 1
Wealth at 0 1
t
Wealth Ratio
t d
=
= =
=
65
http://actuary88.com
o is the force of the interest, the interest rate you earn during a tiny interval such as one
second. If you deposit $1 into a bank account that earns a constant force of interest o ,
then for a tiny interval of time [ ] , t t dt + , youll earn dt o interest. If you deposit $1 at
0 t = , then this $1 will grow into
1
0
ds
e e
o
o
}
= at 1 t = (here we assume a constant o ).

Next, memorize the following diagram:

0 t = 1 t =
$1 $
1
0
ds
e e
o
o
}
=
$e
o
$1
$1 at 0 t = is worth $e
o
at 1 t = . $1 at 1 t = is worth $e
o
at 0 t = .
Wealth at 1
Wealth at 0
t
Wealth Ratio e
t
o
=
= =
=
66
http://actuary88.com
( ) 2
i means that instead of paying the interest once a year at 1 t = , you are paying the
interest twice a year, 1
st
time at 0.5 t = (end of 6-th month) and the 2
nd
time at 1 t = (end
of 12
th
month). Your effective interest rate per 6 month is
( ) 2
2
i
. Similarly,
( ) 12
i means that
instead of paying the interest once a year at 1 t = , you are paying the interest monthly at
1
12
t = (end of Month 1),
2
12
(end of Month 1), ,
11
12
(end of Month 11), and
12
12
(end of
Month 12). Your effective interest rate per month is
( ) 2
12
i
.
( ) m
i means that interest is paid
m-thly per year at
1 2 3 1
, , ,..., ,
m m
t
m m m m m

= .
Next, memorize the following diagram:

0 t =
1
t
m
=
2
t
m
=
1 m
t
m

= 1
m
t
m
= =
$1
( )
1
m
i
m
(
+
(

( )
2
1
m
i
m
(
+
(

( )
1
1
m
m
i
m

(
+
(

( )
1
m
m
i
m
(
+
(

( )
1
m
m
i
m

(
+
(

( )
( ) 1
1
m
m
i
m

(
+
(


( )
( ) 2
1
m
m
i
m

(
+
(

...
( )
1
1
m
i
m

(
+
(

$1
$1 at 0 t = is worth
( )
1
m
i
m
(
+
(

at
1
t
m
= ,
( )
2
1
m
i
m
(
+
(

at
2
t
m
= , , and
( )
1
m
m
i
m
(
+
(

at 1 t = .
$1 at 1 t = is worth
( )
1
m
m
i
m

(
+
(

at 0 t = .
( )
Wealth at 1
1
Wealth at 0
m
m
t i
Wealth Ratio
t m
(
=
= = +
(
=

67
http://actuary88.com
Imagine that $1 at 0 t = travels through the time going through m consecutive doors.
When going through each door, $1 expands to
( )
1
m
i
m
(
+
(

. After going through m doors, it
lands at 1 t = becoming
( )
1
m
m
i
m
(
+
(

.
Finally, lets look at the most difficult symbol
( ) m
d . You walk into a bank to borrow $1.
The bank charges you,
( ) 2
d , a nominal discount rate convertible semiannually.
( ) 2
d
means that cash is deducted twice from your declining principal before it is lent to you.

Imagine a loan officer puts $1 worth of coins into a jar. This $1 is the amount you want to
borrow. You really want to take away this $1 and go home, but the loan officer says,
Wait a minute. Let me deduct your interest payment twice. Then you can take away the
rest. The 1
st
time, the loan officer takes out
( ) 2
2
d
portion of coins out of the jar. Because
the initial principal is just $1,
( ) 2
2
d
portion of $1 is
( ) ( ) 2 2
1
2 2
d d
= . This is your 1
st
interest
payment in advance. After this payment, your principal shrinks from $1 to
( ) 2
1
2
d
. So
now the jar has only
( ) 2
1
2
d
worth of coins. You really want to take
( ) 2
1
2
d
and go
home, the loan officer says, Wait a minute. I need to deduct your interest payment the
2
nd
time. So he takes out
( ) 2
2
d
portion whats left in the jar. So
( ) ( ) 2 2
1
2 2
d d
(

(

worth of
coins is taken out from the jar, leaving only
( ) ( ) ( ) ( )
2
2 2 2 2
1 1 1
2 2 2 2
d d d d
( ( (
=
( ( (

in the
jar. Your 2
nd
interest payment in advance is
( ) ( ) 2 2
1
2 2
d d
(

(

. Now your principle shrinks
from
( ) 2
1
2
d
to
( )
2
2
1
2
d
(

(

.
Your total interest payment in advance is:

( ) ( ) ( )
( )
( ) ( )
2
2 2 2 2 2
2
1 1 1
2 2 2 4 2
d d d d d
d
( (
+ = =
( (

68
http://actuary88.com
The result should make intuitive sense. Originally, the jar has $1 worth of coins. After
two deductions, the jar has only
( )
2
2
1
2
d
(

(

worth of coins left. So the total coins deducted
(i.e. your total interest payment in advance) is simply
( )
2
2
1 1
2
d
(

(

. So you walk away
with
( )
2
2
1
2
d
(

(

in your pocket.

At 1 t = , you pay $1 back to the bank, paying off the loan. Now you owe your bank
nothing. Lets draw a diagram to describe your loan:

0 t = 1 t =
( )
2
2
1
2
d
(

(

$1
( )
2
2
1
2
d
(

(

at 0 t = is worth $1 at 1 t = .
( )
2
2
1
2
d
(

(

at 0 t = is equivalent to $1 1 t = . Then it follows that $1 at 0 t = is equivalent
to
( )
2
2
1
2
d

(

(

at 1 t = . Now we can change the diagram into:

69
http://actuary88.com
Cash flow diagram under
( ) 2
d
0 t = 1 t =
( )
2
2
1
2
d
(

(

$1
$1
( )
2
2
1
2
d

(

(

( )
2
2
1 1
2
d
(

(

at 0 t = is worth $1 at 1 t = .
( )
2
2
1
1 1
2
d
(

(

at 1 t = is worth $1 at 0 t = .
( )
2
2
Wealth at 1 1
Wealth at 0
1 1
2
t
Wealth Ratio
t
d
=
= =
=
(

(

We can easily extend the above reasoning to
( ) m
d . If you want to borrow $1 from a bank
under a nominal discount rate
( ) m
d , then your loan officer immediately deducts interest
payments in advance m times from your initial principal of $1. In the 1
st
deduction,
( ) m
d
m
portion of your initial $1 is deducted, leaving
( )
1
m
d
m
left. In the 2
nd
deduction,
( ) m
d
m
portion of the remaining
( )
1
m
d
m
is deducted, leaving
( )
2
1
m
d
m
(

(

left. After m
70
http://actuary88.com
deductions, only
( )
1
m
m
d
m
(

(

is left. You walk away with
( )
1
m
m
d
m
(

(

in your pocket.
Then one year later at 1 t = , you pay $1 back to the bank. After this payment, you owe
the bank nothing.

Cash flow diagram under
( ) m
d :
0 t = 1 t =
( )
1
m
m
d
m
(

(

$1
$1
( )
1
m
m
d
m

(

( )
1
m
m
d
m
(

(

at 0 t = is worth $1 at 1 t = .
( )
1
m
m
d
m

(

at 1 t = is worth $1 at 0 t = .
( )
Wealth at 1
1
Wealth at 0
m
m
t d
Wealth Ratio
t m

(
=
= =
(
=

71
http://actuary88.com
Now, we are ready to derive the relationship between i , , d o ,
( ) m
i ,
( ) m
d . You should
memorize the following diagram:
0 t = 1 t =
$1 $( ) 1 i +
$1 $
1
1 d
$1 $e
o
$1
( )
1
m
m
i
m
(
+
(

$1
( )
1
m
m
d
m

(

Immediately, we see that
( ) ( )
1
1 1 1
1
m m
m m
i d
i e
d m m
o

( (
+ = = = + =
( (


From here you can derive all sorts of formulas. For example,
1
1
1
d
i
=
+
,
1
1 1
1
d v
i
= =
+
1 i e
o
+ = , ( ) ln 1 i o = +
( )
1 1
m
m
i
i
m
(
+ = +
(

,
( )
1 1
m
m
i
i
m
(
= +
(


( )
1 1
m
m
d
i
m

(
+ =
(

,
( )
1 1
m
m
d
i
m

(
=
(


( )
1
1
1
m
m
d
d m

(
=
(


,
( )
1 1
m
m
d
d
m
(
=
(

72
http://actuary88.com
You can also derive the relationships using the following diagram:

0 t = 1 t =
1
1
v
i
=
+
1
1 d 1
e
o
1
( )
1
m
m
i
m

(
+
(

1
( )
1
m
m
d
m
(

(

1
Now you should clearly see that :

( ) ( )
1
1 1 1
1
m m
m m
i d
v d e
i m m
o

( (
= = = = + =
( (
+

From here, you can derive all sorts of relationship formulas.

Yet, theres the third approach to use the wealth ratio. The wealth ratio should the same
under different measurements of interest. Hence:

( ) ( )
Wealth at 1 1
1 1 1
Wealth at 0 1
m m
m m
t i d
Wealth Ratio i e
t d m m
o

( (
=
= = + = = = + =
( (
=

73
http://actuary88.com
Finally, lets solve the problem: If
( ) 2
10% d = . Calculate d , i , o ,
( ) 12
i .
( )
2
2
2
10%
1 1 1
2 2
d
d
(
(
= =
(
(


, d =9.75%

1
1
1
d
i
=
+
= 1- 9.75%, i =10.8% 1 e i
o
= + , ( ) ln 1 ln1.108 0.10259 i o = + = =
( )
12
12
1 1 1.108
12
i
i
(
+ = + =
(

,
( ) 12
i =10.3%

Example 2. You invest $150 at time zero in an account that earns the following interest:
During Year 1,
( ) 4
10% i = .
During Year 2,
( ) 12
10% d = .
During Year 3, 10% o =
During Year 4, ( ) 2 0.1 t t o = +
Calculate your account value at the end of Year 4.

Solution

Let ( ) A t represent your account value at time t .
( ) 0 100 A = ( ) ( )
4
10%
1 0 1
4
A A
| |
= +
|
\ .
( ) ( )
12
10%
2 1 1
12
A A

| |
=
|
\ .
( ) ( )
10%
3 2 A A e = ( ) ( ) ( )
4
3
4 3 exp 2 0.1 A A t dt
| |
= +
|
\ .
}
( ) ( ) ( ) ( )
4
3
2 2 2
4
3
2 0.1 2 0.05 2 4 3 0.05 4 3 2.35 t dt t t + = + = + =
}
( ) ( )
2.35
4 3 e A A =
( ) ( )
4 12 4
10%
3
10% 10%
4 100 1 1 exp 2 0.1
4 12
A e t dt

| |
| | | |
= + +
|
| |
\ . \ .
\ .
}
4 12
10% 2.35
10% 10%
100 1 1 1, 414.26
4 12
e e

| | | |
= + =
| |
\ . \ .

74
http://actuary88.com
Fut ure value
http://www.youtube.com/watch?v=3ZmiOGsL8G4
Fu t u r e va lu e is t omor r ows va lu e of t oda ys depos it . You depos it $100
in t o a ba n k a ccou n t a n d you r mon ey gr ows a t a compou n d in t er es t r a t e
of 6%. Th en a t t h e en d of Yea r 1, you r or igin a l depos it of $100 will gr ow
in t o $100(1+6%)=$106. $106 is t h e fu t u r e (1 yea r fr om n ow) va lu e of
you r or igin a l $100 depos it .

Simila r ly, a t t h e en d of Yea r 2, you r or igin a l depos it of $100 will become
$100(1+6%)
2
=$112.36. So t h e fu t u r e (2 yea r s fr om n ow) va lu e of you r
or igin a l depos it is $112.36.

Gen er a lly, if you depos it ( ) 0 A a t 0 t = a n d you r depos it gr ows a t a
compou n d in t er es t r a t e of i , t h en t h e fu t u r e va lu e of you r or igin a l
depos it t yea r s fr om n ow is ( ) ( ) ( ) 0 1
t
A t A i = + .
Pre s e nt value
h t t p:/ / www.you t u be.com/ wa t ch ?v=zGdu 2DHu 6yA
Pr es en t va lu e is t oda ys va lu e of t omor r ows mon ey. It is t h e depos it you
mu s t ma ke t oda y in or der t o r eceive s ome mon ey in t h e fu t u r e. For
exa mple, on e yea r fr om n ow you will r eceive $100. As s u me t h e in t er es t
r a t e is 6%. How mu ch mon ey do you n eed t o depos it in t o a ba n k a ccou n t
in or der for you t o r eceive $100 on e yea r fr om n ow?

Let ( ) 0 A r epr es en t t h e a mou n t of mon ey you mu s t depos it in t o a ba n k
a ccou n t n ow. Th en
( )( ) 0 1 6% $100 A + = ( )
$100
0 $94.34
1 6%
A = ~
+
So t o r eceive $100 on e yea r fr om n ow, you mu s t depos it $94.34 t oda y
a n d h a ve it gr ow a t 6%. $94.34 is t h e pr es en t va lu e of r eceivin g $100 on e
yea r fr om n ow.

You a r e r eceivin g ( ) A t a mou n t of mon ey in t yea r s . Th e in t er es t r a t e is i .
Wh a t s t h e pr es en t va lu e of ( ) A t a mou n t of mon ey t yea r s fr om n ow?
( ) ( ) ( ) 0 1
t
A t A i = + ( )
( )
( )
( )( ) 0 1
1
t
t
A t
A A t i
i

= = +
+
75
http://actuary88.com
So t h e pr es en t va lu e of r eceivin g ( ) A t a mou n t of mon ey in t yea r s is
( )( ) 1
t
A t i

+ .
Example 1
Ca lcu la t e t h e pr es en t va lu e of $100 a t 10 t = a t t h e followin g r a t es :
(1) a r a t e of in t er es t r a t e of 6% per yea r con ver t ible mon t h ly
(2) a r a t e of dis cou n t of 6% con ver t ible mon t h ly
(3) a for ce of in t er es t r a t e of 6%

Solut ion
(1)
12 10
6%
100 1 54.96
12

| |
+ =
|
\ .
(2)
12 10
6%
100 1 54.80
12

| |
=
|
\ .

(3)
10 10 6%
100 100 54.88 e e
o
= =
Example 2 (SOA May 2 0 0 0 EA-1 #2 )
A loa n of $1,800 is t o be r epa id by a s in gle pa ymen t of $2,420.8 t wo
yea r s a ft er t h e da t e of t h e loa n . Th e t er ms of t h e loa n a r e qu ot ed u s in g a
n omin a l a n n u a l in t er es t r a t e of 15%.

Wh a t s t h e fr equ en cy of compou n din g?
(A) mon t h ly
(B) ever y t wo mon t h s
(C) qu a r t er ly
(D) s emia n n u a lly
(E) a n n u a lly

Solut ion
Fir s t , let s ca lcu la t e t h e effect ive a n n u a l in t er es t r a t e.

( )
2
1, 800 1 2, 420.8 i + = , 15.9693% i =
We a r e given t h a t t h e n omin a l in t er es t r a t e is 15%. On e wa y t o fin d t h e
fr equ en cy of compou n din g is t o t es t differ en t fr equ en cies .

The ge ne ral formula:
Given
( ) p
i , t h e n omin a l in t er es t r a t e compou n din g p -t h ly a yea r , t h e
equ iva len t a n n u a l effect ive r a t e is :
( )
1 1
p
p
i
i
p
(
= +
(


76
http://actuary88.com
Fir s t , let t r y a n in t er es t r a t e compou n din g mon t h ly. Th e equ iva len t
a n n u a l effect ive r a t e of 15% compou n din g mon t h ly is ( 12 p = ):

12
15%
1 1 16.075% 15.9693%
12
| |
+ = =
|
\ .

So t h e mon t h ly compou n din g is n ot good.

Next , let s t r y a n in t er es t r a t e compou n din g ever y t wo mon t h s . Th e
equ iva len t a n n u a l effect ive r a t e of 15% compou n din g ever y t wo mon t h s
is ( ) 6 p = :
6
15%
1 1 15.9693 15.9693%
6
| |
+ = ~
|
\ .

Th is is good. So t h e a n s wer is B.

Conve rt i nt e re s t rat e t o di s c ount rat e or vi c e ve rs a

Proble m 1 (EA-1 #1 2 0 0 1 )
Select ed va lu es :
( )
1, 000 85.256
m
d =
( ) 2
1, 000 85.715
m
d =
Ca lcu la t e
( ) 3
1, 000
m
i
Solut ion
In s t ea d of memor izin g complex for mu la s , let s u s e t h e dis cou n t $1
met h od. If we h a ve $1 a t t ime on e, wh a t s it s va lu e a t t ime zer o?

Dis c ount ing Me t hod 1 dis cou n t
1
thly
m
fr om t ime zer o t o t ime 1.
Th e t ot a l # of dis cou n t in g per iods is m. Per per iod dis cou n t fa ct or is
( )
1
m
d
m
(

(

.
( )
85.256
0.085256
1, 000
1 1 1
m
m
m
m
d
PV
m m m
(
(
(
(
= = = (
(
(

(

(


77
http://actuary88.com
Dis c ount ing Me t hod 2 Dis cou n t
1
2
thly
m
fr om t ime zer o t o t ime
1. Th e t ot a l # of per iods is 2m. Per per iod dis cou n t fa ct or is
( ) 2
1
2
m
d
m
(

(

.
( )
2
2
2
2
85.715
0.085715
1, 000
1 1 1
2 2 2
m
m
m
m
d
PV
m m m
(
(
(
(
= = = (
(
(

(

(


Th e PV s h ou ld be t h e s a me:
2
0.085256 0.085715
1 1
2
m m
m m
( (
=
( (

2
0.085256 0.085715
1 1
2 m m
( (
=
( (

Let
1
x
m
=
( ) ( )
2
2 2
1 0.085256 1 0.0428575 0.0428575 2 0.0428575 1 x x x x = = +
( )
2 2
0.085256 0.0428575 2 0.0428575 x x x =
( )
2
0.085256 0.0428575 2 0.0428575 x =
0.25 x =
1
4 m
x
= =
Dis c ount ing Me t hod 3 Dis cou n t
1
3
thly
m
fr om t ime on e t o t ime
zer o. Th e t ot a l # of per iods is 3m. Per per iod dis cou n t fa ct or is
( )
1
3
1
3
m
i
m

(
+
(

.
( )
3
3
1
3
m
m
i
PV
m

(
= +
(

On ce a ga in , we s h ou ld get t h e s a me PV:

( )
3
3
0.085256
1 1
3
m
m
m
i
m m

(
(
+ =
(
(



( )
3
3
0.085256
1 1
3
m
i
m m

(
(
+ =
(
(



( )
3
3
0.085256
1 1
3 4 4
m
i

(
+ =
(



( )
1
3
3
0.085256
1 1
3 4 4
m
i

| |
+ =
|

\ .
78
http://actuary88.com
( ) 3
0.08648788
m
i = ,
( ) 3
1, 000 86.49
m
i ~
Proble m 2
If t h e a n n u a l effect ive r a t e 8% i = , ca lcu la t e
( ) 12
i ,
( ) 4
d , a n d t h e for ce of
in t er es t o .
Solut ion
Well u s e t h e a ccu mu la t e $1 met h od. As s u me we h a ve $1 a t 0 t = .
Wh a t s t h e FV of t h is $1?

If we a ccu mu la t e $1 u s in g 8% i = , t h en 1 FV i = + .
If we a ccu mu la t e $1 u s in g
( ) 12
i , t h en
( )
12
12
1
12
i
FV
(
= +
(

If we a ccu mu la t e $1 u s in g
( ) 4
d , t h en
( )
4
4
1
4
d
FV

(
=
(

If we a ccu mu la t e $1 u s in g o , t h en FV e
o
=
We s h ou ld a ccu mu la t e t h e s a me a mou n t of wea lt h .

( ) ( )
( )
12
12 12
1
12
1 1 , 1 1
12 12
i i
i i
(
+ = + + = +
(

( )
( )
1
1
12
12
12
12 1 1 12 1.08 1 7.7208% i i
(
(
= + = =
(
(


( ) ( )
( )
4
4 4
1
4
1 1 , 1 1
4 4
d d
i i

(
= + = +
(

( )
( )
( )
( )
4 1
1 1
4
4
4 4
1 1 , 4 1 1 4 1 1.08 7.6225%
4
d
i d i


(
(
= + = + = =
(
(



( ) 1 , ln 1 ln1.08 7.696% e i i
o
o = + = + = =
79
http://actuary88.com
Proble m 3 Pr ove t h e followin g equ a t ion

( ) ( )
1 1 1
m m
m d i
= +
Solut ion
As s u me we h a ve $1 a t 0 t = a n d we wa n t t o a ccu mu la t e t h is $1 t o
1
t
m
= .
Accu mu la t e $1 fr om 0 t = t o
1
t
m
= u s in g
( ) m
d : FV=
( )
1
1
m
d
m

(

Accu mu la t e $1 fr om 0 t = t o
1
t
m
= u s in g
( ) m
i : FV=
( )
1
m
i
m
+
( ) ( ) ( )
( ) ( )
1
1
1 1 , 1
1
m m m
m m
d i d m
m m m i m i
m

(
= + = =
(
+

+
( )
( )
( )
( )
1
m m
m m
d m i
m m i m i
= =
+ +

( )
( )
( ) ( )
1
m
m m m
m m i m
d i i
+
= = +
( ) ( )
1 1 1
m m
m d i
= +
Proble m 4 Pr ove
( ) ( )
lim lim
m m
m m
d i o

= =
Solut ion
Ba s ed on Pr oblem 3, we h a ve:

( ) ( ) ( ) ( )
1 1 1 1 1 1
lim lim lim lim
lim
m m m m
m m m m
m
m d i i i o

= + = = =
( ) ( )
lim lim
m m
m m
d i o

= =
80
http://actuary88.com
Proble m 5

Lis t t h e r ela t ion s h ip a mon g o , i , v , d .
Solut ion

o i v d
o = o
( ) ln 1 i +
ln v
( ) ln 1 d
i =
1 e
o

i 1
1
v

1
d
d
v =
e
o
1
1 i +
v 1 d
d =
1 e
o

1
i
i +
1 v d
No n eed t o memor ize t h e a bove t a ble. However , ma ke s u r e you ca n der ive
t h e t a ble.

81
http://actuary88.com
PV of a s t re am of c as h flows

Time t 0 1 k n
Cash flow ( ) 0 CF ( ) 1 CF ( ) CF k ( ) CF n
( )
( ) ( )
( )
( )
( )
2
1 2
0 ...
1
1 1
n
CF CF CF n
PV CF
i
i i
= + + + +
+
+ +

Ne t Pre s e nt Value
h t t p:/ / www.you t u be.com/ wa t ch ?v=IH1Uh 2_XFbM
h t t p:/ / www.you t u be.com/ wa t ch ?v=PCr Bvh TJ iAw
NPV is ju s t t h e PV of fu t u r e ca s h flows min u s t h e in it ia l cos t a t t ime zer o.

To ma ke mon ey in t h e fu t u r e, oft en we h a ve t o s pen d mon ey a t t ime zer o
(t o bu y ma ch in es or bu ild a fa ct or y for exa mple). Th en t o ca lcu la t e ou r
t ot a l wea lt h a t t ime zer o, we n eed t o s u bt r a ct , fr om t h e PV of fu t u r e ca s h
flows , ou r in it ia l cos t a t t ime zer o. Th e r es u lt is ca lled NPV.

NPV = - Cos t + PV

Example .
If you in ves t $10 t oda y, you ll get $6 a t t h e en d of Yea r 1 a n d $8 a t t h e
en d of Yea r 2. Th e in t er es t r a t e is 12%. Wh a t s you r NPV?

2
6 8
10 1.7347
1.12 1.12
PV = + + =
Int e rnal rat e of re t urn (IRR)

h t t p:/ / www.you t u be.com/ wa t ch ?v=B89vwIt BFfk
IRR is t h e in t er es t r at e s u ch t h a t t h e n et pr es en t va lu e of fu t u r e ca s h
flows is equ a l t o zer o. To fin d t h e IRR, we n eed t o s olve t h e followin g
equ a t ion :

82
http://actuary88.com
( )
( ) ( )
( )
( )
( )
2
1 2
0 0 ...
1
1 1
n
CF CF CF n
CF
i
i i
= + + + +
+
+ +

You ca n u s e BA II Plu s / BA II Plu s Pr ofes s ion a l Ca s h Flow Wor ks h eet t o
ca lcu la t e IRR.

Example . Ca lcu la t e IRR of t h e followin g ca s h flows :

Time t (yea r ) 0 1 2 3 4 5
Ca s h flow -100 24 35 20 16 13
Solut ion
En t er t h e followin g in t o BA II Plu s / BA II Plu s Pr ofes s ion a l Ca s h Flow
Wor ks h eet :
CF0 C01 C02 C03 C04 C05
Ca s h flow
-$100 24 35 20 16 13
F01 F02 F03 F04 F05
Fr equ en cy
1 1 1 1 1
Pr es s IRR CPT. You s h ou ld get : IRR=3.01%.

Mult iple IRRs . Th e n u mber of pos it ive r oot s in

( )
( ) ( )
( )
( )
( )
2
1 2
0 0 ...
1
1 1
n
CF CF CF n
CF
i
i i
= + + + +
+
+ +

is equ a l t o t h e n u mber of s ign ch a n ges . For exa mple, if ( ) 0 CF is n ega t ive
a n d ( ) 1 CF , ( ) 2 CF ,, ( ) CF n a r e a ll pos it ive, t h en t h er es on ly on e s ign
ch a n ge. As a r es u lt , t h er e is on ly on e pos it ive r oot in polyn omia l
equ a t ion ; t h er e is a u n iqu e IRR.

If ( ) 0 CF is n ega t ive, ( ) 1 CF is pos it ive, ( ) 2 CF is n ega t ive, a n d a ll t h e
r ema in in g ca s h flows a r e pos it ive, t h en we h a ve t wo s ign ch a n ges . As a
r es u lt , t h er e a r e t wo IRRs .

Plea s e n ot e t h a t BA II Plu s / BA II Plu s Pr ofes s ion a l Ca s h Flow Wor ks h eet
gen er a t es on ly on e IRR va lu e.

83
http://actuary88.com
Example . Ca lcu la t e IRR of t h e followin g ca s h flows :

Time t (yea r ) 0 1 2
Ca s h flow - $100 230 -132
Solut ion
We n eed t o s olve t h e followin g equ a t ion :
( )
2
230 132
0 100
1
1
r
r
= +
+
+
( )
1 2 2
1.32 2.3 1 1 1 1
1 0 0 10%, 20%
1 1 1.1 1 1.2
1
r r
r r r
r
| || |
+ = = = =
| |
+ + +
\ .\ . +
Mu lt iple IRRs a r e ha r d t o expla in . Th is is on e of t h e dr a wba cks of u s in g
IRR t o det er min e pr ofit a bilit y.

As s e t and i t s pric e
As s et = a s t r ea m of ca s h flows
Th e pr ice of a n a s s et = PV of fu t u r e ca s h flows

Time t 0 1 k n
Asset ( ) 0 CF ( ) 1 CF ( ) CF k ( ) CF n
( )
( ) ( )
( )
( )
( )
2
1 2
0 ...
1
1 1
n
CF CF CF n
PV CF
i
i i
= + + + +
+
+ +

Th e pr ice of a n a s s et = PV of fu t u r e ca s h flows
Th is is a cr it ica l con cept for Exa m FM. Th is con cept is u s ed in pr icin g
bon ds a n d s t ocks .

84
http://actuary88.com
Conve rt a c as h flow from one point of t ime t o
anot he r point of t ime

Time t 0 1
1
t
2
t
Cash flow $ A ???
As s u me t h a t t h e in t er es t r a t e is i . Th e dis cou n t r a t e is ( )
1
1 1 1 d v i

= = + .
We h a ve a ca s h flow of $A a t
1
t a n d we wa n t t o con ver t it t o a ca s h flow
a t
2
t , wh er e
2 1
t t > .
( ) ( )
( )
1 2
2 1 2 1
$ @ $ 1 1 @
t t t t
A t A i A d t

+ = (a ccu mu la t in g)
Alt er n a t ively, we h a ve a ca s h flow $B a t
2
t a n d we wa n t t o con ver t it t o a
ca s h flow a t
1
t wh er e
2 1
t t > .
Time t 0 1
1
t
2
t
Cash flow ??? $ B
( )
2 1
2 1 2 1
$ @ $ 1 @
t t t t
B t B v B d t

= (dis cou n t in g)
Example .

As s u me t h e dis cou n t r a t e 10% d = .
Time t 0 1 2 3 4 5 6 7
Cash flow $X $10 $Y
( ) ( )
3 1 2
$10 @ 3 $10 1 10 1 10% 8.1 @ 1 t d t

= = = =
( )
( )
( )
7 3 4
$10 @ 3 $10 1 10 1 10% 15.24157903 @ 7 t d t

= = = =
85
http://actuary88.com
Alt er n a t ive ca lcu la t ion :
1 1 10% 0.9 v d = = = ,
1 1
1 0.9 i v

+ = =
( )
3 1 2 2
$10 @ 3 $10 10 10 0.9 8.1 @ 1 t v v t

= = = = =
( ) ( )
1
4
7 3
$10 @ 3 $10 1 10 0.9 15.24157903 @ 7 t i t

= + = = =
Collaps e mult i ple c as h flows int o a s i ngle c as h flow
Ma n y t imes we n eed t o colla ps e (i.e. con s olida t e) mu lt iple ca s h flows
occu r r in g a t differ en t t imes in t o a s in gle ca s h flow occu r r in g a t a
common poin t of t ime. To colla ps e a s t r ea m of ca s h flows in t o a s in gle
ca s h flow, we ca n NOT s imply a dd u p mu lt iple ca s h flows (u n les s t h e
in t er es t r a t e is zer o).

Pr ocedu r es t o colla ps e mu lt iple ca s h flows in t o a s in gle ca s h flow:

Ch oos e a common poin t of t ime.
Con ver t , eit h er by dis cou n t in g or by a ccu mu la t in g, ea ch ca s h flow
t o a n equ iva len t ca s h flow occu r r in g a t t h is common poin t of t ime.
Add u p t h e con ver t ed va lu es of a ll ca s h flows . Th is is t h e s in gle
ca s h flow in t o wh ich mu lt iple ca s h flows colla ps e.
Example .

Time t 0 1 2 3 4 5 6 7

Cash flow $1 $1
As s u me t h e in t er es t r a t e is 10%.

We h a ve 2 ca s h flows : $1 a t t =2 a n d $1 a t t =6. We wa n t t o colla ps e t h es e
2 ca s h flows in t o on e ca s h flow, per h a ps beca u s e we wa n t t o ca lcu la t e
t h e t ot a l va lu e of t h es e 2 ca s h flows .

(1 ) If we c hoos e t =0 as t he c ommon t ime .

2
$1 @ 2 $1.1 0.82644628 @ 0 t t

= = =
6
$1 @ 6 $1.1 0.56447393 @ 0 t t

= = =
Th e t ot a l va lu e of t h e 2 ca s h flows @ 0 t = :
$0.82644628 + $0.56447393 = $1.39092021
86
http://actuary88.com
(2 ) If we c hoos e t =2 as t he c ommon t ime .

$1 @ 2 $1 @ 2 t t = =
4
$1 @ 6 $1.1 0.68301346 @ 2 t t

= = =
Th e t ot a l va lu e of t h e 2 ca s h flows @ 2 t = :
$1 + $0.68301346 = $1.68301346

(3 ) If we c hoos e t =4 as t he c ommon t ime .

2
$1 @ 2 $1.1 1.21 @ 0 t t = = =
2
$1 @ 6 $1.1 0.82644628 @ 4 t t

= = =
Th e t ot a l va lu e of t h e 2 ca s h flows @ 4 t = :
$1.21 + $0.82644628 = $2.03644628

(4 ) If we c hoos e t =6 as t he c ommon t ime .

4
$1 @ 2 $1.1 1.4641 @ 6 t t = = =
$1 @ 6 $1 @ 6 t t = =
Th e t ot a l va lu e of t h e 2 ca s h flows @ 6 t = :
$1.4641 + $1 = $2.4641

(5 ) If we c hoos e t =7 as t he c ommon t ime .

5
$1 @ 2 $1.1 1.61050 @ 7 t t = = =
$1 @ 6 $1.1 @ 7 t t = =
Th e t ot a l va lu e of t h e 2 ca s h flows @ 7 t = :
$1.61050 + $1.1 = $2.71051

So fa r , we h a ve colla ps ed t h e t wo ca s h flows in t o five s in gle ca s h flows
occu r r in g a t t =0, 2, 4, 6, a n d 7 r es pect ively. Sin ce t h es e five s in gle ca s h
flows ea ch r epr es en t t h e t ot a l va lu e of t h e iden t ica l ca s h flows of $1 @
t =2 a n d $1 @ t =6, t h ey s h ou ld con ver t t o ea ch ot h er followin g t h e
s t a n da r d con ver s ion r u le.

87
http://actuary88.com
Time t 0 1 2 3 4 5 6 7
Cash flow
$1.39092021
$1.68301346
$2.03644628 $2.4641 $2.71051
For exa mple:

( )
2
@ 0 $1.39092021 1. $1.39092021 1.6830134 1 2 5 @ t t = = =
We did n ot get $1.68301346 @ 2 t = du e t o r ou n din g.
( )
3
@ 4 $2.03644628 1.1 $2.03644628 2.7105 @ 7 1 t t = = = (OK)

88
http://actuary88.com
Annui t y c ollaps i ng n paralle l c as h flows i nt o a
s ingle c as h flow

n pa r a llel even ly-s pa ced ca s h flows of $1 ea ch

$1 $1 $1 $1 $1 $1
i i i i
1 2 3 4 1 n n
cash flows n

i n
a
i n
a
i n
s
i n
s
The total value of n parallel cash flows of $1 each at one step to the left of the 1
st

cash flow is
1
n
i n
v
i
a

= . In other words, we can collapse n parallel cash flows
of $1 each into a single cash flow
i n
a at one step to the left of the 1
st
cash flow.
The total value of n parallel cash flows of $1 each at the 1
st
cash flow time is
1
n
i n
v
d
a

= . We can collapse n parallel cash flows of $1 each into a single cash
flow
i n
a at the 1
st
cash flow time.
The total value of n parallel cash flows of $1 each at the final cash flow time is
( ) 1 1
n
i n
i
s
i
+
= . We can collapse n parallel cash flows of $1 each into a single
cash flow
i n
s at the final cash flow time.
The total value of n parallel cash flows of $1 each at one step to right of the
final cash flow is
( ) 1 1
i
n
n
i
s
d
+
= . We can collapse n parallel cash flows of $1
each into a single cash flow
i n
s at one step to the right of the final cash flow.
89
http://actuary88.com
Plea s e n ot e t h a t i is t h e effect ive in t er es t r a t e bet ween t wo con s ecu t ive
ca s h flows . For exa mple, if t wo con s ecu t ive ca s h flows a r e 3 mon t h s
a pa r t , t h en i is t h e effect ive in t er es t r a t e for t h e 3 mon t h per iod (i.e.. 3
mon t h s = 1 u n it t ime); if t wo con s ecu t ive ca s h flows a r e 3 yea r s a pa r t ,
t h en i is t h e effect ive in t er es t r a t e for t h e 3 yea r per iod (i.e.. 3 yea r s = 1
u n it t ime).

Sin ce t h e fou r s in gle ca s h flows
i n
a ,
i n
a ,
i n
s , a n d
i n
s ea ch r epr es en t t h e
t ot a l va lu e of t h e iden t ica l n pa r a llel ca s h flows , t h ey s h ou ld con ver t t o
ea ch ot h er followin g t h e s t a n da r d con ver s ion r u le. So we h a ve:

i i n n
v a a = , ( ) 1
i i n n
i a a = +
i i n n
s v s = , ( ) 1
i i n n
s s i = +
n
i i n n
v s a = , ( ) 1
n
i i n n
s i a = +
i
n
i n n
v s a = , ( ) 1
i
n
i n n
s i a = +
1
i
n
i n n
v s a
+
= , ( )
1
1
i
n
i n n
s i a
+
= +
1 n
i i n n
v s a

= , ( )
1
1
n
i i n n
s i a

= +
Avoid t he c ommon pi t fall

Qu es t ion :
Wh a t s t h e differ en ce bet ween a n n u it y du e a n d a n n u it y immedia t e?

An s wer :
In a n n u it y du e, t h e 1
s t
pa ymen t is a t 0 t = ;
in a n n u it y immedia t e, t h e 1
s t
pa ymen t is a t 1 t = .
If you a gr ee wit h t h is a n s wer , you a r e wr on g!

Th is is a common mis t a ke ma de by ma n y. Th is common mis t a ke
or igin a t es fr om t h e fa ct t h a t wh en t ext books der ive t h e for mu la for
a n n u it y du e a n d a n n u it y immedia t e, t h ey a lwa ys dr a w t h e followin g
dia gr a m (ma kin g t h e a n n u it y du e h a ve t h e 1
s t
pa ymen t a t 0 t = a n d t h e
a n n u it y immedia t e h a ve t h e 1
s t
pa ymen t a t 1 t = ):

90
http://actuary88.com
Common diagram for annuity duepotentially misleading
Payment $1 $1 $1 $1
Time 0 t = 1 t = 2 t = . 1 t n = t n =
..
2 1
1 1
1 ...
1
n n
n
n i
v v
a v v v
v d


= + + + + = =

The truth, however, is that in an annuity due, the 1


st
payment can start at any time. For
example, the 1
st
payment can be at 2 t = :
Payment $1 $1 $1 $1
Time 0 t = 1 t = 2 t = 3 t = t n = 1 t n = + 2 t n = +
..
2 1
1 1
1 ...
1
n n
n
n i
v v
a v v v
v d


= + + + + = =

To avoid the faulty thinking that in an annuity due the 1


st
payment is at 0 t = , you should
use the following diagram(i.e. dont include the time in your diagram):
Payment $1 $1 $1 $1
1
st
2nd 3
rd
n-th payment
..
2 1
1 1
1 ...
1
n n
n
n i
v v
a v v v
v d


= + + + + = =

Rule to remember: If you discount n evenly spaced cash flows back to the 1
st
cash
flow time, youll have an annuity due, regardless of when the 1
st
payment is made.

91
http://actuary88.com
Common diagram for annuity immediatepotentially misleading

Payment $1 $1 $1 $1
Time 0 t = 1 t = 2 t = . 1 t n = t n =
1
2 1
1 1 1
...
1 1
1
1
n n n n
n n
n i
v v v v v
a v v v v
v
v i
v v
+


= + + + + = = = =


The truth, however, is that in an annuity immediate, the 1
st
payment can start at any time.
For example, the 1
st
payment can be at 2 t = :
Payment $1 $1 $1 $1
Time 0 t = 1 t = 2 t = 3 t = t n = 1 t n = +
1
2 1
1 1 1
...
1 1
1
1
n n n n
n n
n i
v v v v v
a v v v v
v
v i
v v
+


= + + + + = = = =


To avoid the faulty thinking that in an annuity immediate the 1
st
payment is at 1 t = , you
should use the following diagram(i.e. dont include the time in your diagram):
Payment $1 $1 $1 $1
Time 1
st
2nd 3
rd
n-th payment
1
2 1
1 1 1
...
1 1
1
1
n n n n
n n
n i
v v v v v
a v v v v
v
v i
v v
+


= + + + + = = = =


Rule to remember: If you discount n evenly spaced cash flows back to one interval
prior to 1
st
cash flow time, youll have an annuity immediate, regardless of when the
1
st
payment is made.
92
http://actuary88.com
Similarly, when draw the diagram for
n i
s and
..
n i s , dont include the timeline:
Payment $1 $1 $1 $1
1
st
2nd 3
rd
n-th
( )
..
1 1
n
n i
i
s
d
+
=
( ) 1 1
n
n i
i
s
i
+
=
If you accumulate n evenly spaced cash flows the final cash flow time, use
n i
s ,
regardless of when the final payment is made. If you accumulate n evenly spaced
cash flows the final cash flow time plus one interval, use
..
n i s , regardless of when the
final payment is made.

Summary: Just memorize the following diagram and youll do fine.
Payment $1 $1 $1 $1
1
st
2nd 3
rd
n-th
n i
a
..
n i a
n i
s
..
n i s
One step before the first cash flow time, use
n i
a ;
at the 1
st
cash flow time, use
..
n i a ;
at the final cash flow time, use
n i
s ;
one step after the final cash flow time, use
..
n i s
93
http://actuary88.com
Example 1 .

You a r e given t h e followin g ca s h flows :

Time t 0 1 2 3 4 5 6 7 8
Cash flow $5 $5 $5 $5 $5 $5 $5
Th e in t er es t r a t e is 10%.

Ca lcu la t e, a t t =4, t h e t ot a l va lu e of t h es e ca s h flows .

Solut ion

We n eed t o colla ps e t h e ca s h flows in t o a s in gle ca s h flow @ t =4. Th er e
a r e ma n y wa ys t o do s o.

Met h od 1
Fir s t , we s plit t h e or igin a l ca s h flows in t o t wo s t r ea ms of ca s h flows . Th e
fir s t s t r ea m con s is t s of ca s h flows a t t =2, 3, a n d 4. Th e s econ d s t r ea m
con s is t s of ca s h flows a t t =5, 6, 7, a n d 8. If we a dd u p t h es e t wo s t r ea ms ,
we s h ou ld get t h e or igin a l ca s h flows .

Next , we colla ps e ea ch of t h e t wo s t r ea ms in t o a s in gle ca s h flow a t t =4.
For t h e 1
s t
s t r ea m of ca s h flows , we n eed t o colla ps e 3 pa r a llel ca s h flows
in t o a s in gle ca s h flow a t t h e fin a l pa ymen t t ime. Th is gives u s a n
equ iva len t s in gle ca s h flow a t t =4 of
3 10%
5 s .
For t h e 2
n d
s t r ea m of ca s h flows , we n eed t o colla ps e 4 pa r a llel ca s h flows
in t o a s in gle ca s h flow a t on e s t ep t o t h e left of t h e 1
s t
ca s h flow. As t h e
r es u lt , t h e equ iva len t s in gle ca s h flow a t t =4 is
4 10%
5a .
Time t 0 1 2 3 4
Cash flow $5 $5 $5
3 10%
5 s
94
http://actuary88.com
Time t 4 5 6 7 8
Cash flow $5 $5 $5 $5
4 10%
5a
Con s equ en t ly, t h e t ot a l va lu e of t h e or igin a l ca s h flows @ t =4 is :

3 10% 4 10%
3 4
1.1 1 1 1.1
5 5 5 5 32.40
0.1 0.1
s a


+ = + ~
Met h od 2
We s plit t h e or igin a l ca s h flows in t o t wo s t r ea ms of ca s h flows . Th e 1
s t

s t r ea m con s is t s of ca s h flows a t t =2 a n d 3. Th e 2
n d
s t r ea m con s is t s of
ca s h flows a t t =4, 5, 6, 7, a n d 8.

Next , we colla ps e ea ch of t h e t wo s t r ea ms of ca s h flows in t o a s in gle ca s h
flow a t t =4. For t h e 1
s t
s t r ea m, we n eed t o colla ps e 2 pa r a llel ca s h flows
t o a s in gle ca s h flow on e s t ep t o t h e r igh t of t h e fin a l ca s h flow. Th is gives
u s a n equ iva len t ca s h flow of
2 10%
5s a t t =4.
For t h e 2
n d
s t r ea m, we n eed t o colla ps e five pa r a llel ca s h flows a t t h e fir s t
ca s h flow t ime. Th is gives u s a n equ iva len t ca s h flow of
5 10
5 a a t t =4.

Time t 0 1 2 3 4
Cash flow $5 $5
2 10%
5s
Time t 4 5 6 7 8
Cash flow $5 $5 $5 $5 $5
5 10
5 a
Fin a lly, we a dd u p t h es e t wo equ iva len t ca s h flows a t t =4.

2 5
1 1 2 10% 5 10
1.1 1 1 1.1
5 5 5 5 32.40
1 1.1 1 1.1
s a



+ = + ~


95
http://actuary88.com
Met h od 3
Th is t ime, we don t s plit t h e or igin a l ca s h flows .

Fir s t , we colla ps e t h e or igin a l ca s h flows t o t =1. Sin ce 7 pa r a llel ca s h
flows a r e colla ps ed in t o a s in gle ca s h flow on e s t ep t o t h e left of t h e 1
s t

ca s h flow, t h e equ iva len t s in gle ca s h flow is
7 10%
5a .
Time t 0 1 2 3 4 5 6 7 8
Cash flow $5 $5 $5 $5 $5 $5 $5
7 10%
5a
Next , well con ver t t h e s in gle ca s h flow of
7 10%
5a @t =1 t o a ca s h flow @
t =4.

( ) ( )
4 1 3
7 10% 7 10% 7 10%
$5 @ 1 $5 1.1 5 1.1 @ 4 a t a a t

= = =
Fin a lly, t h e t ot a l va lu e of t h e or igin a l ca s h flow @ 4 t = :
( ) ( )
3 3
7 10%
7
1 1.1
5 1.1 5 1.1 32.40
0.1
a

= ~
Met h od 4
Fir s t , we colla ps e t h e or igin a l ca s h flows t o t =2. Sin ce 7 pa r a llel ca s h
flows a r e colla ps ed in t o a s in gle ca s h flow a t t h e 1
s t
ca s h flow t ime, t h e
equ iva len t s in gle ca s h flow is
7 10
5 a .
Time t 0 1 2 3 4 5 6 7 8
Cash flow $5 $5 $5 $5 $5 $5 $5
7 10
5 a
Next , well con ver t t h e s in gle ca s h flow of
7 10
5 a @t =2 t o a ca s h flow @
t =4.

( ) ( )
4 2 2
7 10 7 10 7 10
$5 @ 2 $5 1.1 5 1.1 @ 4 a t a a t

= = =
96
http://actuary88.com
Fin a lly, t h e t ot a l va lu e of t h e or igin a l ca s h flow @ 4 t = :
( ) ( )
2 2
1 7 10
7
1 1.1
5 1.1 5 1.1 32.40
1 1.1
a

= ~


Met h od 5
Fir s t , we colla ps e t h e or igin a l ca s h flows t o t =8. Sin ce 7 pa r a llel ca s h
flows a r e colla ps ed in t o a s in gle ca s h flow a t t h e fin a l ca s h flow t ime, t h e
equ iva len t s in gle ca s h flow is
7 10%
5 s .
Time t 0 1 2 3 4 5 6 7 8
Cash flow $5 $5 $5 $5 $5 $5 $5
7 10%
5 s
Next , well con ver t t h e s in gle ca s h flow of
7 10%
5 s @t =8 t o a ca s h flow @
t =4.
( ) ( )
4 8 4
7 10% 7 10% 7 10%
$5 @ 8 $5 1.1 5 1.1 @ 4 s t s s t

= = =
Fin a lly, t h e t ot a l va lu e of t h e or igin a l ca s h flow @ 4 t = :
( ) ( )
7
4
7 10%
4
1.1 1
5 1.1 5 1.1 32.40
0.1
s

= ~
Met h od 6
Fir s t , we colla ps e t h e or igin a l ca s h flows t o t =9. Sin ce 7 pa r a llel ca s h
flows a r e colla ps ed in t o a s in gle ca s h flow a t on e s t ep a ft er t h e fin a l ca s h
flow t ime, t h e equ iva len t s in gle ca s h flow is
7 10%
5 s .
Time t 0 1 2 3 4 5 6 7 8 9
Cash flow $5 $5 $5 $5 $5 $5 $5
7 10%
5 s
Next , well con ver t t h e s in gle ca s h flow of
7 10%
5 s @t =9 t o a ca s h flow @
t =4.
97
http://actuary88.com
( ) ( )
4 9 5
7 10% 7 10% 7 10%
$5 @ 9 $5 1.1 5 1.1 @ 4 s t s s t

= = =
Fin a lly, t h e t ot a l va lu e of t h e or igin a l ca s h flow @ 4 t = :
( ) ( )
5 5
1 7 10%
7
1.1 1
5 1.1 5 1.1 32.40
1 1.1
s

= ~


Th e a bove 6 met h ods ma y s eem a n over kill, bu t t h ey a r e good exer cis es
on h ow t o colla ps e a n iden t ica l ca s h flow s t r ea m in va r iou s wa ys .

Example 2
Expla in wh y
( )( )
( )
2 3
4 4
1
i
n n n
n n i n i j
a v v v a a a = + + + = , wh er e n is a pos it ive
in t eger a n d ( ) 1 1
n
j i = + .
Solut ion

We ca n colla ps e 4n pa r a llel ca s h flows of $1 in t o
4n i
a :

4 cash flows n

4n i
a
98
http://actuary88.com
Next , we s plit t h e 4n pa r a llel ca s h flows in t o 4 s et s of n pa r a llel ca s h
flows . We colla ps e ea ch s et of n pa r a llel ca s h flows in t o a s in gle ca s h flow
of
n i
a , a r r ivin g a t 4 s et s of pa r a llel ca s h flows of
n i
a ea ch . Fin a lly, we
colla ps e t h e 4 pa r a llel ca s h flows of
n i
a ea ch in t o a s in gle ca s h flow of
( )( )
4 n i j
a a .

cash flows n

cash flows n

cash flows n

cash flows n

n i
a
n i
a
n i
a
n i
a
( )( )
4 n i j
a a
We u s e a n a n n u it y fa ct or of
4 j
a . j r epr es en t s t h e in t er es t r a t e bet ween
t wo con s ecu t ive ca s h flows of
n i
a . Amon g t h e fou r pa r a llel ca s h flows of
n i
a , t wo con s ecu t ive ca s h flows a r e n t ime a pa r t . Con s equ en t ly,
( ) 1 1
n
j i = + .
Beca u s e 4n pa r a llel ca s h flows ca n be colla ps ed in t o t wo s in gle ca s h
flows
4n i
a a n d
( )( )
4 n i j
a a t h a t occu r a t t h e s a me t ime, it follows t h a t
( )( )
4 4 n i n i j
a a a = .
Beca u s e ( ) ( ) ( )
1 2 3
2 3
4
1 1 1 1 1
j
n n n
a j j j v v v

= + + + + + + = + + + , we h a ve:
( )( )
( )
2 3
4 4
1
n i
n n n
n i n i k
a v v v a a a = + + + =
Ma ke s u r e you u n der s t a n d t h e a bove logic. Th is s h a r pen s you r s kill t o
qu ickly colla ps e a complex s t r ea m of ca s h flows in t o a s in gle ca s h flow.
99
http://actuary88.com
We ca n a ls o pr ove
( )( )
4 4 n i n i j
a a a = u s in g t h e s t a n da r d a n n u it y for mu la .

4
4
1
n
n i
v
d
a

= ,
1
n
n i
v
d
a

=
( )
( )
( )
( )
2 3
1
4 4
4
4
1 1 1 1
1
1
1
1 1 1 1
n
n
n
n n n
n
j
j i
v
v v v
v
j i
a


+ +

= = = = + + +

+ +

( ) ( )
4 4
4
1 1 1
1
n n n
n n i j
v v v
d v d
a a

= =

( )( )
( )
2 3
4 4
1
n i
n n n
n i n i j
a v v v a a a = + + + =
Gen er a lly, for a n y pos it ive in t eger k :
( )( )
( )
1
2
1 ...
k n
n n
n i n i
k n i k j
a a a a v v v

(
(

= = + + + +
( )( )
( )
1
2
1 ...
k n
n n
n i n i
k n i k j
a a a a v v v

(
(

= = + + + +
( )( )
( ) ( ) ( )
( )
2 1
1 1 1 1 ...
n n k n
n i n i
k n i k j
s s s s i i i

(
(

= = + + + + + + +
( )( )
( ) ( ) ( )
( )
2 1
1 1 1 1 ...
n n k n
n i n i
k n i k j
s s s s i i i

(
(

= = + + + + + + +
wh er e ( ) 1 1
n
j i = +
You don t n eed t o memor ize t h e a bove for mu la . J u s t ma ke s u r e t h a t you
ca n der ive t h e a bove for mu la s by colla ps in g ca s h flows .

100
http://actuary88.com
Example 3 (SOA May 2 0 0 2 EA-1 #1 1 )
S1 = Th e a ccu mu la t ed va lu e a s of 12/ 31/ 2002 of $500 in ves t ed a t t h e
en d of ea ch mon t h du r in g 2002 a t a n omin a l in t er es t r a t e of 8%
per yea r , con ver t ible qu a r t er ly.

A1 = Th e pr es en t va lu e a s of 1/ 1/ 2002 of S1, a t a n omin a l dis cou n t r a t e
of 6% per yea r , con ver t ible s emia n n u a lly.

S2 = Th e a ccu mu la t ed va lu e a s of 12/ 31/ 2002 of $1,500 in ves t ed a t t h e
en d of ea ch qu a r t er du r in g 2002 a t a n omin a l dis cou n t r a t e of 6%
per yea r , con ver t ible mon t h ly.

A2 = Th e pr es en t va lu e a s of 1/ 1/ 2002 of S2, a t a n omin a l in t er es t r a t e
of P% per yea r , con ver t ible on ce ever y t wo yea r s .

In wh a t r a n ge is P% s u ch t h a t A1 = A2?
[A] Les s t h a n 4.60%
[B] 4.60% bu t les s t h a n 4.70%
[C] 4.70% bu t les s t h a n 4.80%
[D] 4.80% bu t les s t h a n 4.90%
[E] 4.90% or mor e

Solut ion
To s implify t h e ca lcu la t ion , let s s et $500 a s on e u n it of mon ey.

Date 1/1/2002 12/31/2002
Time t (months) 0 1 2 11 12
Payments $1 1 1 1
1
12 i
S s =
Th e qu a r t er ly effect ive in t er es t r a t e is
( ) 4
/ 4 i =8%/ 4=2%
Remember 1 yea r = 4 qu a r t er s

Th e mon t h ly effect ive in t er es t i is ca lcu la t ed a s follows :
( )
3
1 i + =1+2% (r emember t h a t 1 qu a r t er = 3 mon t h s )
( )
1
3
1 2% 1 0.66227096% i = + =
1
12
12.44689341
i
S s = =
101
http://actuary88.com
Beca u s e t h e n omin a l dis cou n t r a t e is 6% per yea r con ver t ible
s emia n n u a lly, t h e dis cou n t r a t e du r in g a 6-mon t h per iod is
( ) 2
6%
1 1 1 3% 0.97
2 2
d
= = =
We ca n fin d t h e a n n u a l dis cou n t r a t e d :
( )
2
2
2
1 1 0.97
2
d
d
(
= =
(
(


Her e we u s e t h e for mu la :
( )
1 1
m
m
d
d
m
(
=
(
(

Let v r epr es en t t h e a n n u a l dis cou n t in g fa ct or for A1. Th en
( ) 1 1 A S v = . However , 1 v d = . So we h a ve:
( ) ( ) ( )
2
6%
1 1 1 1 12.44689341 1 11.7112820
2
A S v S d
(
= = = =
(


Next , we ca lcu la t e 2 S .
Date 1/1/2002 12/31/2002
Time t (quarters) 0 1 2 3 4
Payments $3 3 3 3
3 units of money=$1,500

2
4
3
j
S s =
Th e n omin a l dis cou n t r a t e is 6% per yea r con ver t ible mon t h ly. We ca n
fin d t h e a n n u a l r a t e of dis cou n t d a n d t h e a n n u a l dis cou n t in g fa ct or v .
( )
12
12
1
1 1
1 12
d
v d
i
(
= = =
(
+
(

On ce a ga in t h e for mu la is
( )
1 1
m
m
d
d
m
(
=
(
(

On ce we h a ve t h e a n n u a l effect ive r a t e i , we ca n fin d t h e qu a r t er ly
effect ive in t er es t r a t e j .
( )
4
1
1 1 j i v

+ = + = (1 yea r =4 qu a r t er s )

102
http://actuary88.com
( ) ( )
( ) ( )
1/ 4
12 3
12 12
1/ 4 1/ 4
1/ 4
1 1 1 1 1
12 12
d d
j i v d

| |
( (
|
+ = + = = = =
( (
|
( (

\ .

( )
3
3
12
6%
1 1 1 1 1.51512594%
12 12
d
j


(
(
= = =
(
(


2
4
3 12.27548783
j
S s = =
To ca lcu la t e
2
A , we n eed t o kn ow h ow t o dea l wit h a biza r r e n omin a l
r a t e: a n omin a l in t er es t r a t e of P% per yea r , con ver t ible on ce ever y t wo
yea r s . Let s s t a r t wit h s omet h in g s imple. If we h a ve a n omin a l in t er es t
r a t e of % P per yea r con ver t ible on ce ever y 6 mon t h s , t h en t h e 6-mon t h -
per iod in t er es t r a t e is
%
2
P
. Her e t h e den omin a t or 2 is t h e # of 6-mon t h -
per iods per yea r (s o on e yea r = t wo 6-mon t h s ). Simila r ly, if we h a ve a
n omin a l in t er es t r a t e of % P per yea r con ver t ible mon t h ly, t h en t h e
mon t h ly in t er es t r a t e is
%
12
P
. Her e t h e den omin a t or 12 is t h e # of mon t h s
per yea r (s o on e yea r = 12 mon t h s ).

Now let s a pply t h is logic t o a n omin a l in t er es t r a t e con ver t ible on ce ever y
t wo yea r s . On e yea r is 0.5 of 2 yea r s . Th en t h e in t er es t r a t e for a 2-yea r
per iod is
%
2 %
0.5
P
P = . Th en t h e dis cou n t in g fa ct or for a 2-yea r per iod is
( )
1 1
1 2 %
1 2 %
P
P

= +
+
. Th en t h e dis cou n t in g fa ct or for on e yea r is
( ) ( )
1
1
1
2
2
1 2 % 1 2 % P P

(
+ = +

.
A2 = Th e pr es en t va lu e a s of 1/ 1/ 2002 of S2, a t a n omin a l in t er es t r a t e
of P% per yea r , con ver t ible on ce ever y t wo yea r s .

Th en ( ) ( )
1 1
2 2
2 2
1 2 % 12.27548783 1 2 % A S P P

= + = +
1 2
A A = ( )
1
2
11.7112820 12.27548783 1 2 % P

= +
% 4.93367417% P = . So t h e a n s wer is E.

103
http://actuary88.com
Pe rpe t uit y

c ollaps ing +pa r a llel ca s h flows in t o a s in gle ca s h flow

+pa r a llel even ly-s pa ced ca s h flows of $1 ea ch

$1 $1 $1 $1 $1 $1
i i i i
1 2 3 4 +
cash flows +

1
i
i
a
+
=
1
i
d
a
+
=
We ca n colla ps e +pa r a llel ca s h flows of $1 ea ch in t o a s in gle ca s h
flow
1
i
a t on e s t ep t o t h e left of t h e 1
s t
ca s h flow.
We ca n colla ps e +pa r a llel ca s h flows of $1 ea ch in t o a s in gle ca s h
flow
1 1
1
d i
= + a t t h e 1
s t
ca s h flow t ime.
104
http://actuary88.com
Example 1 (SOA May 2 0 0 0 EA-1 #1 )
Pu r ch a s e da t e of a per pet u it y-du e: 1/ 1/ 2000
Level pa ymen t a mou n t : $100
Fr equ en cy of pa ymen t s : An n u a l
Cos t of per pet u it y: $1,100
In t er es t r a t e for per pet u it y: % i , compou n ded a n n u a lly

Immedia t ely following t h e pa ymen t on 1/ 1/ 2014, t h e r ema in in g fu t u r e
pa ymen t s a r e s old a t a yield r a t e of % i . Th e pr oceeds a r e u s ed t o
pu r ch a s e a n a n n u it y cer t a in a s follows :
Ter m of a n n u it y: 10 yea r s
1
s t
pa ymen t of a n n u it y: 1/ 1/ 2018
Fr equ en cy of a n n u it y pa ymen t : s emi-a n n u a l on J a n u a r y 1
a n d J u ly 1
In t er es t r a t e for a n nu it y: %
2
i
compou n ded a n n u a lly

In wh a t r a n ge is t h e s emi-a n n u a l a n n u it y pa ymen t ?
(A) Les s t h a n $75
(B) $75 bu t les s t h a n $77
(C) $77 bu t les s t h a n $79
(D) $79 bu t les s t h a n $81
(E) $81 or mor e
Solut ion

Per pet u a l a n n u it y du e:
Time t (years) 0 1 2 3
Payment $100 $100 $100 $100 $100 $100
100
1,100 PV
d
= = ,
1 1
1
1 % 11
d
i
= =
+
, % 10% i =
Per pet u a l a n n u it y du e s old on 1/ 1/ 2014:
# of payments 1 2 3 4
Time t 1/1/2014 7/1/2014 1/1/2015 7/1/2015
Payment $100 $100 $100 $100
100 100
selling price 1, 000
% 10% i
= = =
105
http://actuary88.com
Th e s a les pr oceeds a r e u s ed t o pu r ch a s e a 10 yea r a n n u it y cer t a in .
Time 1/1/2018 7/1/2018 1/1/2019 7/1/2019 1/1/2027 7/1/2027
# of payments 1 2 3 4 19 20
Payment X X X X X X X
20 j
PV X a =
Wh er e j is t h e s emi-a n n u a l effect ive in t er es t r a t e.

0.5 0.5
% 10%
1 1 1 1 2.47%
2 2
i
j
| | | |
= + = + =
| |
\ . \ .

At 1/ 1/ 2018, t h e a ccu mu la t ed va lu e of t h e s a les pr oceeds s h ou ld be
equ a l t o t h e PV of t h e 10 yea r a n n u it y cer t a in .

4
20
1, 000 1 %
2
j
i
X a
| |
+ =
|
\ .
, ( )
4
20 2.47%
1, 000 1.05 X a = , 75.87 X =
So t h e a n s wer is C.

106
http://actuary88.com
Annui t y payable m-t hly i n advanc e

Time t 0 1 1 n n
0
m
1
m
2
m
...
1 m
m
m
m

0
m
1
m
2
m
...
1 m
m
m
m
1
$
m
1
$
m
1
$
m

1
$
m

1
$
m
1
$
m
1
$
m

1
$
m
1 unit of time

-th unit of time n

units of time n

( )
( ) ( ) ( )
1 1
n n
m
n i n i m m m
v d v d
a a
d
d d d

= = = , wh er e
( )
1 1
m
m
d v
m
d
(
= = (
(

To der ive t h e a bove for mu la , we fir s t colla ps e mca s h flows of


1
$
m
ea ch
t h a t occu r in ea ch u n it of t ime in t o a n equ iva len t s in gle ca s h flow. We
s h ou ld h a ve n equ iva len t s in gle ca s h flows (beca u s e we h a ve a t ot a l of
n u n it s of t ime). Next , we colla ps e t h es e n s in gle ca s h flows in t o on e
s in gle ca s h flow of
( )
n i m
d
a
d
.
107
http://actuary88.com
1
$
m
1
$
m
1
$
m

1
$
m
m payments

1
$
j m
m
a wh er e
( )
1
1 1
m
j i = +
( )
( )
( )
( )
( )
1
1 1
1
1 1 1 1
1 1 1
$
1 1
1 1
1
m
m
m
j m m
j i
d
m m m
j
i
m d
d
d
a


=
| |
|
|
\ .
+ +
= = =
+
+


Time t 0 1 1 n n
0
m
1
m
2
m
...
1 m
m
m
m

0
m
1
m
2
m
...
1 m
m
m
m
1
$
m
1
$
m
1
$
m

1
$
m

1
$
m
1
$
m
1
$
m

1
$
m
1 unit of time

-th unit of time n

$
( ) m
d
d
$
( ) m
d
d
$
( ) m
d
d
cash flows n

( )
n i m
d
a
d

108
http://actuary88.com
Annui t y payable m-t hly i n arre ars

Time t 0 1 1 n n
0
m
1
m
2
m
...
1 m
m
m
m

0
m
1
m
2
m
...
1 m
m
m
m
1
$
m
1
$
m
1
$
m

1
$
m

1
$
m
1
$
m
1
$
m

1
$
m
1 unit of time

-th unit of time n

units of time n

( )
( ) ( ) ( )
1 1
n n
m
n i n i m m m
v i v i
a a
i
i i i

= = = , wh er e
( )
1 1
m
m
i
i
m
(
+ = + (
(


( )
( ) ( )
1
n
m
n i n i m m
v d
a a
d d

= = a n d
( )
( ) ( )
1
n
m
n i n i m m
v i
a a
i i

= = a r e t h e on ly t wo for mu la s
you n eed t o memor ize for a n n u it ies wh er e mu lt iple ca s h flows occu r in a
on e u n it t ime. If a problem a s ks you t o fin d
( ) m
n i
s a n d
( ) m
n i
s , you ca n
ca lcu la t e t h is wa y:

( ) ( )
( )
1
n m m
n i n i
s a i = + ,
( ) ( )
( )
1
n m m
n i n i
s a i = +
If t h e s ymbols a n d t h e r ela t ed for mu la s of
( )
m
n i
a ,
( )
m
n i
a ,
( )
m
n i
s , a n d
( )
m
n i
s look
t oo u gly a n d complex, you ca n a lwa ys u s e t h e ca s h flow fr equ en cy
1
m
a s
t h e u n it t ime, t h u s for cin g t h e ca s h flow fr equ en cy a n d t h e in t er es t
compou n din g fr equ en cy t o be iden t ica l. Th is gr ea t ly s implifies t h e
n u mber of con cept s a n d for mu la s you n eed t o memor ize.
If t h e u n it t ime is of
1
m
lon g, t h en t h e effect ive in t er es t r a t e du r in g t h e
u n it t ime is
( )
1
1 1
m
j i = + .
109
http://actuary88.com
Un der t h is s implifyin g t ech n iqu e, t h e PV of a n a n n u it y pa ya ble m-t h ly in
a dva n ce in ea ch u n it t ime is :

Time t 0 1 1 n n
0
m
1
m
2
m
...
1 m
m
m
m

0
m
1
m
2
m
...
1 m
m
m
m
1
$
m
1
$
m
1
$
m

1
$
m

1
$
m
1
$
m
1
$
m

1
$
m
cash flows mn

1
j mn
m
a
1
j mn
s
m

Time t 0 1 1 n n
0
m
1
m
2
m
...
1 m
m
m
m

0
m
1
m
2
m
...
1 m
m
m
m
1
$
m
1
$
m
1
$
m

1
$
m

1
$
m
1
$
m
1
$
m

1
$
m
cash flows mn

1
j mn
m
a
1
j mn
s
m
110
http://actuary88.com
Example 1

A loa n of $100,000 bor r owed a t 6% a n n u a l effect ive is r epa id by level
mon t h ly pa ymen t s in a dva n ce over t h e n ext 30 yea r . Aft er 10 yea r s , t h e
ou t s t a n din g ba la n ce of t h e loa n is r efin a n ced a t 4% a n n u a l effect ive a n d
is pa id by level mon t h ly pa ymen t s in a dva n ce over 20 yea r s .

Ca lcu la t e:
Th e mon t h ly pa ymen t of t h e or igin a l loa n .
Th e pr in cipa l por t ion a n d t h e in t er es t por t ion of t h e 37
t h
pa ymen t .
Th e mon t h ly pa ymen t of t h e r efin a n ced loa n .
Th e a ccu mu la t ed va lu e of t h e r edu ct ion in mon t h ly pa ymen t s
in ves t ed a t 4% a n n u a l effect ive.

Solut ion

Met h od 1 - u s e a yea r a s t h e compou n din g per iod
Find t he mont hly payme nt of t he original loan

Time t 0 1 30
(Year)
1
12
2
12
3
12
4
12
5
12
6
12
7
12
8
12
9
12
10
12
11
12
12
12
12
X
12
X
12
X
12
X
12
X
12
X
12
X
12
X
12
X
12
X
12
X
1 Year's payments

30 years' payments

( ) 12
30 6%
X a
Let
12
X
r epr es en t t h e mon t h ly pa ymen t of t h e or igin a l loa n .

( ) 12
30 6%
100, 000 X a = ,
( )
( )
30
12
12 30 6%
1 v
d
a

= ,
( )
12
12
1 1
12
d
d
(
=
(

111
http://actuary88.com

( )
( ) ( )
1
1 1
12
12
12 12
12 1 1 12 1 12 1 1.06 5.812767% d d v

(
( (
= = = =
(
( (



( )
( )
30 30
12
12 30 6%
1 1 1.06
14.2082053
5.812767%
v
d
a


= = =
( ) 12
30 6%
100, 000 100, 000
7, 038.186588
14.2082053
X
a
= = =


7, 038.186588
586.52
12 12
X
= =
Find t he princ ipal port ion and t he int e re s t port ion of t he 3 7
t h

payme nt

Th e 37
t h
pa ymen t is t h e 1
s t
pa ymen t in t h e 4
t h
yea r .

Time t 0 1 2 3 4 27 28 29 30
(Year)
27 years' payments

( ) 12
27 6%
X a
Th e # of compou n din g per iod r ema in in g immedia t ely a ft er t h e 37
t h

pa ymen t is 30 3 =27.

( )
( )
( )
27 27
12
12 27 6%
1 1 1.06
7, 038.186588 95, 973.09
5.812767%
v
X X
d
a


= = =
Th e in t er es t por t ion of t h e 37
t h
pa ymen t is :

( )
( )
( )
12
5.812767%
95, 973.09 95, 973.09 464.89
12 12
d
= =
Th e pr in cipa l por t ion is :

586.52 464.89 121.63 =
112
http://actuary88.com
Calc ulat e t he mont hly payme nt of t he re financ e d loan.

Time t 0 1 2 10 27 28 29 30
(Year)
20 years' payments

( ) 12
20 6%
X a
Th e ou t s t a n din g ba la n ce of t h e or igin a l loa n a t t =10:

( )
( )
20
12
12 20 6%
6%
1
i
v
X X
d
a
=
(
=
(


( )
20 20
12
20 1 1
12 12
6%
6%
1 1 1.06
11.83937535
12 1 12 1 1.06
i
v
v
a

=
(
(

(
= = =
(
| | | |
(
| |
(
\ . \ .

Let
12
A
r epr es en t t h e mon t h ly pa ymen t of t h e r efin a n ced loa n .

( ) ( ) 12 12
20 20 4% 6%
A X a a =
( )
20 20
12
20 1 1
12 12
4%
4%
1 1 1.04
13.88301906
12 1 12 1 1.04
i
v
v
a

=
(
(

(
= = =
(
| | | |
(
| |
(
\ . \ .

( )
( )
( )
12
20
12
20
6%
4%
11.83937535
7, 038.186588 6, 002.1334
13.88301906
A X
a
a
= = =


So t h e mon t h ly pa ymen t in a dva n ce of t h e r efin a n ced loa n is :

6, 002.1334
500.1777833
12
=
113
http://actuary88.com
Find t he ac c umulat e d value of t he re duc t ion in mont hly payme nt s
inve s t e d at 4 % annual e ffe c t ive .

Time t 0 1 2 10 27 28 29 30
(Year)
20 years' payments

( )
( ) 12
20 4%
X A s
Redu ct ion of t h e mon t h ly pa ymen t du e t o r efin a n cin g:

( ) ( )
1 1 1, 036.0532
7, 038.1866 6, 002.1334 86.3377667
12 12 12
X A = = =
Th e a ccu mu la t ed va lu e of t h e r edu ct ion s a t 4%: ( )
( ) 12
20 4%
X A s
( )
( )
( )
( )
20 20
20
12
12 20 1 1
12 12
4%
1 1 1 1
1.04 1
30.4194036
12 1 12 1 1.04
i i
s
d
v

+ +

= = = =
| | | |

| |
\ . \ .

( )
( )
( )
12
20 4%
1, 036.0532 30.4194036 31, 516.12044 X A s = =
Met h od 2 u s e a mon t h a s t h e compou n din g per iod
Find t he mont hly payme nt of t he original loan

Time t 0 1 2 357 358 359 360
(Year)
8
Y Y Y Y Y Y
360 monthly payments

360 j
Y a
Let Y r epr es en t t h e mon t h ly pa ymen t in a dva n ce.
114
http://actuary88.com
Th e n u mber of compou n din g per iods : ( ) 30 12 360 =
Th e in t er es t r a t e per per iod:
1
12
1.06 1 0.48675506% j = =
360
100, 000
j
Y a =
360 360
1 360
1 1 1.0048675506
170.4984712
1 1.0048675506
j
v
d
a


= = =


360
100, 000
586.5155230 586.52
j
Y
a
= = ~


Find t he princ ipal port ion and t he int e re s t port ion of t he 3 7
t h

payme nt

Time t 0 1 2 36 357 358 359 360
(Month)
Y Y Y Y Y Y Y
324 monthly payments

324 j
Y a
Th e # of compou n din g per iod r ema in in g immedia t ely befor e t h e 37
t h

pa ymen t is ( ) 27 12 324 = .
Th e ou t s t a n din g loa n is :

( )
324 324
1 324
1 1 1.0048675506
586.5155230 95, 973.09
1 1.0048675506
j
v
Y Y
d
a


= = =


Th e in t er es t por t ion of t h e 37
t h
pa ymen t is :

( ) 95, 973.09 d , wh er e
1
1 1 1.0048675506 d v

= =
Th e in t er es t is :
( )
1
95, 973.09 1 1.0048675506 464.8909

=
Th e pr in cipa l por t ion is : 586.52 464.89 121.63 =
115
http://actuary88.com
Calc ulat e t he mont hly payme nt of t he re financ e d loan.

Time t 0 1 2 120 357 358 359 360
(Month)
Y Y Y Y Y Y Y
240 monthly payments

240 j
Y a
Th e ou t s t a n din g ba la n ce of t h e or igin a l loa n a t t =120 is :

( )
240 240
1 240
0.48675506%
1 1 1.0048675506
586.5155230 83, 327.73367
1 1.0048675506
j
j
v
Y Y
d
a

=
| |
= = =
|

\ .

Let B r epr es en t t h e mon t h ly pa ymen t of t h e r efin a n ced loa n .

240 240 k j
B Y a a = wh er e
1
12
1.04 1 0.32737398% k = =
240 240
1
240
1 1 1.0032737398
166.5962287
1 1.0032737398
k
k
v
d
a

| |
= = =
|

\ .

240
240
83, 327.73367
500.177791
166.5962287
j
k
Y
B
a
a
= = =


Find t he ac c umulat e d value of t he re duc t ion in mont hly payme nt s
inve s t e d at 4 % annual e ffe c t ive .

Time t 0 1 2 120 357 358 359 360
(Month)
Y Y Y Y Y Y Y
240 monthly payments

( )
240 k
Y B s
Redu ct ion of t h e mon t h ly pa ymen t du e t o r efin a n cin g:
116
http://actuary88.com
586.5155230 500.177791 86.337732 Y B = =
Th e a ccu mu la t ed va lu e of t h e r edu ct ion s a t 4%:

( )
240 k
Y B s
( )
( )
( )
( )
1 1
240 240
240
1 1 1 0.32737398% 1
365.0329421
1 1 1 1 0.32737398%
k
k
s
k

+ +
= = =
+ +

Th e a ccu mu la t ed va lu e is :

( ) ( )
240
86.337732 365.0329421 31, 516.11632
k
Y B s = =
Example 2 (2 0 0 2 May EA-1 #3 )

Given values:

. s
2n
m b g
= 180 24943

d
(m)
= 0.08

In wh a t r a n ge is

? s
4n
m b g
[A] Les s t h a n 2,930
[B] 2,930 bu t les s t h a n 2,970
[C] 2,970 bu t les s t h a n 3,010
[D] 3,010 bu t les s t h a n 3,050
[E] 3,050 or mor e

Solut ion B

( )
( )
( )
2
m
2n
1 1
s 180.24943
n
m
i
d
+
= = . Us in g d
(m)
= 0.08, we fin d t h a t

( ) ( )
2
1 180.24943 0.08 1 15.42
n
i + = + ~
( )
( )
( )
( )
( )
4 4
2
1 1 1 1
15.42 1
2, 959.71
0.08
m
4n
s
n n
m m
i i
d d
+ +

= = = =
117
http://actuary88.com
Example 3 (2 0 0 2 May EA-1 #4 )
A 20-yea r immedia t e a n n u it y cer t a in is pa ya ble mon t h ly. Immedia t ely
a ft er t h e 43
r d
pa ymen t h a s been ma de, t h e pr es en t va lu e of t h e
r ema in in g a n n u it y pa ymen t s is ca lcu la t ed t o be X .
N is t h e n u mber of t h e pa ymen t a ft er wh ich t h e pr es en t va lu e of t h e
r ema in in g a n n u it y pa ymen t s is les s t h a n
2
X
for t h e fir s t t ime.
( ) 4
0.08 d =
Wh a t is N ?
[A] 67
[B] 68
[C] 171
[D] 172
[E] 173

Solut ion

Fir s t , let s ca lcu la t e t h e mon t h ly in t er es t r a t e i . We a r e given
( ) 4
0.08 d = .
( )
( ) 4
3
3
1 1
4
d
v i

= + = , 0.99328839 v = (mon t h ly dis cou n t fa ct or )
Let P r epr es en t t he mon t h ly pa ymen t .

Time t
(months) 0 1 2 43 45 46 240
Payments
P P P P
240 43 197 i i
P a P a X

= =
We a r e a s ked t o fin d N s u ch t h a t
240
2
N i
X
P a

< . Fir s t , let s fin d N s u ch


t h a t
240
2
N i
X
P a

= .
240
197
1
2
2
N i
i
X
P a
P a X

= =
118
http://actuary88.com

240
240 240
240
197 197 197
197
1
1 1 0.99328839 1
1 1 1 0.99328839 2
N
N N
N i
i
v
P a
v
i
v P a v
i


= = = =

.
We n eed t o s olve t h e equ a t ion :

( )
240 197
1
0.99328839 1 1 0.99328839 0.63268311
2
N
= =
( ) 240 ln0.99328839 ln 0.63268311 N =
ln 0.63268311
240 67.9788
ln 0.99328839
N = =
240 67.9788 172.02 N = ~
As N in cr ea s es , t h e pr es en t va lu e of t h e r ema in in g a n n u it y pa ymen t s
decr ea s es . As t h e ext r eme, if 240 N = , t h en t h er es n o pa ymen t s left a n d
pr es en t va lu e of t h e r ema in in g a n n u it y pa ymen t s is zer o.

So 173 N = is t h e 1
s t
t ime t h a t r ema in in g a n n u it y pa ymen t s is les s t h a n
2
X
. Th e a n s wer is E.

119
http://actuary88.com
Example 4 (2 0 0 4 May EA-1 #8 )
Th e pr es en t va lu e of a 15-yea r mon t h ly a n n u it y-immedia t e is $20,600.
Pa ymen t s a r e a s follows :
Yea r s 1 7: X per mon t h
Yea r s 7 15: $300 X + per mon t h
In t er es t r a t e: 8%, compou n ded a n n u a lly.
Ca lcu la t e X
Solut ion

Time t (year) 0 7 15
Time t
(month) 0 1 2 . 84 85 86 180
payment X
X X X 300 X + 300 X + 300 X + 300 X +
Well br ea k down t h e or igin a l ca s h flows in t o t h e followin g t wo s t r ea ms :

St re am #1
Time t
(month) 0 1 2 . 84 85 86 180
payment

X X X X X X X X
180 i
PV X a =
Wh er e
1
12
1.08 1 0.6430301% i = = (mon t h ly effect ive in t er es t r a t e)

St re am #2
Time t (year) 0 7
Time t
(month) 0 1 2 . 84 85 86 180
Payment
300 300 300 300
( )
7
96
300 1.08
i
PV a

=
180 84 96
300 300
i i
a a

=
So t h e t ot a l PV of t h e pa ymen t s is :

( )
7
180 96
300 1.08 20, 600
i i
X a a

+ =
180
106.4275863
i
a = ,
96
71.45305627
i
a =
Solvin g t h e equ a t ion , we get : 76.03619 X =
120
http://actuary88.com
Inc re as ing annuit y

$1 $2 $3 $4 $( ) 1 n $ n
cash flows n

( )
i n
Ia
( )
i n
Ia
( )
i n
Is
( )
i n
Is
J u s t r emember on e in cr ea s in g a n n u it y for mu la : ( )
n
i
n
n
nv
i
a
Ia

=

Th en , ca lcu la t e t h e r ema in in g in cr ea s in g a n n u it y fa ct or s a s follows :


( ) ( )( )
1
n i n i
Ia i Ia = + ,
( ) ( ) ( )
1
n
n i n i
Is i Ia = + ,
( ) ( ) ( )
1
1
n
n i n i
Is i Ia
+
= +
Cont i nuous ly i nc re as i ng annuit y

( )
0
n n
i
n
n
t
nv
Ia t e dt
a
o
o

= =
}
121
http://actuary88.com
De c re as i ng annuit y

$ n $( ) 1 n $4 $3 $2 $1

cash flows n

( )
n i
Da ( )
i n
Da
( )
i n
Ds ( )
i n
Ds
( )
n i
n i
n a
Da
i

= ,
( )
n i
n i
n a
Da
d

=
( )
( ) 1
n
n i
n i
i n s
Ds
i
+
= ,
( )
( ) 1
n
n i
n i
i n s
Ds
d
+
=
J u s t r emember on e decr ea s in g a n n u it y for mu la :
( )
n i
n i
n a
Da
i

=
Th en , ca lcu la t e t h e r ema in in g in cr ea s in g a n n u it y fa ct or s a s follows :
( ) ( )( ) 1
n i n i
Da Da i = + , ( ) ( ) ( ) 1
n
n i n i
Ds Da i = + , ( ) ( ) ( )
1
1
n
n i n i
Ds Da i
+
= +
122
http://actuary88.com
Example 1
A compa n y is pa r t icipa t in g in a pr oject . Th e ca s h flows of t h e pr oject a r e
a s follows :
Th e compa n y will in ves t $10 million per yea r for t h e 1
s t
t h r ee yea r s
of t h e pr oject . Th e in ves t men t will be ma de con t in u ou s ly.

Th e compa n y will r eceive a ca s h flow a t t h e en d of ea ch yea r
s t a r t in g fr om Yea r 4.

At t h e en d of Yea r 4, t h e compa n y will r eceive t h e 1
s t
ca s h flow of
$9 million . Th is a mou n t will be r edu ced by $0.5 million for ea ch
s u bs equ en t yea r , u n t il t h e compa n y r eceives $5 million in a yea r .

St a r t in g fr om t h a t yea r , t h e ca s h flow r eceived by t h e compa n y will
be r edu ced by $1 million ea ch yea r , u n t il t h e compa n y r eceives
zer o ca s h flow.

Ca lcu la t e t h e NPV of t h e pr oject if t h e dis cou n t r a t e is 12%.

Solut ion

Time t 0 1 2 3 4 5 6 7 8 9 10 11 12 13 14 15 16

cash $9.0 $8.5 $8.0 $7.5 $7.0 $6.5 $6.0 $5.5 $5.0 $4.0 $3.0 $2.0 $1.0

( )
12% 3
10 a
Th e in it ia l in ves t men t a t t =0 is :
( )
3
12%
3
3
12%
1 1 1.12
10 10 10 25.43219763
ln1.12
i
v
a
o

=
| |
= = =
|
\ .

Next , let s ca lcu la t e t h e PV of ca s h flows fr om t =4 t o t =12.

Time t 0 1 2 3 4 5 6 7 8 9 10 11 12 13 14 15 16

ca s h $9.0 $8.5 $8.0 $7.5 $7.0 $6.5 $6.0 $5.5 $5.0

( )
9 12% 8 5%
9 0.5 I a a
123
http://actuary88.com
1 9 12%
9
1 1.12
9 9 53.7087579
1 1.12
a

= =


( )
( )
( )
8
8
8
1
12%
12
8
8 %
1 1.12
8 1.12
8 1.12
1 1.12
0.5 0.5 0.5
0.12 0.12
I
a
a


= =


( )
8
5.56375654 8 1.12
0.5 9.71954465
0.12

= =
( )
9 12% 8 5%
9 0.5 53.7087579 9.71954465 43.98921325 I a a = =
PV of ca s h flows fr om t =4 t o t =12:
( )
4
1.12 43.98921325 27.95594028

=
Next , let s ca lcu la t e t h e PV of ca s h flows fr om t =13 t o t =16:

Time t 0 1 2 3 4 5 6 7 8 9 10 11 12 13 14 15 16

ca s h $4.0 $3.0 $2.0 $1.0

( )
12% 4
Da
( )
12%
12% 4
4
4
4 3.03734935
8.02208878
12% 12%
n
n
i
a a
Da


= = = =
PV is :
( )
12
1.12 8.02208878 2.05907038

=
So t h e PV of a ll ca s h in flows is :

27.95594028 2.05907038 30.01501066 = + =
Fin a lly, t h e NPV of t h e pr oject is :

30.01501066 25.43219763 4.58281303 =
124
http://actuary88.com
Example 2 (May 2 0 0 4 SOA EA-1 #3 )

Type of An n u it y An n u it y immedia t e, wit h 19 a n n u a l
pa ymen t s .
An n u a l Pa ymen t s Fir s t pa ymen t is $1, in cr ea s in g ea ch yea r by
$1 u n t il pa ymen t r ea ch es $10, t h en
decr ea s in g by $1 ea ch yea r t o t h e fin a l
pa ymen t of $1
In t er es t r a t e 5% a n n u a l effect ive
In wh a t r a n ge is t h e pr es en t va lu e of t h is a n n u it y a t t h e da t e of t h e
pu r ch a s e?
[A] Les s t h a n $57
[B] $57 bu t les s t h a n $60
[C] $60 bu t les s t h a n $63
[D] $63 bu t les s t h a n $66
[E] $66 or mor e

Solut ion C
Th e fa s t es t s olu t ion is t o u s e t h e ca s h flow wor ks h eet in BA II Plu s / BA II
Plu s Pr ofes s ion a l. En t er t h e followin g in t o Ca s h Flow Wor ks h eet :

Time Cas h flow Amt Fre que nc y Fre que nc y
0 CF0
1 C0 1 $ 1 F01 1
2 C0 2 $ 2 F02 1
3 C0 3 $ 3 F03 1
4 C0 4 $ 4 F04 1
5 C0 5 $ 5 F05 1
6 C0 6 $ 6 F06 1
7 C0 7 $ 7 F07 1
8 C0 8 $ 8 F08 1
9 C0 9 $ 9 F09 1
1 0 C1 0 $ 1 0 F10 1
1 1 C1 1 $ 9 F11 1
1 2 C1 2 $ 8 F12 1
1 3 C1 3 $ 7 F13 1
1 4 C1 4 $ 6 F14 1
1 5 C1 5 $ 5 F15 1
1 6 C1 6 $ 4 F16 1
1 7 C1 7 $ 3 F17 1
1 8 C1 8 $ 2 F18 1
1 9 C1 9 $ 1 F19 1
125
http://actuary88.com
Th ou gh t h e a bove t a ble lis t s t h e ca s h flow fr equ en cies fr om F01 t o F19,
you r ea lly don t n eed t o en t er a n y ca s h flow fr equ en cy. If a u s er does n t
en t er t h e # of ca s h flows , BA II Plu s / BA II Plu s Pr ofes s ion a u t oma t ica lly
s et s t h e # of a ca s h flow t o on e. Th is s h ou ld be fin e beca u s e a ll ou r ca s h
flow fr equ en cies a r e on e.

Next , s et t h e in t er es t r a t e t o 5%. You s h ou ld get :

62.60644983 NPV =
Alt e rnat ive me t hod (a lit t le s lowe r, but not t oo bad)
We br ea k down t h e or igin a l ca s h flows in t o t wo s t r ea ms :

St r ea m #1: In cr ea s in g a n n u it y fr om t =1 t o t =9
St r ea m #2: Decr ea s in g a n n u it y fr om t =10 t o t =19

We t h en s epa r a t ely ca lcu la t e t h e pr es en t va lu e of ea ch s t r ea m a n d fin d
t h e t ot a l pr es en t va lu e.

Th e pr es en t va lu e of St r ea m #1: ( )
5% 9
Ia
To fin d t h e pr es en t va lu e of St r ea m #2, we fir s t ca lcu la t e t h e pr es en t
va lu e of t h is s t r ea m a t t =9; t h e PV s h ou ld be ( )
5% 10
Da . Th en , we
dis cou n t t h is PV t o t =0.

Th e pr es en t va lu e of St r ea m #2: ( )
9
5% 10
v Da
Th e PV of t h e or igin a l ca s h flows is : ( ) ( )
9
5% 5% 9 10
v Ia Da +
( )
( )
( )
5%
9
9
5% 9
9
9 1.05
7.46321276 9 1.05
33.23465027
5% 5%
a
Ia


= = =

( )
9 9 9
5%
10 5%
10
10
10 7.72173493
1.05 1.05 29.37179956
5% 5%
v
a
Da


= = =
33.23465027 29.37179956 62.60644983 + =
126
http://actuary88.com
Example 3 (May 2 0 0 1 SOA EA-1 #7 )

Repa ymen t s ch edu le for a loa n :

End of each odd numbered year Amount of repayment
1 $100
3 $300
5 $500


X $100X


25 $2,500
In t er es t r a t e: 6% per yea r , compou n ded a n n u a lly

A is t h e t ot a l of t h e pa ymen t s t o be ma de a ft er t h e 15
t h
yea r .

B is t h e pr es en t va lu e of t h e r ema in in g pa ymen t s a s of t h e begin n in g of
t h e 16
t h
yea r .

In wh a t r a n ge is A B ?
(A) Les s t h a n $3,120
(B) $3,120 bu t les s t h a n $3,150
(C) $3,150 bu t les s t h a n $3,180
(D) $3,180 bu t les s t h a n $3,210
(E) $3,210 or mor e

Solut ion C
Let s s et $100 a s on e u n it of mon ey.
Th e in t er es t r a t e per 2 yea r s is :
2
1.06 1 12.36% i = =
A =17+19+21+23+25=105

Next , well ca lcu la t e B . Plea s e n ot e t h a t B is t h e PV a s of t h e be ginning
of t h e 16t h e yea r (i.e. en d of t h e 15
t h
yea r ). So B is t h e PV a t 15 t = , n ot a t
16 t = .
Time t
(year) 15 16 17 18 19 20 21 22 23 24 25
Reset time t
(2 years) 0 1 2 3 4 5
Repayment $17 $19 $21 $23 $25
127
http://actuary88.com
You ca n u s e t h e in cr ea s in g a n n u it y for mu la t o ca lcu la t e B . However ,
t h a t ca lcu la t ion is over ly complex. A s imple a ppr oa ch is t h is :

2 3 4 5
17 19 21 23 25 B v v v v v = + + + +
2 3 4 5
17 19 21 23 25
73.37424439
1.1236 1.1236 1.1236 1.1236 1.1236
= + + + + =
105 73.37 31.63 $3,163 A B = = =
Th e fa s t es t s olu t ion is t o u s e BA II Plu s Ca s h Flow Wor ks h eet . En t er t h e
followin g in t o Ca s h Flow Wor ks h eet :

CF0 C01 C02 C03 C04 C05
$0 $17 $19 $21 $23 $25
Th en s et I=12.36 (s o t h e in t er es t r a t e is 12.36%). You s h ou ld get :

NPV=73.37424439. 73.37424439 B = .
To ca lcu la t e B , s imply s et I=0 (s o t h e in t eres t r a t e is zer o). You s h ou ld
get :

NPV=105.

105 A = 105 73.37 31.63 $3,163 A B = = =
Moral of t his proble m:
Ha vin g a n in cr ea s in g a n n u it y does n t mea n you h a ve t o u s e t h e
in cr ea s in g a n n u it y for mu la .
Example 4 (May 2 0 0 4 SOA EA-1 #2 5 )
Smit h bu ys a 10-yea r decr ea s in g a n n u it y-immedia t e wit h a n n u a l
pa ymen t s of 10, 9 , 8, , 2, 1.

On t h e s a me da t e, Smit h bu ys a per pet u it y-immedia t e wit h a n n u a l
pa ymen t s . For t h e fir s t 11 yea r s , pa ymen t s a r e 1, 2, 3, , 11. Aft er yea r
11, pa ymen t s r ema in con s t a n t a t 11.

At a n a n n u a l effect ive in t er es t r a t e of i , bot h a n n u it ies h a ve a pr es en t
va lu e of X .
Ca lcu la t e X .
128
http://actuary88.com
Solut ion
As u s u a l, we dr a w a ca s h flow dia gr a m:

Time t 0 1 2 3 4 5 6 7 8 9 10 11
Annuity #1 $10 $9 $8 $7 $6 $5 $4 $3 $2 $1
Annuity #2 $1 $2 $3 $4 $5 $6 $7 $8 $9 $10 $11 $11 $11
Sin ce we don t kn ow t h e in t er es t r a t e, we ca n t u s e BA II Plu s Ca s h Flow
Wor ks h eet . We h a ve t o u s e t h e for mu la s for t h e in cr ea s in g a n d
decr ea s in g a n n u it y.

( )
10
10
10
i
i
a
Da
i

=
To ca lcu la t e t h e PV of a n n u it y 2, well br ea k it down in t o 2 s t r ea ms :
St r ea m #1
Time t 0 1 2 3 4 5 6 7 8 9 10 11
Annuity #2 $11 $11 $11
10
11
v
i
11
i
At 10 t = , t h e PV of t h is s t r ea m is
11
i
. At 0 t = , t h e PV of t h is s t r ea m is
10
11
v
i
.
St r ea m #2
Time t 0 1 2 3 4 5 6 7 8 9 10
Annuity #2 $1 $2 $3 $4 $5 $6 $7 $8 $9 $10
( )
10
10
10
10
i
v
i
a
Ia

=

So a t 0 t = , t h e PV of An n u it y #2 is
10
10
10
10
11
v
v
i i
a
+

.
We a r e t old t h a t :

10
10
10 10
10
11
10
i
v
v X
i i
a
i
a
= + =

10
10
10
10
10 11 10
i
a v v a = +
129
http://actuary88.com
10
10 10
10 0
i
a v a + + =
To qu ickly s olve t h is equ a t ion , well con ver t t h e equ a t ion in t o ca s h flows :

Con ver t
10
a in t o a s t r ea m of ca s h flows :
Time t 0 1 2 3 4 5 6 7 8 9 10
$1 $1 $1 $1 $1 $1 $1 $1 $1 $1
10
a
Con ver t
10 i
a in t o a s t r ea m of ca s h flows :
Time t 0 1 2 3 4 5 6 7 8 9 10
$1 $1 $1 $1 $1 $1 $1 $1 $1 $1
10 i
a
Con ver t
10
10 v in t o a s t r ea m of ca s h flows :
Time t 0 1 2 3 4 5 6 7 8 9 10
$1
10
10
v
Su m u p t h e a bove 3 s t r ea ms of ca s h flows :
Time t 0 1 2 3 4 5 6 7 8 9 10
- $9 $2 $2 $2 $2 $2 $2 $2 $2 $2 $2
10
10 10
10 0
i
a v PV a + + = =
To fin d t h e in t er es t r a t e i , well u s e Ca s h Flow Wor ks h eet . En t er t h e
followin g in t o Ca s h Flow Wor ks h eet :

CF0= - 9, C01= 1, F01 = 10

Pr es s IRR CPT. You s h ou ld get : IRR=17.96301385.

So 17.96301385% i =

10 10 17.96301385%
10 10
30.62092178
17.96301385%
i
a a
X
i

= = =
130
http://actuary88.com
Example 5 (May 2 0 0 0 SOA EA-1 #1 0 modifie d)
Ter m of a 20-yea r a n n u it y-cer t a in :
In it ia l pa ymen t : $300 du e 1/ 1/ 200

Pa ymen t pa t t er n s :
All pa ymen t s a r e ma de J a n u a r y 1

Pa ymen t s in cr ea s e by $300 ea ch yea r begin n in g
1/ 1/ 2001 t h r ou gh 1/ 1/ 2009

Pa ymen t s decr ea s e by $200 ea ch yea r begin n in g
1/ 1/ 2001 t h r ou gh 1/ 1/ 2019

In t er es t r a t e: 7% per yea r , compou n ded a n n u a lly for t h e 1
s t
9 yea r s .
6% per yea r , compou n ded a n n u a lly t h er ea ft er .

Ca lcu la t e t h e pr es en t va lu e of t h e a n n u it y.

Solut ion

Fir s t , let s lis t a ll of t h e ca s h flows .

Time t Date
Payment (Use $100 as one
unit of money)
0 1/1/2000 $3
1 1/1/2001 $6
2 1/1/2002 $9
3 1/1/2003 $12
4 1/1/2004 $15
5 1/1/2005 $18
6 1/1/2006 $21
7 1/1/2007 $24
8 1/1/2008 $27
9 1/1/2009 $30
10 1/1/2010 $28
131
http://actuary88.com
11 1/1/2011 $26
12 1/1/2012 $24
13 1/1/2013 $22
14 1/1/2014 $20
15 1/1/2015 $18
16 1/1/2016 $16
17 1/1/2017 $14
18 1/1/2018 $12
19 1/1/2019 $10
Her e we h a ve t wo in t er es t r a t es : 7% fr om 0 t = t o 9 t = a n d 6% fr om 9 t =
t o 19 t = . As a r es u lt , we h a ve t o br ea k down t h e ca s h flows in t o t wo
s t r ea ms . For ea ch s t r ea m, well dir ect ly en t er t h e ca s h flows in t o BA II
Plu s Ca s h Flow Wor ks h eet t h is is t h e fa s t es t wa y.

St re am #1 (En t er t h e followin g in t o Ca s h Flow Wor ks h eet )
Time t Date
Payment (Use $100 as one
unit of money)

9 1/1/2009

CF0
10 1/1/2010 $28

C01
11 1/1/2011 $26

C02
12 1/1/2012 $24

C03
13 1/1/2013 $22

C04
14 1/1/2014 $20

C05
15 1/1/2015 $18

C06
16 1/1/2016 $16

C07
17 1/1/2017 $14

C08
18 1/1/2018 $12

C09
19 1/1/2019 $10

C10
132
http://actuary88.com
Us in g t h e in t er es t r a t e of 6%, t h e PV a t 9 t = of St r ea m #1 is :
146.8777947

St re am #2 At 9 t = , we a dd t h e PV of St r ea m #1 t o t h e ca s h flow of #30
Time t Date

0 1/1/2000 $3

CF0
1 1/1/2001 $6

C01
2 1/1/2002 $9

C02
3 1/1/2003 $12

C03
4 1/1/2004 $15

C04
5 1/1/2005 $18

C05
6 1/1/2006 $21

C06
7 1/1/2007 $24

C07
8 1/1/2008 $27

C08
9 1/1/2009 $30+146.8777947=176.8777947

C09
Us in g t h e in t er es t r a t e of 7%, you s h ou ld get :
NPV=191.4044063 = $19,140.44 (on e u n it =$100)

133
http://actuary88.com
If you pr efer t h e formu la -dr iven a ppr oa ch , t h is is h ow. We s t ill u s e $100
a s on e u n it of mon ey. We br ea k down t h e ca s h flows in t o t wo s t r ea ms .
On e s t r ea m con s is t s of in cr ea s in g a n n u it y pa ymen t s fr om 0 t = t o 9 t = ;
t h e ot h er con s is t s of decr ea s in g a n n u it y pa ymen t s fr om 10 t = t o 19 t =
At 9 t = , t h e PV of t h e decr ea s in g a n n u it y
Time t 9 10 11 12 18 19
Cash flow 30-2(1) 30-2(2) 30-2(3) 30-2(9) 30-2(10)
( )
10 6% 10 6%
30 2 146.8777947 a Ia =
At 0 t = , t h e PV of t h e in cr ea s in g a n n u it y
Time t 0 1 2 3 8 9
Cash flow 3(1) 3(2) 30(3) 3(4) 3(9) 3(10)
( )
10 7%
3 111.51261773 Ia =
Th e t ot a l PV a t 0 t = is :

( ) ( ) ( )
9 9
10 6% 10 7% 10 6%
3 30 2 1.07 111.51261773 146.8777947 1.07 Ia a Ia

(
+ = +


191.4044063 $19,140.44 = =
Example 6 (May 2 0 0 5 SOA EA-1 #8 )

2
8.00407
n n i
a =
1 2 1
8.63279
n n i
a
+
=
i =t h e a n n u a l effect ive in t er es t , compou n ded a n n u a lly.

Ca lcu la t e i .
Solut ion

2 n n i
a a n d
1 2 1 n n i
a
+
a r e s ymbols for defer r ed a n n u it ies . In a defer r ed
a n n u it y, a ll t h e ca s h flows a r e s h ift ed r igh t wa r ds . For exa mple, t h is is
t h e ca s h flow dia gr a m for
2n i
a :
134
http://actuary88.com
Time t 0 1 2 3
2 1 n 2n
Cash flow $1 $1 $1 $1
2n i
a
To dr a w t h e dia gr a m for
2 n n i
a , a n n -yea r defer r ed a n n u it y, we s imply
s h ift a ll t h e a bove ca s h flows r igh t wa r ds by n u n it s of t ime (s o t h e 1
s t

ca s h flow s t a r t s a t 1 n + , in s t ea d of 1 t = ):

Time t 0 1 2 n 1 n + 2 n +
2 1 n n + 2n n +
Cash flow $1 $1 $1
( )
2 2
n
n n i n i
a a v =
Simila r ly, we dr a w t h e dia gr a m for
1 2 1 n n i
a
+
(a n 1 n yea r defer r ed
a n n u it y) by s h ift in g t h e ca s h flows in
2 1 n i
a
+
t o t h e r igh t by 1 n u n it s of
t ime (s o t h e 1
s t
ca s h flow s t a r t s a t n , in s t ea d of 1 t = ):

Time t 0 1 2 3
2 1 n 2n 2 1 n +
Cash flow $1 $1
$1 $1 $1
2 1 n i
a
+
Time t 0 1 2 n 1 n + 2 n +
( ) ( ) 2 1 1 n n + +
Cash flow
$1 $1 $1 $1 $1
( )
1
1 2 1 2 1
n
n n i n i
a a v

+ +
=
Come ba ck t o t h e pr oblem. Let s compa r e
2 n n i
a a n d
1 2 1 n n i
a
+
:
Time t 0 1 2 n 1 n + 2 n +
2 1 n n + 2n n +
Cash flow $1 $1 $1
2 n n i
a
Time t 0 1 2 n 1 n + 2 n +
( ) ( ) 2 1 1 n n + +
Cash flow
$1 $1 $1 $1 $1
1 2 1 n n i
a
+
135
http://actuary88.com
1 2 1 n n i
a
+
h a s 2 1 n + ca s h flows s t a r t in g fr om t n = a n d en din g a t
( ) ( ) 2 1 1 3 t n n n = + + = .
2 n n i
a h a s 2n ca s h flows s t a r t in g fr om 1 t n = + a n d en din g a t 2 3 t n n n = + = .
So
1 2 1 n n i
a
+
h a s a ll t h e ca s h flows in
2 n n i
a , except
1 2 1 n n i
a
+
h a s on e
a ddit ion a l ca s h flow a t t n = . So we h a ve:

1 2 1 2
n
n n n i n i
a a v
+
= +
1 2 1 2
8.63279 8.00407 0.62872
n
n n n i n i
v a a
+
= = =
( )
2
2 2
1
n
n n
n n i n i
v
a a v v
i

= = ,
( )
2
1 0.62872
8.00407 0.62872
i

= , 4.75% i =
136
http://actuary88.com
Chapt e r 4 Calc ulat or t ips

Be s t c alc ulat ors for Exam FM

SOA/ CAS a ppr oved ca lcu la t or s :
BA-35, BA II Plu s , BA II Plu s Pr ofes s ion al, TI-30X, TI-30Xa , TI-30X II (IIS
s ola r or IIB ba t t er y).
Bes t ca lcu la t or s for Exa m P: BA II Plu s , BA II Plu s Pr ofes s ion a l, TI-30X
IIS.
You s h ou ld br in g t wo ca lcu la t or s t o t h e exa m r oom -- BA II Plu s
Pr ofes s ion a l a n d TI-30 IIS. BA II Plu s Profes s ion a l is good for gen er a l
ca lcu la t ion s a n d t h e t ime-va lu e-of mon ey ca lcu la t ion s . TI-30 IIS is good
for gen er a l ca lcu la t ion s .

Even if you h a ve BA II Plu s , you migh t wa n t t o bu y a BA II Plu s
Pr ofes s ion a l.

Ne w fe at ure s adde d in BA II Plus Profe s s ional.

We a r e on ly con cer n ed wit h fea t u r es r eleva n t t o Exa m FM:

BA II Plus Profe s s ional BA II Plus
Net Fu t u r e Va lu e (n ice fea t u r e) Does n t h a ve t h is .
Modified Du r a t ion Does n t h a ve t h is .
In TVM, t h e defa u lt va lu es a r e P/ Y
=1 a n d C/ Y=1 (n ice impr ovemen t
over BA II Plu s )
In TVM, t h e defa u lt va lu es a r e
P/ Y =12 a n d C/ Y=12 (Th is is a
pa in )
How t o re s e t c alc ulat ors t o t he ir be s t c ondit ions
for FM

Accor din g t o exa m r u les , wh en you u s e BA II Plu s , BA II Plu s
Pr ofes s ion a l, a n d TI-30X IIS for a n SOA or CAS exa m, exa m pr oct or s on
s it e will n eed t o clea r t h e memor ies of you r BA II Plu s , BA II Plu s
Pr ofes s ion a l, a n d TI-30X IIS. Typica lly, a pr oct or will clea r you r
ca lcu la t or s memor ies by r es et t in g t h e ca lcu la t or t o it s defa u lt s et t in g.
Th is is don e by pr es s in g 2
n d
Res et En t er for BA II Plu s an d by
s imu lt a n eou s ly pr es s in g On a n d Clea r for TI-30X IIS a n d TI-30X IIB.
You will n eed t o kn ow h ow t o a dju s t t h e s et t in gs of BA II Plu s a n d TI-30X
IIS t o you r bes t a dvan t a ge for t h e exa m.
137
http://actuary88.com
Bes t s et t in gs for BA II Plu s
Default setting Optimal setting Keystrokes to change the default
setting to the optimal setting
Display 2 decimal places. Enter
0.123452, youll get 0.25.
Display 8 decimal places. Enter
0.123452, youll get 0.24690000.

2
nd
Format 8 Enter
Use the chain method. The
calculator calculates numbers
in the order that you enter
them. If you enter 2+310,
youll get 50. The calculator
first calculates 2+3=5. Then it
calculates 510=50.
Use AOS (Algebraic operating
system). The calculator follows
the standard rules of algebraic
hierarchy in its calculation. If you
enter 2+310, youll get 32. The
calculator first calculates 310.
Then it calculates 30+2=32.

2nd [FORMAT],
keep pressing + multiple times
until you see Chn. Press 2nd
[ENTER]

(if you see AOS, your calculator
is already in AOS, in which case
press
[CLR Work] )
Set P/Y=12 and C/Y=12 in
TVM (time value of money)
Worksheet. P stands for
payment. C stands for
compounding. Y stands for
year.

C/Y=12 means 12
compounding periods per year.
If you enter I/Y=6 (i.e.. set 6%
annual interest rate), the
calculator interprets this as a
nominal rate compounding
monthly and uses an interest
rate of 6%/12=0.5% per month
in its calculation.

P/Y=12 means 12 payment in a
year. If you enter 30 2
nd
xP/Y,
youll get 360. This means that
you are paying off a loan
through 360 monthly
payments.

The setting of P/Y=12 and
C/Y=12 is useful occasionally
and harmful in majority of
the times. In the heat of the
exam, you can easily forget to
switch settings.
Set P/Y=1 and C/Y=1.

If you enter 6% per period (per
year, per month, per day, etc),
youll get 6% per period (per year,
per month, per day, etc).

If you enter 30 payments, youll
get 30 payments, not 360
payments.

Always use the setting P/Y=1
and C/Y=1. Under no
circumstance should you change
this setting.

What if you are paying off a loan
through monthly payments?
Simply enter the monthly interest
rate and the # of monthly
payments into BA II Plus TVM.

2
nd
P/Y 1 Enter
2
nd
C/Y 1 Enter
BA II Plu s Pr ofes s iona l t h e defa u lt s et t in g is 2 decima l dis pla y, t h e
ch a in met h od, a n d P/ Y=1 a n d C/ Y=1. As a r es u lt , you ju s t n eed t o s et
BA II Plu s Pr ofes s iona l t o dis pla y 8 decima l pla ces a n d t h e AOS. You
don t n eed t o s et P/ Y=1 a n d C/ Y=1 beca u s e t h is is t h e defa u lt s et t in g.

138
http://actuary88.com
AOS is mor e power fu l t h a n t h e ch a in met h od. For exa mple, if you wa n t
t o fin d
3
1 2 4 5 e + + , u n der AOS, you n eed t o en t er

1 + 2 3 2
n d
x
e + 4 5 x (t h e r es u lt is a bou t 50.1153)

Un der t h e ch a in met h od, t o fin d
3
1 2 4 5 e + + , you h a ve t o en t er :

1+(23 2
n d
x
e )+(45 x )
AOS is bet t er beca u s e t h e ca lcu la t ion s equ en ce u n der AOS is t h e s a me
a s t h e ca lcu la t ion s equ en ce in t h e for mu la . In con t r a s t , t h e calcu la t ion
s equ en ce in t h e ch a in met h od is cu mber s ome.

TI-30X IIS --- You n eed ch a n ge on ly on e it em on you r on ce t h e pr oct or
r es et s it . In it s defa u lt s et t in gs , TI-30X IIS dis pla ys t wo decima l pla ces .
You s h ou ld s et it t o dis pla y 8 decima l pla ces . Pr es s 2
n d
Fix. Th e ch oos e
8.

Th e power of t h e TI-30X IIS lies in it s a bilit y t o dis pla y t h e da t a a n d
for mu la en t er ed by t h e u s er . Th is wh a t you t ype is wh a t you s ee fea t u r e
a llows you t o dou ble ch eck t h e a ccu r a cy of you r da t a en t r y a n d of you r
for mu la . It a ls o a llows you t o r edo ca lcu la t ion s wit h modified da t a or
modified for mu la s .

For exa mple, if you wa n t t o ca lcu la t e
2.5
2 1 e

, a s you en t er t h e da t a in
t h e ca lcu la t or , you will s ee t h e dis pla y:

.
2 ( 2.5) 1 e
If you wa n t t o fin d ou t t h e fin a l r es u lt , pr es s t h e En t er key a n d you will
s ee:

.
2 ( 2.5) 1
-0.835830003
e
So
.
= 2 ( 2.5) 1 -0.835830003 e
Aft er get t in g t h e r es u lt of -0.835830003 , you r ea lize t h a t you ma de a n
er r or in you r da t a en t r y. In s t ea d of ca lcu la t in g
.
2 ( 2.5) 1 e , you r ea lly
wa n t ed t o ca lcu la t e
.
2 ( 3.5) 1 e . To cor r ect t h e da t a en t r y er r or , you
s imply ch a n ge 2.5 t o 3.5 on you r TI-30X IIS. Now you s hou ld s ee:

139
http://actuary88.com
.
2 ( 3.5) 1
-0.939605233
e
Wit h t h e on lin e dis pla y fea t u r e, you ca n a ls o r eu s e for mu la s . For
exa mple, a pr oblem r equ ir es you t o ca lcu la t e

y = 2 1
x
e for
= = =
1 2 3
5, 6, a nd 7. x x x Th er e is n o n eed t o do t h r ee s epa r a t e
ca lcu la t ion s fr om s cr a t ch . You en t er
.
2 (5) 1 e in t o t h e ca lcu la t or t o
ca lcu la t e y wh en 5 x = . Th en you modify t h e for mu la t o ca lcu la t e y wh en
6 x = a n d 7 x = .
Compound i nt e re s t

Proble m 1

Toda y Ma r y depos it s $23.71 in t o a ba n k a ccou n t a n d ea r n s 6% a n n u a l
effect ive. Ca lcu la t e t h e ba la n ce of h er ba n k a ccou n t 2 yea r s fr om t oda y.

Solut ion

Th e for mu la for s imple in t er es t r a t e is : ( ) ( )( ) 0 1
n
A n A i = +
We a r e given : ( ) 0 23.71, 6%, 2 A i n = = = .
( ) ( )
2
2 23.71 1 6% A = +
Me t hod 1 BA II Plus / BA II Plus Profe s s ional

Pr ocedu r e Keys t r okes Dis pla y
As s u me you a lr ea dy s et
t h e ca lcu la t or t o dis pla y
8 decima l pla ce a n d t o
u s e AOS.
Omit t ed Omit t ed
Ca lcu la t e ( )
2
23.71 1 6% +
23.71 23.71000000
( 6% 0.06000000
+1 1
) 1.06000000
2
x
1.12360000
= 26.64055600
140
http://actuary88.com
Wa t ch ou t t h e % oper a t or . In BA II Plu s / BA II Plu s Pr ofes s ion a l,
% a b is ca lcu la t ed a s ( ) 1 % a b . For exa mple, if you en t er 100+5%,
you ll get 105, n ot 100.05. If you en t er 100-5%, you ll get 95, n ot
99.95%. Th is is n ot a pr oblem if you wa n t t o ca lcu la t e ( ) 1 6% + .
However , if you wa n t t o ca lcu la t e ( ) 2 6% + a n d you en t er (2+6%),
you ll get 2.12. To ca lcu la t e 2+6%, you ca n en t er 6%+2, wh ich
gives you 2.06. So t o a void a n y mis t a kes , a lwa ys en t er % b a + if you
wa n t t o ca lcu la t e % a b + .
Plea s e n ot e t h a t if you a r e t o ca lcu la t e 23.71(1.06)
5.6
, you ca n en t er
23.711.06
x
y 5.6=. You s h ou ld get 32.85824772.

In BA II Plu s / BA II Plu s Pr ofes s ion a l, ) is t h e s a me a s =. You
ca n s olve t h e pr oblem u s in g t h e followin g ca lcu la t or key s t r okes :

Pr ocedu r e Keys t r okes Dis pla y
Ca lcu la t e
( )
2
23.71 1 6% +
23.71 23.71000000
(1+ 1.00000000
6% 0.06000000
) 1.06000000
2
x
1.12360000
) 26.64055600
Me t hod 2 --- TI-3 0 IIS

Set t o dis pla y 8 decima l pla ces
En t er 23.71(1+6%)
2
. Pr es s En t er . You s h ou ld get 26.64055600.
Dou ble ch eck you r da t a en t r y. Pr es s | . Dou ble ch eck t h a t you
in deed en t er ed 23.71(1+6%)
2
. Yes , you did. So t h e r es u lt
26.64055600 is cor r ect .

Plea s e n ot e t h a t if you a r e t o ca lcu la t e 23.71(1+6%)
5.6
, you can en t er
23.71(1+6%)^5.6. You s h ou ld get 32.85824772. Plea s e a ls o n ot e t h a t TI-
30 IIS ca lcu la t es a +b% a s a +b%. For exa mple, if you en t er 2+6%, you ll
get 2.06.

141
http://actuary88.com
Me t hod 3 --- Us e % A Works he e t of BA II Plus / BA II Plus Profe s s ional

Pr ocedu re Keys t roke Dis play
Ch oos e % A Wor ks h eet
2n d [ % A ]
OLD= (old con t en t )
Clear wor ks h eet
2n d [CLR Wor k]
OLD=0.00000000
En t er pr in cipal.
23.71 En t er
OLD=23.71000000
En t er t h e # of compou n din g
period
(can de a in t eger or fr a ct ion )

|
#PD=1.00000000
(Th e defau lt period is
on e)
2 En t er
#PD=2.00000000

En t er t h e in t eres t r at e
|
% CH=0.00000000
(Th e defau lt is zer o)
6 En t er (En t er 6, n ot
6%)
% CH=6.00000000

Calcu lat e t h e balan ce
|
NEW=0.00000000
(Th e defau lt is zer o)
CPT
NEW=26.64055600
Not e Th e # of compou n din g per iods ca n be a fr a ct ion . If you a r e t o
ca lcu la t e 23.71(1+6%)
5.6,
you s imply en t er t h e followin g:

OLD=23.71, % CH=6, # PD=5.6.

You s h ou ld get NEW=32.85824772

142
http://actuary88.com
Me t hod 4 --- Us e TVM Works he e t of BA II Plus / BA II Plus
Profe s s ional

Pr ocedu re Keys t roke Dis play
Set C/ Y=1 an d P/ Y=1. We
always s et C/ Y=1 an d P/ Y=1
before u s in g TVM.

Clear TVM Wor ks h eet
2n d [CLR TVM]
0.00000000
En t er pr in cipal.
23.71 PV En t er
PV=23.71000000
En t er in t eres t r at e.
6 I/ Y En t er
I/ Y=6.00000000
En t er t h e # of compou n din g
period
(can de a in t eger or fr a ct ion )

2 N En t er
N=2.00000000
Calcu lat e t h e accu mu lat ed
valu e.

CPT FV
FV= - 26.64055600

Not e:
(1) Th e n ega t ive s ign in FV= - 26.64055600 mea n s t h a t you a r e pa yin g
off a loa n of $26.64055600. As a gen er a l r u le, a pos it ive s ign in TVM
mea n s mon ey comin g fr om s omeon e els es pocket a n d goin g t o you r
pocket ; a n ega t ive s ign in TVM mea n s mon ey lea vin g you r pocket a n d
goin g t o s omeon e els es pocket .

Toget h er PV=23.71, I/ Y=6%,N=2, a n d FV= -26.64055600 mea n t h is :

a t t ime zer o $23.71 flows fr om s omeon es pocket t o you r pocket ; in
ot h er wor ds , you bor r ow $23.71 fr om s omeon e.

you u s e t h e mon ey for 2 yea r s a n d a r e ch a r ged wit h 6% in t er es t
r a t e.

a t t h e en d of Yea r 2, $26.64055600 comes fr om you r pocket t o
s omeon e els es pocket . In ot h er wor ds , you pa y t h e len der
$26.64055600 a t t h e en d of Yea r 2. $26.64055600 is gr ea t er t h a n
$23.71 beca u s e it in clu des in t er es t pa ymen t .

(2) You ca n a ls o s et PV= - 23.71, I/ Y=6%,N=2. Th is will give you FV=
26.64055600. You len d $23.71 for 2 yea r s wit h 6% in t er es t . At t h e en d of
Yea r 2, you r eceive $26.64055600.

(3) Beca u s e t h er e a r e n o r egu la r pa ymen t s in t h is pr oblem, it does n ot
ma t t er wh et h er you s elect en d-of-per iod pa ymen t s by s et t in g 2
n d
END or
you s elect begin n in g-of-per iod pa ymen t s by s et t in g 2
n d
BGN.
143
http://actuary88.com
(4) Th e # of compou n din g per iods ca n be a fr a ct ion . If you a re t o
ca lcu la t e 23.71(1+6%)
5.6,
you s imply en t er t h e followin g:

PV=23.71, I/ Y=6%,N=5.6

You s h ou ld get FV= - 32.85824772

Annui t y

Proble m 1

Ca lcu la t e
10 6%
a
Me t hod 1 Us e TVM of BA II Plus / BA II Plus Profe s s ional
Procedure Keystroke Display
Set to display 8 decimal places.
Make sure that you set C/Y=1 and
P/Y=1. This is the golden rule. Never
break this rule under any
circumstances.
Clear TVM Worksheet
2
nd
[CLR TVM] 0.00000000
If your calculator screen
displays

0.00000000

Then its already in the
immediate annuity mode.
You do nothing.

Use the immediate annuity function
(rather than annuity due).

If last time when TVM was used and
TVM was in the annuity due mode,
you need to set it to the immediate
annuity.

Forgetting to do so gives you a wrong
result without you knowing it.
If your calculator screen
displays

BGN
0.00000000

Then its in the annuity due
mode. You need to set it to
the annuity due mode by:

2nd BGN 2nd Set
CE/C (This enables the
calculator to the standard
calculation mode)

END

0.00000000
Enter the level payment.
1 Payment Enter
PMT=1.00000000

(We enter a positive 1 to indicate
that we want to receive a payment
of $1.)
Enter # of payments
10 N Enter N=10.00000000
144
http://actuary88.com
Calculate PV when the interest rate is
zero. This is an extra step to double
check whether we entered the right
cash flow amounts and the correct # of
payments.

CPT PV PV= - 10.00000000
(The result is correct. The PV of
10 level payments of $1 @ i=0 is
10. )

The negative sign indicates cash
outgo. In other words, if we
spend $10 now to buy a 10 year
annuity immediate @ i=0, well
receive $1 per year.

Enter interest rate.
6 I/Y Enter
I/Y=6.00000000
Enter the # of compounding period
(can de a integer or fraction).

10 N Enter
N=10.00000000
Calculate the present value.
CPT PV
PV= - 7.36008705

So
10 6%
7.36008705 a =
Me t hod 2 Us e Cas h Flow Works he e t of BA II Plus / BA II Plus
Profe s s ional

Befor e u s in g CF Wor ks h eet , let s iden t ify a ll of t h e ca s h flows :

Time t 0 1 2 3 4 5 6 7 8 9 10
Cash flow $0 $1 $1 $1 $1 $1 $1 $1 $1 $1 $1
10 6%
? a =
Procedure Keystroke Display
Set to display 8 decimal places.
Use Cash Flow Worksheet
CF CF0=(old content)
Clear Worksheet
2
nd
[CLR WORK] CF0=0.00000000
Enter the cash flow at t=0. Because
the cash flow is zero, we dont
need to enter anything. Just
press the down arrow.
+
CF0=0.00000000
145
http://actuary88.com
+
C01 0.00000000
Enter the dollar amount of the level
payments.

1 Enter C01=1.00000000
+
F0= 1.00000000
(The default # is 1.)
Enter the # of level payments. We
have 10 level payments from t=1 to
t=10.
10 Enter F0= 10.00000000

Use NVP portion of Cash Flow
Worksheet.

NPV I=0.00000000
(The default interest rate is zero.)
+
NPV= 0.00000000
(The default # is 0.)
Calculate NPV when the interest rate
is zero. This is an extra step to double
check whether we entered the right
cash flow amounts and the correct # of
payments.

CPT NPV= 10.00000000
(The result is correct. The PV of
10 level payments of $1 @ i=0 is
10.)
|
I=0.00000000
(The default interest rate is zero.)

Enter the interest rate.

6 Enter I=6.00000000

+
NPV= 10.00000000
(This is the previous NPV we
calculated last time @ i=0.)
Calculate NPV when the interest rate
is zero. This is an extra step to double
check whether we entered the right
cash flow amounts and the correct # of
payments.
CPT NPV= 7.36008705

So
10 6%
7.36008705 a =
Proble m 2

Ca lcu la t e
10 6%
a
Solut ion

146
http://actuary88.com
Me t hod 1 Us e TVM Works he e t . Th e ca lcu la t ion pr ocedu r es a r e
iden t ica l t o t h e pr ocedu r e for ca lcu la t in g
10 6%
a , except t h a t we n eed t o
u s e t h e a n n u it y du e mode.

You s h ou ld get :
10 6%
7.80169227 a =
Me t hod 2 Us e CF Works he e t .

Time t 0 1 2 3 4 5 6 7 8 9
Cash flow $1 $1 $1 $1 $1 $1 $1 $1 $1 $1
10 6%
? a =
Th e ca lcu la t ion pr ocedu r es a r e iden t ica l t o t h e pr ocedu r e for ca lcu la t in g
10 6%
a , except t h a t we n eed t o en t er t h e followin g ca s h flows :

CF0=1; C01=1,F01=9.
You s h ou ld get :
10 6%
7.80169227 a =
Proble m 3 Ca lcu la t e
10 6%
s
Solut ion

Me t hod 1 Us e TVM

Dis pla y 8 decima l pla ces .
Set C/ Y=1, P/ Y=1.
Us e t h e immedia t e a n n u it y mode.
Set PMT=1, N=10.
CPT FV. You s h ou ld get FV= - 10 @ i =0. Th e r es u lt is cor r ect .
Set I/ Y=6. Pr es s : CPT, FV.
You s h ou ld get FV= - 13.18079494. So
10 6%
13.18079494 s =
147
http://actuary88.com
Me t hod 2 us e Cas h Flow Works he e t .

Time t 0 1 2 3 4 5 6 7 8 9 10
Cash flow $0 $1 $1 $1 $1 $1 $1 $1 $1 $1 $1
10 6%
? s =
Us e BA II Plus Profe s s ional
Keys t r okes
Procedure Keystroke Display
Set to display 8 decimal places.
Use Cash Flow Worksheet
CF CF0=(old content)
Clear Worksheet
2
nd
[CLR WORK] CF0=0.00000000
Enter the cash flow at t=0.
+
CF0=0.00000000
+
C01 0.00000000
Enter the dollar amount of the level
payments.

1 Enter C01=1.00000000
+
F01= 1.00000000
(The default # is 1.)
Enter the # of level payments. We
have 10 level payments from t=1 to
t=10.
10 Enter F01= 10.00000000

NPV I=0.00000000
(The default interest rate is zero.)
+
NPV= 0.00000000
(The default # is 0.)
Use FVP portion of Cash Flow
Worksheet.

+
NFV= 0.00000000
(The default net future value is
zero.)
Calculate NPV when the interest rate
is zero. This is an extra step to double
check whether we entered the right
cash flow amounts and the correct # of
payments.

CPT NPV= 10.00000000
(The result is correct. The NFV
of 10 level payments of $1 @ i=0
is 10.)
Enter the interest rate.
148
http://actuary88.com
NPV I=0.00000000
(The default interest rate is zero.)

6 Enter I=6.00000000

+
NPV= 10.00000000
(NPV is 10 @ i=0. Even though
we entered i=6%, we didnt press
CPT. If CPT is not pressed, BA II
Plus Professional does not update
the NPV using the latest interest
rate. It merely displays the old
NPV value.)
+
NFV= 10.00000000
(NFV is 10 @ i=0. Even though
we entered i=6%, we didnt press
CPT. If CPT is not pressed, BA II
Plus Professional does not update
the NFV using the latest interest
rate. It merely displays the old
NPV value.)
Calculate NPV when the interest rate
is zero. This is an extra step to double
check whether we entered the right
cash flow amounts and the correct # of
payments.

CPT NFV= 13.18079494 (BA II Plus
Professional calculates NFV
using the latest interest rate.)

So
10 6%
13.18079494 s =
Us e BA II Plus
BA II Plu s does n ot h a ve NFV. However , we ca n s t ill ca lcu la t e
10 6%
s u s in g
t h e followin g r ela t ion s h ip:

( ) 1
i i
n
n n
i s a = +
We h a ve
10 6%
7.36008705 a =
( ) ( )
10
6% 10
7.36008705 1.06 13.18079494 Is = =
Proble m 4

Ca lcu la t e
10 6%
s
149
http://actuary88.com
Solut ion

Me t hod 1 Us e TVM

Dis pla y 8 decima l pla ces .
Set C/ Y=1, P/ Y=1.
Us e t h e a n n u it y du e mode.
Set PMT=1, N=10.
CPT FV. You s h ou ld get FV= - 10 @ i =0. Th e r es u lt is cor r ect .
Set I/ Y=6. Pr es s : CPT, FV.
You s h ou ld get FV= - 13.97164264. So
10 6%
13.97164264 s =
Me t hod 2 us e Cas h Flow Works he e t .

BA II Plus Profe s s ional
Time t 0 1 2 3 4 5 6 7 8 9 10
Cash flow $1 $1 $1 $1 $1 $1 $1 $1 $1 $1 $0
10 6%
? s =
Plea s e n ot e BA II Plus Profe s s ional Cas h Flow Works he e t always
ac c umulat e s c as h flows t o t he final payme nt t ime . In or der t o
ca lcu la t e
10 6%
s (wh ich a ccu mu la t es va lu e t o t=10), we n eeded t o a dd a n
a ddit ion a l ca s h flow of zer o a t t =10. Th is t ells BA II Plu s Pr ofes s ion a l
Ca s h Flow Wor ks h eet t o u s e t=10 a s t h e en din g t ime t o a ccu mu la t e ca s h
flows . If we don t a dd t h is a ddit ion a l ca s h flow of zer o t o t=10, BA II Plu s
will ca lcu la t e t h e a ccu mu la t ed va lu e t o t=9.

Keys t r okes BA II Plu s Pr ofes s ion a l on ly (n ot for BA II Plu s ):

Procedure Keystroke Display
Set to display 8 decimal places.
Use Cash Flow Worksheet
CF CF0=(old content)
Clear Worksheet
2
nd
[CLR WORK] CF0=0.00000000
150
http://actuary88.com
Enter the cash flow at t=0.
1 Enter
CF0=1.00000000
+
C01 0.00000000
Enter the dollar amount of the
payments.

1 Enter C01=1.00000000
+
F0= 1.00000000
(The default # is 1.)
Enter the # of level payments. We
have 9 level payments from t=1 to
t=9.
9 Enter F0= 9.00000000

+
C02 0.00000000
(The default # is zero. At this
point, BA II Plus Professional
assumes that cash flows end at
t=9 and theres no cash flow at
t=10. You can verify this by
pressing + . If you press + ,
youll see that F02 is zero. In
other words, the # of cash flow at
t=10 is zero.)
Enter zero cash flow at t=10,
increasing the # of time periods by
one.

Enter
If we omit this step, BA II
Plus Professional will
accumulate values to t=9 and
gives us the final result of
NPV=13.18079494. You can
verify this yourself.

C02 =0.00000000

Once Enter is pressed, the
display C02 0.00000000
becomes C02 =0.00000000.

By pressing Enter, you tell BA
II Plus Professional to increase
the time period by one.

You can verify this by pressing
+ . If you press + , youll see
that F02=1. In other words, the #
of cash flow at t=10 is one.

NPV I=0.00000000
(The default interest rate is zero.)
+
NPV= 0.00000000
(The default # is 0.)
Use FVP portion of Cash Flow
Worksheet.

+
NFV= 0.00000000
(The default net future value is
zero.)
151
http://actuary88.com
Calculate NPV when the interest rate
is zero. This is an extra step to double
check whether we entered the right
cash flow amounts and the correct # of
payments.

CPT NPV= 10.00000000
(The result is correct. The NFV
of 10 level payments of $1 @ i=0
is 10.)
NPV I=0.00000000
(The default interest rate is zero.)

Enter the interest rate.

6 Enter I=6.00000000

+
NPV= 10.00000000
(NPV is 10 @ i=0. Even though
we entered i=6%, we didnt press
CPT. If CPT is not pressed, BA II
Plus Professional does not update
the NPV using the latest interest
rate. It merely displays the old
NPV value.)
+
NFV= 10.00000000
(NFV is 10 @ i=0. Even though
we entered i=6%, we didnt press
CPT. If CPT is not pressed, BA II
Plus Professional does not update
the NFV using the latest interest
rate. It merely displays the old
NPV value.)
Calculate NPV when the interest rate
is zero. This is an extra step to double
check whether we entered the right
cash flow amounts and the correct # of
payments.

CPT NFV= 13.97164264 (BA II Plus
Professional calculates NFV
using the latest interest rate.)
So
10 6%
13.97164264 s =
If we omit t h e s t ep of en t er in g t h e ca s h flow of zer o a t t =10, BA II Plu s
Pr ofes s ion a l will give u s NFV=13.18079494. We ca n ver ify t h a t

( )
1
13.18079494 13.97164264 1.06

=
You s ee t h a t 13.18079494 is t h e a ccu mu la t ed va lu e a t t =9, wh ile
13.97164264 is t h e a ccu mu la t ed va lu e a t t =10.
152
http://actuary88.com
How ca n we ca lcu la t e t h e NFV if we n eed t o a ccu mu la t e t h e s a me ca s h
flows t o t=11? Now t h e ca s h flows dia gr a m is :

Time t 0 1 2 3 4 5 6 7 8 9 10 11

Cash flow $1 $1 $1 $1 $1 $1 $1 $1 $1 $1 $0 $0

NFV=?

We s imply s et C02=0, F02=2. Th is will move t h e en din g t ime fr om t o t =9
t o t =11. Well get t h e r es u lt t h a t NPF = 14.80994120.

We ca n ver ify t h a t ( ) 14.80994120 13.97164264 1.06 = .
Us e BA II Plus
BA II Plu s does n ot h a ve NFV. However , we ca n s t ill ca lcu la t e
10 6%
s u s in g
t h e followin g r ela t ion s h ip:

( ) 1
i i
n
n n
s i a = +
We h a ve
10 6%
7.80169227 a =
( ) ( )
6% 6%
10 10
10 10
1.06 7.80169227 1.06 13.97164263 s a = = =
Loan/ bond amort i zat ion

In BA II Plu s / BA II Plu s Pr ofes s ion a l, TVM Wor ks h eet is a u t oma t ica lly
t ied t o Amor t iza t ion Wor ks h eet . For a given loa n or bon d, if you fin d PV
(wh ich is t h e pr in cipa l of a loa n ), PMT (r egu la r pa ymen t ), N (# of
pa ymen t s ) a n d I/ Y (t h e in t er es t r a t e) t h r ou gh TVM Wor ks h eet ,
Amor t iza t ion Wor ks h eet ca n gen er a t e a n a mor t iza t ion s ch edu le for you ,
s plit t in g ea ch pa ymen t in t o pr in cipa l a n d in t er es t .

153
http://actuary88.com
Proble m 1 (#10, Sa mple FM)

A 10,000 pa r va lu e 10-yea r bon d wit h 8% a n n u a l cou pon s is bou gh t a t
pr emiu m t o yield a n a n n u a l effect ive r a t e of 6%.

Ca lcu la t e t h e in t er es t por t ion of t h e 7
t h
pa ymen t .

(A) 632 (B) 642 (C) 651 (D) 660 (E) 667

Solut ion

On e key t h in g t o r emember is t h a t a bon d is a loa n . Wh en you bu y t h is
$10,000 pa r 10 yea r bon d wit h 8% a n n u a l cou pon s yieldin g 6%, you
len d you r mon ey t o wh oever is s u ed t h e bon d. Th e mon ey you r len d
(pr in cipa l) is t h e pr es en t va lu e of a ll t h e fu t u r e ca s h flows (10 cou pon s of
$800 ea ch fr om t =1 t o t =10 plu s a fin a l ca s h flow of $10,000 a t t =10)
dis cou n t ed a t a n a n n u a l effect ive 6%. An d t h e bor r ower (bon d is s u er )
pa ys ba ck you r loa n t h r ou gh 10 cou pon s of $800 ea ch fr om t =1 t o t =10
plu s a fin a l ca s h flow of $10,000 a t t =10.

We fir s t dr a w a ca s h flow dia gr a m:

Time t 0 1 2 3 4 5 6 7 8 9 10
Cash flow $800 $800 $800 $800 $800 $800 $800 $800 $800 $800
$10,000
For BA II Plu s / BA II Plu s Pr ofes s ion a l Amor t iza t ion Wor ks h eet t o
gen er a t e a n a mor t iza t ion s ch edu le, we fir s t n eed t o ca lcu la t e PV. Let s
ca lcu la t e PV of t h e bon d.

Pr ocedu r es for u s in g TVM:
Dis pla y 8 decima l pla ces .
Set C/ Y=1, P/ Y=1.
Us e t h e a n n u it y immedia t e mode.
Set PMT=800, N=10, FV=10,000. By ma kin g PMT a n d FV pos it ive,
we a r e get t in g 10 level pa ymen t s of $800 ea ch yea r a n d a fin a l
pa ymen t of $10,000 a t t =10. So we bou gh t t h e bon d a n d s h ou ld
r eceive ca s h flows in t h e fu t u r e. Th is will gen er a t e a n ega t ive PV,
154
http://actuary88.com
wh ich is ou r pu r ch a s e pr ice of t h e bon d; we pa y n ow t o get ca s h
flows in t h e fu t u r e.
Alt er n a t ively, we ca n ma ke PMT= - 800 a n d FV= - 10,000; we s old
t h e bon d. Well get a pos it ive PV, wh ich is ou r s ellin g pr ice of t h e
bon d; we get ca s h n ow bu t pa y ca s h flows in t h e fu t u r e. Eit h er wa y
is fin e a s lon g a s we ma ke PMT a n d FV h a ve t h e s a me s ign s .
CPT PV. Th is ca lcu la t es PV @ i =0. We s h ou ld get PV= - 18,000.
Th e r es u lt is cor r ect . If t h e in t er es t r a t e is zer o, PV is ju s t t h e s u m
of a ll ca s h flows . PV=800(10)+10,000=18,000.
Set I/ Y=6. Pr es s : CPT, PV.
We s h ou ld get PV= - 11,472.01741.
Now Amor t iza t ion Wor ks h eet is r ea dy t o gen er a t e a n a mor t iza t ion
s ch edu le for u s , s plit t in g ea ch pa ymen t in t o pr in cipa l a n d in t er es t .

Procedure Keystroke Display
Use Amortization Worksheet.
2nd Amort P1=old content

7 Enter
P1=7.00000000

So the payment to be split begins
at t =7.
+
P2= old content.
Tell the calculator that we are
interested in the 7
th
payment.

7 Enter
P2=7.00000000

So the payment to be split ends at
t =7.
Find the outstanding balance after the
7
th
payment

+
BAL= - 10,534.60239

BA II Plus/BA II Plus
Professional always calculates the
outstanding balance after the P2
level payment is made.
So the outstanding balance
AFTER the 7
th
payment and
immediately before the 8
th

payment is $10,534.60239. Since
PV is negative, the outstanding
balance should also be negative.
155
http://actuary88.com
Find the principal portion of the 7
th

level payment.

+
PRN= 158.4187327

BA II Plus/BA II Plus
Professional splits the total
payments starting from P1 and
ending with P2 into the principal
repayment and interest payment.
Because we set P1=P2=7, the
calculator splits only the 7
th
payment into principal and
interest.

Of the $800 payment at t=7,
$158.42 is the repayment of the
principal. A positive $158.42
means that we receive $158.42.
This makes sense. Our PV is
negative (we lent our money at
t=0). As a result, we will receive
repayment of our principal.
Find the interest portion of the 7
th

payment.

+
INT= 641.5812674

Of the $800 payment at t=7,
$641.58 is the interest payment.
A positive $641.58 means that we
receive $641.58.
So t h e in t er es t por t ion of t h e 7
t h
pa ymen t is a bou t $642.

We ca n ver ify t h a t t h e r es u lt s a r e cor r ect :

Time t 0 1 2 3 4 5 6 7 8 9 10
Cash flow $800 $800 $800 $800 $800 $800 $800 $800 $800 $800
$10,000
Th e ou t s t a n din g pr in cipa l
immedia t e a ft er t h e 6
t h
pa ymen t
( )
4
4 6%
800 10, 000 1.06 10, 693.02112 a

+ =
To ver ify, in t h e a mor t iza t ion
wor ks h eet , If you s et P1=P2=6, you
s h ou ld get BAL= - 10,693.02112.
Th e in t er es t a ccr u ed fr om t =7 t o
t =8
( ) 10, 693.02112 6% 641.58 =
Th e pr in cipa l por t ion of t h e 7
t h

pa ymen t
800 641.58 158.42 =
Plea s e n ot e t h a t Amor t iza t ion Wor ks h eet u s es da t a you en t er ed or
ca lcu la t ed in TVM Wor ks h eet . Wh en ever you u pda t e TVM Wor ks h eet ,
156
http://actuary88.com
Amor t iza t ion Wor ks h eet is a u t oma t ica lly u pda t ed. As a r es u lt , you don t
n eed t o u s e 2
n d
CLR Wor k t o clea r Amor t iza t ion Wor ks h eet . Amor t iza t ion
Wor ks h eet is a lwa ys in s yn c wit h TMV Wor ks h eet . As lon g a s t h e da t a in
TVM Wor ks h eet is cor r ect , Amor t iza t ion Wor ks h eet will gen er a t e t h e
cor r ect a mor t iza t ion s ch edu le.

Addit ional c alc ulat ions on t his proble m:

Wh a t s t h e ou t s t a n din g ba la n ce immedia t ely BEFORE t h e 7
t h

pa ymen t ? Remember t h a t BA II Plu s / BA II Plu s Pr ofes s ion a l
Amor t iza t ion Wor ks h eet a lwa ys ca lcu la t es t h e ou t s t a n din g ba la n ce
a ft er t h e P2 level pa ymen t is ma de. So fir s t we fin d t h e ou t s t a n din g
ba la n ce immedia t ely a ft er t h e 7
t h
pa ymen t is ma de. So we s et
P2=7. Wh a t a bou t P1? We ca n s et P1=1,2,3,4,5, 6, or 7. In ot h er
wor ds , P1 n eeds t o be a pos it ive in t eger equ a l t o or s ma ller t h a n
P2. We s h ou ld get BAL= - 10.534.60239. Immedia t ely a ft er t h e 7
t h

pa ymen t is ma de, t h e ou t s t a n din g ba la n ce is 10.534.60239. So t h e
ou t s t a n din g ba la n ce immedia t ely befor e t h e 7
t h
pa ymen t is ma de is
10.534.60239+800=11,334.60239. (Her es a n ot h er a ppr oa ch . We
a lr ea dy kn ow t h a t t h e ou t s t a n din g loa n ba la n ce immedia t ely a ft er
t h e 6
t h
pa ymen t is 10,693.02112. Accu mu la t in g t h is a mou n t wit h
in t er es t for on e yea r is ( ) 10, 693.02112 1.06 11, 334.60239 = . So t h e
ou t s t a n din g loa n ba la n ce immedia t ely befor e t h e 7
t h
pa ymen t is
11,334.6023.)

Wh a t s t h e ou t s t a n din g ba la n ce a ft er t h e fin a l cou pon is pa id a t
t =10? Wit h ou t u s in g a n y ca lcu la t or s , we kn ow t h a t t h e
ou t s t a n din g ba la n ce mu s t be $10,000 a ft er t h e 10
t h
cou pon of
$800 is pa id. Will Amor t iza t ion Wor ks h eet pr odu ce t h is r es u lt ?
Let s ch eck. Set P2=10; P1 ca n be a n y pos it ive in t eger n o gr ea t er
t h a n 10. We get BAL= - 10,000.

Wh a t s t h e t ot a l pr in cipa l a n d in t er es t pa id by t h e bon d is s u er
du r in g t h e life of t h e bon d? Remember BA II Plu s / BA II Plu s
Pr ofes s ion a l a lwa ys s plit s t h e pa ymen t s s t a r t in g fr om P1 a n d
en din g wit h P2 in t o pr in cipa l a n d in t er es t . Let s s et P1=1 a n d
P2=10. Well get PRN=1,472.017410 a n d INT=6,527.982590. Let s
ch eck:

Th e t ot a l r epa ymen t of t h e loa n
(s imple s u m of a ll t h e fu t u r e
ca s h flows )

10(800)+10,000=18,000
Th e pr in cipa l of t h e loa n (t h e
in it ia l pr ice of t h e bon d)
PV=11,472.01741=PRN (OK)
157
http://actuary88.com
How t o s plit in t o pr in cipa l a n d in t er es t t h e t ot a l pa ymen t s ma de
du r in g t h e 2
n d
, 3
r d
, 4
t h
, a n d 5
t h
pa ymen t s ? Ea s y! Set P1=2 a n d
P2=5. We get PRN=517.8656978 a n d INT=2,682.134302. So ou t of
t h e t ot a l pa ymen t s ma de du r in g t h e 2
n d
, 3
r d
, 4
t h
, a n d 5
t h
pa ymen t s ,
t h e pr in cipa l por t ion is 517.8656978; t h e in t er es t por t ion is
2,682.134302. Let s ch eck:

Time t 0 1 2 3 4 5 6 7 8 9 10
Cash flow $800 $800 $800 $800 $800 $800 $800 $800 $800 $800
$10,000
Th e ou t s t a n din g loa n
immedia t ely AFTER t h e 1
s t

pa ymen t is ma de
( )
9
9 6%
800 10, 000 1.06 11, 360.33845 a

+ =
To ver ify, in t h e a mor t iza t ion
wor ks h eet , s et P1=1 a n d P2=1. You
s h ou ld get BAL= - 11,360.33845.
Th e ou t s t a n din g loa n
immedia t ely AFTER t h e 5
t h

pa ymen t is ma de
( )
5
5 6%
800 10, 000 1.06 10, 842.47276 a

+ =
To ver ify, in t h e a mor t iza t ion
wor ks h eet , s et P1=5 a n d P2=5. You
s h ou ld get BAL= - 10,842.47276
Redu ct ion of pr in cipa l du e t o
t h e 2
n d
, 3
r d
, 4
t h
, a n d 5
t h

pa ymen t s
11, 360.33845 10,842.47276 517.8656979 =
(Th e r es u lt ma t ch es
PRN=517.8656978. Th e s ligh t
differ en ce is du e t o r ou n din g)
Tot a l r epa ymen t ma de in t h e
2
n d
, 3
r d
, 4
t h
, a n d 5
t h
pa ymen t s
800(4)=3,200
Tot a l in t er es t pa id in t h e 2
n d
,
3
r d
, 4
t h
, a n d 5
t h
pa ymen t s
3,200 517.8656979=2,682.134302

Th e r es u lt ma t ch es INT=2,682.134302
Th e t ot a l in t er es t pa id du r in g
t h e life of t h e bon d
18,000 -11,470.01741
=6,57.982590 (OK)
158
http://actuary88.com
Compare Cas h Flow Works he e t wit h TVM
Works he e t

Pr os of u s in g Ca s h Flow Wor ks h eet over TVM:
Avoid in a dver t en t ly u s in g C/ Y=12 a n d P/ Y=12
Avoid pa in fu l s wit ch in g bet ween t h e a n n u it y du e mode a n d t h e
a n n u it y immedia t e mode
Ha n dle level a n d n on -level pa ymen t s
Con s of u s in g Ca s h Flow Wor ks h eet over TVM
A ca n dida t e ca n for get t h a t t h e 1
s t
ca s h flow in Ca s h Flow
Wor ks h eet is t h e CF0 (wh ich t a kes pla ce a t t=0), n ot C01 (wh ich
t a kes pla ce a t t=1).
A ca n dida t e n eeds t o ca r efu lly t r a ck down t h e t imin g of ea ch ca s h
flow.
TVM is a u t oma t ica lly t ied t o Amor t iza t ion Wor ks h eet a n d ca n
gen er a t e a n a mor t iza t ion s ch edu le; Ca s h Flow Wor ks h eet is NOT
t ied t o Amor t iza t ion Wor ks h eet a n d ca n NOT gen er a t e a n
a mor t iza t ion s ch edu le.
I r ecommen d t h a t you ma s t er bot h met h ods . For a n on -a mor t iza t ion
exa m pr oblem, you ca n u s e bot h met h ods for t h e s a me pr oblem a n d
dou ble ch eck you r ca lcu la t ion s . If you a re good a t u s in g BA II Plu s / BA II
Plu s Pr ofes s ion a l, ea ch met h od t a kes you on ly a bou t 10 s econ ds . You
s h ou ld h a ve t ime t o u s e bot h met h ods for t h e s a me pr oblem.

Inc re as ing annuit y

Proble m 1
Ca lcu la t e
( )
10 5%
Ia
Solut ion

Time t 0 1 2 3 4 5 6 7 8 9 10
Cash flow $0 $1 $2 $3 $4 $5 $6 $7 $8 $9 $10
( )
10 5%
Ia
Well u s e BA II Plu s / BA II Plu s Pr ofes s ion a l Ca s h Flow Wor ks h eet .
159
http://actuary88.com
En t er t h e followin g:

CF0=0; C01=1; C02=2; C03=3; C04=4; C05=5;
C06=6; C07=7; C08=8; C09=9; C010=10.
You don t n eed t o en t er F01=1, F02=1, ,F10=1. BA II Plu s / BA II Plu s
Pr ofes s ion a l will a u t oma t ica lly s et t h em t o on e.

Th is is h ow BA II Plu s / BA II Plu s Pr ofes s ion a l s et s t h e # of level ca s h
flows :

If a ca s h flow is zer o, t h e defa u lt # of level ca s h flows is zer o.
If a ca s h flow is n one zer o, t h e defa u lt # of level ca s h flows is 1.
If, for a n y ca s h flow, a u s er does n ot s pecifica lly en t er t h e # of level
ca s h flows , Ca s h Flow Wor ks h eet u s es t h e defa u lt # of level ca s h
flows .

Next , ca lcu la t e NPV. You s h ou ld get 55. Th is is NPV @ i=0. We ca n ver ify
t h is is cor r ect :
( ) ( )
1
1 2 3 ... 10 10 11 55
2
+ + + + = =
We h a ve cor r ect ly en t er ed t h e ca s h flow a mou n t s a n d t h e # of ca s h flows .
Next , s et I=6. We get NPV=36.96240842.
Let s ch eck. Well u s e t h e for mu la :

( )
n
n
n
nv
i
a
Ia

=

( )
( )
10
6%
6%
10
10
10 1.06
7.80169227 5.58394777
36.96240842
6% 6%
a
Ia


= = =

Wh y bot h er u s in g t h e ca lcu la t or wh en a for mu la wor ks fin e? Th e


ca lcu la t or r edu ces a complex in cr ea s in g a n n u it y for mu la t o s imple
keys t r okes . As lon g a s you en t er t h e cor r ect da t a , t h e ca lcu la t or will
gen er a t e t h e r es u lt 100% r igh t . In con t r as t , if you u s e t h e in cr ea s in g
a n n u it y for mu la , you migh t mis ca lcu la t e.

Th e s ecr et t o doin g er r or -fr ee ca lcu la t ion s for a complex pr oblem in t h e
h ea t of t h e exa m is t o r edu ce a complex pr oblem in t o a s imple mech a n ic
s olu t ion . We ma y n ot a lwa ys be a ble t o do s o. However , if s ome pr oblems
160
http://actuary88.com
h a ve mech a n ic s olu t ion s , we pr efer t o give ou r br a in a r es t a n d u s e
mech a n ic s olu t ion s t o s olve t h e complex pr oblems .

Kn owin g t h a t BA II Plu s / BA II Plu s Pr ofes s ion a l ca n ca lcu la t e a n
in cr ea s in g a n n u it y for u s , do we s t ill n eed t o memor ize t h e in cr ea s in g
a n n u it y? Yes . SOA ca n a lwa ys s et u p a pr oblem in s u ch wa y t h a t a
mech a n ic s olu t ion is impos s ible a n d s ome a mou n t of t h in kin g is n eeded.
As a r es u lt , well s t ill n eed t o memor ize t h e in cr ea s in g a n n u it y for mu la .

Proble m 2
Ca lcu la t e
( )
10 5%
Ia
Solut ion

Time t 0 1 2 3 4 5 6 7 8 9
Cash flow $1 $2 $3 $4 $5 $6 $7 $8 $9 $10
( )
10 5%
Ia
Well u s e BA II Plu s / BA II Plu s Pr ofes s ion a l Ca s h Flow Wor ks h eet .

En t er t h e followin g:

CF0=1; C01=2; C02=3; C03=4; C04=5; C05=6;
C06=7; C07=8; C08=9; C09=10.
Ca lcu la t e NPV. We get NPV=55 @ i=0 (OK).

Set I=6. We get NPV=39.18015293.
Let s ch eck. Well u s e t h e for mu la :

( ) ( )
n
n
n n
nv
i
d d
a
Ia Ia

= =


( )
1 6% 10
0.06
36.96240842 39.18015293
1 1.06
Ia

= =


161
http://actuary88.com
Proble m 3

Ca lcu la t e
( )
10 5%
Is
Solut ion

Time t 0 1 2 3 4 5 6 7 8 9 10
Cash flow $0 $1 $2 $3 $4 $5 $6 $7 $8 $9 $10
( )
10 5%
Is
BA II Plu s Pr ofes s iona l Ca s h Flow Wor ks h eet :
En t er t h e followin g:

CF0=0; C01=1; C02=2; C03=3; C04=4; C05=5;
C06=6; C07=7; C08=8; C09=9; C010=10.

Next , ca lcu la t e NFV. You s h ou ld get NFV=55 @ i=0 (OK).

Fin a lly, s et I=6. We get NFV=66.19404398.

Let s ch eck. Well u s e t h e for mu la :
( ) ( ) ( ) 1
n
n n
i Is Ia = +
( ) ( )
10
6% 10
36.96240842 1.06 66.19404398 Is = =
Us e BA II Plu s Ca s h Flow Wor ks h eet (wh ich does n t h a ve t h e FPV
fu n ct ion )
We u s e t h e for mu la : ( ) ( ) ( ) 1
n
n n
i Is Ia = +
Us in g Ca s h Flow Wor ks h eet , we get : ( )
6% 10
36.96240842 NPV Ia = =
( ) ( )
10
6% 10
36.96240842 1.06 66.19404398 Is = =
162
http://actuary88.com
Proble m 4

Ca lcu la t e
( )
10 5%
Is
Solut ion

Time t 0 1 2 3 4 5 6 7 8 9 10
Cash flow $1 $2 $3 $4 $5 $6 $7 $8 $9 $10 $0
( )
10 5%
Is
Us e BA II Plu s Pr ofes s ion a l Ca s h Flow Wor ks h eet .
En t er t h e followin g:

CF0=1; C01=2; C02=3; C03=4; C04=5; C05=6;
C06=7; C07=8; C08=9; C09=10.
C10=0;F10=1
(Th is t ells BA II Plu s Pr ofes s ion a l t o a ccu mu la t e va lu e a t t o t =10.)
Ca lcu la t e NPV. We get NFV=55 @ i=0 (OK).
Set I=6. We get NFV=70.16568662.
Let s ch eck:
( ) ( ) ( ) 1
i n
n
i n
i Is Ia = +
( ) ( ) ( ) ( )
6% 10
10 10
6% 10
1.06 39.18015293 1.06 70.16568662 Is Ia = = =
Us e BA II Plu s Ca s h Flow Wor ks h eet .
Fir s t , we ca lcu la t e ( )
6% 10
39.18015293 Ia =
( ) ( ) ( ) ( )
6% 10
10 10
6% 10
1.06 39.18015293 1.06 70.16568662 Is Ia = = =
163
http://actuary88.com
Proble m 5

Ca lcu la t e
( ) 12
1 6%
a
Solut ion

Let s fir s t dr a w a ca s h flow dia gr a m:

Time t 0 1 2 3 4 5 6 7 8 9 10 11 12
(Months)
Cash flow $0 $
1
12
$
1
12
$
1
12
$
1
12
$
1
12
$
1
12
$
1
12
$
1
12
$
1
12
$
1
12
$
1
12
$
1
12
( ) 12
1 6%
a
Th is is a ca s e wh er e level pa ymen t s of
1
12
a r e ma de mon t h ly yet t h e
in t er es t r a t e of 6% is compou n din g a n n u a lly. Wh en t h e pa ymen t s
fr equ en cy a n d t h e compou n din g fr equ en cy do n ot ma t ch , we always u s e
t h e pa ymen t fr equ en cy a s t h e compou n din g per iod for ou r ca lcu la t ion s
in BA II Plu s / BA II Plu s Pr ofes s ion a l. Remember t h is r u le. Never devia t e
fr om t h is r u le.

Let s con ver t t h e a n n u a l in t er es t r a t e t o t h e mon t h ly in t er es t r a t e:

( )
1
12
12
1 1.06 1.06 1 0.48675506% i i + = = =
Th ou gh h er e I explicit ly wr it e t h e mon t h ly in t er es t r a t e a s 0.48675506%,
wh en you s olve t h is pr oblem in t h e exa m, you ca n ca lcu la t e t h e mon t h ly
in t er es t r a t e bu t s t or e t h e r es u lt in on e of t h e ca lcu la t or s memor ies (we
t a lked a bou t t h is befor e). Th is elimin a t es t h e er r or -pr on e pr oces s of
t r a n s fer r in g a lon g decima l n u mber ba ck a n d for t h bet ween you r
ca lcu la t or a n d t h e s cr a p pa per .

Next , well ch a n ge t h e complex a n n u it y
( ) 12
1 6%
a t o a s t a n da r d a n n u it y:
( ) 12
1 6% 12 0.48675506%
1
12
i
a a
=
=
Us in g TVM Wor ks h eet or Ca s h Flow Wor ks h eet , we ca n ea s ily ca lcu la t e:
164
http://actuary88.com
12 0.48675506%
11.62880032
i
a
=
=
( ) 12
1 6% 12 0.48675506%
1
0.96906669
12
i
a a
=
= =
You ca n u s e t h e s a me met h od a n d ca lcu la t e
( ) m
n i
a ,
( ) m
n i
a ,
( ) m
n i
s ,
( ) m
n i
s :
( )
1
m
n i j mn
m
a a = ,
( )
1
m
n i j mn
m
a a = ,
( )
1
m
n i j mn
s s
m
= ,
( )
1
m
n i j mn
m
s s =
Wh er e ( )
1
1 1
m
j i = +
Proble m 6

Ca lcu la t e
( ) 12
1
a , wh er e t h e in t er es t r a t e is
( ) 12
6% i =
Solut ion

Let s fir s t dr a w a ca s h flow dia gr a m:

Time t 0 1 2 3 4 5 6 7 8 9 10 11 12
(Months)
Cash flow $0 $
1
12
$
1
12
$
1
12
$
1
12
$
1
12
$
1
12
$
1
12
$
1
12
$
1
12
$
1
12
$
1
12
$
1
12
( ) 12
1
a
In t h is pr oblem, t h e pa ymen t fr equ en cy is mon t h ly a n d t h e in t er es t r a t e
given is t h e n omin a l r a t e compou n din g mon t h ly. Wh en t h e fr equ en cy of
level pa ymen t s ma t ch es t h e fr equ en cy by wh ich a n omin a l in t er es t
compou n ds , BA II Plu s / BA II Plu s Pr ofes s ion a l TVM Wor ks h eet h a s a
s h or t cu t wa y t o ca lcu la t e t h e a n n u it y va lu e. However , n ever u s e t h is
s h or t cu t ; it ca u s es mor e t r ou bles t h a n good.
Let s fir s t go t h r ou gh t h e s h or t cu t in TVM Wor ks h eet :

165
http://actuary88.com
Da n ger ou s pr ocedu r e t o ca lcu la t e
( ) 12
1
a :
Dis pla y 8 decima l pla ces .
Set C/ Y=12 a n d P/ Y=12. (C=compou n d. C/ Y=12 mea n s t h a t
t h e in t er es t r a t e compou n ds 12 t imes in a yea r ; P=pa ymen t .
P/ Y=12 mea n s pa yin g 12 t imes in a yea r ).
Us e t h e a n n u it y immedia t e mode.
Set PMT=1/ 12, N=12.
CPT PV. Th is ca lcu la t es PV @ i =0. We s h ou ld get PV= - 1 (OK).
Set I/ Y=6 (h er e we en t er t h e n omin a l in t er es t r a t e in s t ea d of
con ver t in g t h e n omin a l r a t e t o t h e mon t h ly in t er es t r a t e).
CPT PV. We s h ou ld get : PV= - 0.96824434
Th ou gh it s r es u lt is cor r ect , t h is pr ocedu r e ch a n ges t h e s a fe s et t in g of
C/ Y=1 a n d P/ Y=1 t o a da n ger ou s s et t in g of C/ Y=12 a n d P/ Y=12. If you
s et C/ Y=12 a n d P/ Y=12 bu t for get t o r es et t o t h e s a fe s et t in g of C/ Y=1
a n d P/ Y=1, wh en you en t er a n in t er es t r a t e in TMV, TVM will t r ea t t h is
in t er es t r a t e a s t h e n omin a l in t er es t r a t e compou n din g mon t h ly. You
migh t s a y, Th is is OK. Ill r emember t o ch a n ge C/ Y=12 a n d P/ Y=12
ba ck t o C/ Y=1 a n d P/ Y=1. However , in t h e h ea t of t h e exa m, it s ver y
ea s y t o for get t o r es et C/ Y=1 a n d P/ Y=1. If you for get t o r es et C/ Y=1 a n d
P/ Y=1, a ll you r a n n u it y ca lcu la t ion s wh er e a n a n n u a l effect ive in t er es t
r a t e is en t er ed will be wr on g. For t h is r ea s on , a lwa ys s t ick t o t h e s a fe
s et t in g C/ Y=1 a n d P/ Y=1. Never s et C/ Y=12 a n d P/ Y=12.

Th e s a fe pr ocedu r e t o ca lcu la t e
( ) 12
1
a
Dis pla y 8 decima l pla ces .
Set C/ Y=1 a n d P/ Y=1.
Us e t h e a n n u it y immedia t e mode.
Set PMT=1/ 12, N=12.
CPT PV. Th is ca lcu la t es PV @ i =0. We s h ou ld get PV= - 1 (OK).
Set I/ Y=6/ 12=0.5 (we s imply en t er t h e mon t h ly in t er es t r a t e).
CPT PV. We s h ou ld get : PV= - 0.96824434
166
http://actuary88.com
Compre he ns ive c alc ulat or e xe rc is e

Proble m 1
A loa n of $100,000 bor r owed a t 6% a n n u a l effect ive is r epa id by level
mon t h ly pa ymen t s in a dva n ce over t h e n ext 30 yea r . Aft er 10 yea r s , t h e
ou t s t a n din g ba la n ce of t h e loa n is r efin a n ced a t 4% a n n u a l effect ive a n d
is pa id by level mon t h ly pa ymen t s in a dva n ce over 20 yea r s .

Ca lcu la t e:
Th e mon t h ly pa ymen t of t h e or igin a l loa n .
Th e pr in cipa l por t ion a n d t h e in t er es t por t ion of t h e 37
t h
pa ymen t .
Th e mon t h ly pa ymen t of t h e r efin a n ced loa n .
Th e a ccu mu la t ed va lu e of t h e r edu ct ion in mon t h ly pa ymen t s
in ves t ed a t 4% a n n u a l effect ive.

Solut ion

Find t he mont hly payme nt of t he original loan

Time t 0 1 2 357 358 359 360
(Month)
Y Y Y Y Y Y Y
360 Months

360 j
Y a
Keys t r okes for TVM:

Dis pla y 8 decima l pla ces .
Set C/ Y=1 a n d P/ Y=1.
Us e t h e a n n u it y du e mode.
Set N=360, PV=100,000.
167
http://actuary88.com
CPT PMT. Th is ca lcu la t es PMT @ i =0. We s h ou ld get PMT= -
277.7777778. Ch eck: 100,000/ 360277.7777778. OK.
Set I/ Y=
1
12
100 1.06 1
(

(

. Remember t o mu lt iple t h e in t er es t r a t e
by 100 (ex. en t er 6 if t h e in t er es t r a t e is 6%).
CPT PMT. We s h ou ld get : PMT= - 586.5155230
Find t he princ ipal port ion and t he int e re s t port ion of t he 3 7
t h

payme nt

Th e 37
t h
pa ymen t is t h e 1
s t
pa ymen t in t h e 4
t h
yea r .

Time t 0 1 2 357 358 359 360
(Month)
Y Y Y Y Y Y Y
360 Months

360 j
Y a
Keys t r okes for TVM:
If you don t clea r TVM, TVM r emember s a ll of t h e va lu es you en t er ed la s t
t ime a n d t h e va lu es it ca lcu la t ed la s t t ime. So you don t n eed t o r een t er
a n yt h in g. You ca n s imply pick u p wh er e you left off wit h TVM.
Keys t r okes for a ddit ion a l ca lcu la t ion s :
En t er 2
n d
AMORT (t h is a ct iva t es t h e a mor t iza t ion wor ks h eet )
En t er P1=37, P2=37 (t h is t ells t h e ca lcu la t or t o look a t t h e 37
t h
pa ymen t )
En t er + , you ll s ee:
BAL=95,386.57701 (Th is is t h e ou t s t a n din g loa n ba la n ce AFTER t h e 37
t h

pa ymen t is ma de. Beca u s e you bor r owed t h e PV of fu t u r e ca s h flows , t h e
PV is a ca s h in flow t o you . So it s pos it ive. )
En t er + , you ll s ee:
PRN= - 121.6245226 (Th is is t h e pr in cipa l por t ion of t h e 37
t h
pa ymen t .
Th e n ega t ive s ign in dica t es ca s h ou t flow; t h is a mou n t is you r r epa ymen t
of t h e pr in cipa l in you r 37
t h
pa ymen t of t h e loa n .)
168
http://actuary88.com
En t er + , you ll s ee:
INT= - 464.8910004 (Th is is t h e in t er es t por t ion of t h e 37
t h
pa ymen t . Th e
n ega t ive s ign in dica t es ca s h ou t flow; t h is a mou n t is you r in t er es t por t ion
of t h e 37
t h
pa ymen t .)
Calc ulat e t he mont hly payme nt of t he re financ e d loan.

Time t 0 1 2 120 357 358 359 360
(Month)
Y Y Y Y Y Y Y Y
240 Months

240 j
Y a
Keys t r okes for TVM:
La s t t ime you u s ed Amor t iza t ion Wor ks h eet . TVM keeps t r a ck of a ll t h e
va lu es in TVM a n d Amor t iza t ion Wor ks h eet . So you don t n eed t o r een t er
wh a t you en t er ed. You s imply pick u p wh er e you left off.
Keys t r okes for a ddit ion a l ca lcu la t ion s :
En t er CE/ C t o lea ve Amor t iza t ion Wor ks h eet .
En t er N=240.
CPT PV. You s h ou ld get : PV = 83,327.72914 (t h e ou t s t a n din g ba la n ce of
t h e or igin a l loa n a t t =240).
Set I/ Y=
1
12
100 1.04 1
(

(

CPT PMT. You s h ou ld get : PMT = - 500.1777638
Ca lcu la t e t h e r edu ct ion of PMT du e t o r efin a n cin g:
PMT @6% = 586.5155230 (ign or ed t h e n ega t ive s ign ). As s u me you
s t or e t h is va lu e in a memor y.
PMT @4%= 500.1777638 (ign or ed t h e n ega t ive s ign ). As s u me you
s t or e t h is va lu e in a n ot h er memor y.
169
http://actuary88.com
Redu ct ion = 586.5155230 - 500.1777638 = 86.33775916
Set PMT = 86.33775916.
By n ow, you a r e pr oba bly los t a s t o wh a t va lu es a r e cu r r en t ly s t or ed in
TVM a n d wh a t keys t r okes t o pr es s n ext . Wh en you a r e los t , you ca n
a lwa ys clea r t h e cu r r en t TVM Wor ks h eet a n d s t a r t a fr es h TVM
Wor ks h eet . Of cou r s e, you n eed t o r een t er ma n y va lu es you en t er ed
befor e.
Un der t h is met h od, you s t a r t a n ew TVM Wor ks h eet . En t er N=240,
I/ Y=
1
12
100 1.04 1
(

(

. PMT=86.33775916. Th en , Pr es s CPT FV. You s h ou ld
get FV= - 31,526.11850. Th is is a ccu mu la t ed va lu e of t h e r edu ct ion s of
mon t h ly pa ymen t s a t 4% t o t =240 (mon t h s ).
As a n ea s ier a lt er n a t ively, you keep u s in g t h e cu r r en t TVM, r eca ll ea ch
in pu t in t h e cu r r en t TVM, a n d ch a n ge a n y in pu t a s n eces s a r y. For
exa mple, t o fin d ou t t h e va lu e of N, you s imply pr es s RCL N. You
s h ou ld get N=240. Next , you r eca ll I/ Y. You s h ou ld get 0.32737398. You
ca n ch eck t h a t t h is is
1
12
100 1.04 1
(

(

. Or if ch eckin g t h e in t er es t r a t e is t oo
mu ch pa in , you s imply r es et I/ Y=
1
12
100 1.04 1
(

(

. So TVM u s es t h e 4%
a n n u a l in t er es t r a t e. Th en you r eca ll PV a n d s h ou ld get PV =
83,327.72914. Fin a lly, you r eca ll FV a n d s h ou ld get FV = 0.
Summary of your re c all:
N=240, I/ Y=
1
12
100 1.04 1
(

(

, PV = 83,327.72914, FV=0, PMT =86.33775916
How do you ca lcu la t e t h e a ccu mu la t ed va lu e? Simply s et PV=0. Pr es s
CPT FV. You s h ou ld get FV= - 31,526.11850.
Wit h ou t s et t in g PV=0, if you pr es s CPT FV, you s h ou ld get a ga r ba ge FV=
- 214,097.4342. Th is ga r ba ge FV in clu des n ot on ly t h e a ccu mu la t ed
va lu e of t h e r edu ct ion of pa ymen t s t o t =240, bu t a ls o t h e a ccu mu la t ed
va lu e of 83,327.72914 t o t =240. You ca n ch eck t h a t :
( )
20
214, 097.4342 83, 327.72914 1.04 31,526.11850 = +
170
http://actuary88.com
I r ecommen d t h a t you wor k t h r ou gh t h is pr oblem mu lt iple t imes wit h
TVM Wor ks h eet . Tr y t o develop a men t a l pict u r e of h ow you r keys t r okes
will ch a n ge t h e in t er n a l s et t in g TVM.
Compre he ns ive c alc ulat or e xe rc is e #2
A compa n y is pa r t icipa t in g in a pr oject . Th e ca s h flows of t h e pr oject a r e
a s follows :
Th e compa n y will in ves t $10 million per yea r for t h e 1
s t
t h r ee yea r s
of t h e pr oject . Th e in ves t men t will be ma de con t in u ou s ly.

Th e compa n y will r eceive a ca s h flow a t t h e en d of ea ch yea r
s t a r t in g fr om Yea r 4.

At t h e en d of Yea r 4, t h e compa n y will r eceive t h e 1
s t
ca s h flow of
$9 million . Th is a mou n t will be r edu ced by $0.5 million for ea ch
s u bs equ en t yea r , u n t il t h e compa n y r eceives $5 million in a yea r .

St a r t in g fr om t h a t yea r , t h e ca s h flow r eceived by t h e compa n y will
be r edu ced by $1 million ea ch yea r , u n t il t h e compa n y r eceives
zer o ca s h flow.

Ca lcu la t e
t h e NPV of t h e pr oject if t h e dis cou n t r a t e is 12%.
t h e IRR of t h e pr oject .

Solut ion

Us e NPV Works he e t (quic k and e as y)

Time t 0 1 2 3 4 5 6 7 8 9 10 11 12 13 14 15 16

cash $9.0 $8.5 $8.0 $7.5 $7.0 $6.5 $6.0 $5.5 $5.0 $4.0 $3.0 $2.0 $1.0

( )
12% 3
10 a
Th e in it ia l in ves t men t a t t =0 is :

( )
3
12%
3
3
12%
1 1 1.12
10 10 10 25.43219763
ln1.12
i
v
a
o

=
| |
= = =
|
\ .

171
http://actuary88.com
Us e NPV Wor ks h eet :

CF0= -25.43219763;
C01=0, F01=3;
C04=9; C05=8.5; C06=8; C07=7.5; C08=7; C09=6.5; C10=6;
C11=5.5; C12=5; C13=4; C14=3; C15=2; C16=1.
CPT NPV. You s h ou ld get NPV=47.56780237 @ i=0.
Dou ble ch eck:
-25.43219763+9+8.5+8+7.5+7+6.5+6+5.5+5+4+3+2+1=47.56780237
(OK)
Set I=12 a n d ca lcu lat e NPV. You s h ou ld get NPV=4.58281303
Next , pr es s IRR CPT. You s h ou ld get : IRR=14.60149476.
So t h e IRR is 14.60149476%.
To ca lcu la t e IRR, we n eed t o s olve t h e followin g equ a t ion :

Time t 0 1 2 3 4 5 6 7 8 9 10 11 12 13 14 15 16

cash $9.0 $8.5 $8.0 $7.5 $7.0 $6.5 $6.0 $5.5 $5.0 $4.0 $3.0 $2.0 $1.0

( )
12% 3
10 25.43219763 a =
4 5 6 7 8 9 10 11
25.43219763 9 8.5 8 7.5 7 6.5 6 5.5 v v v v v v v v + + + + + + + +
12 13 14 15 16
5 4 3 2 0 v v v v v + + + + + =
We ca n NOT s olve t h is equ a t ion ma n u a lly. We n eed t o u s e t h e IRR
fu n ct ion of BA II Plu s / BA II Plu s Pr ofes s ion a l. We get :

IRR = 14.60149476%

172
http://actuary88.com
Compre he ns ive c alc ulat or e xe rc is e #3 (May 2 0 0 5 FM, #3 5 )
A ba n k cu s t omer t a kes ou t a loa n of 500 wit h a 16% n omin a l in t er es t
r a t e con ver t ible qu a r t er ly. Th e cu s t omer ma kes pa ymen t s of 20 a t t h e
en d of ea ch qu a r t er .

Ca lcu la t e t h e a mou n t of pr in cipa l in t h e fou r t h pa ymen t .

[A] 0.0
[B] 0.9
[C] 2.7
[D] 5.2
[E] Not en ou gh in for ma t ion

Solut ion

Well s olve t h is pr oblem wit h ou r ima gin a r y ca s h flow met h od. To u s e
t h is met h od, we n eed t o fin d n , t h e # of level pa ymen t s . On ce we fin d n ,
well s et u p a n ima gin a r y ca s h flow of $20 a t 1 t n = + . Next , well dis cou n t
t h is ima gin a r y ca s h flow fr om 1 t n = + t o 4 t = t o fin d t h e pr in cipa l por t ion
of t h e pa ymen t ma de a t 4 t = :
Pr in cipa l of t h e 4
t h
pa ymen t =
( ) 1 4
20
n
v
+
So we n eed t o ca lcu la t e n by s olvin g t h e followin g equ a t ion :

4%
500 20
n
a =
Th is equ a t ion s a ys t h a t t h e PV of t h e a ll t h e qu a r t er ly pa ymen t s of $20 @
4% qu a r t er ly in t er es t r a t e is t h e $500 (t h e t ot a l pr in cipa l). Th e qu a r t er ly
effect ive in t er es t r a t e is 4% beca u s e we a r e given t h a t
( ) 4
16% i = .
Well u s e BA II Plu s / BA II Plu s Pr ofes s ion a l TVM t o s olve
4%
500 20
n
a = .
Set PV= - 500, PMT = 20, I/ Y=4. CPT N.

We get a n er r or mes s a ge. Per h a ps weve en t er ed wr on g n u mber s . On ce
a ga in , we en t er :

Set PV= - 500, PMT = 20, I/ Y=4. CPT N.

We get a n er r or mes s a ge a ga in . Not kn owin g wh er e t h e pr oblem is , let s
t h r ow a wa y t h e ca lcu la t or a n d s olve t h e equ a t ion ma n u a lly.

4% 4%
500 20 25
n n
a a = =
173
http://actuary88.com
1 1.04
25, 1 1.04 1, 1.04 0,
4%
n
n n
n

= = = =
Now we s ee wh er e t h e t r ou ble is . Th e loa n is r epa id t h r ou gh a per pet u a l
immedia t e a n n u it y. BA II Plu s or BA II Plu s Pr ofes s ion a l TVM ca n t
h a n dle per pet u a l a n n u it y (immedia t e or du e).

Now we h a ve n = .
Plea s e n ot e t h a t ou r ima gin a r y ca s h flow wor ks even wh en n = .
Pr in cipa l of t h e 4
t h
pa ymen t is :

( ) 1 4 1 4
20 20 0
n
v v
+ +
= =
Mor a l of t h is pr oblem:

SOA ca n pu r pos ely des ign a pr oblem t o ma ke ou r ca lcu la t or s u s eles s . In
s t u dyin g for FM, we n eed t o lea r n h ow t o s olve a pr oblem wit h a
ca lcu la t or a n d h ow t o s olve it wit h ou t a ca lcu la t or .
174
http://actuary88.com
Chapt e r 5 Ge ome t ric ally inc reas ing
annuit y

Ke y point s :

Un der s t a n d a n d memor ize t h e followin g geomet r ic a n n u it y s h or t cu t s :

( ) ( ) ( )
2 1
payments
1 1 1 ...... 1
n
n
k k k

+ + +

1
1
1 j
i k
n
k
k
a
=

+
+
( )
1
1
j
i k
n
k
a
=

+

( )
1
1
1
i k
k
n
j n
k s

+

=
+
( )
1
1
i k
k
n
n j
k s

+
=
+
In t er pr et a t ion of t h is dia gr a m:
For a geomet r ica lly in cr ea s in g a n n u it y wh er e
n geomet r ica lly in cr ea s in g pa ymen t s a r e ma de a t a r egu la r in t er va l;
t h e 1
s t
pa ymen t is $1;
t h e n ext pa ymen t is a lwa ys ( ) 1 k + t imes t h e pr eviou s pa ymen t .
Th en
(1) Th e pr es en t va lu e a t on e in t er va l pr ior t o t h e 1
s t
pa ymen t is
1
1
1 j
i k
n
k
k
a
=

+
+
. Th is va lu e h a s a fa ct or of
1
1 k +
beca u s e t h e geomet r ic
pa ymen t pa t t er n on e in t er va l pr ior t o t h e 1
s t
pa ymen t is
1
1 k +
.
175
http://actuary88.com
(2) Th e pr es en t va lu e immedia t ely a ft er t h e 1
s t
pa ymen t is ( )
1
1
j
i k
n
k
a
=

+
. Th e
pr es en t va lu e h a s a fa ct or of 1 beca u s e t h e 1
s t
pa ymen t is 1.
(3) Th e a ccu mu la t ed va lu e immedia t ely a ft er t h e fin a l pa ymen t is
( )
1
1
1
i k
k
n
j n
k s

+

=
+ . Th is va lu e h a s a fa ct or of ( )
1
1
n
k

+ beca u s e t h e fin a l
pa ymen t is ma de is ( )
1
1
n
k

+ .
(4) Th e a ccu mu la t ed va lu e a t on e in t er va l a ft er t h e fin a l pa ymen t is
( )
1
1
i k
k
n
n j
k s

+
=
+ . Th is va lu e h a s a fa ct or of ( ) 1
n
k + beca u s e t h e geomet r ic
pa ymen t pa t t er n on e in t er va l a ft er t h e fin a l pa ymen t is ( ) 1
n
k + .
Explanat ions

Pre s e nt value of ge ome t ric annuit y due :

Amou n t $1 $( ) 1 k + $( )
2
1 k + $( )
3
1 k + $( )
1
1
n
k

+
Time 0 1 2 3 1 n
( )
1 1
1
j j
i k i k
n n
k k
a a
= =

+ +
=
Proof.

Th e pr es en t va lu e is : ( ) ( ) ( )
2 1
2 1
1 1 1 ... 1
n
n
k v k v k v


+ + + + + + + .
Let s s et ( ) 1 V k v = + wh er e V is t h e n ew dis cou n t fa ct or .

Th e n ew in t er es t is
( )
1 1 1 1
1 1 1 1
1
1 1 1
1
i i k
j
k
V k v k k
i
+
= = = = =
+
+ + +
+
Th e pr es en t va lu e is
2 1
1
1 ...
n
j
i k
n
k
V V V a

=

+
+ + + + =
176
http://actuary88.com
How t o memor ize t h is for mu la :

Geomet r ic a n n u it y du e is s t ill a n n u it y du e a n d it s pr es en t va lu e a t
t ime 0 s h ou ld h a ve a n a n n u it y du e fa ct or
j n
a wh er e j is t h e n ew
in t er es t r a t e.

Th e pa ymen t a t t ime zer o is 1. We s a y t h a t t h e pa ymen t fa ct or =1.
We t h en mu lt iply
j n
a wit h 1.
Ac c umulat e d value of ge ome t ric annuit y due :

Amou n t $1 $( ) 1 k + $( )
2
1 k + $( )
3
1 k + $( )
1
1
n
k

+
Time 0 1 2 3 1 n n
( )
1
1
i k
k
n
n j
k s

+
=
+
Proof.

Th e a ccu mu la t ed va lu e is

( ) ( ) ( ) ( ) ( ) ( )( )
( )
1 2 2 1
1 2
1 1 1 1 1 ... 1 1
1 1 1 1
1 ...
1 1 1 1
n n n n
n n n
n
i i k i k i k
i i i i
k
k k k k


+ + + + + + + + + + +
(
+ + + + | | | | | | | |
= + + + + +
(
| | | |
+ + + +
\ . \ . \ . \ .
(


Let s s et
1
1
1
i
j
k
+
+ =
+
wh er e
1
1
1 1
i i k
j
k k
+
= =
+ +
is t h e n ew in t er es t r a t e.

Th en t h e a ccu mu la t ed va lu e is :

( ) ( ) ( ) ( ) ( ) ( )
1 2
1
1 1 1 1 ... 1 1
n n n n
i k
k
n
n j
k j j j j k s

+
=
(
+ + + + + + + + + = +


177
http://actuary88.com
How t o memor ize t h is for mu la :

Geomet r ic a n n u it y du e is s t ill a n n u it y du e a n d it s a ccu mu la t ed
va lu e a t t ime n h a s a n a n n u it y fa ct or
n j
s wh er e
1
i k
j
k

=
+
is t h e n ew
in t er es t r a t e.
If we ext en d t h e geomet r ica lly in cr ea s in g pa t t er n t o t ime n , t h en
t h e pa ymen t a t t ime n will be ( ) 1
n
k + . We s a y t h a t t h e pa ymen t
fa ct or is ( ) 1
n
k + .
Fu t u r e va lu e=( )( ) ( )
1
Payment Factor Annuity Factor 1
n
i k
n j
k
k s

=
+
= +
Pre s e nt value of ge ome t ric annuit y imme diat e :

Amou n t $1 $( ) 1 k + $( )
2
1 k + $( )
2
1
n
k

+ $( )
1
1
n
k

+
Time 0 1 2 3 1 n n
1
1
1 j
i k
n
k
k
a
=

+
+
Proof.

Th e pr es en t va lu e is :

( ) ( ) ( ) ( ) ( ) ( )
{ }
2 1 2 1
2 3
1
1 1 ... 1 1 1 ... 1
1
n n
n
v k v k v k v v k v k v k
k

+ + + + + + + = + + + + + + ( ( (

+
Let s s et ( ) 1 V v k = + wh er e V is t h e n ew dis cou n t fa ct or .
Th e n ew in t er es t r a t e is
( )
1 1 1 1
1 1 1 1
1
1 1 1
1
i i k
j
k
V k v k k
i
+
= = = = =
+
+ + +
+
Th e pr es en t va lu e of geomet r ic a n n u it y immedia t e is :
178
http://actuary88.com
2 1
1
1 1
...
1 1
n
i k
k
j n
V V V
k k
a

+
=
( + + + =

+ +

How t o memor ize t h is for mu la :

Geomet r ic a n n u it y immedia t e is s t ill a n n u it y immedia t e a n d it s
pr es en t va lu e a t t ime 0 h a s a n a n n u it y immedia t e fa ct or
j n
a wh er e
1
i k
j
k

=
+
is t h e n ew in t er es t r a t e.
If we ext en d t h e geomet r ica lly in cr ea s in g pa t t er n t o t ime zer o, t h en
t h e pa ymen t a t t ime zer o will be
1
1 k +
. We s a y t h a t t h e pa ymen t
fa ct or is
1
1 k +
.
( )( )
1
1
Payment Factor Annuity Factor
1
i k
k
j n
PV
k
a

+
=
= =
+
Ac c umulat e d value of ge ome t ric annuit y imme diat e :

Amou n t $1 $( ) 1 k + $( )
2
1 k + $( )
2
1
n
k

+ $( )
1
1
n
k

+
Time 0 1 2 3 1 n n
( )
1
1
1
i k
k
n
j n
k s

+

=
+
Proof.

Th e a ccu mu la t ed va lu e is

( ) ( ) ( ) ( ) ( ) ( ) ( ) ( )
( )
2 3 2 2 1 1
1 2 3
1
1 1 1 1 1 ... 1 1 1
1 1 1 1
1 ... 1
1 1 1 1
n n n n n
n n n
n
i i k i k i k k
i i i i
k
k k k k

+ + + + + + + + + + + + +
(
+ + + + | | | | | | | |
= + + + + + +
(
| | | |
+ + + +
\ . \ . \ . \ .
(


179
http://actuary88.com
Let s s et
1
1
1
i
j
k
+
+ =
+
wh er e
1
1
1 1
i i k
j
k k
+
= =
+ +
is t h e n ew in t er es t r a t e.

Th en t h e a ccu mu la t ed va lu e is :

( ) ( ) ( ) ( ) ( )
1
1 1 2 1
1 1 1 ... 1 1 1
i k
k
n n n
j
n
n
k j j j k s

+

=

(
+ + + + + + + + = +


How t o memor ize t h is for mu la :

Geomet r ic a n n u it y du e is s t ill a n n u it y du e a n d it s a ccu mu la t ed
va lu e a t t ime n h a s a n a n n u it y fa ct or
j n
s wh er e
1
i k
j
k

=
+
is t h e n ew
in t er es t r a t e.
Th e pa ymen t a t t ime n is ( )
1
1
n
k

+ . We s a y t h a t t h e pa ymen t fa ct or
is ( )
1
1
n
k

+ .
( ) ( ) ( )
1
1
Payment Factor Annuity Factor 1
i k
k
j
n
n
FV k s

+
=

= = +
Sample Proble ms and Solut ions

Proble m 1

You a r e given t h e followin g ca s h flows

( ) ( ) ( )
2 3
payments
1 1 1 ...... 1
n
n
k k k k + + + +

A
B
C
D
180
http://actuary88.com
Ca lcu la t e t h e va lu e of t h e a n n u it y a t fou r differ en t poin t s :
on e s t ep befor e t h e 1
s t
pa ymen t (poin t A)
1
s t
pa ymen t t ime ( poin t B)
fin a l pa ymen t t ime (poin t C), a n d
on e s t ep a ft er t h e fin a l pa ymen t (poin t D).

Solut ion

( ) ( ) ( )
2 3
payments
1 1 1 ...... 1
n
n
k k k k + + + +

1
j
i k
n
k
a
=

+
( )
1
1
j
i k
n
k
k a
=

+
+
( )
1
1
i k
k
n
j n
k s

+
=
+
( )
1
1
1
i k
k
n
n j
k s

+
+
=
+
Va lu e of a geomet r ic a n n u it y = pa ymen t fa ct or * a n n u it y fa ct or

Th e pa ymen t fa ct or a t on e s t ep befor e t h e 1
s t
pa ymen t is $1. Th e 1
s t

pa ymen t is 1 k + . If we ext en d t h e geomet r ic pa ymen t pa t t er n t o on e s t ep
befor e t h e 1
s t
pa ymen t t ime, we get $1. So t h e va lu e of t h e a n n u it y on e
s t ep befor e t h e 1
s t
pa ymen t t ime is
1
j
i k
n
k
a
=

+
.
Th e pa ymen t fa ct or a t t h e 1
s t
pa ymen t is 1 k + .
Th e va lu e of t h e a n n u it y a t t h e 1
s t
pa ymen t t ime is ( )
1
1
j
i k
n
k
k a
=

+
+ .
Th e pa ymen t fa ct or a t t h e fin a l pa ymen t is ( ) 1
n
k + .
Th e va lu e of t h e a n n u it y a t t h e 1
s t
pa ymen t t ime is ( )
1
1
i k
k
n
j n
k s

+
=
+ .
181
http://actuary88.com
Th e pa ymen t fa ct or a t on e s t ep a ft er t h e fin a l pa ymen t is ( )
1
1
n
k
+
+ . If we
ext en d t h e geomet r ic pa t t er n t o on e s t ep a ft er t h e fin a l pa ymen t t ime, we
get ( )
1
1
n
k
+
+ . So t h e va lu e of t h e a n n u it y a t t h e 1
s t
pa ymen t t ime is
( )
1
1
1
i k
k
n
n j
k s

+
+
=
+ .
Proble m 2

You a r e given t h e followin g ca s h flows

( ) ( ) ( ) ( )
2 3 4 1
payments
1 1 1 ...... 1
n
n
k k k k
+
+ + + +

A
B
C
D
Ca lcu la t e t h e va lu e of t h e a n n u it y a t fou r differ en t poin t s :
on e s t ep befor e t h e 1
s t
pa ymen t (poin t A)
1
s t
pa ymen t t ime ( poin t B)
fin a l pa ymen t t ime (poin t C), a n d
on e s t ep a ft er t h e fin a l pa ymen t (poin t D).

182
http://actuary88.com
Solut ion

( ) ( ) ( ) ( )
2 3 4 1
payments
1 1 1 ...... 1
n
n
k k k k
+
+ + + +

( )
1
1
j
i k
n
k
k a
=

+
+
( )
2
1
1
j
i k
n
k
k a
=

+
+
( )
1
1
1
i k
k
n
j n
k s

+
+
=
+
( )
1
2
1
i k
k
n
n j
k s

+
+
=
+
Proble m 3

An a n n u it y immedia t e h a s 15 pa ymen t s . Th e 1
s t
pa ymen t is $100. Ea ch
followin g pa ymen t is 8% la r ger t h a n t h e pr eviou s pa ymen t . Th e a n n u a l
effect ive in t er es t r a t e is 14%. Ca lcu la t e t h e pr es en t va lu e of t h is a n n u it y.

Solut ion

( ) ( )
2
15 payments
$100 $100 1 8% $100 1 8% ...... + +

PV=?
We a r e a s ked t o fin d t h e pr es en t va lu e of t h is geomet r ic a n n u it y a t on e
in t er va l pr ior t o t h e 1
s t
pa ymen t t ime.

183
http://actuary88.com
Fr om t h e s h or t cu t , we kn ow

( ) ( ) Payment Factor Annuity Factor PV =
To fin d t h e pa ymen t fa ct or , we ext en d t h e geomet r ic a n n u it y pa ymen t
pa t t er n t o on e in t er va l pr ior t o t h e 1
s t
pa ymen t t ime. Beca u s e t h e 1
s t

pa ymen t is $100 a n d ea ch pa ymen t is 8% la r ger t h a n t h e pr eviou s
pa ymen t , t h e pa ymen t t h a t wou ld h a ve been ma de on e in t er va l pr ior t o
t h e 1
s t
pa ymen t is :
$100
1.08
. So ( )
$100
Payment Factor
1.08
=
To fin d t h e a n n u it y fa ct or , we s imply ch a n ge t h e or igin a l in t er es t of 14%
t o t h e n ew in t er es t r a t e:

14% 8%
5.5556%
1 1 8%
i k
j
k

= = =
+ +

( )
15 5.5556%
Annuity Factor a =
( )( )
15 5.5556%
$100
Payment Factor Annuity Factor $925.98
1.08
PV a = = =
Plea s e n ot e t h a t $925.98 is ca lcu la t ed u s in g BA II Plu s or BA II Plu s
Pr ofes s ion a l. Th er e is n o n eed for you t o ma n u a lly ca lcu la t e t h e a n n u it y
u s in g t h e for mu la :

1
n
i n
v
i
a

=
Plea s e a ls o n ot e t h a t you don t n eed t o dr a w t h e t ime lin e or t o s pell ou t
ea ch of t h e 15 pa ymen t s ; doin g s o is t ime-con s u min g a n d er r or -pr on e.
So don t dr a w t h e followin g dia gr a m:

Amou n t $1 $( ) 1 8% + $( )
2
1 8% + $( )
13
1 8% + $( )
14
1 8% +
Time 0 1 2 3 14 15
Th is dia gr a m is good for you t o in it ia lly pr ove t h e geomet r ic a n n u it y
for mu la . On ce you h a ve pr oven t h e for mu la , don t dr a w t h is dia gr a m a n y
mor e.
184
http://actuary88.com
All you n eed t o kn ow t h a t t h e pr es en t va lu e of a n a n n u it y immedia t e is
t h e pr es en t va lu e on e in t er va l pr ior t o t h e 1
s t
pa ymen t .

Alt er n a t ive met h od:

( ) ( )
2
15 payments
$100 $100 1 8% $100 1 8% ...... + +

PV=?

We fir s t ca lcu la t e t h e pr es en t va lu e of t h e geomet r ic a n n u it y du e. Th en
we dis cou n t t h is pr es en t va lu e on e in t er va l pr ior t o t h e 1
s t
pa ymen t .

( )( ) geometric annuity due Payment Factor Annuity Factor PV =
( ) Payment Factor $100 =
( )
15 5.5556%
Annuity Factor
n j
a a = =
15 5.5556%
geometric annuity due $100 PV a =
Next , we dis cou n t t h is va lu e u s in g t h e dis cou n t fa ct or
1
1 14% +
.
Th e pr es en t va lu e of t h e geomet r ic a n n u it y immedia t e is :
15 5.5556%
1
$100 $925.98
1 14%
a
| |
=
|
+
\ .

Proble m 4

An a n n u it y immedia t e h a s 15 pa ymen t s . Th e 1
s t
pa ymen t is $100. Ea ch
followin g pa ymen t is 8% la r ger t h a n t h e pr eviou s pa ymen t . Th e a n n u a l
effect ive in t er es t r a t e is 14%. Immedia t ely a ft er t h e 1
s t
pa ymen t is ma de,
t h is a n n u it y is s old a t a pr ice of X . Ca lcu la t e X .
185
http://actuary88.com
Solut ion

$100 ( ) ( )
2
14 payments
$100 1 8% $100 1 8% ...... + +

PV=?

Immedia t ely a ft er t he 1
s t
pa ymen t , t h er e a r e 14 geomet r ic pa ymen t s left .
We a r e a s ked t o fin d t h e pr es en t va lu e of t h es e 14 geomet r ic pa ymen t s
on e in t er va l pr ior t o t h e 1
s t
pa ymen t of $100(1+8%). Th is s h ou ld be t h e
s a les pr ice X .
On ce a ga in , we u s e t h e s h or t cu t :

( ) ( ) Payment Factor Annuity Factor PV =
Plea s e n ot e t h a t a mon g t h e r ema in in g 14 geomet r ic pa ymen t s , t h e 1
s t

pa ymen t is $100(1+8%). As a r es u lt , if we ext en d t h e geomet r ic pa ymen t
pa t t er n on e in t er va l pr ior t o t h is 1
s t
pa ymen t , well get $100.

( ) Payment Factor $100 = , ( )
14 5.5556%
Annuity Factor a =
14 5.5556%
100 955.62 PV a = =
Proble m 5

An a n n u it y immedia t e h a s 15 pa ymen t s . Th e 1
s t
pa ymen t is $100. Ea ch
followin g pa ymen t is 8% la r ger t h a n t h e pr eviou s pa ymen t . Th e a n n u a l
effect ive in t er es t r a t e is 14%. Ca lcu la t e t h e a ccu mu la t ed va lu e of t h is
a n n u it y immedia t ely a ft er t h e 15
t h
pa ymen t .

Solut ion

( ) ( ) ( )
2 14
15 payments
$100 $100 1 8% $100 1 8% ......$100 1 8% + + +

FV=?

186
http://actuary88.com
( ) ( ) Payment Factor Annuity Factor FV =
( ) ( )
14
Payment Factor 100 1 8% = +
( )
15
Annuity Factor
j
s = ,
14% 8%
5.5556%
1 8%
j

= =
+
( )
14
15 5.5556%
100 1 8% 6, 609.64 FV s = + =
Proble m 6

In a per pet u a l a n n u it y immedia t e, t h e 1
s t
pa ymen t is $100 a n d ea ch
followin g pa ymen t is 8% la r ger t h a n t h e pr eviou s pa ymen t . Th e a n n u a l
effect ive in t er es t r a t e is 14%. Ca lcu la t e t h e pr es en t va lu e of t h is a n n u it y.

Solut ion

( ) ( )
2
payments
$100 $100 1 8% $100 1 8% ......
+
+ +

PV=?
( ) ( ) Payment Factor Annuity Factor PV =
( )
100
Payment Factor
1.08
= , ( )
1
Annuity Factor
j
j
a
+
= =
14% 8% 6%
1 8% 1.08
j

= =
+
100 1 100
1, 666.67
6%
1.08 6%
1.08
PV
| |
= = =
|
\ .

Gen er a lly, t h e pr es en t va lu e of a per pet u a l geomet r ic a n n u it y immedia t e
is
1
i k
.
187
http://actuary88.com
Amou n t $1 $( ) 1 k + $( )
2
1 k + $( )
2
1
n
k

+ $( )
1
1
n
k

+
Time 0 1 2 3 1 n n +
1
1 1 1 1
1 1
1
j
i k
k
i k
k k i k
k
a
=

+
+
= =

+ +
+
Proble m 7

In a per pet u a l a n n u it y immedia t e, t h e 1
s t
pa ymen t is $100 a n d ea ch
followin g pa ymen t is 8% la r ger t h a n t h e pr eviou s pa ymen t . Th e a n n u a l
effect ive in t er es t r a t e is 14%. Ca lcu la t e t h e pr es en t va lu e of t h is a n n u it y
immedia t ely befor e t h e 1
s t
pa ymen t is ma de.

Solut ion

( ) ( )
2
payments
$100 $100 1 8% $100 1 8% ......
+
+ +

PV=?

( ) ( ) Payment Factor Annuity Factor PV =
( ) Payment Factor 100 =
( )
1
1
Annuity Factor
j
i k
k
d
a
=

+
+
= =
188
http://actuary88.com
1 1
1 1
1 1
1
1
i k
d
i k
j i
k

= = =

+ +
+
+
( )
1
1 1
Annuity Factor
j
i k
k
i
d i k
a
=

+
+
+
= = =


1 1 14%
100 100 1, 900
14% 8%
i
PV
i k
+ +
= = =


Gen er a lly, t h e pr es en t va lu e of a per pet u a l geomet r ic a n n u it y du e is
1 i
i k
+

.
Amou n t $1 $( ) 1 k + $( )
2
1 k + $( )
3
1 k + $( )
1
1
n
k

+
Time 0 1 2 3 1 n n +
1
1
j
i k
k
i
i k
a
=

+
+
+
=


Proble m 8

You pla n t o pa y off you r $150,000 mor t ga ge by mon t h ly in s t a llmen t s for
15 yea r s . You r 1
s t
pa ymen t is on e mon t h fr om n ow. You pla n t o in cr ea s e
you r mon t h ly pa ymen t s by 10% ea ch yea r . You pa y a n omin a l in t er es t
r a t e of 12% compou n din g qu a r t er ly.

Ca lcu la t e you r 1
s t
mon t h ly pa ymen t .

Solut ion

To s implify ou r ca lcu la t ion , let s fin d t h e equ iva len t a n n u a l pa ymen t s t o
r epla ce ea ch yea r s 12 mon t h ly pa ymen t s .

Let X =you r 1
s t
mon t h ly pa ymen t . Let Y =t h e pr es en t va lu e of t h e a n n u it y
immedia t e of you r 1
s t
yea r pa ymen t .

189
http://actuary88.com
12 payments
...... X X X

12 i
PV Y X a = = , wh er e i is t h e mon t h ly in t er es t r a t e.
Let s fin d t h e mon t h ly in t er es t r a t e.

( )
( ) 4
3
1 1
4
i
i + = +
( )
1
1
4
3
3
12%
1 1 1 1 0.99016%
4 4
i
i
(
(
= + = + =
(
(



12 0.99016%
11.2621 Y X X a = =
Beca u s e you r mon t h ly pa ymen t s in cr ea s e by 10% per yea r , t h e 12
mon t h ly pa ymen t s in t h e 2
n d
yea r ca n be r epla ced by a s in gle ca s h flow
of 1.1Y a t t h e begin n in g of t h e 2
n d
yea r . Similar ly, t h e 12 pa ymen t s in t h e
3
r d
yea r ca n be r epla ced by a s in gle ca s h flow of ( )
2
1.1 Y a t t h e begin n in g
of t h e 3
r d
yea r . As a r es u lt , you ll h a ve a geomet r ic a n n u it y immedia t e for
15 pa ymen t s (yea r s ).

( )
2
15 payments
1.1 1.1 ...... Y Y Y

15
1
150, 000
r k
j
k
PV Y a

=
+
= =
wh er e r is t h e a n n u a l effect ive in t er es t r a t e a n d 10% k = . Th e in t er es t
r a t e per 3-mon t h is 12%/ 4=3%. So

4
1.03 1 12.5509% r = =
12.5509% 10%
2.319%
1 1 10%
r k
j
k

= = =
+ +

15 2.319%
150, 000 12.8388 , 11, 683.3307 Y Y Y a = = =
11, 683.3307
1, 037.40
11.2621 11.2621
Y
X = = =
190
http://actuary88.com
Alt e rnat ive me t hod:

We ca n u s e a s in gle ca s h flow Z a t t h e en d of Yea r 1 t o r epla ce t h e 1
s t

yea r s 12 mon t h ly pa ymen t s . Th en , t h e 12 mon t h ly pa ymen t s in t h e 2
n d

yea r ca n be r epla ced by a s in gle ca s h flow of 1.1Z a t t h e en d of Yea r 2.
An d t h e 12 mon t h ly pa ymen t s in t h e 3
r d
yea r ca n by r epla ced by a s in gle
ca s h flow of ( )
2
1.1 Z a t t h e en d of Yea r 3. An d s o on .

( )
2
15 payments
1.1 1.1 ...... Z Z Z

15
1
1
150, 000
1.1
r k
j
k
PV Z a

=
+
= = , wh er e
1
1.1
is t h e pa ymen t fa ct or .
15
15
1
2.319%
1 1
150, 000 , 13,149.6670
1.1 1.1
r k
j
k
Z Z Z a a

=
+
= = =
Next , we ca n ca lcu la t e t h e 1
s t
mon t h ly pa ymen t X .
12 payments
...... X X X X

12 12 0.99016% i
FV Z X s X s = = =
12 0.99016% 12 0.99016%
13,149.6670
1, 037.40
Z
X
s s
= = =
191
http://actuary88.com
Proble m 9

An a n n u it y immedia t e h a s 15 pa ymen t s . Th e 1
s t
pa ymen t is $100. Ea ch
followin g pa ymen t is 20% la r ger t h a n t h e pr eviou s pa ymen t . Th e a n n u a l
effect ive in t er es t r a t e is 14%. Ca lcu la t e t h e pr es en t va lu e of t h is a n n u it y.

Solut ion

( ) ( )
2
15 payments
$100 $100 1 20% $100 1 20% ...... + +

PV=?
To fin d t h e pa ymen t fa ct or , we ext en d t h e geomet r ic a n n u it y pa ymen t
pa t t er n t o on e in t er va l pr ior t o t h e 1
s t
pa ymen t t ime. Beca u s e t h e 1
s t

pa ymen t is $100 a n d ea ch pa ymen t is 20% la r ger t h a n t h e pr eviou s
pa ymen t , t h e pa ymen t t h a t wou ld h a ve been ma de on e in t er va l pr ior t o
t h e 1
s t
pa ymen t is :
$100
1.2
. So ( )
$100
Payment Factor
1.2
=
To fin d t h e a n n u it y fa ct or , we s imply ch a n ge t h e or igin a l in t er es t of 14%
t o t h e n ew in t er es t r a t e:

14% 20%
5%
1 1 20%
i k
j
k

= = =
+ +
(OK if t h e n ew in t er es t r a t e is n ega t ive)

( )
15 5%
Annuity Factor a

=
( )( )
15 5%
$100
Payment Factor Annuity Factor $1, 930.78
1.2
PV a

= = =
wh er e
15 5%
15
1
1
1 5%
23.1694
5%
a

| |

\ .
= =

192
http://actuary88.com
Proble m 1 0

An a n n u it y immedia t e h a s 15 pa ymen t s . Th e 1
s t
pa ymen t is $100. Ea ch
followin g pa ymen t is 20% la r ger t h a n t h e pr eviou s pa ymen t . Th e a n n u a l
effect ive in t er es t r a t e is 14%. Ca lcu la t e t h e a ccu mu la t ed va lu e of t h is
a n n u it y immedia t ely a ft er t h e 15
t h
pa ymen t .

Solut ion

( ) ( ) ( )
2 14
15 payments
$100 $100 1.2 $100 1.2 ......$100 1.2

FV=?

( ) ( ) Payment Factor Annuity Factor FV =
( ) ( )
14
Payment Factor 100 1.2 =
( )
15
Annuity Factor
j
s = ,
14% 20%
5%
1 20%
j

= =
+
( )
14
15 5%
100 1.2 13, 781.81 FV s

= =
Wh er e
15
15 5%
0.95 1
10.7342
5%
s


= =

Proble m 1 1

In a per pet u a l a n n u it y immedia t e, t h e 1
s t
pa ymen t is $100. Ea ch
followin g pa ymen t is 20% la r ger t h a n t h e pr eviou s pa ymen t . Th e a n n u a l
effect ive in t er es t r a t e is 14%. Ca lcu la t e t h e pr es en t va lu e of t h is a n n u it y.

193
http://actuary88.com
Solut ion

( ) ( )
2
payments
$100 $100 1 20% $100 1 20% ......
+
+ +

1 1
20
14% 20%
PV
i k
= = =


We got a n on s en s e n u mber of n ega t ive 20. Th is r ea lly mea n s t h a t t h e
pr es en t va lu e is n ot defin ed.

We ca n ea s ily ch eck t h a t t h e pr es en t va lu e is n ot defin ed. Pa ymen t s
in cr ea s e by 20%, wh ich is fa s t er t h a n t h e dis cou n t r a t e 14%. As a r es u lt ,
t h e pr es en t va lu e of t h is per pet u a l a n n u it y becomes +.
Mor a l of t h is pr oblem: If 0
1
i k
j
k

= <
+
, t h en t h e pr es en t va lu e of t h e
per pet u a l geomet r ic a n n u it y is u n defin ed.

Proble m 1 2

Ca lcu la t e
2 3 4 5
1 4 4 4 4 4 + + + + +
Solut ion

Th is is a geomet r ic a n n u it y pr oblem wit h 0 i = a n d 3 k = .
( ) ( )
2
6 payments
1 1 3 1 3 ...... + +

6 j
PV a =
0 3
75%
1 1 3
i k
j
k

= = =
+ +
,
6 75%
1, 365
j
PV a
=
= =
194
http://actuary88.com
We ca n ch eck t h a t t h e r es u lt is cor r ect :

6
2 3 4 5
1 4
1 4 4 4 4 4 1, 365
1 4

+ + + + + = =

Gen er a lly,
2
1 ...
n
j n
q q q a + + + + = wh er e
1
1 j
q
= (for 1 q = ).

Alt er n a t ively, we ca n u s e t h e followin g for mu la :

( ) ( ) ( ) ( )
2 3 1
1 1 1 1 ... 1
n
r n
r r r r s

+ + + + + + + + + =
( )
6
2 3 4 5
300% 6
1 1
4 1
1 4 4 4 4 4 1, 365
3
n
i
s
i
+

+ + + + + = = = =
If you t h in k it s a joke t o u s e a n n u it y t o ca lcu la t e t h e s u m of a geomet r ic
pr ogr es s ion , t h in k a ga in . In t h e h ea t of t h e exa m, it ca n be fa s t er t o
ca lcu la t e a n n u it y t o ca lcu la t e t h e s u m of a power s er ies .

Proble m 1 3

St a r t in g fr om t h e cu r r en t yea r , ea ch yea r J oh n will t a ke ou t a level
per cen t a ge of h is s a la r y a n d depos it it in t o a r et ir emen t fu n d. His goa l is
t o a ccu mu la t e $250,000 immedia t ely a ft er h e r et ir es .

Fa ct s :

J oh n s cu r r en t a ge 40
Ret ir emen t a ge 65
Cu r r en t s a la r y $50,000
An n u a l gr owt h of s a la r y 3%
In t er es t r a t e ea r n ed by t h e
r et ir emen t fu n d
7% a n n u a l effect ive
Alt er n a t ive depos it pla n #1 At t h e en d of ea ch yea r depos it
% X of t h e s a la r y in t o t h e
r et ir emen t fu n d.
Alt er n a t ive depos it pla n #2 At t h e begin n in g of ea ch yea r ,
depos it % Y of t h e s a la r y in t o t h e
r et ir emen t fu n d.
Ca lcu la t e X Y .
195
http://actuary88.com
Solut ion

Well u s e $1,000 a s t h e u n it mon ey t o s implify ou r ca lcu la t ion .

Age 40 41 42 43 64 65
Time t 0 1 2 3 24 25
( ) % 50 X ( ) % 50 1.03 X ( )
2
% 50 1.03 X ( )
23
% 50 1.03 X ( )
24
% 50 1.03 X
250
At t =25
t h e pa ymen t fa ct or = ( )
24
% 50 1.03 X
t h e a n n u it y fa ct or =
25 i
s wh er e
7% 3%
3.8835%
1 3%
i

= =
+
( )
24
25
% 50 1.03 250
i
X s =
( )
24
25
250
% 6%
50 1.03
i
X
s
= =
Age 40 41 42 43 64 65
Time t 0 1 2 3 24 25
( ) % 50 Y ( ) % 50 1.03 Y ( )
2
% 50 1.03 Y ( )
23
% 50 1.03 Y ( )
24
% 50 1.03 Y
250
At t =25
t h e pa ymen t fa ct or = ( )
25
% 50 1.03 Y
t h e a n n u it y fa ct or =
25 i
s wh er e
7% 3%
3.8835%
1 3%
i

= =
+
( )
25
25
% 50 1.03 250
i
Y s =
( )
25
25
250
% 5.607%
50 1.03
i
Y
s
= =


6 5.607 0.393 X Y = =
196
http://actuary88.com
Proble m 1 2 (SOA May 2 0 0 1 EA-1 #4 )
Da t e of loa n : 1/ 1/ 2001
Amou n t of loa n : $100,000
Fr equ en cy of pa ymen t s : Qu a r t er ly
Da t e of 1
s t
pa ymen t : 3/ 31/ 2001
# of pa ymen t s : 120

Amou n t of ea ch of t h e 1
s t
110 r epa ymen t s : $3,100

Amou n t of ea ch of t h e la s t 10 r epa ymen t s :
In it ia l r epa ymen t of $ X , t h en dou blin g ever y qu a r t er t h er ea ft er

In t er es t r a t e: 12% per yea r , compou n din g qu a r t er ly

Ca lcu la t e t h e a mou n t of t h e fin a l pa ymen t .

Solut ion
Let s u s e $1,000 a s on e u n it of mon ey a n d a qu a r t er a s on e u n it of t ime.
Time 0 1 2 3 110 111 112 113 120
Amount $3.1 $3.1 $3.1 $3.1 $ X $ 2X $
2
2 X $
9
2 X
$100 PV =
Th e in t er es t r a t e per qu a r t er is
12%
3%
4
i = = .
Let s br ea k down t h e ca s h flows in t o t wo s t r ea ms :

St r ea m #1
Time 0 1 2 3 110
Amount $3.1 $3.1 $3.1 $3.1
110
3.1
i
PV a =
St r ea m #2
Time 0 1 2 3 110 111 112 113 120
Amount $ X $ 2X $
2
2 X $
9
2 X
10 j
PV X a =
Wh er e
3% 1
48.5%
1 1 1
i k
j
k

= = =
+ +
(h er e 1 k = )
197
http://actuary88.com
Th e pr es en t va lu e of t h es e t wo s t r ea ms of ca s h flows s h ou ld be $100 a t
0 t = .
111
110 3% 10 48.5%
100 3.1 1.03 a X a

= +
110 3%
32.04275602 a =
10 48.5%
808.0213752 a

=
( )
( )
110 3%
111 111
10 48.5%
100 3.1
100 3.1 32.04275602
0.02197516
1.03 1.03 808.0213752
a
X
a


= = =


Alt er n a t ively, we ca n ca lcu la t e t h e pr es en t va lu e of t h e 2
n d
s t rea m of t h e
ca s h flows a t 110 t = :
St r ea m #2
Time 0 1 2 3 110 111 112 113 120
Amount $ X $ 2X $
2
2 X $
9
2 X
10
2
j
X
PV a =
110
110 3% 10
100 3.1 1.03
2
j
X
a a

= +
10 10 48.5%
1, 568.973544
j
a a

= =
( )
110 3%
110
10 110
100 3.1
100 3.1 32.04275602
0.02197516
1, 568.973544
1.03
1.03
2
2
j
a
X
a


= = =

Th e fin a l r epa ymen t (i.e. t h e 120


t h
r epa ymen t ) is :

( )
9 9
2 2 0.02197516 11.25127973 X = =
Sin ce on e u n it of mon ey r epr es en t s $1,000, t h e fin a l r epa ymen t is
$11,151.28. So t h e a n s wer is B.
198
http://actuary88.com
Proble m 1 3 (SOA May 2 0 0 5 EA-1 #1 7 )
Smit h r et ir es on 1/ 1/ 2005 a n d r eceives h is r et ir emen t ben efit a s
mon t h ly a n n u it y pa ya ble a t t h e en d of ea ch mon t h for a per iod cer t a in of
20 yea r s .

Th e ben efit for t h e 1
s t
yea r is $2,000 per mon t h . Th is mon t h ly ben efit is
in cr ea s ed a t t h e begin n in g of ea ch yea r t o be 5% la r ger t h a n t h e mon t h ly
pa ymen t in t h e pr ior yea r .

X is t h e pr es en t va lu e on 1/ 1/ 2005 of t h e r et ir emen t ben efit a t a
n omin a l in t er es t r a t e of 6%, con ver t ible mon t h ly.

In wh a t r a n ge is X ?
(A) Les s t h a n $405,000
(B) $405,000 bu t les s t h a n $410,000
(C) $410,000 bu t les s t h a n $415,000
(D) $415,000 bu t les s t h a n $420,000
(E) $420,000 or mor e

Solut ion
Well do t wo t h in gs t o s implify ou r ca lcu la t ion . Fir s t , well u s e $1,000 a s
on e u n it of mon ey. Secon d, well con ver t t h e 12 mon t h ly pa ymen t s a t
Yea r 1 in t o on e equ iva len t ca s h flow.

Con ver t t h e 12 mon t h ly pa ymen t s in Yea r 1 in t o on e ca s h flow a t 0 t = :
( )
12
2 2 11.61893207 23.2379
i
P a = = = wh er e
6%
0.5%
12
i = =
Well dr a w a ca s h flow dia gr a m for 20 yea r s pa ymen t s . Th e n ext yea r s
pa ymen t is 5% la r ger t h a n t h e pr eviou s yea r s pa ymen t .

Time t (years) 0 1 2 3 19
Payment
P
1.05P
2
1.05 P
3
1.05 P
19
1.05 P
Th e pr es en t va lu e of 20 yea r s pa ymen t s is :

20 j
P a , wh er e
5%
1 1 5%
r k r
j
k

= =
+ +

Time t (months) 0 1 2 3 11 12
Payment

$2 $2 $2 $2 $2 $2
199
http://actuary88.com



Because the unit time is one year, r is the annual effective return.

12
6%
1 1 6.16778%
12
r

= + =




6.16778% 5%
1.11217%
1 1 5%
r k
j
k

= = =
+ +


So the present value is:

( )
20 20 1.11217%
23.2379 23.2379 18.0418 419.2538
j
P a a = = =

Because one unit of money is really $1,000, so the present value is
$419,253.8.

The answer is D.

Problem 14 (SOA May 2002 EA-1 #2)

Annual payments into a fund: $10,000 at the end of year one, increasing
by $500 per year in the 2
nd
through the 10
th
years. After the 10
th
year,
each payment increases by 3.5% over the prior payment.

Interest rate: 7% compounded annually.

Calculate the accumulated value of the fund at the end of year 20.

Solution

Well use $100 as one unit of money to keep our calculation simple.

First, lets use a table to keep track of cash flows:











200
http://actuary88.com
Time t payment
0
1 $100
2 105
3 110
4 115
5 120
6 125
7 130
8 135
9 140
10 145
11 145*1.035
12 145*1.035
2
13 145*1.035
3
14
145*1.035
4
15
145*1.035
5
16
145*1.035
6
17
145*1.035
7
18
145*1.035
8
19
145*1.035
9
20
145*1.035
10

We a r e a s ked t o ca lcu la t e t h e fu n ds va lu e a t 20 t = . Th er e a r e ma n y wa ys
t o fin d t h e a n s wer . On e wa y is t o br ea k t h e ca s h flows in t o t wo s t r ea ms :

201
http://actuary88.com
St r ea m #1
Time t payment
0
1 $100
2 105
3 110
4 115
5 120
6 125
7 130
8 135
9 140
10 145
St r ea m #2
11 145*1.035
12 145*1.035
2
13 145*1.035
3
14
145*1.035
4
15
145*1.035
5
16
145*1.035
6
17
145*1.035
7
18
145*1.035
8
19
145*1.035
9
20
145*1.035
10

A fa s t wa y t o fin d t h e a ccu mu la t ed va lu e of s t r ea m #1 is t o u s e BA II Plu s
Ca s h Flow Wor ks h eet . En t er t h e followin g in t o Ca s h Flow Wor ks h eet :

202
http://actuary88.com
Time t payment

0 $0
CF0
1 100
C01
2 105
C02
3 110
C03
4 115
C04
5 120
C05
6 125
C06
7 130
C07
8 135
C08
9 140
C09
10 145
C10
Set I=7 (s o t h e in t er es t r a t e is 7%). You s h ou ld get NPV=840.9359126.
Th is is t h e PV of St r ea m #1 a t 0 t = . Th e a ccu mu la t ed va lu e of St r ea m #1
a t 20 t = is :

20
840.9359126 1.07 3, 254.156635 =
Th e a ccu mu la t ed va lu e of St r ea m #2 a t 20 t = is :

( )
10
10
145 1.035
j
s , wh er e
7% 3.5%
3.38164251%
1 3.5%
j

= =
+
Us in g BA II Plu s TVM, you s h ou ld get :

( )
10
10
145 1.035 2, 386.418027
j
s =
So a t 20 t = t h e a ccu mu la t ed va lu e of t h e en t ir e fu n d is :

3, 254.156635 2, 386.418027 5, 640.574662 + =
Beca u s e on e u n it is equ a l t o $100, t h e a ccu mu la t ed va lu e is
$564,000.57.

203
http://actuary88.com
Chapt e r 6 Re al vs . nominal int ere s t
rat e

Ke y point s :

1. Nomin a l in t er es t r a t e
t h e gr owt h r a t e of you r mon ey
in t er es t r a t e qu ot ed in t h e ma r ket

2. Rea l in t er es t r a t e
t h e gr owt h r a t e of you r pu r ch a s in g power

3. For mu la s

1 nominal interest rate
1 real interest rate
1 inflation rate
+
+ =
+
nominal interest rate -- inflation rate
real interest rate=
1+ inflation rate

real interest rate nominal interest rate -- inflation rate


Nominal Cash Flow
Real Cash Flow
1 inflation rate
=
+
4. Con s t a n t dolla r vs . r ea l dolla r

Cu r r en t dolla r s mon ey r eceived. If you get $100 on you r
pa r t t ime job la s t week, you get $100 cu r r en t dolla r s . You r
$100 cu r r en t dolla r s h a ve n ot been a dju s t ed for in fla t ion .

Con s t a n t or r ea l dolla r s -- dolla r s r epor t ed in t er ms of t h e
va lu e t h ey h a d on a pr eviou s da t e. Th e $100 you got la s t
week cou ld bu y 90 Big Ma cs fr om Ma cDon a lds . However ,
s ever a l yea r s a go, $50 cou ld bu y 90 Big Ma cs . So you r
cu r r en t $100 is wor t h on ly $50 in con s t a n t or r ea l dolla r s .
Con s t a n t or r ea l dolla r s a r e cu r r en t dolla r s a dju s t ed for
in fla t ion .

204
http://actuary88.com
5. Dis cou n t in g r u le

Dis cou n t n omin a l dolla r ca s h flows by t h e n omin a l in t er es t
r a t es

Dis cou n t r ea l dolla r ca s h flows by t h e r ea l in t er es t r a t es

Sample Proble ms

Proble m 1

St a r t in g fr om t h e cu r r en t yea r , ea ch yea r J oh n will t a ke ou t mon ey fr om
h is s a la r y a n d depos it it in t o a r et ir emen t fu n d. Th e r et ir emen t fu n d
ea r n s t h e ma r ket in t er es t r a t e. His goa l is t o a ccu mu la t e $100,000
con s t a n t dolla r s immedia t ely a ft er h e r et ir es .

Fa ct s :
J oh n s cu r r en t a ge 40
J oh n s Ret ir emen t a ge 65
Ma r ket in t er es t r a t e 7% a n n u a l effect ive
In fla t ion 3% per yea r
Alt er n a t ive depos it Pla n #1 Depos it X con s t a n t dolla r s a t t h e en d of
ea ch yea r in t o t h e r et ir emen t fu n d.
Alt er n a t ive depos it Pla n #2 Depos it Y n omin a l dolla r s a t t h e en d of
Yea r 1, ea ch s u bs equ en t a n n u a l depos it is
5% la r ger t h a n t h e pr eviou s on e.
[1] Us e t h e n omin a l dolla r met h od, ca lcu la t e t h e a n n u a l depos it a t t h e
en d of Yea r 1 for Pla n #1 a n d Pla n #2

[2] Us e t h e con s t a n t dolla r met h od, ca lcu la t e t h e a n n u a l depos it a t t h e
en d of Yea r 1 for Pla n #1 a n d Pla n #2

Solut ion

[1 ] Nominal dollar me t hod Plan #1
Us e n omin a l dolla r ca s h flows
Dis cou n t a ll n omin a l ca s h flows a t t h e n omin a l in t er es t r a t e

Th e n omin a l in t er es t r a t e is 7%. Th e in fla t ion is 3%.

205
http://actuary88.com
J oh n wa n t s t o a ccu mu la t e, fr om t =0 t o t =25, $100,000 in con s t a n t
dolla r s . $100,000 con s t a n t dolla r s a r e equ iva len t t o t h e followin g
n omin a l dolla r s : ( ) ( )
25 25
$100, 000 1 inflation 100, 000 1.03 209, 377.79 + = =
Age 40 41 42 43 65
Time t 0 1 2 3 25
Constant dollars X X X X
Nominal dollars 1.03X 1.03
2
X 1.03
3
X 1.03
25
X
Nominal dollars ( )
25
100, 000 1.03 209, 377.79 =
We h a ve a geomet r ic a n n u it y. Well a pply t h e 3 min u t e s cr ipt .

( )
25
25
25
annuity
factor
payment
factor
at 25
1.03 100, 000 1.03
j
t
X s
=
=

, wh er e
7% 3%
3.8835%
1 3%
j

= =
+
25
100, 000, 2, 439.12
j
Xs X = = (con s t a n t dolla r s )

Th e a ct u a l a n n u a l depos it a t t h e en d of Yea r 1 is :

1.03 2, 512.29 X = (n omin a l dolla r s )

Cons t ant dollar me t hod Plan #1

Us e con s t a n t dolla r ca s h flows
Dis cou n t con s t a n t dolla r ca s h flows a t t h e r ea l in t er es t r a t e

Th e r ea l in t er es t r a t e is
7% 3%
3.8835%
1 3%
j

= =
+
.
Age 40 41 42 43 65
Time t 0 1 2 3 25
Constant dollars X X X X
Constant dollars 100, 000
206
http://actuary88.com
Th is t ime, we h a ve a s imple a n n u it y.

25
100, 000, 2, 439.12
j
Xs X = = (con s t a n t dolla r s )

Th is ma t ch es t h e r es u lt in t h e n omin a l dolla r met h od.

In t h is pr oblem, t h e con s t a n t dolla r met h od is s impler . Th is is beca u s e
t h e pr oblem gives u s t wo figu r es , t h e level a n n u a l depos it a n d t h e
a ccu mu la t e va lu e of t h e fu n d, in con s t a n t dolla r t o begin wit h .

[2 ] Nominal dollar me t hod Plan #2

Th e fu n ds a ccu mu la t ed va lu e t =25:

( ) ( )
25 25
$100, 000 1 inflation 100, 000 1.03 209, 377.79 + = = (n omin a l dolla r s )

Age 40 41 42 43 65
Time t 0 1 2 3 25
Nominal dollars Y 1.05Y
2
1.05 Y
24
1.05 Y
Nominal dollars ( )
25
100, 000 1.03 209, 377.79 =
On ce a ga in , we u s e t h e 3 min u t e s cr ipt for t h e geomet r ic a n n u it y:

( )
25
24
25
annuity
factor
payment
factor
at 25
1.05 100, 000 1.03
k
t
s Y
=
=

, wh er e
7% 5%
3.8835%
1 5%
k

= =
+
2, 051.64 Y = (n omin a l dolla r s )

So t h e a ct u a l a n n u a l depos it a t t h e en d of Yea r 1 is $2,051.64

207
http://actuary88.com
Cons t ant dollar me t hod Plan #2

Age 40 41 42 43 65
Time t 0 1 2 3 25
Nominal dollars Y 1.05Y
2
1.05 Y
24
1.05 Y
Constant dollars 1.03
-1
Y (1.03
-2
)1.05Y (1.03
-3
)
2
1.05 Y (1.03
-25
)
24
1.05 Y
Constant dollars 100, 000
Ca s h flows in cr ea s e by t h e followin g r a t e:

1.05
1 1.9417%
1.03
r = =
We n eed t o a ccu mu la t e t h e ca s h flows a t t h e r ea l in t er es t r a t e:

7% 3%
3.8835%
1 3%
j

= =
+
On ce a ga in , we h a ve a geomet r ic a n n u it y.

( )

25 24
25
annuity
factor
payment
factor
at 25
1.03 1.05 100, 000
R
t
s Y

=
=

, wh er e
3.8835% 1.9417%
1.9048%
1 1 1.9417%
j r
R
r

= = =
+ +

Ma ke s u r e you a r e n ot los t in t h e a bove equ a t ion .

2, 051.63 Y = (n omin a l dolla r s )

Proble m 2

You in ves t ed in a for eign cou n t r y a n d ea r n ed a n a n n u a l effect ive r et u r n
of 2% for 5 yea r s . Th e cou n t r y exper ien ced defla t ion (n ega t ive in fla t ion ) of
3% per yea r effect ive du r in g t h e per iod.

Ca lcu la t e t h e r ea l r a t e of r et u r n per yea r over t h e 5 yea r per iod.

Solut ion

208
http://actuary88.com
Th e equ a t ion is :

1 nominal interest rate
1 real interest rate
1 inflation rate
+
+ =
+
We a r e given :

Th e n omin a l in t er es t r a t e = 2%
Th e in fla t ion r a t e = - 3%

So t h e r ea l in t er es t r a t e is :

( )
1 nominal interest rate 1 2%
real interest rate 1 1 5.15%
1 inflation rate 1 3%
+ +
= = =
+ +
209
http://actuary88.com
Chapt e r 7 Loan re payme nt and
amort izat ion

Th e bor r ower bor r ows a loa n a t 0 t = a n d pa ys ba ck t h e loa n
t h r ou gh n in s t a llmen t s

Loa n pr in cipa l bor r owed = PV of fu t u r e pa ymen t s

At a n y t ime t wh er e 0 t n s s a n d t is a n in t eger befor e t h e loa n is
fu lly pa id off, t h e ou t s t a n din g ba la n ce of t h e loa n ca n be ca lcu la t ed
u s in g on e of t h e t wo met h ods

1. Pros pe c t ive me t hod. Th e ou t s t a n din g ba la n ce is equ a l t o
t h e PV of r ema in in g pa ymen t s t o be pa id in t h e fu t u r e.

2. Re t ros pe c t ive me t hod. Th e ou t s t a n din g ba la n ce is equ a l t o
t h e or igin a l loa n pr in cipa l a ccu mu la t ed t o t les s t h a n t h e
pr eviou s pa ymen t s a ccu mu la t ed t o t .
(A s pecia l ca s e you n eed t o r emember ) A loa n is pa id off by level
pa ymen t s X pa ya ble a t t h e en d of ea ch per iod for n per iods . For
a n y t ime t wh er e 0 t n s s a n d t is a n in t eger , t h en
1. PV of t h e loa n a t t ime zer o =
n
X a
2. Th e ou t s t a n din g ba la n ce immedia t ely a ft er t h e t -t h pa ymen t
is
n t
X a

3. Th e pr in cipa l por t ion of t h e 1


s t
, 2
n d
, 3
r d
, , t -t h , .. n-t h
pa ymen t is
n
Xv ,
1 n
Xv

,
2 n
Xv

,,
1 n t
Xv
+
,, Xv . (As t ime
pa s s es , mor e a n d mor e goes t o t h e pr in cipa l). Not ice t h a t t h e
pr in cipa l r epa ymen t s n icely for m a geomet r ic pr ogr es s ion .
(However , t h e pr in cipa l r epa ymen t s n icely for m a geomet r ic
pr ogr es s ion on ly wh en t h e r epa ymen t s a r e level.)

4. Th e in t er es t por t ion of t h e 1
s t
, 2
n d
, 3
r d
, , k -t h , .. n-t h
pa ymen t is
( )
1
n
X v ,
( )
1
1
n
X v

,
( )
2
1
n
X v

, ,
( )
1
1
n t
X v
+
,
, ( ) 1 X v . (As t ime pa s s es , les s a n d les s goes t o t h e
pr in cipa l)

210
http://actuary88.com
Example 1
A loa n of $100,000 is r epa id t h r ou gh mon t h ly level pa ymen t s over t h e
n ext 20 yea r s , t h e 1
s t
pa ymen t du e on e mon t h fr om t oda y. Th e mon t h ly
pa ymen t s a r e ca lcu la t ed u s in g a n omin a l in t er es t r a t e of 12%
compou n din g mon t h ly.

Ca lcu la t e
Th e mon t h ly pa ymen t s
Th e pr in cipa l ou t s t a n din g immedia t ely a ft er 15
t h
pa ymen t
Th e pr in cipa l a n d in t er es t s plit of t h e 16
t h
pa ymen t

Solut ion
Us e a mon t h a s t h e compou n din g per iod

Time t 0 1 2 237 238 239 240
(Month)
Y Y Y Y Y Y
240 month payments


240
100, 000 Y a =
Th e mon t h ly in t er es t r a t e is :

( ) 12
12%
1%
12 12
i
i = = =
240 1%
240 1%
100, 000
100, 000 1,101.086 a Y
a
Y = = =
Time t 0 1 2 15 16 237 238 239 240
(Month)
Y Y Y Y Y Y Y
225 month payments

15
240 15 225
P
B Ya Ya

= =
211
http://actuary88.com
Next , well fin d t h e ou t s t a n din g ba la n ce immedia t ely a ft er t h e 15
t h

pa ymen t .

Pros pe c t ive me t hod:

15
1% 240 15 225 225
1,101.086 98, 372.815
P
B Ya Ya a

= = = =
Re t ros pe c t ive me t hod:

Time t 0 1 2 15 16 237 238 239 240
(Month)
Y Y Y Y Y Y Y
15 month payments

15
Ys
Th e va lu e of t h e pr in cipa l a ccu mu la t ed t o 15: t =
( )
15
100, 000 1.01 116, 096.896 =
Pr eviou s pa ymen t s fr om 1 t = t o 15 t = a ccu mu la t ed t o 15: t =
15 15 1%
1,101.086 17, 724.066 Ys s = =
So t h e ou t s t a n din g ba la n ce immedia t ely a ft er t h e 15
t h
pa ymen t is :
116, 096.896 17, 724.066 98, 372.83 =
Th e s ligh t differ en ce bet ween t h e r es u lt s gen er a t ed by t h e pr os pect ive
met h od a n d t h e r et r os pect ive met h od is du e t o r ou n din g.

Th e in t er es t por t ion of t h e 16
t h
pa ymen t : ( ) 98, 372.8 1% 983.73 =
Th e pr in cipa l por t ion of t h e 16
t h
pa ymen t : 1,101.09 983.73 117.36 =
Alt er n a t ively, we ca n u s e t h e memor ized r u le on h ow t o s plit a pa ymen t
in t o pr in cipa l a n d in t er es t .

Th e pr in cipa l por t ion :
( )
240 16 1
1 1
1,101.09 1.01 117.36
n t
Yv
+
+
= =
Th e in t er es t por t ion :
( ) ( )
240 16 1
1 1
1 1,101.09 1 1.01 983.73
n t
Y v
+
+
(
= =
(


212
http://actuary88.com
Example 2 (SOA 2 00 5 May EA-1 #3 )
Ter ms of a $1,000 loa n is s u ed by Smit h :
Len gt h of loa n : 20 yea r s
Pa ymen t s : Level a n n u a l pa ymen t s a t t h e en d of ea ch yea r
In t er es t : 5% n omin a l, con ver t ible s emi-a n n u a lly

Wh en Smit h r eceives ea ch pa ymen t , it is immedia t ely r ein ves t ed a t 6%,
compou n ded a n n u a lly.

R is t h e effect ive a n n u a l r a t e of in t er es t ea r n ed by Smit h on h is
combin ed in ves t men t s over t h e 20 yea r per iod.

In wh a t r a n ge is R ?
(A) Les s t h a n 5.57%
(B) 5.57%, bu t les s t h a n 5.62%
(C) 5.62%, bu t les s t h a n 5.67%
(D) 5.67%, bu t les s t h a n 5.72%
(E) 5.72% or mor e

Solut ion
At 0 t = Smit h in ves t s (i.e. givin g ou t ) $1,000 ca s h . In r et u r n , h e get s 20
a n n u a l pa ymen t s . To fin d Smit h s a n n u a l r a t e of r et u r n , we n eed t o fin d
Smit h s wea lt h a t 20 t = . Th en , we ca n ca lcu la t e Smit h s r et u r n by s olvin g
t h e followin g equ a t ion :

( )
20
1, 000 1 Smith's wealth @ 20 R t + = =
Let X r epr es en t t h e a n n u a l pa ymen t .

20
1, 000
i
Xa = , wh er e
2
5%
1 1 5.0625%
2
i
| |
= + =
|
\ .

Us in g BA II TVM or t h e a n n u it y immedia t e for mu la , we get 80.668 X = .
Smit h s wea lt h @ 20 t = is
20 6%
2, 967.4338 Xs = , u s in g TVM or a for mu la .

( )
20
1, 000 1 2, 967.4338 R + = , 5.59% R =
So t h e a n s wer is B.

213
http://actuary88.com
Example 3 (SOA 2 00 2 May EA-1 #1 )
Loa n r epa ymen t per iod: 5 yea r s
Begin n in g loa n a mou n t : $75,000

Repa ymen t Pla n #1: Level a n n u a l pa ymen t s a t t h e begin n in g of ea ch yea r

Repa ymen t Pla n #2: Level s emi-a n n u a l pa ymen t s a t t h e en d of ea ch 6
mon t h per iod

A =An n u a l pa ymen t u n der Repa ymen t Pla n #1

B =Tot a l pa ymen t s in a yea r u n der Repa ymen t Pla n #2

( ) 4
1, 000 76.225 d =
In wh a t r a n ge is t h e a bs olu t e va lu e of A B ?
(A) Les s t h a n $1,000
(B) $1,000 bu t les s t h a n $1,025
(C) $1,025 bu t les s t h a n $1,050
(D) $1,050 bu t les s t h a n $1,075
(E) $1,075 or mor e

Solut ion
Let
X =t h e level a n n u a l pa ymen t u n der Repa ymen t Pla n #1
Y =t h e level s emi-a n n u a l pa ymen t u n der Repa ymen t Pla n #2

Repa ymen t Pla n #1
time t (year) 0 1 2 3 4
payment
X X X X X
5
75, 000
i
X a =
wh er e i is t h e a n n u a l effect ive r a t e

Repa ymen t Pla n #2
time t
(6 month) 0 1 2 3 4 5 6 7 8 9 10
payment
Y Y Y Y Y Y Y Y Y Y
10
75, 000
j
Y a =
214
http://actuary88.com
Wh er e j is t h e s emi-a n n u a l effect ive r a t e

Next , we n eed t o ca lcu la t e i a n d j .
( )
4
4
4
1 0.076225
1 1
1 4 4
d
v
i
(
(
= = =
(
(
+


, 8% i =
( )
2
1 1 1.08 j i + = + = , 3.923% j =
Th en u s in g eit h er BA II Plu s TVM or a n n u it y for mu la s , we ca n s olve for
X a n d Y :
5
75, 000
i
X a = , 17, 392.81 X =
10
75, 000
j
Y a = , 9, 211.43 Y =
Th en A X = , 2 B Y =
( ) 2 17, 392.81 2 9, 211.43 1, 030.06 A B X Y = = =
Th e a n s wer is C.

Example 4 (SOA May 2 0 0 2 EA-1 #5 )
Two $10,000 loa n s h a ve t h e followin g r epa ymen t ch a r a ct er is t ics :

Loa n 1: Level qu a r t er ly pa ymen t s a t t h e en d of ea ch qu a r t er for five
yea r s .

Loa n 2: Mon t h ly in t er es t pa ymen t s on t h e or igin a l loa n a mou n t a t t h e
en d of ea ch mon t h for 48 mon t h s plu s a ba lloon r epa ymen t of
pr in cipa l a t t h e en d of t h e fou r t h yea r . Th e ba lloon r epa ymen t
will be ma de u s in g t h e a ccu mu la t ed va lu e of a s in kin g fu n d
cr ea t ed by level a n n u a l depos it s a t t h e begin n in g of ea ch of t h e
fou r yea r s .

Effect ive a n n u a l in t er es t r a t e on t h e loa n : 8%.
Effect ive a n n u a l in t er es t r a t e on t h e s in kin g fu n d: 9%.

A = Su m of r epa ymen t s u n der Loa n 1.
B = Su m of in t er es t pa ymen t s on Loa n 2 plu s s u m of s in kin g fu n d
pa ymen t s .

In wh a t r a n ge is t h e a bs olu t e va lu e of A B ?
215
http://actuary88.com
[A] Les s t h a n $875
[B] $875 bu t les s t h a n $950
[C] $950 bu t les s t h a n $1,025
[D] $1,025 bu t les s t h a n $1,100
[E] $1,100 or mor e

Solut ion

Loa n #1
time t (quarter) 0 1 2 3 19 20
payment X X X X X X
20
10, 000
i
X a = ,
1
4
1.08 1 1.94265% i = =
608.19 X = (u s in g BA II Plu s TVM or a n n u it y for mu la )

Loa n #2 in t er es t pa ymen t
time t (month) 0 1 2 47 48
Interest payment Y Y Y Y Y
( )
1 12
10, 000 1.08 1 64.34 Y = =
Loa n #2 in s t a llmen t in t h e s in kin g fu n d
time t (year) 0 1 2 3 4
installment
Z Z Z Z
4 9%
10, 000 Z s =
2, 006.13 Z =
20 A X = , 48 4 B Y Z = +
( ) ( ) ( ) 20 20 608.19 48 64.34 4 2, 006.13 1, 050.96 A B X B = = + = (


So t h e a n s wer is D.

216
http://actuary88.com
Example 5 (SOA May 2 0 0 2 EA-1 #6 )
Smit h obt a in s a loa n for $10,000 wit h 40 a n n u a l pa ymen t s a t a n
effect ive a n n u a l in t er es t r a t e of 7%. Th e fir s t pa ymen t is du e on e yea r
fr om n ow.

A = Su m of in t er es t pa id in t h e even -n u mber ed pa ymen t s .
B = Su m of pr in cipa l pa id in t h e odd-n u mber ed pa ymen t s .

In wh a t r a n ge is A B + ?
[A] Les s t h a n $13,800
[B] $13,800 bu t les s t h a n $14,200
[C] $14,200 bu t les s t h a n $14,600
[D] $14,600 bu t les s t h a n $15,000
[E] $15,000 or mor e

Solut ion D
Well u s e t h e ima gin a r y ca s h flow met h od in t r odu ced in Ch a pt er 1 How
t o bu ild a 3 min u t e s olu t ion s cr ipt .

time t (year) 0 1 2 3 4 39 40 41
Total Payment X
X X X X X X
Principal portion
40
Xv
39
Xv
38
Xv
37
Xv
2
Xv
Xv
Interest portion
( )
40
1 X v
( )
39
1 X v
( )
38
1 X v
( )
37
1 X v
( )
2
1 X v ( ) 1 X v
A (even #)
( )
39
1 X v
( )
37
1 X v ( ) 1 X v
B (odd #)
40
Xv
38
Xv
2
Xv
40 7%
10, 000 X a = , 750.09 X =
( ) ( ) ( )
39 37
1 1 ... 1 A X v X v X v = + + +
( )
41
39 37 35
2
20 ... 20
1
v v
X X v v v v X
v
| |
= + + + + =
|

\ .
1 41
2
1.07 1.07
750.09 20 9, 832.75
1 1.07

| |
= =
|

\ .

( )
2 42 2 42
40 38 2
2 2
1.07 1.07
... 750.09 4, 830.92
1 1 1.07
v v
B X v v v X
v

| |
= + + + = = =
|

\ .

9,832.75 4, 830.92 14, 663.37 A B + = + =
ima gin a r y
ca s h flow
217
http://actuary88.com
Example 6 (SOA May 2 0 0 2 EA-1 #7 )
Smit h pu r ch a s es a h ou s e for $120,000 a n d a gr ees t o pu t 20% down . He
t a kes ou t a 30-yea r mor t ga ge, wit h mon t h ly pa ymen t s , wit h t h e fir s t
pa ymen t on e mon t h a ft er t h e da t e of t h e mor t ga ge. Th e in t er es t r a t e is
8% compou n ded mon t h ly.

Immedia t ely following t h e 180
t h
pa ymen t , Smit h r efin a n ces t h e
ou t s t a n din g ba la n ce wit h a n ew 10-yea r mor t ga ge, a ls o wit h mon t h ly
pa ymen t s , wit h t h e fir s t pa ymen t on e mon t h a ft er t h e da t e of t h e n ew
mor t ga ge. Th e n ew in t er es t r a t e is 7.5% compou n ded mon t h ly.

A = Amou n t of in t er es t pa id in t h e 100
t h
pa ymen t of t h e fir s t mor t ga ge.
B = Amou n t of pr in cipa l pa id in t h e 100
t h
pa ymen t of t h e r efin a n ced
mor t ga ge.
In wh a t r a n ge is [A + B]?
[A] Les s t h a n $1,300
[B] $1,300 bu t les s t h a n $1,325
[C] $1,325 bu t les s t h a n $1,350
[D] $1,350 bu t les s t h a n $1,375
[E] $1,375 or mor e

Solut ion

time t (month) 0 1 2 180 181 360
payment X X X X X X X
( )
360
120, 000 80%
i
X a = , wh er e
8%
0.66667%
12
i = =
704.41 X = ,
( )
360 100 1
704.41 1 580 A v
+
= = @0.66667%

Th e ou t s t a n din g ba la n ce immedia t ely befor e t h e r efin a n cin g
time t (month) 0 1 2 180 181 360
payment X X X X X X X
180
73.710.30
i
P X a = =
Th e mon t h ly pa ymen t in t h e r efin a n ced mor t ga ge
time t (month) 0 1 2 180 181 360
payment Y Y Y
180
73.710.30
j
Y a =
218
http://actuary88.com
Wh er e
7.5%
0.625%
12
j = =
Th e n ew mon t h ly pa ymen t is
120 0.625%
73, 710.3
875
a
=
120 100 1
875 768 B v
+
= = @0.625%

1, 348 A B + =
Example 7 (SOA May 2 0 0 1 EA-1 #3 )

Amou n t of t h e loa n $100,000
# of or igin a lly s ch edu led level
a n n u a l r epa ymen t s
30
Time of 1
s t
r epa ymen t 1 yea r fr om t h e da t e of t h e loa n
Addit ion a l pa ymen t s ma de wit h t h e
5
t h
a n d 10
t h
s ch edu led r epa ymen t s
$5,000 ea ch
Effect ive a n n u a l in t er es t r a t e 6%
Su bs equ en t t o t h e t wo a ddit ion a l pa ymen t s , t h e loa n con t in u es t o be
r epa id by a n n u a l r epa ymen t s of t h e or igin a l s ize, plu s a s ma ll fin a l
r epa ymen t on e yea r a ft er t h e la s t fu ll r epa ymen t .

In wh a t r a n ge is t h e t ot a l a mou n t of in t er es t s a ved du e t o t h e t wo
a ddit ion a l pa ymen t s ?
(A) Les s t h a n $23,500
(B) $23,500 bu t les s t h a n $23,600
(C) $23,600 bu t les s t h a n $23,700
(D) $27,000 bu t les s t h a n $27,800
(E) $23,800 or mor e

Solut ion

Th is pr oblem is difficu lt . To s olve it , you ll n eed t o fin d ou t t h e or igin a lly
s ch edu led pa ymen t s a n d t h e a ct u a l pa ymen t s . To s implify ou r
ca lcu la t ion , well u s e $1,000 a s on e u n it of mon ey.

Let X r epr es en t t h e or igin a lly s ch edu led level a n n u a l r epa ymen t .

219
http://actuary88.com
Sch edu led r epa ymen t s :
Time t
(years) 0 1 2 5 10 .. .. .. 30
Repayments X X X X X X X X X X
30 6%
100 X a = , 7.26489115 X =
Act u a l r epa ymen t s :
Time t
(years) 0 1 2 5 10 .. m 1 m+
Repayments X X X 5 X + X 5 X + X X Y
last full repayment
small final repayment
To fin d m a n d Y , we ca n ca lcu la t e t h e ou t s t a n din g loa n ba la n ce a t 10 t =
immedia t ely a ft er bor r ower pa ys 5 X + . Fir s t , t h ou gh , well ca lcu la t e t h e
ou t s t a n din g loa n ba la n ce a t 10 t = immedia t ely a ft er t h e bor r ower pa ys X
a s s u min g n o ext r a r epa ymen t s a t 5 t = or 10 t = :
Time t
(years) 0 1 2 5 10 .. .. .. 30
Repayments
(scheduled)
X X X X X X X X X X
Extra 5 5
20 6%
scheduled
83.32772914 P X a = =
To fin d t h e a ct u a l loa n ba la n ce a t 10 t = , well n eed t o a ccou n t for t h e t wo
ext r a r epa ymen t s of 5 ea ch a t 5 t = or 10 t = :
( )
5
accumulating two extra
payments to 10
actual scheduled
5 1.06 5 71.63660125
t
P P
=
= + =


Next , we a r e r ea dy t o ca lcu la t e t h e # of a ct u a l fu ll r epa ymen t s a ft er 10 t = :
6%
actual
n
P X a = ,
6%
71.63660125 7.26489115
n
a =
15.37019964 n = (u s in g BA II Plu s TVM)
220
http://actuary88.com
So t h er e a r e 15 fu ll r epa ymen t s a ft er 10 t = .
Act u a l r epa ymen t s :
Time t
(years) 0 1 2 5 10 11 25 26
Repayments X X X 5 X + X 5 X + X X Y
last full repayment
small final repayment
actual
71.63660125 P =
At 10 t = , t h e PV of t h e 15 fu ll r epa ymen t s fr om 11 t = t o 25 t = a n d t h e PV
of Y mu s t a dd u p t o
actual
P :
( )
16
15 6%
actual
1.06 Y Xa P

+ =
( ) ( )
16 16
15 6% 15 6%
actual
1.06 71.63660125 7.26489115 1.06 2.73892982 Y P Xa a = = =
Th e t ot a l s ch edu led r epa ymen t s : ( ) 30 30 7.26489115 $217.94673447 X = =
Tot a l in t er es t pa id if s ch edu led r epa ymen t s a r e ma de:
30 100 217.94673447-100=117.94673447 X =
Th e t ot a l a ct u a l r epa ymen t s :
( ) 25 5 5 25 7.26489115 5 5 2.73892982 194.361208544 X Y + + + = + + + =
Tot a l a ct u a l in t er es t pa id:
194.361208544 100 94.361208544 =
In t er es t s a ved:
1 23.585525926=$23,585.525926 17.94673447 - 94.361208544 =
So t h e a n s wer is B.

Th e followin g is t h e s ch edu led vs . a ct u a l pa ymen t s (r ou n ded t o 2
decima ls ). In t h e exam, you don t r ea lly n eed t o wr it e ou t t h e a bove t a ble.
Her e I wr ot e t h e t a ble ju s t t o ma ke t h in gs clea r .

221
http://actuary88.com
time t
scheduled
repayments
actual
repayments
0
1 $7.26 $7.26
2 $7.26 $7.26
3 $7.26 $7.26
4 $7.26 $7.26
5 $7.26 $12.26
6 $7.26 $7.26
7 $7.26 $7.26
8 $7.26 $7.26
9 $7.26 $7.26
10 $7.26 $12.26
11 $7.26 $7.26
12 $7.26 $7.26
13 $7.26 $7.26
14 $7.26 $7.26
15 $7.26 $7.26
16 $7.26 $7.26
17 $7.26 $7.26
18 $7.26 $7.26
19 $7.26 $7.26
20 $7.26 $7.26
21 $7.26 $7.26
22 $7.26 $7.26
23 $7.26 $7.26
24 $7.26 $7.26
25 $7.26 $7.26
26 $7.26 $2.74
27 $7.26
28 $7.26
29 $7.26
30 $7.26
Total $217.95 $194.36
Th e in t er es t s a ved: $217.95 - $194.36 = 23.59=$23,590.

222
http://actuary88.com
Example 8 (SOA May EA-1 2 0 0 1 #6 )
Amou n t of t h e loa n : $25,000
Ter m of loa n : 8 yea r s
Loa n r epa ymen t s : Qu a r t er ly, a t t h e en d of ea ch qu a r t er
In t er es t r a t e: 8% per yea r , compou n ded s emia n n u a lly

Th e 11
t h
a n d 12
t h
s ch edu led pa ymen t s a r e n ot ma de.

Th e loa n is r en egot ia t ed immedia t ely a ft er t h e du e da t e of t h e 12
t h
(2
n d

mis s ed) s ch edu led r epa ymen t wit h t h e followin g pr ovis ion s :

13
t h
(1
s t
r en egot ia t ed) s ch edu led r epa ymen t : $ X
14
t h
t h r ou gh 32
n d
r epa ymen t s :

Ea ch even -n u mber ed r epa ymen t is $200 gr ea t er t h a n t h e
Immedia t ely pr oceedin g (odd-n u mber ed) r epa ymen t .

Ea ch odd-n u mber ed r epa ymen t is equ a l t o t h e immedia t ely
Pr ecedin g even -n u mber ed r epa ymen t .

Th e loa n is t o be complet ely r epa id over t h e or igin a l t er m.

In wh a t r a n ge is X ?
(A) Les s t h a n $250
(B) $250 bu t les s t h a n $255
(C) $255 bu t les s t h a n $260
(D) $260 bu t les s t h a n $265
(E) $265 or mor e

Solut ion C
Th e difficu lt y of t h is pr oblem is t o n ea t ly keep t r a ck of t h e complex
r epa ymen t s in t h e r en egot ia t ed loa n . On e s imple a ppr oa ch is t o
exh a u s t ively lis t a ll of t h e r epa ymen t s :

Time
(quarters)
loan repayments in the
original plan loan repayments in the revised plan
0 (borrow 250)
1 P P
2 P P
3 P P
4 P P
5 P P
6 P P
7 P P
8 P P
223
http://actuary88.com
9 P P
10 P P
11 P 0 (1
st
missed repayment)
12 P 0 (2
nd
missed repayment)
13 P X
14 P 2 X +
15 P 2 X +
16 P 4 X +
17 P 4 X +
18 P 6 X +
19 P 6 X +
20 P 8 X +
21 P 8 X +
22 P 10 X +
23 P 10 X +
24 P 12 X +
25 P 12 X +
26 P 14 X +
27 P 14 X +
28 P 16 X +
29 P 16 X +
30 P 18 X +
31 P 18 X +
32 P 20 X +
In t h e a bove t a ble, P is t h e level qu a r t er ly r epa ymen t or igin a lly
s ch edu led. Plea s e n ot e t h a t I u s ed $100 a s on e u n it of mon ey. Th is wa y,
t h e in cr emen t a l r epa ymen t of $200 in t h e r evis ed loa n becomes 2; t h e
loa n a mou n t ch a n ges fr om 25,000 t o 250. Th is ma kes it ea s ier for u s t o
keep t r a ck of t h e loa n r epa ymen t s in t h e r evis ed r epa ymen t pla n .

32
250
i
P a = ,
0.5
8%
1 1 1.98%
2
i
| |
= + =
|
\ .
10.6223 P =
Th e ou t s t a n din g loa n ba la n ce a t 10 t = immedia t ely a ft er t h e 10
t h

r epa ymen t is ma de is :
22 22 1.98%
10.6223 187.9560
i
P a a = = .Th is a mou n t
a ccu mu la t es t o ( )
3
187.9560 1 199.3454 i + = a t 13 t = . Th is ou t s t a n din g
pr in cipa l mu s t be pa id in t h e r en egot ia t ed loa n .

Th e r epa ymen t s in t h e r en egot ia t ed loa n con s is t of t h e followin g t wo ca s h
flow s t r ea ms :

224
http://actuary88.com
Time
(quarters)
loan repayment stream #1
in the renegotiated loan
loan repayment stream #2
in the renegotiated loan
13 X
14 X
2
15 X
2
16 X
4
17 X
4
18 X
6
19 X
6
20 X
8
21 X
8
22 X
10
23 X
10
24 X
12
25 X
12
26 X
14
27 X
14
28 X
16
29 X
16
30 X
18
31 X
18
32 X
20
Th e PV of t h e 1
s t
s t r ea m @ 13 t = :
20 1.98%
16.7069 X a X =
To ca lcu la t e t h e PV of t h e 2
n d
s t r ea m @ 13 t = , we s imply en t er t h e
followin g ca s h flows in t o BA II Plu s Ca s h Flow Wor ks h eet :
Time (quarters)
13

CF0=0
14 2
15 2
C01=2, F01=2

16 4
17 4
C02=4, F02=2
18 6
19 6
C03=6, F03=2
20 8
21 8
C04=8, F04=2
22 10
23 10
C05=10, F05=2
24 12
25 12
C06=12, F06=2
26 14
27 14
C07=14, F07=2
28 16
29 16
C08=16, F08=2
225
http://actuary88.com
30 18
31 18
C09=18, F09=2
32 20 C10=20, F10=1
Set I=1.98 (t h e in t eres t r a t e). Th e NPV=156.1209.
Th e PV of t h e t wo s t r ea ms s h ou ld be t h e ou t s t a n din g loa n ba la n ce a t
13 t = :
199.3454 156.1209+16.7069X = , 2.5872 $258.72 X = =
Example 9 (SOA May 2 0 0 1 EA-1 #5 )
On 1/ 1/ 2002, Smit h con t r ibu t es $2,000 in t o a n ew s a vin g a ccou n t t h a t
ea r n s 5% in t er es t , compou n ded a n n u a lly. On ea ch J a n u a r y 1 t h er ea ft er ,
h e ma kes a n ot h er depos it t h a t is 97% of t h e pr ior depos it . Th is
con t in u es u n t il h e h a s 20 depos it s in a ll.

On ea ch J a n u a r y 1 begin n in g on 1/ 1/ 2025, Smit h ma kes a n n u a l
wit h dr a wa ls . Th er e is t o be a t ot a l of 25 wit h dr a wa ls , wit h ea ch
wit h dr a wa l 4% mor e t h a n t h e pr ior wit h dr a wa l, a n d t h e 25
t h
wit h dr a wa l
exa ct ly deplet es t h e a ccou n t .

In wh a t r a n ge is t h e s u m of t h e wit h dr a wa ls ma de on 1/ 1/ 2025 a n d
1/ 1/ 2026?

(A) Les s t h a n $5,410
(B) $5,410 bu t les s t h a n $5,560
(C) $5,560 bu t les s t h a n $5,710
(D) $5,710 bu t les s t h a n $5,860
(E) $5,860 or mor e

Solut ion
Fir s t , let s or ga n ize t h e in fo in t o a t a ble. Well u s e $1,000 a s on e u n it of
mon ey. In t h e followin g t a ble, t h e re d n u mber s a r e depos it s ; t h e blue
n u mber s a r e wit h dr a wa ls . Let X r epr es en t t h e 1
s t
wit h dr a wa l.

1/1/2002 1/1/2003 1/1/2021 1/1/2025 1/1/2026 1/1/2049
Time t (years) 0 1 .. 19 23 24 47
Deposits
(withdrawals) 2
( ) 2 0.97

( )
19
2 0.97
X 1.04X
24
1.04 X
1
20
2
i
PV a =
2
25 j
PV X a =
Us in g t h e geomet r ic a n n u it y s h or t cu t , we kn ow t h a t a t 0 t = , t h e PV of
t h e depos it s is :

226
http://actuary88.com
1
20
2
i
PV a = , wh er e
( )
( )
5% 3%
8.24742268%
1 3%
i

= =
+
Plea s e n ot e t h a t t h e 97% r a t io is equ iva len t t o -3% in cr ea s e r a t e.

1
20 8.24742268%
2 20.87005282 PV a = =
At 23 t = , t h e PV of t h e t ot a l wit h dr a wa ls is :

2
25 j
PV X a = , wh er e
5% 4%
0.96153824%
1 4%
j

= =
+
2
25
22.34096995
j
PV X a X = =
Beca u s e t h e 25
t h
wit h dr a wa l deplet es t h e s a vin g a ccou n t , we h a ve:

( )
2 3
1 2

PV of withdrawals
accumulate deposits
at 23
to 23
1.05
t
t
PV PV
=
=
=


( )
2 3
20.870 22 0528 .340 2 1.05 96995X = , 2.8692963269 X =
So t h e s u m of t h e wit h dr a wa ls ma de on 1/ 1/ 2025 (1
s t
wit h dr a wa l) a n d
1/ 1/ 2026 (2
n d
wit h dr a wa l) is :

( ) 1.04 2.04 2.04 2.8692963269 5.853364507 $5, 853.364507 X X X + = = = =
Th e s u m is $5,853. Th e a n s wer is D.

227
http://actuary88.com
Example 1 0 (SOA May 2 0 0 1 EA-1 #8 )

Da t e of t h e loa n 1/ 1/ 2001
Da t e of 1
s t
r epa ymen t 12/ 31/ 2001
Fr equ en cy of r epa ymen t s An n u a l
Ter m of loa n 4 yea r s
Amou n t of ea ch r epa ymen t $1,000
1
1
v
i
=
+
Th e s u m of t h e pr in cipa l r epa ymen t s in yea r s on e a n d t wo is equ a l t o
2
10v t imes t h e s u m of t h e in t er es t r epa ymen t s in yea r s t h r ee a n d fou r .

Ca lcu la t e v .
Solut ion

Well u s e t h e ima gin a r y ca s h flow met h od. In a ddit ion , we u s e $1,000 a s
on e u n it of mon ey.

Date 1/1/2001 12/31/200212/31/200312/31/200412/31/200512/31/2006
time t (year) 0 1 2 3 4 5
repayment $1 $1 $1 $1 $1
principal portion
5 1 4
v v

=
5 2 3
v v

=
5 3 2
v v

=
5 4
v v

=
interest portion
4
1 v
4
1 v
4
1 v
4
1 v
Imaginary cash flow

We a r e t old:
( )
4 3 2 2
10 1 1 v v v v v + = +
2
20
0
11
v v + = ,
0.938 v = (ch oos e t h e pos it ive r oot beca u s e 0 v > )
228
http://actuary88.com
Example 1 1 (SOA May 2 0 0 1 EA-1 #9 )

Amou n t of a loa n $1,000
Da t e of loa n 1/ 1/ 2001
Ter m of loa n 30 yea r s
Da t e of 1
s t
r epa ymen t 1/ 1/ 2004
Fr equ en cy of r epa ymen t s Ever y 3 yea r s
In t er es t r a t e 4% per yea r , compou n ded a n n u a lly
Ca lcu la t e t h e pr in cipa l r epa id in t h e 5
t h
r epa ymen t .

Solut ion

Well u s e $1,000 a s on e u n it of mon ey.
Date 1/1/2001 1/1/2004 1/1/2007
time t 0 1 2 5 .. 10
repayment X X X X X X
principal repayment
6
X v
Th e in t er es t r a t e per 3 yea r s is
3
1.04 1 12.4864% i = =
10
1
i
X a = , 0.18052244 X =
( )
6
6
0.18052244 1 12.4864% 0.08911095 $89.11 X v

= + = =
So t h e pr in cipa l r epa id in t h e 5
t h
r epa ymen t is $89.11.

229
http://actuary88.com
Example 1 2 (SOA May 2 0 0 1 EA-1 #1 2 )
A $200,000, 30-yea r va r ia ble r a t e mor t ga ge loa n is obt a in ed. Th e 1
s t

mon t h ly pa ymen t is du e on e mon t h fr om t h e da t e of t h e loa n . At t h e t ime
t h e loa n is obt a in ed, t h e in t er es t r a t e is 7%, compou n ded mon t h ly.

On t h e 2
n d
a n n iver s ar y of t h e loa n , t h e in t er es t r a t e is in cr ea s ed t o 7.5%,
compou n ded mon t h ly.

On t h e 4
t h
a n n iver s a r y of t h e loa n , t h e in t er es t r a t e is in cr ea s ed t o 8%,
compou n ded mon t h ly, a n d r ema in s fixed for t h e r ema in der of t h e
mor t ga ge r epa ymen t per iod.

Ca lcu la t e t h e t ot a l in t er es t pa id on t h e loa n .

Solut ion
Well u s e $1,000 a s on e u n it of mon ey.
St ep 1 Ca lcu la t e t h e mon t h ly pa ymen t wh en t h e in t er es t r a t e is 7%

Time t
(month) 0 1 2 24 48 360
payment X
X X X X X X X
360
200
i
X a = , where
7%
12
i = .
In BA II Plu s TVM, s et PV= - 200, N=360, I/ Y=7/ 12. CPT PMT.

1.33060499 X =
Aft er fin din g X , don t er a s e t h e da t a in pu t s in TVM. You ll n eed t o r eu s e
t h es e in pu t s .

St ep 2 Ca lcu la t e t h e ou t s t a n din g loa n ba la n ce wh en t h e in t er es t
r a t e ch a n ges t o 7.5%
Time t
(month) 0 1 2 24 48 360
payment X
X X X X X X X
24
360 24 336 i i
P X a X a

= =
In TVM, s imply r es et N=336. CPT PV. You s h ou ld get :
24
195.7898947 P =
230
http://actuary88.com
St ep 3 Reca lcu la t e t h e level mon t h ly pa ymen t u n der t h e n ew
in t er es t r a t e of 7.5%

Time t
(month) 0 1 2 24 48 360
payment X
X X X Y Y Y Y
24
360 24 336 j j
P Y a Y a

= = , where
7.5%
12
j =
In TVM, s imply r es et I/ Y=7.5/ 12. CPT PMT. You s h ou ld get :

1.39572270 Y =
St ep 4 Ca lcu la t e t h e ou t s t a n din g loa n ba la n ce wh en t h e in t er es t
r a t e ch a n ges t o 8%
Time t
(month) 0 1 2 24 48 360
payment X
X X X X Y Y Y
48
360 48 312 j j
P Y a Y a

= =
In TVM, s imply r es et N=312. CPT PV. You s h ou ld get :
48
191.3502131 P =
St ep 5 Reca lcu la t e t h e level mon t h ly pa ymen t u n der t h e n ew
in t er es t r a t e of 8%

Time t
(month) 0 1 2 24 48 360
payment X
X X X Y Y Y Y
48
360 48 312 k k
P Z a Z a

= = , where
8%
12
k =
In TVM, s imply r es et I/ Y=8/ 12. CPT PMT. You s h ou ld get :
1.45923312 Z =
Th e t ot a l loa n r epa ymen t s : 24 24 312 X Y Z + +
Th e t ot a l in t er es t pa id on t h e loa n :
24 24 312 200 320.7125968 $320, 712.60 X Y Z + + = =
231
http://actuary88.com
Example 1 3 (SOA May 2 0 0 4 EA-1 #2 7 )
A loa n is ma de on 1/ 1/ 2004.

Loa n r epa ymen t s : 120 level mon t h ly pa ymen t s of in t er es t a n d pr in cipa l
wit h 1
s t
pa ymen t a t 2/ 1/ 2004.

In t er es t is ch a r ged on t h e loa n a t a r a t e of
( ) 12
7.5% i = .
Amou n t of in t er es t pa id in t h e 54
t h
pa ymen t of loa n = $100

P =Pr in cipa l ou t s t a n din g on t h e loa n a ft er t h e 90
t h
pa ymen t .

Ca lcu la t e P .
Solut ion

time t
(month) 0 1 2 54 90 120
payments X X X X X X X X
Us in g t h e ima gin a r y ca s h flow met h od, we kn ow t h a t t h e in t er es t por t ion
of t h e 54
t h
pa ymen t is :

( ) ( )
121 54 67
1 1 X v X v

= , wh er e X is t h e level mon t h ly pa ymen t



Next , we n eed t o fin d t h e mon t h ly effect ive in t er es t r a t e:

( ) 12
7.5%
0.625%
12 12
i
i = = =
( ) ( )
67
67
1 1 1 0.625% 100 X v X

(
= + =

, 293.02 X =
time t
(month) 0 1 2 54 90 120
payments X X X X X X X X
120 90 30 i i
P X a X a

= =
30 30
293.02 7, 992.98
i i
P X a a = = =
232
http://actuary88.com
Example 1 4 (SOA May 2 0 0 4 EA-1 #7 )
Det a ils of a loa n ma de on 1/ 1/ 2004:

# of pa ymen t s : 10
Amou n t of ea ch pa ymen t : $5,000
Da t e of 1
s t
pa ymen t : 12/ 31/ 2004
In t er es t r a t e: 8% compou n ded a n n u a lly

Immedia t ely a ft er t he 6
t h
pa ymen t , a n a ddit ion a l $10,000 pa ymen t is
ma de. Th e loa n is r e-a mor t ized over a lon ger t er m t o pr ovide for a n n u a l
pa ymen t of $1,000 a n d a fin a l s ma ller pa ymen t of X on e yea r a ft er t h e
la s t $1,000 pa ymen t .

Ca lcu la t e X .
Solut ion

Well u s e $1,000 a s on e u n it of mon ey.
Th e ou t s t a n din g loa n ba la n ce immedia t ely a ft er t h e 6
t h
pa ymen t is ma de:
Time t (year) 0 1 2 . 6 7 10
payment 5 5 5
4 8%
5 16.5606342 a =
At 6 t = , t h e bor r ower s pa ys a ddit ion a l $10 a n d immedia t ely r e-a mor t ize
t h e r ema in in g loa n ba la n ce. Aft er t h a t , h e pa ys $1 per yea r for n yea r s :

16.5606342 10
n i
a =
En t er t h e followin g in t o BA II Plu s TVM:
PV= - 6.5606342, PMT=1, I/ Y=8, FV=0. Pr es s CPT N.

You s h ou ld get : 9.66886977 N =
So a ft er t h e r e-a mor t iza t ion , t h e bor r ower pa ys 9 level pa ymen t s of $1
ea ch a n d pa ys a fin a l pa ymen t of X a ft er t h e 9
t h
pa ymen t

10
9
16.5606342 10
i
a Xv = +
Solvin g t h e a bove equ a t ion , we get :

0.67735471 $677.35471 X = =
233
http://actuary88.com
Example 1 5 (SOA May 2 0 0 0 EA-1 #2 4 )
Amou n t of a loa n : $250,000
Fr equ en cy of r epa ymen t s : qu a r t er ly, a t t h e en d of ea ch qu a r t er
# of r epa ymen t s : 100
In t er es t r a t e: 8% per yea r , compou n ded con t in u ou s ly

In wh ich r epa ymen t does t h e pr in cipa l compon en t fir s t exceed t h e
in t er es t compon en t ?

Solut ion
Let X r epr es en t t h e level qu a r t er ly r epa ymen t .

Time t
(quarter) 0 1 2 k . 100
Payment X X X X X X
As s u me t h a t in t h e k -t h r epa ymen t t h e pr in cipa l compon en t fir s t
exceeds t h e in t er es t compon en t .

Th e pr in cipa l compon en t of t h e k -t h r epa ymen t is
101 k
X v

; t h e in t er es t
compon en t is
( )
101
1
k
X v

.
We n eed t o s olve t h e followin g equ a t ion :

( )
101 101
1
k k
X v X v

>
101 101
1
k k
v v

> ,
101
2 1
k
v

> ,
101
0.5
k
v

>
We a r e given t h e for ce of in t er es t 8% o = . So t h e qu a r t er ly dis cou n t in g
fa ct or is :

8%
4 4
0.02
v e e e
o


= = =
101
0.5
k
v

>
( ) 0.02 101
0.5
k
e

>
( ) 0.02 101 ln 0.5 k > ,
ln 0.5
101 34.657
0.02
k < =

, 66.3426 k >
So t h e 67
t h
r epa ymen t is t h e 1
s t
t ime t h e pr in cipa l compon en t exceeds
t h e in t er es t compon en t .

234
http://actuary88.com
Example 1 6 (SOA May 2 0 0 0 EA-1 #1 3 )
Da t e of a loa n : 1/ 1/ 1990
Amou n t of loa n : $100,000
In t er es t r a t e: 12% per yea r , compou n ded mon t h ly
Ter m of loa n : 360 level mon t h ly r epa ymen t s
Fir s t r epa ymen t da t e: 2/ 1/ 1990

Immedia t ely a ft er ma kin g t h e 120
t h
r epa ymen t , t h e bor r ower decides t o
a dd $Q t o ea ch mon t h ly r epa ymen t s o t h a t t h e loa n will be r epa id a ft er
h a vin g ma de a t ot a l of 160 mon t h ly r epa ymen t s .

Ca lcu la t e $Q.
Solut ion

Or igin a l r epa ymen t s ch edu le wh er e X is t h e mon t h ly r epa ymen t
Time t
(month) 0 1 2 120 121 360
Cash flow X X X X X X X
360
100, 000
i
X a = wh er e
12%
1%
12
i = = .
1.028.612597 X =
240
93, 417.9957
i
P X a = =
So t h e ou t s t a n din g ba la n ce immedia t ely a ft er t h e 120
t h
pa ymen t ma de is
93,417.9957.

Act u a l r epa ymen t
Time t
(month) 0 1 2 120 121 160
Cash flow
X X X X X Q + X Q + X Q +
( )
40
93, 417.9957
i
X Q a = +
2, 845.100921 X Q + = 1, 816.488324 Q =
235
http://actuary88.com
Example 1 7
Pa ymen t s a r e ma de a t t h e en d of t h e yea r for 30 yea r s , wit h t h e pa ymen t
equ a l t o $12 for ea ch of t h e fir s t 20 pa ymen t s a n d $9 for ea ch of t h e la s t
10 pa ymen t s . Th e in t er es t por t ion of t h e 11
t h
pa ymen t is t wice t h e
in t er es t por t ion of t h e 21
s t
pa ymen t .

Ca lcu la t e t h e in t er es t por t ion of t h e 21
s t
pa ymen t .

Solu t ion

Fir s t defin e s ome s ymbols :

t
OB is t h e ou t s t a n din g loa n ba la n ce immedia t ely a ft er t h e
t th pa ymen t is ma de

t
K is t h e pa ymen t ma de a t t h e t th pa ymen t .
t t t
K I P = +

t
I is t h e in t er es t por t ion of t h e t th pa ymen t

t
P is t h e pr in cipa l por t ion of t h e t th pa ymen t

i is t h e effect ive in t er es t r a t e per pa ymen t per iod (1 yea r in t h is
pr oblem)

Th en
1 t t
I OB i

= a n d
1 t t t t t
P K I OB OB
+
= =
Time t (Yr) 0 1 2 10 11 12 20 21 22 30
Payment $12 $12 $12 $12 $12 $12 $9 $9 $9
10
10
10 10
12 9 OB a v a = +
20
10
9 OB a =
Th e in t er es t por t ion of t h e 11
t h
pa ymen t is
( )
10
11 10
10 10
12 9 I OB i a v a i = = +
Th e in t er es t por t ion of t h e 21
t h
pa ymen t is
( ) 21 20
10
9 I OB i a i = =
( ) ( )
10
10 10 10
12 9 2 9 a v a i a i + =
236
http://actuary88.com
( )
10
12 9 2 9 v + =
10
6
9
v =

( ) ( )
10
10
21 20
10
1 6
9 9 9 1 9 1 3
9
v
I OB i a i i v
i
| |
= = = = = =
|
\ .

Alt er n a t ive met h od t o ca lcu la t e
21
I a ft er we fin d t h a t
10
6
9
v = .
Pr os pect ively, t h e pa ymen t s a t t =21,22,,30 a r e level. So we ca n u s e t h e
ima gin a r y ca s h flow met h od t o ca lcu la t e t h e in t er es t por t ion of t h e
pa ymen t s ma de a t t =21,22,,30.

So we a dd a n ima gin a r y ca s h flow of $9 a t 31 t = .
Time t (Yr) 0 1 2 10 11 12 20 21 22 30 31
Payment $9 $9 $9 $9
Th e pr in cipa l por t ion of t h e 21
s t
pa ymen t is

31 21 10
21
6
9 9 9 $6
9
P v v

= = = =
Th e in t er es t por t ion of t h e 21
s t
pa ymen t is
21 21 21
9 6 $3 I K P = = =
Example 1 8
A loa n of $100 a t a n omin a l in t er es t r a t e of 12% con ver t ible mon t h ly is t o
be r epa id by 6 mon t h ly pa ymen t s . Th e 1
s t
mon t h ly pa ymen t s t a r t s on e
mon t h fr om t oda y. Th e fir s t 3 mon t h ly pa ymen t s a r e x ea ch ; t h e la s t 3
mon t h ly pa ymen t s a r e 3x ea ch .

A=t h e pr in cipa l por t ion of t h e 3
r d
pa ymen t
B =t h e in t er es t por t ion of t h e 5
t h
pa ymen t

Ca lcu la t e A B +
237
http://actuary88.com
Solut ion

Time t (month) 0 1 2 3 4 5 6
Payment x x x 3x 3x 3x
3
3 3
100 3 xa xv a = + wh er e i =1%

( ) ( ) ( )
3 3 3
3 3
100 100 100
8.692
1 3 1 3 1 3 1.01 2.941
x
v a v a

= = = =
+ + +

Time t (month) 0 1 2 3 4 5 6
Payment x 3x 3x 3x
2
OB
( ) ( )( )
1
2
3
3 1.01 8.692 3 8.692 2.941 84.536 OB v x xa

= + = + =
The interest portion of the 3
rd
payment:
3 2
84.536 1% 0.845 I OB i = = =
The principal portion of the 3
rd
payment:
3 3 3
8.692 0.845 7.847 P K I = = =
Time t (month) 0 1 2 3 4 5 6
Payment 3x 3x
4
OB
( )
( )
4
2
3 3 8.692 1.9704 51.38 OB xa = = =
Th e in t er es t por t ion of t h e 5
t h
pa ymen t is :
5 4
51.38 1% 0.514 I OB i = = =
Alternative method to calculate
5
I . Since prospectively the payments are level at 5, 6 t = ,
we can use the imaginary cash flow method. So we set a fake cash flow at 7 t = .
Time t (month) 0 1 2 3 4 5 6 7
Payment 3x 3x 3x
7 5 2
5
3 3 8.692 1.01 25.562 P x v

= = =
7 5
5 5 5
3 3 8.692 25.562 0.514 I K P x v

= = = =

3 5
7.847 0.514 8.361 P I + = + =
238
http://actuary88.com
Example 1 9
A loa n is r epa id by a n n u a l in s t a llmen t s of X a t t h e en d of ea ch yea r for
20 yea r s . Th e a n n u a l in t er es t r a t e is 6%.

Let A=t h e t ot a l pr in cipa l r epa id in t h e fir s t 5 yea r s
Let B=t h e t ot a l pr in cipa l r epa id in t h e la s t 5 yea r s

Ca lcu la t e A/ B.

Solut ion
( )
20 19 18 17 16
A X v v v v v = + + + +
( )
5 4 3 2
B X v v v v v = + + + +
15 15
/ 1.06 0.4173 A B v

= = =
Example 2 0
Pa ymen t s a r e ma de a t t h e en d of t h e yea r for 30 yea r s , wit h pa ymen t
equ a l t o 120 for ea ch of t h e fir s t 20 yea r s a n d 90 for ea ch of t h e la s t 10
yea r s . Th e in t er es t por t ion of t h e 11
t h
pa ymen t is t wice t h e in t er es t
por t ion of t h e 21
s t
pa ymen t . Ca lcu la t e t h e in t er es t por t ion of t h e 21
s t

pa ymen t .

Solut ion

Time t (Yr) 0 1 2 10 11 12 20 21 22 30
Payment 120 120 120 120 120 120 120 120 90 90 90 90
Th e ou t s t a n din g ba la n ce immedia t ely a ft er t h e 10
t h
pa ymen t is ma de:
( )
10
10
10 10
120 9 OB a v a = +
Th e in t er es t por t ion of t h e 11
t h
pa ymen t is
( )
10
11 10
10 10
120 90 I OB i a a v i = = +
Simila r ly, t h e in t er es t por t ion of t h e 21
s t
pa ymen t is :
( ) 21 20
10
90 I OB i a i = =
( ) ( )
10
10 10 10
120 90 2 90 a a v i a i + = ( )
10
120 90 2 90 v + =
10
6/ 9 v =
1/10
1
6
1 1 4.1380%
9
i v

| |
= = =
|
\ .

10
10
1 1 6/ 9
8.0555
4.1380%
v
a
i

= = =

( ) 21
10
90 90 8.0555 4.1380% 30 I a i = = =
239
http://actuary88.com
Chapt e r 8 Sinking fund

Ke y point s

At 0 t = , t h e bor r ower bor r ows t h e pr in cipa l.

At ea ch yea r for n yea r s , t h e bor r ower pa ys on ly t h e in t er es t
a ccu mu la t ed du r in g t h a t yea r , s t oppin g t h e ou t s t a n din g pr in cipa l
fr om gr owin g or declin in g. Th u s , t h e ou t s t a n din g pr in cipa l
immedia t ely a ft er t h e a n n u l in t er es t pa ymen t is a lwa ys equ a l t o
t h e or igin a l pr in cipa l a t 0 t = . (Th e bor r ower is on ly dea lin g wit h t h e
in t er es t du e on yea r by yea r ba s is . He is n ot wor r yin g a bou t pa yin g
t h e pr in cipa l a t t h is s t a ge.)

At t h e en d of t h e t er m of t h e loa n , t h e bor r ower pa ys a lu mp s u m
equ a l t o t h e bor r owed pr in cipa l, t er min a t in g t h e loa n . (Even t u a lly,
t h e bor r ower h a s t o pa y t h e pr in cipa l.).

To ma ke s u r e h e ca n even t u a lly pa y off t h e pr in cipa l, t h e bor r ower
per iodica lly depos it s mon ey in t o a fu n d. Th is fu n d a ccu mu la t es
wit h in t er es t (ca n be differ en t fr om t h e in t er es t r a t e u s ed t o
ca lcu la t e t h e a n n u a l in t er es t pa ymen t ). At t h e en d of t h e t er m of
t h e loa n , t h is fu n d a ccu mu la t es en ou gh mon ey t o pa y off t h e
pr in cipa l.

Sample proble ms

Proble m 1 (#1 2 SOA May 2 0 0 3 EA-1 )

A t en -yea r loa n of $10,000 a t a n 8% a n n u a l effect ive r a t e ca n be r epa id
u s in g a n y of t h e followin g met h ods :

I. Amor t iza t ion met h od, wit h level a n n u a l pa ymen t s a t t h e en d of
ea ch yea r .

II. Repa y t h e pr in cipa l a t t h e en d of t h e yea r s wh ile pa yin g 8%
a n n u a l effect ive in t er es t on t h e loa n a t t h e en d of ea ch yea r . Th e
pr in cipa l is r epa id by ma kin g equ a l a n n u a l depos it s a t t h e en d
of ea ch yea r in t o a s in kin g fu n d ea r n in g in t er es t a t 6% a n n u a l
effect ive s o t h a t t h e s in kin g fu n d a ccu mu la t es t o $10,000 a t t h e
en d of t h e 10
t h
yea r .

III. Sa me a s II, except t h e s in kin g fu n d ea r n s 8% a n n u a l effect ive.

IV. Sa me a s II, except t h e s in kin g fu n d ea r n s 12% a n n u a l effect ive.
240
http://actuary88.com
Ra n k t h e a n n u a l pa ymen t a mou n t s of ea ch met h od.

[A] I < II < III < IV
[B] II < I = III < IV
[C] I < IV < III < II
[D] IV < I < III < II
[E] Th e cor r ect a n s wer is n ot given by [A], [B], [C], or [D] a bove.

Solut ion

Wit h ou t doin g a n y ma t h , you s h ou ld kn ow

I = III
IV < III < II

Rea s on for t h e 1
s t
equ a t ion :
Th e loa n a n d s in kin g fu n d h a ve t h e s a me in t er es t r a t e. Con s equ en t ly,
ou r a n n u a l pa ymen t s u n der I a n d III s h ou ld be s a me. If we h a ve t h e
s a me in t er es t r a t e, it does n t ma t t er wh et h er we s epa r a t ely pa y t h e
a n n u a l in t er es t a n d t h e pr in cipa l (t h e s in kin g fu n d met h od) or we
combin e t h e a n n u a l in t er es t a n d t h e a n n u a l pr in cipa l in t o on e a n n u a l
pa ymen t (loa n a mor t iza t ion ). We h a ve t h e s a me pie, n o ma t t er h ow we
s lice it .

Rea s on for t h e 2
n d
equ a t ion :
II, III, a n d IV differ on ly in a n n u a l s in kin g fu n d pa ymen t s . An n u a l
depos it s t o t h e s in kin g fu n d u n der II, III, a n d IV mu s t ea ch accu mu la t e
t o $10,000 a t t h e en d of Yea r 10. As a r es u lt , t h e h igh er t h e in t er es t r a t e
in t h e s in kin g fu n d, t h e s ma ller t h e a n n u a l depos it . Beca u s e t h e in t er es t
r a t es ea r n ed in t h e s in kin g fu n d a r e 6%, 8%, a n d 12% r es pect ively in II,
III, a n d IV, t h e a n n u a l depos it t o t h e s in kin g fu n d is la r ges t in II, s ma ller
in III, a n d s ma lles t in IV.

So t h e cor r ect a n s wer is (E).

Alt er n a t ively, you ca n ca lcu la t e t h e a n n u a l pa ymen t for ea ch of t h e fou r
pla n s .

241
http://actuary88.com
Plan I
Time t 0 1 2 3 4 10
$X X X X X
8 10 %
10, 000 Xa = 1490.29 X = (Tot a l a n n u a l pa ymen t )
Plan II
Time t 0 1 2 3 4 10
$800 $800 $800 $800 $800
Time t 0 1 2 3 4 10
$A A A A A
6% 10
10, 000 As =
758.68 A = 1, 558.68 X A + = (Tot a l a n n u a l pa ymen t )
Plan III
Time t 0 1 2 3 4 10
$800 $800 $800 $800 $800
Time t 0 1 2 3 4 10
$B B B B B
8% 10
10, 000 Bs =
690.29 B = 1, 490.29 X B + = (Tot a l a n n u a l pa ymen t )
Plan IV
Time t 0 1 2 3 4 10
$800 $800 $800 $800 $800
Time t 0 1 2 3 4 10
$C C C C C
12% 10
10, 000 Cs =
569.84 C = 1, 369.84 X C + = (Tot a l a n n u a l pa ymen t )
242
http://actuary88.com
Proble m 2 (#1 7 SOA May 2 0 0 4 EA-1 )

Loa n t er m $100,000 bor r owed on 1/ 1/ 2004, is s u ed a t a 5%
a n n u a l effect ive in t er es t r a t e.
Loa n r epa ymen t No r epa ymen t s of pr in cipa l or in t er es t a r e ma de on
t h e loa n u n t il a s in kin g fu n d h a s a ccu mu la t ed t o pa y
t h e ba la n ce in fu ll. Th is occu r s 12/ 31/ 2019.
Sin kin g fu n d $10,000 a n n u a l depos it s fr om 12/ 31/ 2004 t h r ou gh
12/ 31/ 2011 a n d $5,000 a n n u a l depos it s begin n in g
on 12/ 31/ 2012. Th e s in kin g fu n d a ccu mu la t ion is
% i u n t il 12/ 31/ 2011 a n d % k t h er ea ft er .
Du r in g ca len da r yea r 2011, in t er es t a ccr u ed on t h e loa n a n d in t er es t
ea r n ed on t h e s in kin g fu n d a r e t h e s a me.

In wh a t r a n ge is % k ?
[A] Les s t h a n 6.10%
[B] 6.10% bu t les s t h a n 6.60%
[C] 6.60% bu t les s t h a n 7.10%
[D] 7.10% bu t les s t h a n 7.60%
[E] 6.60% or mor e

Solut ion

Th is is n ot a t ypica l s in kin g fu n d. In a t ypica l s in kin g fu n d, t h e bor r ower
pa ys t h e a n n u a l in t er es t du e. In a ddit ion , h e s et s u p a s in kin g fu n d t o
a ccu mu la t e t h e pr in cipa l a t t h e en d of t h e loa n t er m.

In t h is pr oblem, h owever , t h e bor r ower s et s u p a s in kin g fu n d t o pa y, a t
t h e en d of t h e loa n t er m, bot h t h e pr in cipa l a n d t h e in t er es t du e.

You s h ou ldn t be s ca r ed. J u s t a pply t h e gen er a l pr in cipa l of t h e t ime
va lu e of mon ey a n d you s h ou ld do fin e.

To n ea t ly t r a ck down t h e t imin g of ea ch ca s h flow, well con ver t 12/ 31 of
a yea r t o 1/ 1 of t h e n ext yea r . For exa mple, we con ver t 12/ 31/ 2019 t o
1/ 1/ 2020. Th is h elps pr even t t h e off-by-on e er r or .

To s implify ou r ca lcu la t ion , well u s e $1,000 a s t h e u n it mon ey.

243
http://actuary88.com
Loan $100
5 %
1/1/04 1/1/05 1/1/06 1/1/11 1/1/12 1/1/13 1/1/14 1/1/20
Time t 0 1 2 7 8 9 10 16
Sinking Fund $10 $10 $10 $10 $5 $5 $5
% i % k
We ca n s et u p t wo equ a t ion s :

Th e s in kin g fu n d a n d t h e loa n s h ou ld a ccu mu la t e t o t h e s a me
a mou n t a t t =16.

( )
8
%
Accumulate fr
value of sinking fund
at 8 (8 parallel cash
flows of $10 each are
collasped to the final
cash flow time at 8);
the interest rate is %
8
10 1 %
i
t
t
i
k s
=
=
+

( )
16
%
Accumulate
loan fro
om
8 parallel cash flows
8 to 16 at %
of $5 each are collasped
to the final cash flow
time at 16; the interest
rate is %
8
5 100 1.05
k
t t k
t
k
s
= =
=
+ =


m 0
to 16 at 5%
t
t
=
=

Th e loa n a n d t h e s in kin g fu n d s h ou ld gen er a t e t h e s a me in t er es t


a mou n t fr om t =7 t o t =8

( )
7
7 %
Loan balance Sinking fund
at 7 balance at 7
100 1.05 5% 10 %
i
t t
s i
= =
=


Th e s econ d equ a t ion is ea s ier .

( )
( ) ( )
7
7
7
7 %
1 1
% % 1 % 1 10 1.05 5%
i
i
i i
i
i s
+
(
= + =

= , 7.907% i =
Solvin g t h e equ a t ion :

( ) ( )
8 16
8 7.907% 8 %
10 1 % 5 100 1.05
k
s k s + + = , ( )
8
8 %
106.021 1 % 5 218.287
k
k s + + =
We t r a n s la t e ( )
8
8 %
106.021 1 % 5 218.287
k
k s + + = in t o t h e follow ca s h flow
dia gr a m:
244
http://actuary88.com
Time t 0 1 2 3 4 5 6 7 8
cash 106.021 5 5 5 5 5 5 5 5
FV=218.287

In TVM, s et PV=106.021, PMT=5, N=8, FV= - 218.287.
Us e a n n u it y immedia t e mode. You s h ou ld get : I/ Y=5.98%
So t h e cor r ect a n s wer is [A].

Proble m 3 (#2 2 SOA May 2 0 0 0 EA-1 )
Da t e of a loa n : 1/ 1/ 2000
Amou n t of loa n : $X
Da t e of t h e 1
s t
r epa ymen t : 12/ 31/ 2000
Fr equ en cy of r epa ymen t s : An n u a lly
# of r epa ymen t s : 10
Amou n t of ea ch r epa ymen t : $1,000

Met h od of r epa ymen t :
On e h a lf of t h e loa n is r epa id by t h e a mor t iza t ion met h od u s in g a n
in t er es t r a t e of 7% per a n n u m compou n ded a n n u a lly.

Th e ot h er h a lf is r epa id by t h e s in kin g fu n d met h od wh er e t h e
len der r eceives 7% per a n n u m, compou n ded a n n u a lly, on t h is
por t ion of t h e loa n , a n d t h e s in kin g fu n d a ccu mu la t es a t 6% per
a n n u m, compou n ded a n n u a lly.

Ca lcu la t e $X .
Solut ion
On e h a lf of t h e loa n is r epa id t h r ou gh a mor t iza t ion met h od. Let A
r epr es en t t h e level a n n u a l r epa ymen t in t h e a mor t iza t ion met h od.

Time t (year) 0 1 2 . 10
Payment A A A A
10 7%
2
X
A a = ,
10 7%
2
X
A
a
=
245
http://actuary88.com
Th e ot h er h a lf is r epa id by t h e s in kin g fu n d met h od. Th e a n n u a l
r epa ymen t in t h is met h od con s is t s of t wo pa r t s :
An n u a l in t er es t pa ymen t of ( ) 7%
2
X
An n u a l depos it B in t o t h e s in kin g fu n d t o a ccu mu la t e t o
2
X
a t
10 t = . Th e in t er es t r a t e is 6%.

Time t (year) 0 1 2 . 10
Payment B B B B
10 6%
2
X
B s = ,
10 6%
2
X
B
s
=
So t h e t ot a l a n n u a l r epa ymen t is :

( )
10 7% 10 6%
7% 1, 000
2 2 2
X X X
a s
+ + =
( )
10 7% 10 6%
1 1
7% 1, 000
2
X
a s
(
+ + = (
(


10 7%
7.02358154 a = ,
10 6%
13.18079494 s =
$6, 938.53077 X =
246
http://actuary88.com
Chapter 9 Callable and non-callable
bonds

Bond is a standardized loan. The borrower borrows money from a lender.
The borrower pays back the borrowed money by paying regular
installments called coupons and a final lump sum (called redemption
value) in the end.
Key concepts:

Par or face value
For calculating the size of the coupon
Not necessarily the price that the investor pays for the bond or
the payment the investor receives at the bond maturity date

Redemption value
The amount the investor receives at the bond maturity date
excluding the coupon
If a problem doesnt specify the redemption value, assume the
redemption value is equal to the par (or face amount)

Coupon
the interest rate on the nominal amount of the bond
fixed throughout the life of the bond

Time to maturity
the length of time until the bond is redeemed

The market price of a coupon bond is calculated by discounting all
the future cash flows at the yield to maturity
The yield to maturity is the investors opportunity cost of
capital (i.e. return earned by investing in other assets).

Because investors demand to earn the prevailing market
interest rate, the yield to maturity is the prevailing market
interest rate. Here we assume a constant interest rate that
doesnt change with time (i.e. a flat yield curve).

YTM is the IRR an investor would realize by purchasing the
bond, holding it to maturity, and reinvesting coupons at YTM.

If the bond is purchased at a premium, YTM is less than the
coupon rate. Coupons will be reinvested a lower rate; the
247
http://actuary88.com
investor will realize a capital loss when the bond matures.

If the bond is purchased at a discount, YTM is greater than the
coupon rate. Coupons will be reinvested a higher rate; the
investor will realize a capital gain when the bond matures.

Bond prices change in the opposite direction from the change of
the prevailing market interest rate
If the market interest rate goes up, the price of a bond goes
down; we have to discount the future cash flows at a higher
discount rate.

If the market interest rate goes down, the price of a bond goes
up; we have to discount the future cash flows at a lower
discount rate.

Selling a bond at par, premium, discount
When the coupon rate is equal to the prevailing market interest
rates, the price of the bond is equal to its par value. The bond
sells at par value.

When coupon rate is below the prevailing market interest rates
at a certain point of time, the price of the bond will fall below its
par value. The bond sells at a discount.

When the coupon rate is above the prevailing market interest
rates at a certain point of time, the price of the bond will be
greater than its par value. The bond sells at a premium.

Bond cash flow diagram make sure you know how to draw one

Unit time = per coupon period
Time t 0 1 2 3 4 n
Coupon Coupon Coupon Coupon Coupon
Redemption
Set per coupon period as the unit time. If a bond pays coupons
every 6 months, then the unit time is 6 months.

Cash flows consist of regular coupons plus a final redemption
amount.
248
http://actuary88.com
If a bond pays 8% coupons convertible semiannually per $100
face amount, this means that you get $4 once every 6 months, not
$8 once every year. Make sure you remember this.

Big ideas:

1. Bond is a loan just like a student loan or home mortgage. If you
buy a bond issued by AT&T, you are lending your money to AT&T. The
principal amount you lend to AT&T is just the present value of the bond
cash flows (regular coupons plus face amount at the end of the loan
term) discounted at YTM. In turn, AT&T repays the loan through
installments (regular coupons plus face amount at the end of the loan
term). Everything you learn about loan (such as amortization) applies to
a bond. This is the most important concept you need to know about a
bond. Knowing this enables you to cut to the chase of many bond
problems.
2. Bond is a standardized loan.

Difference Bond Typical loan
Borrower
Government and corporations (called bond
issuers). They borrow money to fund special
projects or business growth.
Can be anyone.
Individuals,
corporations,
government.
Lender
Anyone who has idle money and wants to
earn interest. When you buy a bond, you
become a lender. Typically banks.
Payback method
Bond issuers pay back loans through regular
payments called coupons and a final payment.
Coupons are typically paid once every 6
months. A final payment is made at the end
of borrowing period.

In old days before computers, investors
actually cut off coupons from a coupon book
and mailed it to the issuing firm to claim
interest payment. This is the origin of the
name coupon.

Some bonds dont pay coupons. When a bond
matures, the borrower pays the principal plus
the interest accrued thus far. These bonds are
called zero-coupons.
The borrower and the
lender can negotiate
and agree on any
payback methods.
Can the lender sell the loan to
another investor who has idle
cash and wants to earn interest?
Bonds are standardized loan and can be easily
bought and sold in the open market.
Hard for a homeowner
to sell his mortgage to
other investors.
3. Why standardizing loan (i.e. inventing bond)?

249
http://actuary88.com
One-off borrowing is time-consuming. Corporations and
government always need extra money to expand business or
finance special projects (such as building highways). If every time
they need money they have to walk to a bank and fill out a loan
application, the borrowing process is time-consuming.

One-off borrowing is expensive. Banks often charge extra fees to
cover costs associated with loan approvals (such as checking
borrowers credits and negotiating loan terms). As a result, banks
often charge a higher interest rate to cover these fees. So
government and corporations rather not borrow money from
banks. By issuing bonds and borrowing money from the general
public, government and corporations lower their borrowing cost.

Standardized loans are easier for the general public to understand
than one-off loans.

Standardized loans can be sold and traded in the open market
(such as New York Stock Exchange and NASDAQ). If a lender
wants to back out, he can quickly sell the bond to someone else
(may incur a loss though). This helps the sales of the bonds.

Standardized loans allow multiple investors to split a large amount
of loan. If a company wants to borrow $100,000,000 cash, it
doesnt have to spend lot of time looking for one wealthy person or
a big bank to lend this amount. Instead, the company can simply
issue 100,000 bonds with each bond selling $1,000.

4. Call a bond = buy back the bond prior to the bonds maturity
= refinance the loan at a lower coupon rate

If you ever refinanced your student loan, or if your family
refinanced the house, youll understand a callable bond.

Refinancing a student loan. Say you borrowed the U.S. government
a $50,000 student loan at 8% interest per year. After you
graduated, you started to pay your student loan through monthly
payments. You were paying 8% interest per year. Gradually, you
reduced your loan balance to $45,000. Then the market interest
rate dropped to 4%. You didnt want to continue paying the U.S.
government 8% interest rate -- the market rate was only 4%. What
could you do? You walked to a bank, borrowed $45,000 at 4%, and
immediately mailed this $45,000 check to the U.S. government.
Now you no longer owed the U.S. government anything. You just
owed the bank $45,000 but at 4% interest rate.
250
http://actuary88.com
Call a bond = Refinance the bond. Once the bond is issued, the
cash flows of the bond are set. Bond issuers pay the set coupon
rates. However, if the interest rate drops in the future, it doesnt
make sense for the bond issuer to continue paying a high coupon
rate. As a result, the bond issuer will refinance the bond by
backing back the higher coupon bond and by issuing a new bond
with a lower coupon rate.

Say AT&T originally took out a loan from you (i.e. you bought a
bond issued from AT&T) and paid you annual coupons of 10%. A
few years later, the interest rate dropped to 4%. Would AT&T still
happily pay you 10% coupons when the market interest rate is
only 4%? No. If the bond is callable, AT&T will refinance the loan.
AT&T simply borrows money from someone else by issuing a bond
that pays a lower coupon rate (such as 4% or 4.5%). Then AT&T
immediately pays back what he still owes you at that time using
the proceeds it got from issuing the new lower coupon bond. The
net result: AT&T gets rid of the 10% coupon bond and replaces it
with 4% coupon bond. AT&T saves lot of money by calling a bond
(i.e. refinancing a bond).

In U.S., most corporate bonds are callable bonds. Call features are
attractive to bond issuers because bond issuers can refinance their
debt in the event that the interest rate drops.
If the interest rate drops, the issuer of a callable bond can issue a
brand new bond in the market at a lower borrowing rate;
simultaneously, the bond issuer buys back the old bond using the
proceeds generated by the brand new bond.
Before refinancing Your debt was $45,000; you were
paying 8% interest.
After refinancing Your debt was $45,000; you will pay
4% interest.
Before refinancing the bond
(i.e. calling the bond)
AT&T pays 10% coupons.
After refinancing the bond AT&T pays 4% coupons.
251
http://actuary88.com
However, if the interest rate stays level or goes up, the bond issuer
will not exercise the call option. It doesnt make sense to refinance
a debt at a higher interest rate. So call feature is an option, not a
duty. The issuer can call a bond if he wants to according to the
contract, but it doesnt have to call.
However, not every bond can be called. To make a bond callable,
the bond issuer must state, in the contract (called bond indenture),
that the bond to be issued is callable. Calling a bond is bad for the
bond holder. After the bond is called, the bond holder must give up
his high-yielding bond and look for another bond (a lower coupon
bond) to invest in.
Typically, when the interest rate drops a lot, callable bonds get
called. This is like student loan refinancing. When the interest rate
drops a lot, the number of student loans refinanced goes up.
5. Determine the highest price or the minimum yield of a callable
bond. SOA likes to test this type of problems.

Pricing approach assume the bond issuer will choose the
redemption date most detrimental to the bond holder (the worse
case scenario).

An investor will pay only the bottom price under the worse case
scenario. This way, the investor wont get burned.

How to determine the floor price and the redemption date for
a callable bond

(1) If the redemption amounts are constant for a range of callable
dates

If the modified coupon rate of the bond > the yield rate per
coupon period, call the bond ASAP. This makes intuitive
sense. If AT&T pays you 10% coupons semiannually, yet the
market rate is only 6% nominal compounding semiannually,
AT&T will gladly refinance the original 10% coupon bond by
issuing a 6% coupon bond.

If the modified coupon rate of the bond < the yield rate per
coupon period, dont call the bond and let it mature. This
makes intuitive sense. If AT&T pays you 6% coupons
semiannually, yet the market rate is 10% nominal
252
http://actuary88.com
compounding semiannually, it will be foolish for AT&T to
refinance the original 6% coupon bond by issuing a 10%
coupon bond. AT&T will simply ignore its call option.

If the modified coupon rate of the bond = the yield rate per
coupon period, calling the bond or not calling the bond
makes no difference.
(2) If the redemption amounts are not constant for a range of
callable dates,
Step #1 For each range of callable dates where the
redemption amount is constant, apply Rule (1) and
determine the redemption date and bond price.

Step #2 Among the prices calculated in Step #1, choose
the minimum price. This is the price to be paid by the
investor. The redemption date associated with this price is
the worst-case-scenario redemption date.

I recommend that you work through Brovermans Example 4.5 and
4.6 on page 246 and 247 (if you use the 3
rd
edition of his
Brovermans textbook).
Sample Problems

Problem 1

Bond face amount $1,000
Maturity 5 years
Coupons Zero
Selling price $725
Calculate the annual rate of return earned by the buyer of the bond.

Solution
This question is about zero coupon bonds.

Key points about zero coupon bonds (also called pure discount bonds):
Pay stated face or par value at maturity
Sold at a discount
Zero coupons are paid in the life of the bond
Like a saving account

253
http://actuary88.com
(Unit time =1 year)
Time t 0 1 2 3 4 5
Cash flow $0 $0 $0 $0 $1,000
$725
A zero coupon bond is like a saving account. To find the annual return i ,
we solve the following equation:

( )
5
725 1 1, 000 6.643% i i + = =
Problem 2

Bond issue date 1/1/2005
Maturity date 1/1/2010
Face amount $1,000
Coupons 8% payable 7/1 and 1/1
Redemption amount Par
Yield to maturity at purchase 10.25% annual effective
Explain whether the bond is a premium bond or discount bond;
Calculate the premium or discount.
Generate an amortization schedule.

Solution

First, we draw a cash flow diagram. Because coupons are paid once every
6 months, well use 6 months as unit time to simply our calculations.

Please note that the term of the bond is 5 years or 10 units of time.

(Unit time =6 months)
Time t 0 1 2 3 9 10
Cash flow $40 $40 $40 $40 $40
$1,000
( )
1
10
2
10
1, 000 @ 1 10.25% 1 5% 40 v i PV a + = + = =
922.78 PV =
254
http://actuary88.com
Please note that the price of the bond is always equal to PV of the bond;
PV of the bond is always equal to the cash flows discounted at YTM (yield
to maturity).

PV=922.78 < Face amount = 1,000

The bond is sold at a discount
- $77.22 Discount 922.78 1, 000 PV Face = = =
We can also intuitively see why this bond sold at a discount. The
coupons of the bond pay the buyer 4% per 6 months, while the market
interest rate is only 5% per 6 months. If the bond is still sold at par, the
bond issuer (the borrower) will underpay the bond buyer, creating
unfairness in the transaction. As a result, the bond issuer charges a
price below the par. The amount by which the selling price is below the
par amount is the discount.

To generate an amortization schedule of the bond, well treat the bond as
a loan. In this loan, the borrower (the bond issuer) borrows the present
value (or price) of the bond; he repays the loan by paying ten semiannual
payments of $40 each plus a final payment of $1,000 at the end of Year
5.

By treating a bond as a loan, we can amortize a bond the same way we
amortize a loan through the following steps:

Find the outstanding balance
t
P of the bond immediately after
repayment ( ) X t is made. We can calculate
t
P using the
retrospective or prospective method. Please note that the
repayment is $40 from 1 t = to 9 t = and $1,040 at 10 t = (the unit
time is 6 months).

Multiple
t
P with the effective interest rate per unit time i (5% in
this problem). This gives us ( )
t
Interest t i P = , the interest portion of
( ) X t .
Calculate ( ) ( ) ( ) Principal t X t Interest t = , the principal portion of
( ) X t .
255
http://actuary88.com
Following the above procedure, well get the follow amortization schedule:
Date Loan
repayment
Interest Principal Outstanding
balance of
the bond
1/1/2005 $922.78
7/1/2005 $40

$46.14
(1)
-$6.14
(2)
928.92
(3)
1/1/2006 40 46.45 -6.45 935.37
7/1/2006 40 46.77 -6.77 942.14
1/1/2007 40 47.11 -7.11 949.25
7/1/2007 40 47.46 -7.46 956.71
1/1/2008 40 47.84 -7.84 964.55
7/1/2008 40 48.23 -8.23 972.78
1/1/2009 40 48.64 -8.64 981.42
7/1/2009 40 49.07 -9.07 990.49
1/1/2010 1,040 49.52 990.48 0

Total 1,400 477.23 922.77

(1) 46.14 = 922.78(5%)
(2) -6.14 = 40 46.14
(3) 928.92 =922.78 - (-6.14)

From the above table, we see that a discount bond has a negative
amortization in all the payments except the final one. The earlier periodic
repayments do not even cover the interest due, creating negative
principal repayments (i.e. increasing the outstanding balance of the
loan).

Make sure you can manually create the above table.

Please also note that we can use the amortization method suggested by
the textbook:

K-th
payment
Outstanding
balance
Payment Interest portion Principal
portion
0
( ) 1
n j
F r j a
(
+

1
( )
1
1
n j
F r j a

(
+
(

F r
( )
( )
1
n
j
F j r j v
(
+

( )
n
j
F r j v
2
( )
2
1
n j
F r j a

(
+
(

F r
( )
( )
1
1
n
j
F j r j v

(
+

( )
1 n
j
F r j v

k
( ) 1
n k j
F r j a

(
+
(

F r
( )
( )
1
1
n k
j
F j r j v
+
(
+

( )
1 n k
j
F r j v
+

n-1
( )
1
1
j
F r j a
(
+
(

F r
( )
( )
2
1
j
F j r j v
(
+

( )
2
j
F r j v
n 0 F r F +
( )
( )
1
j
F j r j v
(
+

( )
j
F r j v
256
http://actuary88.com
Under this method, we have 1, 000, 4%, 5%, 10 F r j n = = = = . Lets
generate an amortization schedule for the first two payments:

t Outstanding balance Payment Interest portion Principal portion
0
( )
10 5%
1, 000 1 4% 5% a
(
+

922.78 =
1
( )
9 5%
1, 000 1 4% 5% a
(
+

928.92 =
( ) 1, 000 4%
40 =
( )( )
10
1, 000 5% 4% 5% 1 1.05

(
+

46.14 =
( )
10
1, 000 4% 5% 1.05

6.14 =
2
( )
8 5%
1, 000 1 4% 5% a
(
+

935.37 =
( ) 1, 000 4%
40 =
( )( )
9
1, 000 5% 4% 5% 1 1.05

(
+

46.45 =
( )
9
1, 000 4% 5% 1.05

6.45 =
We can also generate the amortization schedule using BA II Plus/BA II
Plus Professional Amortization Worksheet. Refer to Chapter 4 on how to
generate the amortization schedule.

Problem 3

Bond issue date 1/1/2005
Maturity date 1/1/2010
Face amount $1,000
Coupons 8% payable 7/1 and 1/1
Redemption amount Par
Yield to maturity at purchase 6.09% annual effective
Explain whether the bond is a premium bond or discount bond;
Calculate the premium or discount.
Generate an amortization schedule.

Solution

(Unit time =6 months)
Time t 0 1 2 3 9 10
Cash flow $40 $40 $40 $40 $40
$1,000
( )
1
10
2
10
1, 000 @ 1 6.09% 1 3% 40 v i PV a + = + = =
1, 085.30 PV =
257
http://actuary88.com
PV=1,085.30 > Face amount = 1,000
The bond is sold at a premium

Premium = PV Face amount = 1,085.30 1,000 = 85.30

We can also intuitively see why this bond sold at a premium. The
coupons of the bond pay the buyer 4% per 6 months, while the market
interest rate is only 3% per 6 months. If the bond is still sold at par, the
bond issuer (the borrower) will overpay the bond buyer, creating
unfairness in the transaction. As a result, the bond issuer charges a
price above and beyond the par. The amount by which the selling price
exceeds the par amount is the premium.

Amortization schedule:
Date Loan
repayment
Interest Principal Outstanding
balance of
the bond
1/1/2005 $1,085.30
7/1/2005 $40

$32.56
(4)
$7.44
(5)
1,077.86
(6)
1/1/2006 40 32.34 7.66 1,070.20
7/1/2006 40 32.11 7.89 1,062.31
1/1/2007 40 31.87 8.13 1,054.18
7/1/2007 40 31.63 8.37 1,045.81
1/1/2008 40 31.37 8.63 1,037.18
7/1/2008 40 31.12 8.88 1,028.30
1/1/2009 40 30.85 9.15 1,019.15
7/1/2009 40 30.57 9.43 1,009.72
1/1/2010 1,040 30.27 1,009.71 0

Total $1,400 $314.69 $1,085.29
(4) 32.56 = 1,085.30 (3%)
(5) 7.44 = 40 32.56
(6) 1,077.86

= 1,085.30 7.44

258
http://actuary88.com
Lets generate an amortization schedule for the first two payments using
the textbook method. We have 1, 000, 4%, 3%, 10 F r j n = = = = .
t Outstanding balance Payment Interest portion Principal portion
0
( )
10 3%
1, 000 1 4% 3% a
(
+

1, 085.30 =
1
( )
9 3%
1, 000 1 4% 3% a
(
+

1, 077.86 =
( ) 1, 000 4%
40 =
( ) ( )
10
1, 000 3% 4% 3% 1 1.03

(
+

32.56 =
( )
10
1, 000 4% 3% 1.03

7.44 =
2
( )
8 3%
1, 000 1 4% 3% a
(
+

1, 070.20 =
( ) 1, 000 4%
40 =
( ) ( )
9
1, 000 3% 4% 3% 1 1.03

(
+

32.34 =
( )
9
1, 000 4% 3% 1.03

7.66 =
Make sure you can also generate the amortization schedule using BA II
Plus/BA II Plus Professional Amortization Worksheet.

Problem 4

Face amount of a
callable bond
$1,000
Coupon 8% annual
Purchase date 7/1/2005
Call date Any time between 7/1/2020 and 7/1/2026
Redemption amount Par
Calculate the maximum price the buyer of the bond will pay to guarantee
a yield of at least 7%.

Solution

The bond issuer pays 8% annual coupons, but the buyer of the bond is
content to lock in only 7%. Why cant the buyer get 8% return? Because
the market interest rate is below 8%. Why is the bond buyer happy to get
a minimum return of 7%? Because the market interest rate is volatile,
going up and down; by locking in the 7% floor rate, the buyer can sleep
well at night assured that hes getting at least 7%.

Because the buyer is happy to lock in 7%, the bond issuer will gladly
refinance the bond at 7% at the earliest callable date. This way, the bond
issuer will pay only a 7% annual interest rate on its refinanced debt,
instead of continuing paying 8% coupons. As a result, the bond issuer
will recall the bond at the earliest recallable date, which is 7/1/2020.

259
http://actuary88.com
(Unit time =1 year)
7/1/2005 7/1/2020
Time t 0 1 2 3 14 15
Cash flow $80 $80 $80 $80 $80
$1,000
15
15
80 1, 000 1, 091.08 PV a v = + = @ 7% i =
So the maximum price the buyer will pay is $1,091.08. If the bond sells
above this price, the buyers return for investing the bond will be lower
than 7%.

260
http://actuary88.com
Problem 5

Bond A Bond B
Face amount $100 $100
Coupons 7% payable
semiannually
4% payable
semiannually
Redemption 100% par 150% par
Term to maturity 10 years 10 years
Bond A and Bond B have the same yield to maturity and sells at the
same price. Calculate the price of each bond.

Solution
Unit time =6 months
A Time t 0 1 2 3 19 20
Cash flow $3.5 $3.5 $3.5 $3.5 $3.5
$100
B Time t 0 1 2 3 19 20
Cash flow $2 $2 $2 $2 $2
$150
A - B Time t 0 1 2 3 19 20
Cash flow $1.5 $1.5 $1.5 $1.5 $1.5
- $50
20
20
1.5 50 0 PV a v = =
5.07615296% i = Using BA II Plus/Professional TVM
Then the price of Bond A is:
20
20
3.5 100 80.48386442 a v + =
Then the price of Bond B is:
20
20
2 150 80.48386442 a v + =
261
http://actuary88.com
Problem 6 (May 2004 EA-1)
On 1/1/2005, Smith purchases a 20-year bond with a par value of
$1,000. The bond pays semi-annual coupons at an annual rate of 6%.
The bond is purchased to yield 5% per annual effective. When each
coupon is received, it is immediately reinvested at a rate of interest of 6%
per annum convertible quarterly.

In what range is Smiths effective annual rate of return over the term of
the bond?

[A] Less than 5.20%
[B] 5.20% but less than 5.30%
[C] 5.30% but less than 5.40%
[D] 5.40% but less than 5.50%
[F] 5.50% or more

Solution

Unit time = 6 months
Time 0 1 2 3 40
Amount $0 $30 $30 $30 $30
$1,000
20 years
Smiths cost of buying
the bond at time zero
40
40
30 1, 000 @ 1.05 1 v i a + =
Smiths total wealth at t=40
= reinvest coupons @ 6% + receiving par
2
40
Receiving
Reinvest
Face
Coupons
amount
6%
30 1, 000 @ 1 1
4
s j
| |
+ = +
|
\ .


Remember that reinvestment rate is
6% per year convertible quarterly
We can solve for r , Smiths annual rate of return:

262
http://actuary88.com
( )
( )
20
40
40 40
Ending Wealth
Initial Investment
30 1, 000 1 30 1, 000
i j
v r s a + + = +


Using BA II Plus/BA II Plus Professional TVM, we have:

40
40
30 1, 000 1,133.854761 @ 1.05 1 v i a + = =
2
40
6%
30 2, 273.610707 @ 1 1
4
s j
| |
= = +
|
\ .

( )
20
40
40
40
30 1, 000
2, 273.610707 1, 000
1 2.88715171
30 1, 000 1,133.854761
s
r
v a
+
+
+ = = =
+
The correct answer is D.

Callable bond

Problem 7
Facts about a callable bond:

Face $100
Coupon 6% semiannually
Redemption value $100
Call dates 10
th
through 15
th
years at par
Calculate
[1] Find the price of the bond to yield 8% convertible semiannually
[2] Find the price of the bond to yield 4% convertible semiannually

Solution
[1] Find the price of the bond to yield 8% convertible semiannually

Assume the bond is called immediately after the n -th coupon is paid.
Because the call dates must be in the 10
th
through 15
th
year, we have:

20, 21, 22,..., 30 n =
The price of the bond is:

3 100 @4% per coupon period
n
n
P v a = +
In the above equation, 3 is the semiannual coupon payment.
263
http://actuary88.com
We need to minimize P , the purchase price of this callable bond. This is
the maximum price the investor is willing to pay in order to lock in the
minimum return of
( ) 2
8% i = .
To minimize P , we need to change the formula:

3 100
n
n
P v a = +
1
1
n
n
n n
v
v i
i
a a

= =
( ) ( )
4% 4% 4%
3 100 1 3 100 1 4% 100
n n n n n
P i a a a a a = + = + =
4% n
a is an increasing function with n . To minimize P , we need to
maximize n . We choose 30 n = .
30
4% 30
3 100 1.04 82.708 P a

= + =
[2] Find the price of the bond to yield 4% convertible semiannually

3 100 @2% per coupon period
n
n
P v a = +
( ) ( )
2% 2% 2%
3 100 1 3 100 1 2% 100
n n n n n
P i a a a a a = + = + = +
To minimize P , we need to minimize n . So we set 20 n = .
( )
20
2% 20
3 100 1.02 116.35 P a

= + =
264
http://actuary88.com
Problem 8

Facts about a callable bond:
Face $1,000
Coupon 4% semiannually
Maturity 20 years if not called
Redemption value $1,000
2
nd
through 5
th
years at $1,050
6
th
through 10
th
years at $1,025
11
th
through 19
th
years at $1,010
Call dates
20
th
year at $1,000
Calculate the maximum price an investor is willing to pay in order to lock
in a yield of 6% convertible semiannually.

Solution

The solution process is similar to the process used for the last problem.

The price of the bond is:

( )
3% 3%
20 1050 1050 20 1050 3% 1050 11.5
n
n n n
P v a a a = + = + =
Where 4, 5, 6, 7, 8, 9,10 n =
P reaches minimum when 10 n = .
( )
10
3% 10
min 20 1050 1.03 951.903 P a

= + =
( )
3% 3%
20 1025 1025 20 1025 3% 1025 10.05
n
n n n
P v a a a = + = + =
Where 12,13,14,..., 20 n =
P reaches minimum when 20 n = .
( )
20
3% 20
min 20 1025 1.03 865.067 P a

= + =
( )
3% 3%
20 1010 1010 20 1010 3% 1010 10.3
n
n n n
P v a a a = + = + =
Where 22, 23, 24,..., 38 n =
P reaches minimum when 38 n = .
( )
38
3% 38
min 20 1010 1.03 778.328 P a

= + =
( )
3% 3%
20 1000 1000 20 1000 3% 1010 10
n
n n n
P v a a a = + = + =
265
http://actuary88.com
Where 39, 40 n =
P reaches minimum when 40 n = .
( )
40
3% 40
min 20 1000 1.03 768.85 P a

= + =
So 768.85 is the maximum price the investor is willing to pay to lock in a
yield of 6% convertible semiannually.

Problem 9 (May 2004 EA-1 #32)

Bond A Bond B
Face amount $100 $100
Coupon rate 6%, payable semi-annually 5%, payable semi-
annually
Redemption Par $125
Length of bond 20 years 20 years
Both bonds have the same purchase price and the same yield rate.

Calculate the annual effective yield on these two bonds.

Solution

Bond A
Time t
(6 months) 0 1 2 3 40
Cash flows $3 $3 $3 $3 $103
Bond B
Time t
(6 months) 0 1 2 3 40
Cash flows $2.5 $2.5 $2.5 $2.5 $127.5
Cash flows of A B
Time t
(6 months) 0 1 2 3 40
Cash flows $0.5 $0.5 $0.5 $0.5 - $24.5
Because Bond A and B have the same price, the present value of A B
should be zero.

266
http://actuary88.com
There are at least two ways to calculate the effective yield.

Method 1 use BA II Plus Cash Flow Worksheet

Enter the following into Cash Flow Worksheet:

CF0=0, C01=0.5, F01=39, C02= - 24.5.

Press IRR CPT.

You should get: IRR=1.10894155

So the 6-month effective yield is 1.10894155%. The annual effective yield
is:

( )
2
1 1.10894155% 1 2.23018061% + =
Method 2 Use BA II Plus TVM Worksheet

We can rewrite the cash flows of A B as follows:
Time t
(6 months) 0 1 2 3 40
Cash flows $0.5 $0.5 $0.5 $0.5 $0.5 - $25
Enter: PMT=0.5, FV= - 25, N=40, PV=0.
Press CPT I/Y.

You should get: I/Y= 1.10894155.

So the 6-month effective yield is 1.10894155%. The annual effective yield
is:

( )
2
1 1.10894155% 1 2.23018061% + =
267
http://actuary88.com
Problem 10 (May 2004 EA-1 #28)

For a given bond:
Par value $1,000
Redemption value $1,100
Term of bond 10 years
Coupons % r per year, payable semiannually
Issue price P if yield to maturity is 4%, compounded annually
Issue price 95.5 P if yield to maturity is 5%, compounded annually
Calculate % r .
Solution

Time t
(6 months) 0 1 2 3 20
Cash flows
5r 5r 5r 5r 1100 5r +
Bond price is P if yield to maturity is 4%, compounded annually:

( )
10
20
5 1100 1.04
i
P r a

= + , where
1
2
1.04 1 1.98039027% i = =
Bond price is 95.5 P if yield to maturity is 5%, compounded annually:

( )
10
20
95.5 5 1100 1.05
j
P r a

= + , where
1
2
1.05 1 2.46950766% j = =
( ) ( )
10 10
20 20
5 1100 1.04 5 1100 1.05 95.5
i j
r a r a

+ = + +
( ) ( )
10 10
20 20
5 95.5 1100 1.04 1.05
i j
r a a

=
20
16.38241895
i
a = ,
20
15.63418569
j
a =
( )
( )
10 10
20 20
95.5 1100 1.04 1.05
7.4%
5
i j
r
a a


= =

268
http://actuary88.com
Problem 11 (May 2000 EA-1 #10)

Issue date of a bond: 1/1/1994
Term of bond; 15 years
Par value of bond: $10,000
Coupons: 8% per year, paid on June 30 and December 31
Amortized value on July 1, 2001: $13,741.11
Amortized value on January 1, 2002: $13,629.67
Calculate the redemption amount to be paid upon maturity.

Solution

Cash flows $400 $400 $400 $400 $400 $400 400 X +
Date 1/1/19946/30/19942/31/1994 6/30/200112/31/2001 1/1/2009
Time t (6months) 0 1 2
15 16
30
15
15
13, 741.11 400
i
a X v = +
14
14
13, 629.67 400
i
a X v = +
Let i represent the 6-month effective interest rate; X represent the
redemption amount at maturity.

Treat this bond as a loan. Then the coupons are just annual repayments
of the loan made by the borrower. The amortized value is just the loan
outstanding balance.

At 15 t = , the loan balance is $13,741.11. This amount accumulates to
( ) 3, 741.11 1 i + at 16 t = , at which time a loan repayment of $400 is made.
Then the loan balance becomes $13,629.67.

( ) 13, 741.11 1 400 13, 629.67 i + = , 2.1% i =
15
15
13, 741.11 400
i
a X v = + , 11,800 X = (using TVM)

Alternatively,
14
14
13, 629.67 400
i
a X v = + , 11,800 X = (using TVM)

269
http://actuary88.com
Problem 12 (May 2000 EA-1 #11)

Issue date of a bond: 1/1/2001
Coupon dates: 12/31/2002 and every two years
thereafter, with the final payment on 12/31/2010.

Coupon amount: $60 each.
Investors yield: 8% year annum
Price of the bond at issue: $691.49
Amortized value on 1/1/2005: $A
Amortized value on 1/1/2007: $B
Calculate A B .
Solution

Cash flow $60 60 60 60 60 F +
Date 1/1/2001 12/31/2002 12/31/2004 12/31/2006 12/31/2008 12/31/2010
Time t (2 years) 0 1 2 3 4 5
$691.49 A B
Here the unit time is 2 years. The effective interest rate per period is:

2
1.08 1 16.64% i = =
Well treat this bond as a loan. Then the amortized value of the bond is
the outstanding balance of the loan. Well use the respective method.
Under this method, the outstanding balance at 2 t = immediately after
coupon payment is the accumulated value of the original loan balance
less the accumulated value of the coupon payments:

( ) ( ) ( ) ( )
2 2
691.49 1 60 1 60 691.49 1.1664 60 1.1664 60 A i i = + + + = + ( (

810.7805110 =
Similarly,
( ) ( ) 1 60 810.7805110 1.1664 60 885.6943881 B A i = + = =
74.91 A B =
270
http://actuary88.com
Problem 13 (May 2002 EA-1 #9)
Face value of a bond: $1,000
Redemption value: $1,050
Time to maturity: 10 years
Coupon rate: 9% per year, convertible semi-annually
Yield rate: 10.25% per year
The bond is not callable.

Calculate the increase in the book value of the bond during the 3
rd
year.

Solution

Cash
flows $45 $45 $45 $45 $45 $45 $45 $45+1,050
Time t
(6 months) 0 1 2 3 4 5 6 20
16
4
16
45 1, 050
i
P a v = +
14
6
14
45 1, 050
i
P a v = +
The 6-month effective interest rate is ( )
0.5
1 10.25% 1 5% i = + = .
The book value of the bond immediately after the 4
th
coupon payment is:

16
4
16
45 1, 050 968.7267
i
P a v = + =
The book value of the bond immediately after the 6
th
coupon payment is:

14
6
14
45 1, 050 975.7602
i
P a v = + =
The increase in the book value of the bond during the 3
rd
year is:

6 2
975.7602 968.7267 7.043 P P = =
271
http://actuary88.com
Chapter 10 Valuation of stocks

Key points:

If you own a stock, you are entitled to receive future dividends. In
addition, you can sell the stock in the future.

You can think of a stock as a series of cash flows:

Time t 0 1 2 T
Cash flow
1
D
2
D
T T
D P +
0
P
The price of the stock is equal to the PV of future cash flows.

1 1
0
1
D P
P
i
+
=
+
(assume you buy the stock at t=0 and sell it at t=1)

2 2
1
1
D P
P
i
+
=
+
(assume the next owner buys the stock at t=1 and sells
it at t=1)

( )
1 1 1 2 1
0 2
1 1 1
1
D P D D P
P
i i i
i
+
= + = +
+ + +
+
3 3
2
1
D P
P
i
+
=
+
(assume the next owner buy the stock at t=2 and sell it
at t=3)

We can continue this line of thinking.

( ) ( ) ( )
1 2
0 2
1
... ...
1
1 1 1
t T
T t
t
t
D D D D
P
i
i i i
=+
=

= + + + + =
+
+ + +

Stock price = PV of future dividends.


272
http://actuary88.com
Constant growth model

Time t 0 1 2 T

Cash flow
1
D ( )
2 1
1 D D g = + ( )
1
1
1
T
T
D D g

= +
0
P
Dividend paid at the end of Year 1 =
1
D
Dividend paid at the end of Year 2 = ( )
2 1
1 D D g = +
Dividend paid at the end of Year 3 = ( )
2
3 1
1 D D g = +

Dividend paid at the end of Year T= ( )
1
1
1
T
T
D D g

= +
If the interest rate r is greater than the dividend growth rate g
( )
( )
( )
( )
1 1
1 1
0 2
1
1 1
... ...
1
1 1
T
T
D g D g
D D
P
i r g
i i

+ +
= + + + + =
+
+ +

Problems

Problem 1

Stock price $50
Dividend at the end of Year 1 $2
Dividends growth rate per year forever 8%
Calculate the return expected by investors.

Solution

1 1
0
0
2
8% 12%
50
D D
P r g
r g P
= = + = + =

So investors are expecting 12% return annually.



273
http://actuary88.com
Problem 2

Stock purchase date 3 months before the next dividend
is due
Next dividend $2 per share
Dividend payment Annual
Dividend growth rate 5% per year in perpetuity
Calculate the purchase price of the stock if investors want an 8% return
annual effective.

Solution

Method #1

Time t (year) 0 0.25 1 2 3 !
cash $2 2*1.05 2*1.05
2
2*1.05
!-1
( )
2 2 2
66.66667
1.05 1.05 1.05 2.85714%
j
a
j

= = = (geometric annuity shortcut)


where
8% 5%
2.85714%
1 5%
j

= =
+
The purchase price of the bond at 0.25 t = (3 month before the next
dividend date) is:
( )
0.25
66.66667 1.08 67.96 =
Method #2

Time t (year) 0 0.25 1 2 3 !
cash $2 2*1.05 2*1.05
2
2*1.05
!-1
..
2 2
2 72
1
1
1 2.85714%
j
j
a
d
= = =

+
Then the price of the bond at 0.25 t = is: 72(1.08
0.75
)=67.96

274
http://actuary88.com
Chapt e r 1 1 Pric e of a bond s old
be t we en t wo coupon payme nt s

Exa m FM Sa mple Qu es t ion s h a ve on e qu es t ion (#50) on t h is t opic.

Proc e dure t o de t e rmine t he marke t pric e s old be t we e n c oupon
dat e s :
Det er min e t h e pu r ch a s e pr ice of t h e bon d a s s u min g t h e 2
n d
own er
h a s 100% own er s h ip of t h e n ext cou pon

Det er min e t h e a ccr u ed in t er es t

Det er min e t h e ma r ket pr ice of t h e bon d u s in g t h e followin g
equ a t ion

Bon d (qu ot ed) ma r ket pr ice
= Bon d pu r ch a s e pr ice (a s s u min g 100% own er s h ip of t h e n ext cou pon )
- Accr u ed in t er es t a s of t h e la s t cou pon da t e
Explanat ionTo ca lcu la t e t h e bon ds ma r ket pr ice, we fir s t ign or e t h e fa ct
t h a t t h e or igin a l own er des er ves a por t ion of t h e n ext cou pon . We
pr et en d t h a t t h e 2
n d
own er pos s es s es 100% of t h e n ext cou pon (plu s a ll
t h e ot h er ca s h flows ). We t h en ca lcu la t e t h e pu r ch a s e pr ice of t h e bon d
a s s u min g t h e 2
n d
own er h a s 100% of t h e n ext cou pon .
Th e pu r ch a s e pr ice of a bon d (a s s u min g 100% own er s h ip of t h e n ext
cou pon )
= PV of t h e bon ds fu t u r e ca s h flows (in clu din g t h e n ext cou pon )
dis cou n t ed a t t h e ma r ket in t er es t r a t e.
Remember , wh en ever we dis cou n t a ca s h flow, we implicit ly a s s u me t h e
100% own er s h ip of t h is ca s h flow. If t h e own er s h ip is n ot 100%, we ca n
NOT dis cou n t t h is ca s h flow by it s fu ll a mou n t .
Next , we con s ider t h e fa ct t h a t t h e or igin a l own er des er ves a fr a ct ion of
t h e n ext cou pon .
If a bon d is s old bet ween t wo cou pon da t es , t h e bu yer of t h e bon d mu s t
compen s a t e t h e s eller for t h e fr a ct ion of t h e n ext cou pon pa ymen t t h e
s eller des er ves bu t will n ot r eceive. Th is a mou n t is ca lled a ccr u ed
in t er es t .
275
http://actuary88.com
Th er e a r e differ en t wa ys of ca lcu la t in g t h e a ccr u ed in t er es t . Let s n ot
wor r y a bou t it n ow.
Fin a lly, we ca lcu la t e t h e r ea l va lu e of t h e bon d by s u bt r a ct in g t h e
a ccr u ed in t er es t fr om t h e pu r ch a s e pr ice of t h e bon d. Th e logic h er e is
s imple. Th e pu r ch a s e pr ice of t h e bon d is over s t a t ed; it s ca lcu la t ed on
t h e a s s u mpt ion t h a t t h e 2
n d
own er get s t h e n ext cou pon 100% a n d t h e
or igin a l own er get s n on e. Sin ce t h e 2
n d
bu yer mu s t pa y t h e or igin a l
own er t h e a ccr u ed in t er es t , we n eed t o dedu ct t h e a ccr u ed in t er es t fr om
t h e pu r ch a s e pr ice of t h e bon d.
Now you kn ow t h a t t h e ma r ket pr ice of a bon d s old bet ween t wo cou pon
da t es is t h e PV of bon ds fu t u r e ca s h flows min u s t h e a ccr u ed in t er es t .
However , t h e a ct u a l ca lcu la t ion is mes s y beca u s e t h e n ext cou pon occu r s
a t a fr a ct ion a l t ime. Well u s e t h e followin g s t eps t o ca lcu la t e t h e ma r ket
pr ice of t h e bon d:
St e p #1 Find t he frac t ional t ime
days between settlement date and last coupon date
Total days per coupon period
a
k
b
= =
b
a
Last coupon date Settlement date Next coupon date
(i.e. Bond purchase date)
276
http://actuary88.com
We n eed t o u s e on e of t h e t wo da y cou n t met h ods t o ca lcu la t e k .
Day count
method
Explanation Example When used
360/360 Assume every month has 30
days and every year has 360
days.
The # of days
between 2/1/2005 and
3/1/2005 is 30 days.
For municipal
and corporate
bonds
Actual/Actual Use the actual # of days in a
month.
The # of days
between 2/1/2005 and
3/1/2005 is 28 days.
For treasury
bonds
In Exa m FM, if a pr oblem does n t s pecify wh ich da y cou n t met h od t o u s e,
u s e Act u a l/ Act u a l met h od.

St e p #2 Calc ulat e t he PV of t he bonds fut ure c as h flows .

Time zero is the last coupon payment date.
Time t 0 k 1 2 3 t n
Cash flow ( ) 1 CF ( ) 2 CF ( ) 3 CF ( ) CF t ( ) CF n
( )
( )
( )
( )
( )
( )
( )
( )
( )
1 2
1 1
1 2
...
1 1 1 1
n n
k k n k
t t
t k
x
t k
CF t CF CF CF n
PV CF t v
i i i i

= =

= = = + + +
+ + + +


277
http://actuary88.com
If we don t like fr a ct ion a l dis cou n t in g per iods s u ch a s 1 k , 2 k , , we
ca n ca lcu la t e t h e PV of t h e fu t u r e ca s h flows a t t=1 a n d t h en dis cou n t
t h e t h is PV t o t= k (s ee t h e dia gr a m below):

Time zero is the last coupon payment date.
Time t 0 k 1 2 3 t n
Cash flow ( ) 1 CF ( ) 2 CF ( ) 3 CF ( ) CF t ( ) CF n
( )
( ) ( )
( )
( )
( )
1 2 1
2 3
1 ...
1
1 1
n
CF CF CF n
PV CF
i
i i

= + + + +
+
+ +
1
Bond's book value @ the next coupon date PV =
( )
( )
1 1
1
Bond's book value @ the next coupon date
1
k
k k
PV
PV v
i

= =
+
Alt er n a t ively, we ca n ca lcu la t e t h e PV a t t =0 by dis cou n t in g a ll fu t u r e
ca s h flows t o t =0. Next , we a ccu mu la t e t h is PV a t t =0 t o t= k (s ee t h e
dia gr a m below):

Time t 0 k 1 2 3 t n
Cash flow ( ) 1 CF ( ) 2 CF ( ) 3 CF ( ) CF t ( ) CF n
( )
0
1
k
k
PV PV i = +
( ) ( ) Bond's book value @ the last coupon date 1
k
i = +
( ) ( )
( )
( )
( )
0 2
1 2
... Bond's book value @ the last coupon date
1
1 1
n
CF CF CF n
PV
i
i i
= + + + =
+
+ +

278
http://actuary88.com
St e p #3 Calc ulat e t he ac c rue d int e re s t .
days between settlement date and last coupon date
Total days per coupon period
a
k
b
= =
b
a
Last coupon date Settlement date Next coupon date
(i.e. Bond purchase date)
( ) ( )
a
Accrued Interest k coupon k Fr Fr
b
= = =
St e p #4 Calc ulat e t he (quot e d) marke t pric e .
Ma r ket Pr ice
= Pu r ch a s e pr ice of t h e bon d a t s et t lemen t a ccr u ed in t er es t .

Th e met h od ou t lin ed in t h e a bove fou r s t eps is t h e met h od mos t oft en
u s ed in t h e r ea l wor ld. Br over ma n des cr ibed t h is met h od in h is t ext book,
except h e didn t men t ion t h e da y cou n t met h od (per h a ps t o keep t h e
con cept s imple).

Kellis on , h owever , des cr ibed a myr ia d of wa ys t o ca lcu la t e t h e bon d
ma r ket pr ice in Ta ble 7.4 (pa ge 223).

Flat price (i. e.
t he bond
purchas e price )
Accrued int eres t Market price
Theoret ical
met hod
( ) 1
k
t
B i +
( )
i k
Fr s
( ) ( ) 1
k
t
i k
B i Fr s +
Pract ical met hod
( ) 1
t
B k i + ( ) k Fr ( ) ( ) 1
t
B k i k Fr +
Semi-t heoret ical
met hod
( ) 1
k
t
B i +
( ) k Fr
( ) ( ) 1
k
t
B i k Fr +
Un der t h e t h eor et ica l met h od,
k
s (wh er e k is a fr a ct ion ) does n t h a ve a n
in t u it ive expla n a t ion . However , SOA loves t h e complex con cept of
k
s . As
279
http://actuary88.com
a r es u lt , you n eed t o memor ize t h e t h eor et ica l met h od of ca lcu la t in g t h e
a ccr u ed in t er es t r a t e.

Plea s e n ot e t h a t TVM Wor ks h eet in BA II Plu s / BA II Plu s Pr ofes s ion a l
a llows N (t h e # of compou n din g per iods ) t o be a fr a ct ion . So you ca n u s e
TVM t o ca lcu la t e
k
s even wh en k is a fr a ct ion .
Let s look a t Kellis on s met h od:

Flat price (i. e. t he
bond purchas e price)
Accrued int eres t
Theoret ical met hod Compounding int eres t Compounding int eres t
Pract ical met hod Simple int eres t Simple int eres t
Semi-t heoret ical
met hod
Compounding int eres t Simple int eres t
So u n der Kellis on s met h od, bot h t h e fla t pr ice a n d t h e a ccr u ed in t er es t
ca n be ca lcu la t ed u s in g eit h er a s imple in t er es t r a t e or a compou n din g
in t er es t r a t e. Th er e is a t ot a l of t h r ee met h ods t o ca lcu la t e t h e ma r ket
pr ice t h eor et ica l, pr a ct ica l, a n d s emi-t h eor et ica l.

Beca u s e Kellis on s t h r ee met h ods a r e in t h e s ylla bu s , you migh t wa n t t o
memor ize h is t h r ee met h ods .

Example 1

Bon d fa ce a mou n t $100
Cou pon r a t e 4% s emia n n u a l
Cou pon pa ymen t s J u ly 1 a n d December 31
Is s u e da t e 1/ 1/ 2000
Ter m t o ma t u r it y 12/ 31/ 2004
Da t e wh en bon d is r es old 5/ 1/ 2002
Yield t o ma t u r it y t o t h e 2
n d
bu yer 6% s emia n n u a l
Wh a t s t h e ma r ket pr ice of t h e bon d a t t h e s et t lemen t da t e a s s u min g t h e
2
n d
bu yer h a s 100% own er s h ip of t h e n ext cou pon a n d a ll t h e ot h er ca s h
flows ? Us e a compou n din g in t er es t r a t e

280
http://actuary88.com
Solut ion

Unit time = 0.5 year
The interest rate per coupon period
6%
3%
2
i = =
12/31 5/1 7/1 12/31 12/31
2001 2002 2002 2002 2004
Time t 0 x 1 2 6
$2 $2 $2
$100
PV
0
PV
X
PV
1
Well dis cou n t fu t u r e ca s h flows occu r r in g a t t =1,2, , 6 t o t=x a t a
dis cou n t r a t e of 3% per cou pon per iod. Th e PV of t h es e ca s h flows is t h e
pu r ch a s e pr ice of t h e bon d a t t=x if we ign or e t h a t fa ct t h a t t h e pr eviou s
own er des er ves a n y por t ion of t h e n ext cou pon .

( ) ( ) ( ) ( ) ( ) ( )
1 2 6 1 2 6
2 2 102 2 2 102
... 1.03 ...
1.03 1.03 1.03 1.03 1.03 1.03
x
x x x x
PV

(
= + + + = + + + (
(

( )
6
3 6 %
1.03 2 100
x
x
PV v a = +
Next , we n eed t o fin d x .
days between settlement date and last coupon date
Total days per coupon period
x =
Met h od #1 360/ 360 da y cou n t met h od (for mu n icipa l a n d cor por a t e
bon ds )
Un der t h is met h od:

# of da ys bet ween 12/ 31/ 2001 a n d 5/ 1/ 2002 is 121 da ys .
281
http://actuary88.com
# of da ys bet ween 12/ 31/ 2001 a n d 7/ 1/ 2002 is 181 da ys .
How do we a ct u a lly cou n t t h e da ys ? Th ou gh we ca n do t h e ma t h
ou r s elves a s s u min g a mon t h h a s 30 da ys , we will let BA II Plu s / BA II
Plu s Pr ofes s ion a l Da t e Wor ks h eet cou n t t h e da ys for u s .

Key s t r okes in BA II Plu s / BA II Plu s Pr ofes s ion a l Da t e Wor ks h eet for
ca lcu la t in g t h e # of da ys bet ween 12/ 31/ 2001 a n d 5/ 1/ 2002:

Procedure Keystrokes Display
Select Date Worksheet 2
nd
[Date] DT1= (old content)
Clear worksheet 2
nd
[CLR Work] DT1 = 12-31-1990
Enter 1
st
date 12.3101 DT1 = 12-31-2001
Enter 2
nd
date 5.0102 DT2 = 5-01-2002
Choose 360/360 day count
method
+ + 2
nd
Set
360
Compute days between
dates
| CPT
DBD = 121
So t h er e a r e 121 da ys bet ween 12/ 31/ 2001 a n d 5/ 1/ 2002 u n der
360/ 360 da y cou n t met h od.

Simila r ly, we fin d t h a t t h er e a r e 181 da ys bet ween 12/ 31/ 2001 a n d
7/ 1/ 2002 u n der 360/ 360 da y cou n t met h od.

days between settlement date and last coupon date 121
Total days per coupon period 181
x = =
( ) ( )
6 6
121
181
3 3 6 % 6 %
1.03 2 100 1.03 2 100
x
x
PV v v a a = + = +
6
3 6 %
2 100 94.58280856 v a + = (u s in g TVM)
( )
( )
6
121
181
3 6 %
1.03 2 100 1.03 94.58280856 96.47038172
x
x
PV v a = + = ~
Ot h er met h ods t o ca lcu la t e
x
PV ;
Alt er n a t ive Met h od A
0
1.03
x
x
PV PV =
6
0
3 6 %
2 100 PV v a = +
( )
6
3 6 %
1.03 2 100 96.47038172
x
x
PV v a = + ~
282
http://actuary88.com
Alt er n a t ive Met h od B
1
1 x
x
PV v PV

=
5
1
35 6
2 100 PV v a = +
( ) ( )
5 5
35 35
1
6 6
2 100 2 100
x x
x
PV v v v v v a a

(
= + = +
(


( ) ( )
6 6
3 3 6 % 6 %
2 100 1.03 2 100 96.47038172
x x
v v v a a

= + = + ~
Met h od #2 Act u a l/ Act u a l da y cou n t met h od (for Tr ea s u r y bon ds )
Sin ce SOA does n t a llow u s t o br in g a ca len da r in t h e exa m r oom, we ca n
NOT look u p a ca len da r a n d cou n t t h e a ct u a l da ys . On ce a ga in , well let
BA II Plu s / BA II Plu s Pr ofes s ion a l do t h e wor k for u s .
Key s t r okes in BA II Plu s / BA II Plu s Pr ofes s ion a l Da t e Wor ks h eet for
ca lcu la t in g t h e # of da ys bet ween 12/ 31/ 2001 a n d 5/ 1/ 2002:

Procedure Keystrokes Display
Select Date Worksheet 2
nd
[Date] DT1= (old content)
Clear worksheet 2
nd
[CLR Work] DT1 = 12-31-1990
Enter 1
st
date 12.3101 DT1 = 12-31-2001
Enter 2
nd
date 5.0102 DT2 = 5-01-2002
Choose 360/360 day count
method
+ + 2
nd
Set
ACT
Compute days between
dates
| CPT
DBD = 121
So t h er e a r e 121 da ys bet ween 12/ 31/ 2001 a n d 5/ 1/ 2002 u n der
Act u a l/ Act u a l da y cou n t met h od.

Simila r ly, we fin d t h a t t h er e a r e 1 8 2 da ys bet ween 12/ 31/ 2001 a n d
7/ 1/ 2002 u n der Act u a l/ Act u a l da y cou n t met h od.

days between settlement date and last coupon date 121
Total days per coupon period 182
x = =
In t h is pr oblem, t h ere is lit t le differ en t bet ween t h e t wo da y cou n t
met h ods . However , in s ome pr oblems , t h e differ en ce bet ween t h e t wo
met h ods ca n be bigger .

Fin a lly, we a r e r ea dy t o ca lcu la t e t h e pu r ch a s e pr ice of t h e 2
n d
bu yer :
283
http://actuary88.com
( )
6
121
182
3 6 %
1.03 2 100 96.45990820
x
PV v a = + ~
An exa m pr oblem ma y n ot t ell you wh ich da t e cou n t met h od t o u s e. In
t h a t ca s e, u s e Act u a l/ Act u a l met h od.

Example 2

Bon d fa ce a mou n t $100
Cou pon r a t e 4% s emia n n u a l
Cou pon pa ymen t s J u ly 1 a n d December 31
Is s u e da t e 1/ 1/ 2000
Ter m t o ma t u r it y 12/ 31/ 2004
Da t e wh en bon d is r es old 5/ 1/ 2002
Yield t o ma t u r it y t o t h e
2
n d
bu yer
6% n omin a l compou n din g
s emia n n u a lly
Da y cou n t Act u a l/ Act u a l
Wh a t s t h e ma r ket pr ice of t h e bon d a t t h e s et t lemen t da t e a s s u min g t h e
2
n d
bu yer h a s 100% own er s h ip of t h e n ext cou pon a n d a ll t h e ot h er ca s h
flows ? Us e a s imple in t er es t r a t e.

Solut ion

Unit time = 0.5 year
The interest rate per coupon period
6%
3%
2
i = =
12/31 5/1 7/1 12/31 12/31
2001 2002 2002 2002 2004
Time t 0 x 1 2 6
$2 $2 $2
$100
PV
0
PV
X
PV
1
284
http://actuary88.com
We a lr ea dy ca lcu la t ed t h a t u n der Act u a l/ Act u a l

days between settlement date and last coupon date 121
Total days per coupon period 182
x = =
To fin d t h e bon d pu r ch a s e pr ice, we fir s t we dis cou n t t h e fu t u r e ca s h
flows t o t =0 (t h e pr ior cou pon da t e):

6
0 0
3 6 %
2 100 B PV v a = = +
Next , we a ccu mu la t e
0
B t o t ime x u s in g a s imple in t er es t r a t e:

( ) ( )
( )
6
0
3 6 %
1 0.03 1 0.03 2 100
x x
B PV x PV x v a = = + = + +
( )
6
3 6 %
121
1 3% 2 100
182
x x
B PV v a
| |
= = + +
|
\ .

( )
121
1 3% 94.58280856 96.46926787
182
x x
B PV
| |
= = + =
|
\ .

Example 3

Bon d fa ce a mou n t $100
Cou pon r a t e 4% s emia n n u a l
Cou pon pa ymen t s J u ly 1 a n d December 31
Is s u e da t e 1/ 1/ 2000
Ter m t o ma t u r it y 12/ 31/ 2004
Da t e wh en bon d is r es old 5/ 1/ 2002
Yield t o ma t u r it y t o t h e 2
n d
bu yer 6% n omin a l compou n din g
s emia n n u a lly
Da y cou n t Act u a l/ Act u a l
Ca lcu la t e t h e ma r ket pr ice of t h e bon d immedia t ely a ft er t h e bon d is s old
on 5/ 1/ 2002, u n der t h e followin g met h ods :
Th eor et ica l met h od
Pr a ct ica l
Semi-t h eor et ica l met h od

285
http://actuary88.com
Solut ion

Unit time = 0.5 year
The interest rate per coupon period
6%
3%
2
i = =
12/31 5/1 7/1 12/31 12/31
2001 2002 2002 2002 2004
Time t 0 x 1 2 6
$2 $2 $2
$100
We a lr ea dy ca lcu la t ed t h a t u n der Act u a l/ Act u a l da y cou n t ,

days between settlement date and last coupon date 121
Total days per coupon period 182
x = =
The ore t ic al

( )
6
0
121
182
3 6 %
1.03 1.03 2 100 96.45990820
x x
x
B PV PV v a = = = + ~
( )
( )
121
182
1 3% 1
2 2 1.32307317
3%
x x
AI coupon s s
+
= = = =
Alt er n a t ive met h od t o ca lcu la t e 2
x
s :
En t er t h e followin g in t o BA II Plu s / BA II Plu s Pr ofes s ion a l TVM:

PMT=2, N=
121
182
, I/ Y=3. FV= -1.32307317

Bon d ma r ket pr ice= pu r ch a s e pr ice a ccr u ed in t er es t

=96.45990820 1.32307317 95.14 =
286
http://actuary88.com
Prac t ic al me t hod
( ) ( )
( )
6
0
3 6 %
1 0.03 1 0.03 2 100
x x
B PV x PV x v a = = + = + +
( )
6
3 6 %
121
1 3% 2 100
182
v a
| |
= + +
|
\ .

( )
121
1 3% 94.58280856 96.46926787
182
| |
= + =
|
\ .

( ) ( )
121
2 1.32967033
182
AI coupon x = = =
Bon d ma r ket pr ice= pu r ch a s e pr ice a ccr u ed in t er es t

=96.46926787 1.32967033 95.14 =
Se mi-t he ore t ic al me t hod
( )
6
0
121
182
3 6 %
1.03 1.03 2 100 96.45990820
x x
x
B PV PV v a = = = + ~
( ) ( )
121
2 1.32967033
182
AI coupon x = = =
Bon d ma r ket pr ice= pu r ch a s e pr ice a ccr u ed in t er es t

=96.45990820 1.32967033 95.13 =
Th e r es u lt s u n der t h e t h r ee met h ods a r e ver y clos e.

Re fe re nc e t o SOA proble ms
Exa m FM Sa mple Qu es t ion s #50
Expla n a t ion of SOAs s olu t ion t o FM Sa mple Qu es t ion s #50. Th is
pr oblem a s ks for t h e pu r ch a s e pr ice, n ot t h e (qu ot ed) ma r ket pr ice. So
t h er es n o n eed t o s u bt r a ct t h e a ccr u ed in t er es t . Th e pr oblem a ls o t ells
you t o u s e a s imple in t er es t bet ween bon d cou pon da t es .

Fir s t , SOA ca lcu la t es t h e bon d pr ice P=
0
PV =906.32 on t h e pr eviou s
cou pon da t e of 4/ 15/ 2005 u s in g
( ) 2
7% i = . Next , u s in g t h e s imple in t er es t
r a t e, we ca lcu la t e t h e pu r ch a s e pr ice of t h e bon d on 6/ 28 a s
( )
0
74
1 0.035 1 0.035 906.32 919.15
183
x x
B PV x PV
| |
= = + = + =
|
\ .
.
287
http://actuary88.com
Chapt e r 1 2 Time we ight e d re t urn
and dollar we ight e d ret urn

Time we ight e d re t urn

To ca lcu la t e t h e t ime weigh t ed r et u r n of a fu n d over a t ime h or izon
[ ] 0, t , we fir s t br ea k down [ ] 0, t in t o n s u b-per iods . Th es e n s u b-
per iods ca n be of equ a l len gt h s or u n equ a l len gt h s .

For exa mple, t o ca lcu la t e t h e t ime weigh t ed r et u r n of a fu n ds
per for ma n ce in a yea r , we ca n br ea k down a yea r in t o 4 qu a r t er s .
We ca n a ls o br ea k down a yea r in t o 3 s u b-per iods : s u b-per iod 1
con s is t s of J a n u a r y, s u b-per iod 2 con s is t s of 4 mon t h s fr om
Febr u a r y t o Ma y, a n d s u b-per iod 3 con s is t s of 7 mon t h s fr om J u n e
t o December . (An SOA pr oblem will t ell you t h e wh ole per iod is
br oken down in t o h ow ma n y s u b-per iods .)

Next , we ca lcu la t e t h e r et u r n for ea ch s u b-per iod a s follows :

Ending asset value of - sub-period
=Return of - sub-period= 1
Beginning asset value of - sub-period
k
k th
R k th
k th

We t h en ca lcu la t e t h e t ime weigh t ed r et u r n R for t h e wh ole per iod
[ ] 0, t by s olvin g t h e followin g equ a t ion :

( ) ( ) ( ) ( )
1 2
1 1 1 ... 1
n
t
R R R R + = + + +
Th e t ime weigh t ed r et u r n is t h e geomet r ic a ver a ge of a ll t h e s u b-
per iods r et u r n s .

Time t 0 t
1
1 R +
2
1 R + . 1
n
R +
Period 1 Period 2 Period n
( ) ( )( ) ( )
1 2
1 1 1 ... 1
n
t
R R R R + = + + +
288
http://actuary88.com
Dollar we ight e d re t urn

To ca lcu la t e t h e dolla r weigh t ed r et u r n of a fu n d over a t ime
h or izon [ ] 0, t , we fir s t fin d t h e complet e h is t or y of a ll t h e ca s h
in flows a n d ca s h ou t flows du r in g [ ] 0, t .
1. We n eed t o kn ow h ow mu ch mon ey t h e fu n d h a s a t t ime
zer o. We ca ll t h is ( ) 0 CF . Th is is t h e begin n in g a ccou n t
va lu e.
2. We n eed t o kn ow h ow mu ch mon ey t h e fu n d h a s a t t ime t .
We ca ll t h is ( ) CF t . Th is is t h e en din g a ccou n t va lu e.

3. We n eed t o kn ow wh a t h a ppen ed in bet ween . If a n y mon ey is
a dded t o or wit h dr a wn fr om t h e fu n d du r in g [ ] 0, t , we n eed t o
fin d ou t (1) wh en t h is h a ppen ed, (2) h ow mu ch mon ey is
a dded t o or wit h dr a wn fr om t h e fu n d.

4. For exa mple, we r ecor d a ca s h flow of ( )
1
CF t a t t ime
1
t wh er e
1
0 t t < < . If ( )
1
CF t is a n in flow (i.e. mor e mon ey is a dded t o
t h e fu n d), we will ma ke ( )
1
CF t pos it ive; If ( )
1
CF t is a n
ou t flow (i.e. mon ey is wit h dr a wn fr om t h e fu n d), we will
ma ke ( )
1
CF t n ega t ive.

Next , we t r a n s la t e t h e fu n d h is t or y du r in g [ ] 0, t in t o t h e followin g
ca s h flow dia gr a m:

Time t 0
1
t
2
t
3
t t
Cash flow ( ) 0 CF ( )
1
CF t ( )
2
CF t ( )
3
CF t ( ) CF t
Fin a lly, we fin d t h e dolla r weigh t ed r et u r n R by s olvin g t h e
followin g equ a t ion (we a s s u me a s imple in t er es t r a t e R )
( )( ) ( ) ( ) ( ) ( ) ( )
1 2
0 1 1 1 2 1 ... CF tR CF t t R CF t t R CF t + + + + + + = ( (


289
http://actuary88.com
Proble m 1

Da t e Fu n s ba la n ce
befor e depos it s
a n d wit h dr a wa ls
Depos it Wit h dr a wa l
1/ 1/ 2004 $100,000
3/ 31/ 2004 $120,000 $30,000
6/ 30/ 2004 $140,000 $50,000
10/ 31/ 2004 $198,000 $90,000 $70,000
12/ 31/ 2004 $220,000
Fin d t h e fu n ds t ime weigh t ed r et u r n a n d dolla r weigh t ed r et u r n du r in g
2004.

Solut ion

Fin d t h e t ime weigh t ed r et u r n
( )
1
1/1 3/ 31 3/ 31 6/30 6/30 10/ 31 10/ 31 12/31
120, 000 140, 000 198, 000 220, 000
1
100, 000 120, 000 30, 000 140, 000 50, 000 198, 000 90, 000 70, 000
R

+ =
+ +

148.66% R =
We ca lcu la t e t h e r et u r n fr om 1/ 1 t o 3/ 31, t h e begin n in g fu n d va lu e of is
$100,000 a n d t h e en din g va lu e is $120,000. Wh en we ca lcu la t e t h e
r et u r n fr om 3/ 31 t o 6/ 30, h owever , t h e begin n in g fu n d va lu e is

$120,000 + $30,000 = $150,000

We a dd $30,000 beca u s e $30,000 fr es h mon ey flows in t o t h e fu n d a t t h e
en d of 3/ 31, per h a ps beca u s e t h e fu n d h a s ea r n ed 20% r et u r n fr om 1/ 1
t o 3/ 31.

Fin d t h e dolla r weigh t ed r et u r n
To ca lcu la t e t h e dolla r weigh t ed r et u r n , we n eed t o kn ow t h e begin n in g
fu n d va lu e, t h e en din g fu n d va lu e, a n d a ll t h e ca s h flows in -bet ween .
However , we don t n eed t o kn ow t h e in t er im fu n d ba la n ces . To s imply ou r
ca lcu la t ion , let s fir s t r emove t h e in t er im fu n d ba la n ces .

290
http://actuary88.com
Date Funs balance before
deposits and
withdrawals
Deposit Withdrawal
1/1/2004 $100,000
3/31/2004 $120,000 $30,000
6/30/2004 $140,000 $50,000
10/31/2004 $198,000 $90,000 $70,000
12/31/2004 $220,000
In a ddit ion , t o h elp u s t r a ck down t h e t ime n ea t ly, we con ver t a mon t h -
en d da t es t o a mon t h -begin da t e. Specifica lly, we ch a n ge 3/ 31/ 2004 t o
4/ 1/ 2004; 6/ 30 t o 7/ 1; 10/ 31 t o 11/ 1. Well keep 12/ 31.

Th is wa y, we kn ow t h a t t h e dis t a n ce bet ween 3/ 31 a n d 1/ 1 is t h e
dis t a n ce bet ween 4/ 1 a n d 1/ 1 (3 mon t h s a pa r t ). If we don t con ver t 3/ 31
t o 4/ 1, we migh t in cor r ect ly con clu de 3/ 31 a n d 1/ 1 a r e 2 mon t h s a pa r t .

Is it OK t o con ver t 3/ 31 t o 4/ 1? Yes . Wh en we cou n t t h e fu n d va lu e on
3/ 31, we a ct u a lly cou n t t h e fu n d va lu e a t t h e en d of 3/ 31, wh ich is t h e
s a me a s t h e fu n d va lu e cou n t ed in t h e begin n in g of 4/ 1.

Time t 1/1 3/31 6/30 10/31 12/31
1/1 4/1 7/1 11/1 12/31
(Year) 0
3
12
6
12
10
12
12
1
12
=
Cash flow $100,000 $30,000 - $50,000 $90,000 $220,000
- $70,000
Next , we s et u p t h e followin g equ a t ion :
( ) ( )
3 6 10
100, 000 1 30, 000 1 1 50, 000 1 1 90, 000 70, 000 1 1 220, 000
12 12 12
i i i i
( ( ( | | | | | |
+ + + + + + =
| | | ( ( (
\ . \ . \ .

( )
9 6 2
100, 000 1 30, 000 1 50, 000 1 20, 000 1 220, 000
12 12 12
i i i i
( ( (
+ + + + + + =
( ( (


220, 000 100, 000 30, 000 50, 000 20, 000
119%
9 6 2
100, 000 30, 000 50, 000 20, 000
12 12 12
i
+
= =
| | | | | |
+ +
| | |
\ . \ . \ .

So t h e dolla r weigh t ed r et u r n is 119%.

291
http://actuary88.com
In t h e a bove expr es s ion , 220, 000 100, 000 30, 000 50, 000 20, 000 120, 000 + = is t h e
t ot a l in t er es t ea r n ed in 2004.
9 6 2
100, 000 30, 000 50, 000 20, 000
12 12 12
| | | | | |
+ +
| | |
\ . \ . \ .
is t h e
t ot a l pr in cipa l t h a t gen er a t ed t h e $120, 000 in t er es t . Th e r a t io r epr es en t s
t h e in t er es t ea r n ed du r in g 2004.

In Exa m FM, SOA wa n t s u s t o ca lcu la t e t h e t ime weigh t ed r et u r n u s in g a
s imple in t er es t r a t e; we s h a ll do s o in t h e exa m.
In t h e r ea l wor ld, h owever , t h e s t a n da r d pr a ct ice is t o ca lcu la t e t h e t ime
weigh t ed r et u r n u s in g a compou n d in t er es t r a t e.
Let s ca lcu la t e t h e t ime weigh t ed r et u r n u s in g a compou n d in t er es t r a t e.

We s et u p t h e followin g equ a t ion :

( ) ( ) ( ) ( )( )
3 6 10
1 1 1
12 12 12
100,000 1 30,000 1 50,000 1 90,000 70,000 1 220,000 i i i i

+ + + + + + =
( ) ( ) ( ) ( )
9 6 2
12 12 12
100,000 1 30,000 1 50,000 1 20,000 1 220,000 i i i i + + + + + + =
We ca n t s olve t h is equ a t ion ma n u a lly; we n eed t o u s e BA II Plu s / BA II
Plu s Pr ofes s ion a l Ca s h Flow Wor ks h eet .

Beca u s e BA II Plu s / BA II Plu s Pr ofes s ion a l Ca s h Flow Wor ks h eet ca n n ot
u s e fr a ct ion a l t ime (a ll ca s h flow t imes n eed t o be en t er ed a s a n on -
n ega t ive in t eger ), we will u s e a mon t h a s t h e u n it t ime a n d ca lcu la t e t h e
mon t h ly effect ive in t er es t r a t e r .
Time 0 1 2 3 4 5 6 7 8 9 10 11 12
$100,000 $30,000 - $50,000 $20,000 - $220,000
( ) ( ) ( ) ( )
12 9 6 2
100,000 1 30,000 1 50,000 1 20,000 1 220,000 r r i i + + + + + + =
( ) ( ) ( ) ( )
3 6 10 12
0 100,000 30,000 1 50,000 1 20,000 1 220,000 1 r i i i

= + + + + + +
To s olve t h e a bove equ a t ion , we en t er t h e followin g in t o Ca s h Flow
Wor ks h eet :
292
http://actuary88.com
To s peed u p ou r ca lcu la t ion , we u s e $1,000 a s t h e u n it mon ey. If a ca s h
flow is $30,000, we en t er 30. Tr y t o lea r n t h is t ech n iqu e in t h e exa m.

Ca s h flow dia gr a m (s et $1,000 a s on e u n it of mon ey):

Time 0 1 2 3 4 5 6 7 8 9 10 11 12
$100 30 - 50 20 - 220
Ca s h Flow Wor ks h eet
CF0 = 100 1
st
cash flow at t=0 is $100.
C01=0, F01=2 Indicate cash flows at t=1, 2 are zero. If a cash flow is
zero, still need to press Enter.
C02=30

Indicate cash flow at t=3 is 30. No need to set F02=1;
Cash Flow Worksheet automatically use 1 as the default
cash flow frequency.
C03=0, F03=2 Indicate cash flows at t=4, 5 are zero
C04= - 50 Indicate cash flow at t=6
C05=0, F05=3 Indicate cash flows at t=7, 8, 9 are zero
C06=20 Indicate cash flow at t=10
C07 =0 Indicate cash flows at t=11 is zero
C08= - 220 Indicate cash flow at t=12
Let Ca s h Flow Wor ks h eet s olve for IRR. Pr es s IRR CPT.

IRR=6.6794%

Fin a lly, we con ver t t h is mon t h ly effect ive r a t e in t o a n a n n u a l effect ive
r a t e:

( )
12
1 6.6794 1 117.25% + ~
Proble m 2

A fu n d h a s t h e followin g t r a n s a ct ion s :

Da t e
Accou n t Va lu e
(in million s )
Con t r ibu t ion
(in million s )
1/ 1/ 2000 1.35
12/ 31/ 2001 X
1/ 1/ 2002 3.95
1/ 1/ 2003 5.8
293
http://actuary88.com
For t h e per iod fr om 1/ 1/ 2000 t o 1/ 1/ 2003, t h e dolla r weigh t ed r et u r n
u s in g t h e compou n d in t er es t r a t e a n d t h e t ime weigh t ed r et u r n a r e t h e
s a me.

Ca lcu la t e X
Solut ion

Dolla r weigh t ed r et u r n
Wh en ca lcu la t in g t h e dolla r weigh t ed r et u r n , we ign or e t h e in t er im
a ccou n t va lu es . So X is n ot n eeded.

Time 0 1 2 3

1/1/2000 1/1/2001 1/1/2002 1/1/2003

1. 35 3.95 5.8

( ) ( )
3
1.35 1 3.95 1 5.8 i i + + + =
Th is is a difficu lt equ a t ion t o s olve. For t u n a t ely, we ca n u s e t h e IRR
fu n ct ion a lit y in BA II Plu s or BA II Plu s Pr ofes s ion a l Ca s h Flow
Wor ks h eet .

Mu lt iplyin g bot h s ides ( ) ( )
3
1.35 1 3.95 1 5.8 i i + + + = wit h ( )
3
3
1 v i

= + , we h a ve:
3
1.35 3.95 5.8 0 v v + =
Th is equ a t ion t r a n s la t es t o t h e followin g ca s h flow dia gr a m:

Time 0 1 2 3

1/1/2000 1/1/2001 1/1/2002 1/1/2003

1. 35 3.95 - 5.8
Plea s e n ot e t h e ca s h flow a t 3 t = is a n ega t ive 5.8.
294
http://actuary88.com
En t er t h e followin g in t o Ca s h Flow Wor ks h eet :
CF0 C01 C02 C03
1.35 0 3.95 -5.8
F01 F02 F03
1 1 1
Pr es s IRR CPT. You s h ou ld get : IRR=6.06031163

So t h e dolla r weigh t ed r et u r n u s in g a compou n din g in t er es t r a t e is :

6.06% i =
Time weigh t ed r et u r n
Come ba ck t o t h e t a ble.
Date
Account Value
(in millions)
Contribution
(in millions)
1/1/2000 1.35
12/31/2001 X
1/1/2002 3.95
1/1/2003 5.8
( )
3
1/1/2000 to 12/31/2001 1/1/2002 to 1/1/2003
5.8
1
1.35 3.95
X
j
X
= +
+


We a r e t old t h a t 6.06% j i = =

3
5.8
1.06
1.35 3.95
X
X
| |
=
|
+
\ .
, 1.515 X =
Proble m 3 (SOA May 2 0 0 5 EA-1 #1 )

A fu n d h a s t h e followin g t r a n s a ct ion s

Da t e Accou n t Va lu e Con t r ibu t ion s Ben efit s pa ymen t s
1/ 1/ 2005 $1,000 1
C
0
3/ 31/ 2005 0 $100
295
http://actuary88.com
4/ 1/ 2005 $1,300 0 0
12/ 31/ 2005 2
C
$150
1/ 1/ 2006 $1,700 0 0
Th e t ime weigh t ed r a t e of r et u r n in 2005 is 6.25%. Th e dolla r weigh t ed
r a t e of r et u r n in 2005 is 6.00%.
In wh a t r a n ge is C1?
(A) Les s t h a n $405
(B) $405 bu t les s t h a n $430
(C) $430 bu t les s t h a n $455
(D) $455 bu t les s t h a n $480
(E) $480 or mor e

Solut ion

Th is pr oblem is s imple con cept u a lly, bu t it in volves mes s y ca lcu la t ion s .

Time weigh t ed r et u r n
We br ea k down Yea r 2005 in t o t h e t wo s egmen t s : [1/ 1, 3/ 31], [4/ 1,
12/ 31].

Segmen t 1/ 1 t o 3/ 31 4/ 1 t o 12/ 31
Begin a cct va lu e 1
1, 000 C +
1,300
En din g a cct va lu e 1,300+100=1,400
(1)
2
1, 700 150 C +
2
1, 850 C =
(2)
(1) On 4/ 1, t h e a ccou n t va lu e is $1,300. Beca u s e $100 is t a ken ou t
on 3/ 31, t h e a ccou n t va lu e on 3/ 31 is 1,400.

(2) On 1/ 1/ 2006, t h e a ccou n t va lu e is $1,700. On 12/ 31/ 2005,
2
C
a mou n t of n ew mon ey comes in a n d $150 flows ou t . So t h e
a ccou n t va lu e on 12/ 31/ 2005 is
2 2
1, 700 150 1, 850 C C + = .
We a r e given t h a t t h e t ime weigh t ed r et u r n du r in g 2005 is 6.25%.

2
1
1, 850 1, 400
1.0625
1, 000 1, 300
C
C

=
+
(Equ a t ion 1)

Dolla r weigh t ed r et u r n
Wh en ca lcu la t in g t h e dolla r weigh t ed r et u r n , we n eed t o t h r ow a wa y t h e
in t er media t e a ccou n t va lu es . So we don t n eed t o u s e t h e a ccou n t va lu e
296
http://actuary88.com
on 4/ 1/ 2005. We on ly ca r e a bou t t h e a ccou n t va lu e on 1/ 1/ 2005; t h e
a ccou n t va lu e on 12/ 31/ 2005; a n d t h e mon ey t h a t flows in or comes ou t
du r in g Yea r 2005.

Th e Accou n t va lu e on 1/ 1/ 2005 is
1
1, 000 C + . Th is ea r n s a fu ll yea r
in t er es t . On 3/ 31/ 2005, $100 flows ou t ; t h is ea r n s 9 mon t h s n ega t ive
in t er es t . Th e a ccou n t va lu e on 12/ 31/ 2005 is
2
1, 700 150 C +
2
1, 850 C = .
We a r e t old t h a t t h e t imed weigh t ed r et u r n du r in g 2005 is 6%:

( )
1 2
9
1, 000 1.06 100 1 6% 1, 850
12
C C
| |
+ + =
|
\ .
(Equ a t ion 2)

Solvin g t h es e t wo equ a t ion s , we h a ve:

1
423.84 C = ,
2
445.23 C =
So t h e cor r ect a n s wer is B.

Proble m 4 (SOA May 2 0 0 4 EA #1 0 )

Date Market value of fund Contributions Withdrawals
1/1/2004 $100,000 None None
4/1/2004 $85,000 $30,000 None
8/1/2004 $100,000 None $20,000
12/31/2004 $80,000 None None
Ma r ket va lu e of t h e fu n d is pr ior t o con t r ibu t ion s a n d wit h dr a wa ls .

A =Time weigh t ed r et u r n
B =Dolla r weigh t ed r et u r n

In wh a t r a n ge is t h e a bs olu t e va lu e of A B + ?
(A) Les s t h a n 45%
(B) 45% bu t les s t h a n 48%
(C) 48% bu t les s t h a n 51%
(D) 51% bu t les s t h a n 54%
(E) 54% or mor e

Solut ion

297
http://actuary88.com
To s peed u p ou r ca lcu la t ion , we u s e $1000 a s on e u n it of mon ey. Th en
t h e or igin a l t a ble becomes :

Date Market value of fund Contributions Withdrawals
1/1/2004 $100 None None
4/1/2004 $85 $30 None
8/1/2004 $100 None $20
12/31/2004 $80 None None
Time weigh t ed r et u r n
8/1/2004 to 12/31/2004 1/1/2004 to 4/1/2004 4/1/2004 to 8/1/2004
85 100 80
1
100 85 30 100 20
i = +
+


26.09% i =
Dolla r weigh t ed r et u r n
We fir s t delet e t h e in t er im fu n d va lu es :

Date Market value of fund Contributions Withdrawals
1/1/2004 $100 None None
4/1/2004 $85 $30 None
8/1/2004 $100 None $20
12/31/2004 $80 None None
( )
9 5
100 1 30 1 20 1 80
12 12
j j j
| | | |
+ + + + =
| |
\ . \ .
, 26.28% j =
26.09% 26.28% 52.37% A B i j + = + = =
So t h e a n s wer is D.

Proble m 5 (SOA May 2 0 0 3 EA-1 #5 )

298
http://actuary88.com
All a s s et s of a pen s ion pla n a r e in ves t ed by ma n a ger Smit h a n d ma n a ger
J on es . Th er e a r e n o ot h er pla n a s s et s . Th e followin g ch a r t s h ows t
ma r ket va lu e of t h e pla n s a s s et s wit h ea ch ma n a ger :

Date Smith Jones
Balance 12/31/2001 $2,500,000 $2,500,000
Contribution 1/1/2002 $0 $1,500,000
Balance 6/30/2002 $2,800,000 $4,500,000
Transfer 7/1/2002 $1,000,000 ($1,000,000)
Balance 12/31/2002 $4,180,000 $3,500,000
X =on e-h a lf of t h e s u m of bot h ma n a ger s t ime-weigh t ed per cen t a ge
r et u r n s for 2002.

Y = dolla r weigh t ed per cen t a ge r et u r n for 2002 for t h e en t ir e pen s ion
pla n .

Z Y X =
In wh a t r a n ge is Z ?
(A) Les s t h a n 0.09%
(B) 0.09% bu t les s t h a n 0.18%
(C) 0.18% bu t les s t h a n 0.27%
(D) 0.27% bu t les s t h a n 0.36%
(E) 0.36% or mor e

Solut ion

Well u s e $1,000,000 a s on e u n it of mon ey. Doin g s o will gr ea t ly
s implifies ou r ca lcu la t ion a n d r edu ces ch a n ces of er r or s . You s h ou ld
lea r n t h is t ech n iqu e.

Th en t h e t a ble is s implified a s follows :
Date Smith Jones
Balance 12/31/2001 $2.5 $2.5
Contribution 1/1/2002 $0 $1.5
Balance 6/30/2002 $2.8 $4.5
Transfer 7/1/2002 $1 ($1)
Balance 12/31/2002 $4.18 $3.5
Time-weigh t ed r et u r n
Smit h :
299
http://actuary88.com
12/31/2001 to 6/30/2002 7/1/2002 to 12/31/2002
2.8 4.18
1
2.5 2.8 1
i = +
+

, 23.2% i =
J on es :

12/31/2001 to 6/30/2002 7/1/2002 to 12/31/2002
4.5 3.5
1
2.5 1.5 4.5 1
j = +
+

, 12.5% i =
( ) ( )
1 1
23.2% 12.5% 17.85%
2 2
X i j = + = + =
Dolla r weigh t ed r et u r n for t h e en t ir e pen s ion pla n
We combin e t h e t r a n s a ct ion s by Smit h a n d by J on es in t o on e:
Date Smith Jones
Entire Pension Plan
(Smith + Jones)
Balance 12/31/2001 $2.5 $2.5 $5
Contribution 1/1/2002 $0 $1.5 $1.5
Balance 6/30/2002 $2.8 $4.5 $7.3
Transfer 7/1/2002 $1 ($1) $0
Balance 12/31/2002 $4.18 $3.5 $7.68
Th e in t er im ba la n ce of $7.3 on 6/ 30/ 2002 is n ot n eeded for t h e
ca lcu la t ion .

( ) ( ) 5 1 1.5 1 7.68 k k + + + = , 18.15% k = , 18.15% Y k = =
18.15% 17.85% 0.3% Z Y X = = =
So t h e a n s wer is D.

Proble m 6 (SOA May 2 0 0 0 EA-1 #1 7 )

Ma r ket va lu e of a pen s ion fu n d:
Date Value
1/1/2000 $50,000
3/31/2000 $60,000
6/30/2000 $45,000
9/30/2000 $40,000
300
http://actuary88.com
12/31/2000 $65,000
Con t r ibu t ion a n d ben efit pa ymen t s :

Date Contributions Benefit Payments
4/1/2000 $0 $P
7/1/2000 $17,000 P
10/1/2000 $55,000 P
Dolla r -weigh t ed r a t e of r et u r n for 2000 u s in g s imple in t er es t : 7%.

In wh a t r a n ge is t h e t ime weigh t ed r a t e of r et u r n for 2000?

Solut ion

Well u s e $1,000 a s on e u n it of mon ey. Fir s t , let s combin e t h e t wo t a bles
in t o on e.

Date Value Contributions Benefit Payments
1/1/2000 $50
3/31/2000 $60
4/1/2000 $0 $P
6/30/2000 $45
7/1/2000 $17 P
9/30/2000 $40
10/1/2000 $55 P
12/31/2000 $65
Dolla r weigh t ed r et u r n
( ) ( ) ( )
9 6 3
50 1 1 17 1 55 1 65
12 12 12
i P i P i P i
| | | | | |
+ + + + + + =
| | |
\ . \ . \ .

We a r e given : 7% i = .
( ) ( ) ( )
9 6 3
50 1.07 1 0.07 17 1 0.07 55 1 0.07 65
12 12 12
P P P
| | | | | |
+ + + + + =
| | |
\ . \ . \ .

Solvin g t h e a bove equ a t ion , we get : 19.986 20 P = ~
Time weigh t ed r et u r n
301
http://actuary88.com
4/1 to 6/30 7/1 to 9/30 10/1 to 12/31 1/1 to 3/31
60 45 40 65
1
50 60 45 17 40 55
j
P P P
= +
+ +


60 45 40 65
1.11
50 60 20 45 17 20 40 55 20
~
+ +
11% j =
So t h e a n s wer is D.

Proble m 6 (FM May 2 0 0 5 # 1 6 )

At t h e begin n in g of t h e yea r , a n in ves t men t fu n d wa s es t a blis h ed wit h a n
in it ia l depos it of 1000. A n ew depos it of 1000 wa s ma de a t t h e en d of 4
mon t h s . Wit h dr a wa ls of 200 a n d 500 wer e ma de a t t h e en d of 6 mon t h s
a n d 8 mon t h s , r es pect ively. Th e a mou n t in t h e fu n d a t t h e en d of t h e
yea r is 1560.

Ca lcu la t e t h e dolla r -weigh t ed (mon ey-weigh t ed) yield r a t e ea r n ed by t h e
fu n d du r in g t h e yea r .

(A) 18.57%
(B) 20.00%
(C) 22.61%
(D) 26.00%
(E) 28.89%

Solut ion

( )
8 6 4
1000 1 1000 1 200 1 500 1 1560
12 12 12
i i i i
| | | | | |
+ + + + + =
| | |
\ . \ . \ .

18.571% i =
So t h e a n s wer is A.

302
http://actuary88.com
Chapt e r 1 3 Inves t me nt year &
port folio me t hod

SOA pr oblems on t h is t opic t en d t o be s imple a n d s t r a igh t for wa r d. Her e
a r e s u pplemen t a l expla n a t ion s .

1. Wh en in s u r a n ce compa n ies s et t h eir pr emiu m r a t es , t h ey do s o
a ft er t a kin g in t o t h e a ccou n t t h a t pr emiu ms collect ed a r e in ves t ed
s omewh er e ea r n in g in t er es t s . If pr emiu m dolla r s ca n be in ves t ed
pr u den t ly a n d ea r n a h igh er in t er es t r a t e, pr emiu m r a t es will be
lower a n d a gen t s ca n s ell t h e in s u r a n ce pr odu ct s mor e ea s ily.
Con ver s ely, if pr emiu ms ea r n a lower in t er es t r a t e, t h en pr emiu m
r a t es will be h igh er a n d t h e in s u r a n ce pr odu ct s will be les s
compet it ive in t h e ma r ket pla ce.

2. In s u r a n ce compa n ies (es pecia lly life in s u r a n ce compa n ies ) oft en
decla r e t wo in t er es t r a t es , t h e gu a r a n t eed in t er es t r a t e a n d t h e
cu r r en t in t er es t r a t e. Th e gu a r a n t eed in t er es t r a t e is s et
per ma n en t ly a n d ca n n ot be ch a n ged. Th e cu r r en t in t er es t r a t e is
t h e a ct u a l in t er es t r a t e cr edit ed t o t h e policyh older a n d is a dju s t ed
on t h e on -goin g ba s is .

3. For exa mple, a wh ole life in s u r a n ce policy ma y h a ve a gu a r a n t eed
in t er es t r a t e of 3% a n d t h e cu r r en t in t er es t r a t e of 5.5%.
Th e gu a r a n t eed 3% mea n s t h a t t h e policyh older will ea r n a t
lea s t 3% per yea r n o ma t t er wh a t .

Th e gu a r a n t eed in t er es t r a t e is wr it t en in t h e in s u r a n ce
con t r a ct ; on ce s et , it ca n n ot be ch a n ged.

Th is gu a r a n t eed r a t e is ju s t t h e lowes t in t er es t pos s ible. It is
wr it t en in t h e con t r a ct t o pr ot ect t h e in s u r a n ce compa n y
a ga in s t r ea lly ba d in ves t men t exper ien ces . Un les s t h e
in ves t men t exper ien ce is r ea lly ba d, t h e in s u r a n ce compa n y
does n t r ea lly u s e t h is r a t e.

Th e cu r r en t in t er es t r a t e of 5.5% is t h e r a t e a ct u a lly cr edit ed t o
t h e policyh older .

In s u r a n ce compa n ies ch a n ge t h eir cu r r en t r a t es per iodica lly (ex
ever y 6 mon t h s or ever y yea r ) t o be lin e wit h t h e pr eva ilin g
ma r ket in t er es t r a t e.

303
http://actuary88.com
4. Pot en t ia l bu yer s of in s u r a n ce pr odu ct s oft en wa n t t o kn ow t h e
in t er es t r a t e ea r n ed by t h eir pr emiu ms . Th ey oft en s h op a r ou n d
a n d compa r e in t er es t r a t es ea r n ed by pr emiu m dolla r s .

5. Wh en s et t in g t h e on -goin g cu r r en t in t er es t r a t es per iodica lly, t h e
in s u r a n ce compa n y oft en u s es on e of t h e t wo met h ods : t h e
por t folio met h od a n d t h e in ves t men t yea r met h od.

Th e por t folio met h od

As s et s a r e combin ed for differ en t pr odu ct s (ex. wh ole life a n d
u n iver s a l life), or differ en t t ime per iods wh en pr emiu ms
come t o t h e in s u r a n ce compa n y (ex. a ll t h e pr emiu ms
collect ed la s t yea r , wh et h er in t h e 1
s t
qu a r t er or ot h er
qu a r t er s ).

A s in gle in t er es t r a t e is u s ed for differ en t pr odu ct s , differ en t
per iods of t ime.

Simple for t h e in s u r a n ce compa n ys IT depa r t men t t o
ma n a ge. For exa mple, t h e IT depa r t men t does n ot n eed t o
u s e complex s oft wa r e t o keep t r a ck of wh en pr emiu ms come
in a n d pr emiu ms a r e for wh a t pr odu ct .

Th e in ves t men t yea r met h od (or t h e n ew mon ey met h od)

As s et s a r e s egmen t ed for differ en t pr odu ct s , or differ en t t ime
per iods du r in g wh ich fu n ds a r e r eceived. Differ en t in t er es t
r a t es a r e u s ed.

Dis a dva n t a ge - IT depa r t men t n eeds t o u s e complex s oft wa r e
t o keep t r a ck of wh en n ew fu n ds come in a n d n ew fu n ds a r e
for wh a t pr odu ct s .

Adva n t a ge Th e in t er es t r a t e cr edit ed t o pr emiu ms or ot h er
fu n ds depen ds wh en pr emiu ms or fu n ds come in . If t h e
in t er es t r a t e is h igh er wh en n ew pr emiu ms come t o t h e
in s u r a n ce compa n y, n ew pr emiu ms will a u t oma t ica lly get
t h e h igh er in t er es t r a t e.

SOA pr oblems on t h e por t folio met h od a n d t h e in ves t men t yea r met h od
a r e s imple, r equ ir in g ca n dida t es t o look u p t h e cor r ect in t er es t r a t e fr om
a given in t er es t r a t e t a ble. Plea s e r efer t o Sample FM #8 t o lea r n h ow t o
s olve t h is t ype of pr oblems .

304
http://actuary88.com
Chapt e r 1 4 Short Sales

Sh or t s a les of s t ocks a r e cover ed in t wo a r ea s : (1) t h eor y of in t er es t
t ext books (s u ch a s Sa m Br over ma n s t ext book Ma t h ema t ics of
In ves t men t a n d cr edit s ect ion 8.2.2), a n d (2) Der iva t ives Ma r ket s (s ect ion
1.4).

Accor din g t o t h e SOAs s ylla bu s , you ju s t n eed t o s h or t s a les a s
expla in ed in Der iva t ives Ma r ket s .

You don t n eed t o wor r y a bou t s h or t s a les a s expla in ed in a n y of t h e fou r
r ecommen ded t ext books for t h e t h eor y of in t er es t . As a ma t t er of fa ct ,
SOA r emoved Sa mple FM #38, #39, a n d #40. All t h es e pr oblems a r e
s h or t s a les of s t ocks a s expla in ed in t h e t h eor y of in t er es t t ext books .

Th is ch a pt er is n ot on t h e s ylla bu s . However , I in clu de t h is ch a pt er t o
h elp you bet t er u n der s t a n d Der iva t ives Ma r ket s s ect ion 1.4. If you
u n der s t a n d Der iva t ives Ma r ket s s ect ion 1.4, t h a t s gr ea t a n d you ca n
ju s t s kip t h is ch a pt er .

Compa n y As s t ocks a r e cu r r en t ly s ellin g $100 per s h a r e. You h a ve a
good r ea s on t o believe t h a t t h is s t ock pr ice is n ot goin g t o la s t lon g a n d
will go down s oon . Per h a ps you h ea r d t h e r u mor t h a t Compa n y A u s es
a ggr es s ive a ccou n t in g t o exa gger a t e it s s a les . Or per h a ps you lea r n ed
fr om t h e Wa ll St r eet J ou r n a l t h a t Compa n y As t op ma n a gemen t is
in compet en t . Or per h a ps you fou n d t h a t on e of Compa n y As ch ief
compet it or s h a s ju s t des ign ed a r evolu t ion a r y pr odu ct wh ich will ma ke
Compa n y As ma in pr odu ct lin e obs olet e.

For wh a t ever r ea s on , you believe t h a t Compa n y As s t ock pr ice will go
down in t h e ver y n ea r fu t u r e, s a y in 15 da ys . How ca n you ma ke mon ey
on Compa n y As s t ocks ba s ed on you r a n a lys is ?

You ca n ma ke mon ey t h is wa y. You ca n u s e a n in t er n et br oker a ge fir m t o
s ell s h or t 1,000 s h a r es of Compa n y A s t ocks . Or if you pr efer t r a dit ion a l
br oker a ge fir m, you ca n a s k you r br oker t o s ell 1,000 s h a r es of Compa n y
A s t ocks s h or t for you . Sellin g s h or t mea n s t h a t you s ell s omet h in g you
don t h a ve. Cu r r en t ly you don t own a n y s t ocks fr om Compa n y A, bu t
you wa n t t o s ell 1,000 s h a r es of Compa n y A s t ocks . So you a r e s ellin g
s h or t . Es s en t ia lly, you a r e bor r owin g 1,000 s h a r es fr om a br oker a ge fir m
a n d s ell t h em in t h e open ma r ket .

J u s t a s you h a ve pr edict ed, 15 da ys la t er , t h e pr ice of Compa n y As
s t ocks dr ops t o $80 per s h a r e. You bu y 1,000 s h a r es of Compa n y A
305
http://actuary88.com
s t ocks in t h e open ma r ket a t $80 per s h a r e a n d r et u r n t h e 1,000 s h a r es
t o t h e br oker a ge fir m.

Let s s ee h ow mu ch mon ey you h a ve ma de. You s ell 1,000 s h a r es a t
$100 per s h a r e. You r cos t is $80 per s h a r e. So you h a ve ea r n ed $20 per
s h a r e. You r t ot a l pr ofit is ($20)(1,000)=$20,000. Nice job.

Th is is t h e s t r ipped-down ver s ion of s h or t s a les ; t h e a ct u a l t r a n s a ct ion s
a r e a lit t le mor e complex. Let s a n a lyze s h or t s a les .

Fir s t , in s h or t s a les , you s ell s h a r es h igh n ow a n d bu y s h a r es ba ck la t er .
You h ope t o bu y s h a r es ba ck in t h e n ea r fu t u r e a t a lower pr ice.
However , if t h e s h a r e pr ice goes u p, you ll h a ve t o bu y s h a r es ba ck a t a
h igh er pr ice a n d in cu r a los s . For exa mple, 15 da ys la t er , Compa n y As
s t ocks s ell $110 per s h a r e. All t h e r u mor s you h ea r d a bou t Compa n y
t u r n ou t t o be fa ls e. Compa n y A is doin g qu it e well. You wa it for a wh ile,
bu t Compa n y A s t ock pr ices s t a y s t a ble a t $110 per s h a r e. Fin a lly, you
h a ve t o bu y ba ck 1,000 s h a r es a t $110 per s h a r e in t h e open ma r ket a n d
r et u r n t h em t o t h e br oker a ge fir m wh ich did t h e s h or t s a le for you . You
in cu r r ed a t ot a l los s of 1,000($110-$100)=$10,000.

Secon d, la ws r equ ir e t h a t s h or t s a les ca n t a ke pla ce on ly if t h e la s t
r ecor ded ch a n ge in t h e pr ice of a s t ock is pos it ive (i.e. t h e s t ock pr ice
wen t u p la s t t ime). You ca n t s h or t s ell a s t ock a ft er t h e s t ock pr ice wen t
down . You ca n s h or t s ell a s t ock a ft er t h e s t ock pr ice wen t u p. Th is r u le
a t t empt s t o dis cou r a ge wild s pecu la t ion s on s t ocks .

Th ir d, in s h or t s a les , you r lia bilit y t o t h e br oker a ge fir m is t h e n u mber of
s h a r es bor r owed, n ot t h e va lu e of t h e bor r owed s h a r es a t t h e t ime wh en
s h or t s a les t a ke pla ce. In ou r exa mple, you bor r ow 1,000 s h a r es of
Compa n y A s t ocks fr om t h e br oker a ge fir m a t $100 per s h a r e. You r
lia bilit y t o t h e br oker a ge fir m is 1,000 s h a r es of s t ocks , n ot $100,000. In
t h e fu t u r e, you ju s t n eed t o r et u r n 1,000 s h a r es t o t h e br oker a ge fir m. If
you ca n r et u r n 1,000 s h a r es of s t ocks t o t h e br oker a ge fir m a t a lower
pr ice, you ll ma ke a pr ofit . If you r et u r n 1,000 s h a r es a t a h igh er pr ice,
you h a ve a los s .

For t h , you ca n bor r ow s h a r es for lon g t ime befor e pa yin g t h em ba ck.
Ha ve you won der ed wh er e t h e br oker a ge fir m get s t h e 1,000 s h a r es of
Compa n y A s t ocks t o len d t h em t o you ? Th e br oker a ge fir m s imply
bor r ows 1,000 s h a r es of Compa n y A s t ocks fr om a n ot h er in ves t or wh o
h olds 1,000 s h a r es of Compa n y A s t ocks in h is a ccou n t a t t h e br oker a ge
fir m. Th is in ves t or will n ot even kn ow t h a t t h e br oker a ge fir m h a s
bor r owed 1,000 s h a r es fr om h is a ccou n t . If t h is in ves t or wa n t s t o s ell
1,000 s h a r es of Compa n y A s t ocks a ft er you h a ve s h or t s old h is 1,000
s h a r es of s t ocks , t h e br oker a ge fir m s imply bor r ows 1,000 s h a r es of
306
http://actuary88.com
Compa n y A s t ocks fr om a n ot h er in ves t or . As a r es u lt , you ca n bor r ow
s omeon e els es s h a r es for a lon g t ime.

However , in s ome s it u a t ion s you s h or t s old s omebody els es 1,000 s h a r es
of s t ocks . Th en t h e or igin a l own er dema n ded 1,000 s h a r es of s t ocks
(beca u s e h e wa n t ed t o s ell t h em), bu t t h e br oker a ge fir m cou ldn t fin d
a n ot h er in ves t or wh o h a d 1,000 s h a r es of s t ocks . If t h is h a ppen s , t h e
br oker a ge fir m will a s k you t o immedia t ely pu r ch a s e 1,000 s h a r es fr om
t h e open ma r ket a n d r et u r n t h em t o t h e br oker a ge fir m.

Fift h , if t h e bor r owed s t ocks pa y dividen ds a ft er t h e s h or t s a le a n d befor e
you r et u r n t h e bor r owed s t ocks t o t h e br oker a ge fir m, you a r e r equ ir ed t o
pa y t h e dividen d t o t h e or igin a l own er of t h e s t ocks . Wh en you wa n t ed t o
s h or t s ell 1,000 s h a r es of Compa n y A s t ocks , you r br oker a ge fir m
decided bor r ow 1,000 s h a r es fr om a n in ves t or (we ca ll h im Smit h ) a n d
len t t h em t o you . You s h or t s old t h es e 1,000 s h a r es of s t ocks a t $100 per
s h a r e t o s omeon e (we ca ll h im J oh n ). On e mon t h la t er , Compa n y A
decided t o pa y a dividen d of 5 cen t s per s h a r e. Compa n y A fou n d ou t
t h a t J oh n wa s t h e own er of t h e 1,000 s h a r es . At t h is t ime you h a d n ot
pa id ba ck t h e 1,000 bor r owed s h a r es t o t h e br oker a ge fir m.

Compa n y A wou ld pa y t h e dividen d wor t h a t ot a l of $0.05(1,000)=$50 t o
t h e n ew own er J oh n , n ot t o t h e or igin a l own er Smit h . However , Smit h
s t ill own ed 1,000 s h a r es of Compa n y A s t ocks a n d s h ou ld get a t ot a l of
$50 dividen d. Con s equ en t ly, you n eeded t o pa y t h e br oker a ge fir m $50
dividen d a n d t h e br oker a ge fir m wou ld give t h e $50 t o t h e or igin a l own er
Smit h .

Does t h is mea n t h a t you pa id $50 dividen d t o Smit h ou t of you r own
pocket ? No. Th e dividen d of 5 cen t s per s h a r e wa s r eflect ed in t h e s t ock
pr ice. In ot h er wor ds , wh en you s h or t s old 1,000 s h a r es of s t ocks a t
$100 per s h a r e, t h e $100 pr ice a lr ea dy r eflect ed t h e pos s ibilit y t h a t
Compa n y wou ld dis t r ibu t e cer t a in a mou n t of dividen d t o it s
s h a r eh older s . Beca u s e Compa n y A s t ocks cou ld pos s ibly gen er a t e a
dividen d of 50 cen t s per s h a r e, you cou ld s h or t s ell Compa n y A s t ocks a t
$100 per s h a r e. If Compa n y wa s n ot expect ed t o dis t r ibu t e a n y dividen d
in t h e n ea r fu t u r e, t h e s a les pr ice of Compa n y A s t ocks wou ld h a ve been
les s t h a n $100 per s h a r e.

Sixt h , you mu s t meet t h e in it ia l ma r gin r equ ir emen t befor e you ca n s h or t
s ell. For exa mple, t h e br oker a ge fir m r equ ir es a 50% in it ia l ma r gin . Th is
mea n s t h a t befor e a s h or t s a le ca n t a ke pla ce, you r colla t er a l a ccou n t in
t h e br oker a ge fir m mu s t h old a t lea s t 50% of t h e t h en cu r r en t ma r ket
va lu e of 1,000 s h a r es of Compa n y A s t ocks .

307
http://actuary88.com
Seven t h , you mu s t meet t h e on goin g ma in t en a n ce ma r gin r equ ir emen t .
Th e ma in t en a n ce ma r gin r equ ir emen t pr ovides a n ot h er la yer of
pr ot ect ion t o t h e br oker a ge fir m. For exa mple, t h e br oker a ge fir m
r equ ir es a 40% ma in t en a n ce ma r gin . Th is mea n s t h a t fr om t h e momen t
wh en a s h or t s a le t a kes pla ce t o t h e momen t immedia t ely befor e you
fin a lly r et u r n 1,000 s h a r es t o t h e br oker a ge fir m, you r colla t er a l a ccou n t
in t h e br oker a ge fir m mu s t h old a t lea s t 40% of t h e t h en cu r r en t ma r ket
va lu e of 1,000 s h a r es of Compa n y A s t ocks .

Ma t h ema t ica lly, a t a n y t ime t (wh er e t is bet ween wh en a s h or t s a le
t a kes pla ce t o t h e momen t immedia t ely befor e you fin a lly r et u r n t h e
bor r owed s h a r es t o t h e br oker a ge fir m)

( )
( )
Your equity
Mainteance margin %
market value of the stocks borrowed
t
t
>
( ) [ ] ( ) Your equity Mainteance margin % market value of the stocks borrowed t t > (

( ) ( ) ( ) Equity Asset Liability t t t =
Plea s e n ot e t h a t t h e 40% ma r gin r equ ir emen t ca n be met by ca s h , s t ocks
a n d bon ds . If you r accou n t a t t h e br oker a ge fir m h a s ca s h , s t ocks of
a n ot h er compa n y, or bon ds wor t h les s t h a n $40,000, you will get a
ma r gin ca ll, in wh ich ca s h you mu s t depos it a ddit ion a l ca s h t o you r
colla t er a l a ccou n t .

For a ddit ion a l in for ma t ion , r efer t o
h t t p:/ / www.in ves t opedia .com/ u n iver s it y/ s h or t s ellin g/
For a det a iled des cr ipt ion of h ow a s h or t s a le wor ks , r efer t o
h t t p:/ / webcompos er .pa ce.edu / pvis wa n a t h / n ot es / in ves t men t s / s h or t s a l.
h t ml#logis t ics
Sample proble ms
Proble m 1 (SOA Cours e 6 #3 May 2 0 0 1 )
(a ) Wit h r es pect t o s h or t s a les of a s ecu r it y:
(1) Des cr ibe t h e pr oces s for execu t in g a s h or t s a le
(2) Ou t lin e a n in ves t or s mot iva t ion for execu t in g s u ch a t r a n s a ct ion

(b) You a r e given t h e followin g:

Da t e Cor por a t e Z Sh a r e Pr ice
J a n u a r y 1, 2001 60
J a n u a r y 15, 2001 63
308
http://actuary88.com
J a n u a r y 31, 2001 58
Cor por a t ion Z pa id a dividen d of 1 on J a n u a r y 15, 2001
Th e ma ximu m pr ice of Cor por a t ion Z s h a r es du r in g t h e mon t h of
J a n u a r y 2001 wa s 63

On J a n u a r y 1, 2001, In ves t or A s old s h or t 100 s h a r es of
Cor por a t ion Z
On J a n u a r y 31, 2001, In ves t or A cover ed t h e s h or t pos it ion

Th e in it ia l ma r gin r equ ir emen t wa s 50%
Th e ma in t en a n ce ma r gin r equ ir emen t wa s 40%

Th er e wer e n o ot h er t r a n s a ct ion cos t s
No in t er es t wa s ea r n ed on t h e ba la n ce wit h t h e br oker

(3) Ou t lin e t h e t r a n s a ct ion on J a n u a r y 1, 2001
(4) Ou t lin e t h e t r a n s a ct ion on J a n u a r y 31, 2001
(5) Det er min e wh et h er a ma r gin ca ll wa s n eces s a r y
(6) Ca lcu la t e t h e r a t e of r et u r n t o In ves t or A for J a n u a r y 2001.

Sh ow a ll t h e wor k.

Solut ion

Down loa d t h e officia l s olu t ion fr om t h e SOA webs it e.

Plea s e n ot e s ome of t h e r equ ir ed con cept s in FM a r e fr om Cou r s e 6.

Proble m 2 (SOA Cours e 6 #3 May 2 0 0 3 )

You a r e given t h e followin g in for ma t ion :

ma r gin r equ ir emen t on s h or t s a les : 50%
ma in t en a n ce ma r gin : 30%
a n in ves t or s a ccou n t wit h a br oker cu r r en t ly h olds :
va lu e of T-bills : 10,000
n u mber of s h a r es of XYZ s t ocks 500
s t ock pr ices :

Da t e ABC St ock Pr ice XYZ St ock Pr ice
J u n e 2, 2003 103 75
J u n e 3, 2003 102 76
J u n e 4, 2003 99 77
309
http://actuary88.com
J u n e 5, 2003 100 75
J u n e 6, 2003 101 80
J u n e 9, 2003 105 72
J u n e 10, 2003 115 65
Th e in ves t or t ells t h e br oker t o s h or t 1,000 s h a r es of t h e ABC s t ock on
J u n e 3, 2003. Th e br oker execu t es t h e or der on t h e fir s t da y a llowed.
Sh a r es a r e t r a ded on ce per da y.

(a ) Ca lcu la t e t h e a ddit ion a l ca s h (if a n y) n eces s a r y t o s a t is fy t h e
ma r gin r equ ir emen t .
(b) Ca lcu la t e t h e a mou n t of t h e ma r gin ca lls (if a n y) bet ween J u n e 3,
2003 a n d J u n e 10.

Sh ow a ll t h e wor k.

Solut ion

Down loa d t h e officia l s olu t ion fr om t h e SOA webs it e.

Proble m 3

You s h or t s ell 100 s h a r es of Compa n y XYZ, wh ich cu r r en t ly s ell for $50
per s h a r e. Th e in it ia l ma r gin r equ ir emen t is 40%.

1. Wh a t s you r r a t e of r et u r n if on e yea r la t er Compa n y XYZs s t ocks
s ell $55, $50, or $45 per s h a r e? As s u me you ea r n n o in t er es t on
you r ma r gin a ccou n t . Als o a s s u me t h a t Compa n y XYZ does n t pa y
dividen d.

2. If t h e ma in t en a n ce ma r gin is 30%, h ow h igh ca n XYZs s t ock pr ice
go u p befor e you get a ma r gin ca ll?

3. Redo t h e a bove t wo qu es t ion s a s s u min g t h a t XYZ pa ys dividen d of
$1 per s h a r e a t t h e en d of yea r 1.

Solut ion

1. To s a t is fy t h e ma r gin r equ ir emen t , you mu s t depos it t h e followin g
a mou n t of ca s h in t o t h e ma r gin a ccou n t a t 0 t = :
$50(100)(40%)=$2,000

You r r a t e of r et u r n is :

310
http://actuary88.com
( ) 100 50
2, 000
P profit
r
Initial deposit

= = , wh er e P is t h e s h a r e pr ice a t 1 t =
You r r et u r n s a r e:

If 55 P = ,
( ) 100 50 55
25%
2, 000
r

= =
If 50 P = ,
( ) 100 50 50
0%
2, 000
r

= =
If 45 P = ,
( ) 100 50 45
25%
2, 000
r

= =
2. You r a s s et con s is t s of:
pr oceeds fr om s h or t s a les : 100(50)=$5,000
in it ia l depos it in t o t h e ma r gin a ccou n t : $2,000

You r t ot a l a s s et is $7,000

You r lia bilit y is t o r et u r n 100 s h a r es of s t ocks : $100P

You r equ it y is : 7,000 100P

You get a ma r gin ca ll if t h e r a t io of you r equ it y t o t h e t h en ma r ket
va lu e of t h e s t ocks is les s t h a n t h e ma in t en a n ce ma r gin :

7000 100
30%
100
P
P

< ,
70
$53.85
1.3
P > =
3. Now you h a ve t o pa y dividen d t o you r br oker . Th e dividen d a mou n t
is 100($1 per s h a r e)=$100. Now you r r et u r n is

( ) 100 50 100
2, 000
P profit
r
Initial deposit

= =
You r r et u r n s a r e:

If 55 P = ,
( ) 100 50 55 100
30%
2, 000
r

= =
If 50 P = ,
( ) 100 50 50 100
5%
2, 000
r

= =
311
http://actuary88.com
If 45 P = ,
( ) 100 50 45 100
20%
2, 000
r

= =
You r a s s et con s is t s of:
pr oceeds fr om s h or t s a les : 100(50)=$5,000
in it ia l depos it in t o t h e ma r gin a ccou n t : $2,000
Tot a l a s s et : $7,000

You r lia bilit y:
To r et u r n 100 s h a r es of s t ocks : $100P
To pa y t h e dividen d: $100
Tot a l lia bilit y: $100P+100

You r equ it y is : 7,000 100P-100

You get a ma r gin ca ll if t h e r a t io of you r equ it y t o t h e ma r ket va lu e
of t h e s t ocks s old is les s t h a n t h e ma in t en a n ce ma r gin :

7000 100 100
30%
100
P
P

< ,
69
$53.08
1.3
P > =
Proble m 4 (#1 7 , Nov 2 0 0 5 FM)

Th eo s ells a s t ock s h or t wit h a cu r r en t pr ice of 25,000 a n d bu ys it ba ck
for X a t t h e en d of 1 yea r . Gover n men t a l r egu la t ion s r equ ir e t h e s h or t
s eller t o depos it ma r gin of 40% a t t h e t ime of t h e s h or t s a le. Th e
pr eva ilin g in t er es t r a t e is a n 8% a n n u a l r a t e, a n d Th eo ea r n s a 25% yield
on t h e t r a n s a ct ion .

Ca lcu la t e X
Solut ion

At 0 t = , Th eo depos it ed ( ) 25, 000 40% 10, 000 = in t o t h e ma r gin a ccou n t . At
1 t = wh en t h e s h or t s a le is clos ed ou t , Th eos wea lt h is

FV of t h e in it ia l depos it + ga in fr om t h e s h or t s a le dividen d pa id
= ( ) ( ) 10, 000 1.08 25, 000 0 X +
Th eo ea r n ed 25% yield on t h e t r a n s a ct ion :

312
http://actuary88.com
( ) ( ) 10, 000 1.08 25, 000 10, 000 1.25 X + = , 23, 300 X =
Proble m 5 (#3 6 , May 2 0 0 3 Cours e 2 ) (als o Sample FM #3 8 )
(SOA r emoved Sa mple #38 t h r ou gh Sa mple #44 fr om t h e Sa mple FM
Qu es t ion s . As a r es u lt , you ca n s kip t h e followin g qu es t ion s . I in clu ded
t h es e qu es t ion s for complet en es s .)

Er ic a n d J a s on ea ch s ell a differ en t s t ock s h or t a t t h e begin n in g of t h e
yea r for a pr ice of 800 . Th e ma r gin r equ ir emen t for ea ch in ves t or is 50%
a n d ea ch will ea r n a n a n n u a l effect ive in t er es t r a t e of 8% on h is ma r gin
a ccou n t .

Ea ch s t ock pa ys a dividen d of 16 a t t h e en d of t h e yea r . Immedia t ely
t h er ea ft er , Er ic bu ys ba ck h is s t ock a t a pr ice of 800 2X , a n d J a s on
bu ys ba ck h is s t ock a t a pr ice of 800 X + .
Er ics a n n u a l effect ive yield, i, on t h e s h or t s a le is t wice J a s on s a n n u a l
effect ive yield.

Ca lcu la t e i.

Solut ion

At 0 t = , Er ic depos it s 800*50%=400.
At 1 t = , Er ics wea lt h is :
FV of t h e in it ia l depos it + ga in fr om t h e s h or t s a le dividen d pa id
= ( ) ( ) ( ) 400 1.08 800 800 2 16 400 1.08 2 16 X X + = + (


Er ics a n n u a l effect ive yield fr om t h e s h or t s a le is i . Th en
( ) ( ) 400 1 400 1.08 2 16 i X + = + ,
2 16
400
X
i
+
=
At 0 t = , J a s on depos it s 800*50%=400.
At 1 t = , J a s on s wea lt h is :
FV of t h e in it ia l depos it + ga in fr om t h e s h or t s a le dividen d pa id
= ( ) ( ) ( ) 400 1.08 800 800 16 400 1.08 16 X X + + = (


J a s on s a n n u a l effect ive yield fr om t h e s h or t s a le is j . Th en
( ) ( ) 400 1 400 1.08 16 j X + =
16
400
X
j

=
We a r e t old t h a t 2 i j = :
313
http://actuary88.com
2 16 16
2
400 400
X X + | |
=
|
\ .
, 4 X =
( ) 2 4 16
2 16
6%
400 400
X
i
+
+
= = =
Sample FM Proble m #3 9
J os e a n d Ch r is ea ch s ell a differ en t s t ock s h or t for t h e s a me pr ice. For
ea ch in ves t or , t h e ma r gin r equ ir emen t is 50% a n d in t er es t on t h e ma r gin
debt is pa id a t a n a n n u a l effect ive r a t e of 6%. Ea ch in ves t or bu ys ba ck
h is s t ock on e yea r la t er a t a pr ice of 760. J os es s t ock pa id a dividen d of
32 a t t h e en d of t h e yea r wh ile Ch r is s s t ock pa id n o dividen ds . Du r in g
t h e 1-yea r per iod, Ch r is s r et u r n on t h e s h or t s a le is i , wh ich is t wice t h e
r et u r n ea r n ed by J os e. Ca lcu la t e i .
Solut ion

Let X r epr es en t t h e pr ice of t h e s t ock wh en t h e s h or t s a le t a kes pla ce
(i.e. a t 0 t = ).

At 0 t = , J os e depos it s 0.5X .
At 1 t = , J os es wea lt h is :
FV of t h e in it ia l depos it + ga in fr om t h e s h or t s a le dividen d pa id
= ( ) ( ) 0.5 1.06 760 32 1.53 792 X X X + =
J os es a n n u a l effect ive yield fr om t h e s h or t s a le is i . Th en
( ) 0.5 1 1.53 792 X j X + = ,
1.53 792
1
0.5
X
j
X

=
At 0 t = , Ch r is depos it s 0.5X .
At 1 t = , Ch r is s wea lt h is :
FV of t h e in it ia l depos it + ga in fr om t h e s h or t s a le dividen d pa id
= ( ) ( ) 0.5 1.06 760 1.53 760 X X X + =
Ch r is s a n n u a l effect ive yield fr om t h e s h or t s a le is i . Th en
( ) 0.5 1 1.53 760 X j X + = ,
1.53 760
1
0.5
X
i
X

=
We a r e t old t h a t 2 i j = :
1.53 792 1.53 760
2 1 1
0.5 0.5
X X
X X
| |
=
|
\ .
, 800 X =
314
http://actuary88.com
( )
( )
1.53 706.8 792
1 16%
0.5 706.8
i

= =
Sample FM Proble m #4 0
Bill a n d J a n e ea ch s ell a differ en t s t ock s h or t for a pr ice of 1000. For
bot h in ves t or s , t h e ma r gin r equ ir emen t is 50%, a n d in t er es t on t h e
ma r gin is cr edit ed a t a n a n n u a l effect ive r a t e of 6%. Bill bu ys ba ck h is
s t ock on e yea r la t er a t a pr ice of P . At t h e en d of t h e yea r , t h e s t ock pa id
a dividen d of X . J a n e a ls o bu ys ba ck h er s t ock a ft er on e yea r , a t a pr ice
of ( P 25). At t h e en d of t h e yea r , h er s t ock pa id a dividen d of 2X . Bot h
in ves t or s ea r n ed a n a n n u a l effect ive yield of 21% on t h eir s h or t s a les .
Ca lcu la t e P .
Solut ion

At 0 t = , Bill depos it s 1000*50%=500.
At 1 t = , Bills wea lt h is :
FV of t h e in it ia l depos it + ga in fr om t h e s h or t s a le dividen d pa id
= ( ) ( ) 500 1.06 1000 1530 P X P X + =
Bills a n n u a l effect ive yield fr om t h e s h or t s a le is 21%. Th en
( ) 500 1 21% 1530 P X + =
At 0 t = , J a n e depos it s 1000*50%=500.
At 1 t = , J a n es wea lt h is :
FV of t h e in it ia l depos it + ga in fr om t h e s h or t s a le dividen d pa id
= ( ) ( ) 500 1.06 1000 25 2 1555 2 P X P X + = (


J a n es a n n u a l effect ive yield fr om t h e s h or t s a le is 21%. Th en
( ) 500 1 21% 1555 2 P X + =
1530 1555 2 P X P X = , 25 X =
( ) 500 1 21% 1530 25 P + = , 900 P =
315
http://actuary88.com
Chapt e r 1 5 Te rm s t ruct ure of
int ere s t rat e , s pot rat e, forward rat e , and
arbit rage

Ke y point s :

1 . Law of one pric e (no arbit rage princ iple )
Two bon ds (or ot h er s ecu r it ies ) wit h iden t ica l ca s h flows s h ou ld
s ell for a n iden t ica l pr ice.

If t h ey don t s ell for t h e s a me pr ice (i.e. if on e bon d s ells a t a h igh er
pr ice t h a n a n ot h er bon d wit h iden t ica l ca s h flows ), a n yon e ca n
ma ke mon ey by bu yin g t h e lower pr iced bon d, t u r n in g a r ou n d, a n d
s ellin g it a t a h igh er pr ice.

Th e s t r a t egy of exploit in g looph oles t o ma ke mon ey is ca lled
a r bit r a ge.

Ch a r a ct er is t ics of a r bit r a ge:
(1) Pr ofit is ma de wit h 100% cer t a in t y
(2) Pr ofit is ma de wit h zer o cos t
(3) Pr ofit is ma de wit h zer o r is ks t a ken

No a r bit r a ge pr in ciple a s s u mes t h er e a r e n o t r a n s a ct ion cos t s s u ch
a s t a x a n d commis s ion s .

2 . Te rm s t ruc t ure of int e re s t rat e s

Ter m s t r u ct u r e of in t er es t r efer s t o t h e ph en omen on t h a t a bon ds
yield-t o-ma t u r it y ch a n ges a s t h e bon ds ma t u r it y ch a n ges .

A h ypot h et ica l exa mple

Ma t u r it y of a bon d Yield t o ma t u r it y
1 yea r 7%
2 yea r 8%
3 yea r s 8.75%
4 yea r s 9.25%
5 yea r s 9.5%
316
http://actuary88.com
Ter m s t r u ct u r e of in t er es t is a ls o ca lled t h e yield cu r ve, wh ich is a
2-D gr a ph s h owin g h ow t h e yield t o ma t u r it y (Y a xis ) is a fu n ct ion
of t h e bon ds ma t u r it y ( X a xis ).

Differ en t t h eor ies expla in wh y a lon ger -t er m bon d h a s a h igh er or
lower yield-t o-ma t u r it y t h a n does a s h or t -t er m bon d. (No n eed t o
lea r n t h os e t h eor ies )

Beca u s e a t r a ded bon ds yield of ma t u r it y is oft en u s ed a s t h e
ma r ket s fa ir dis cou n t r a t e (t h e r a t e u s ed t o dis cou n t fu t u r e ca s h
flows ), t h e dis cou n t r a t e is a fu n ct ion of t h e len gt h of t h e
in ves t men t .

3 . Spot rat e .

Spot r a t e is t h e r et u r n you ca n lock in a t t ime zer o. It s t h e r et u r n
you ca n get by len din g (or depos it in g) you r mon ey a t 0 t = a n d
h a vin g you r mon ey r epa id in a fu t u r e t ime
1
t .
Sign con t r a ct ; Len d mon ey Loa n r epa id

Time 0 spot rate
1
t
For exa mple, a t 0 t = you bou gh t a t wo yea r zer o-cou pon bon d wit h
$100 fa ce a mou n t for a pr ice of $85. In t h is t r a n s a ct ion , you len t
$85 a t 0 t = . You r mon ey is t o be r epa id a t 2 t = . Th e r et u r n you
h a ve locked in a t 0 t = ca n be s olved a s follows :

( )
2
85 1 100 8.465% r r + = =
So a t 0 t = you locked in a n a n n u a l r et u r n of 8.465% for t wo yea r s .
8.465% r = is t h e 2-yea r s pot r a t e.

Th e wor d s pot comes fr om t h e ph r a s e t h e s pot ma r ket . Th e s pot
ma r ket (or ca lled ca s h ma r ket ) is wh er e t h e s eller immedia t ely
deliver s t h e pr odu ct t o t h e bu yer on t h e s pot . Exa mple. You pa y
$10 t o a fa r mer a n d h e immedia t ely gives you 15 t oma t oes on t h e
s pot . Remember , s pot ma r ket = immedia t e deliver y. (In t h e r ea l
wor ld, h owever , ma n y s pot t r a n s a ct ion s a r e complet ed wit h in a few
h ou r s or a cou ple of da ys .)

317
http://actuary88.com
Th e ph r a s e s pot r a t e is u s ed beca u s e a t t ime zer o wh en you bu y
t h e in ves t men t , t h e in ves t veh icle (a bon d, a CD, or a n y ot h er
in ves t men t oppor t u n it y) is immedia t ely deliver ed t o you a t t ime
zer o.
A s pot r a t e a n s wer s t h is qu es t ion , If a on e bu ys a bon d n ow a t
t ime zer o fr om a ma r ket , wh a t r et u r n ca n h e get ?

Spot r a t e is oft en den ot ed a s
t
s . Th is is t h e r et u r n you ca n get if
you in ves t you r mon ey a t t ime zer o for t yea r s .

4 . Forward rat e

At t ime zer o, you s ign a n in ves t men t con t r a ct , wh ich r equ ir es you
t o len d you r mon ey a t a fu t u r e t ime
1
t . You r mon ey is t o be r epa id
in a n ot h er fu t u r e t ime
2
t wh er e
2 1
t t > . Th e r et u r n you ea r n by
len din g you r mon ey fr om
1
t t o
2
t is a for wa r d r a t e.

Sign con t r a ct Len d mon ey Loa n r epa id

Time 0
1
t forward rate
2
t
Th e wor d for wa r d comes fr om t h e for wa r d ma r ket . In a for wa r d
ma r ket , a n a gr eemen t is ma de a t t ime zer o bu t t h e deliver y da t e is
a t a fu t u r e t ime
1
t . Exa mple. You pa y $10 t o a fa r mer t oda y for h im
t o deliver 15 t oma t oes t o you in 3 mon t h s . You do s o per h a ps you
r ea lly u s e t oma t oes a n d you wor r y t h a t t oma t o pr ice ma y go u p in
t h e fu t u r e. Mos t likely, people wh o bu y for wa r d pr odu ct s a r e n ot
dir ect ly con s u mer s , bu t a r e pr ofit ma ker s . Th ey h ope t o r es ell a
for wa r d con t r a ct a t a h igh er pr ice.

For wa r d ma r ket = fu t u r e deliver y.
A for wa r d r a t e a n s wer s t h e qu es t ion , If a n -yea r t er m bon d (or
a n ot h er in ves t men t veh icle) is deliver ed a t
1
t (wh er e
1
0 t > ) t o a n
in ves t or , wh a t r et u r n will t h e in ves t or get for len din g mon ey fr om
1
t
t o
1
t n + ? ( n is n ot n eces s a r ily a n in t eger )

For wa r d r a t e is oft en den ot ed a s
, j k
f . It s t h e in t er es t r a t e ch a r ged
for len din g mon ey fr om j t o j k + .
318
http://actuary88.com
5 . Re lat ions hip be t we e n s pot rat e s and forward rat e s (as s uming no
arbit rage )

4
s
3
s
2
s
1 0 1 ,
s f =
1 1 ,
f
2 1 ,
f
3 1 ,
f
Time 0 1 2 3 4

1 0 1 ,
1 1 s f + = +
( ) ( )( ) ( ) ( )
2
2 1 1 1 0 1 1 1 , , ,
1 1 1 1 1 s s f f f + = + + = + +
( ) ( ) ( ) ( )( )( )
3 2
3 2 2 1 0 1 1 1 2 1 , , , ,
1 1 1 1 1 1 s s f f f f + = + + = + + +
( ) ( ) ( ) ( ) ( )( ) ( )
4 3
4 3 3 1 0 1 1 1 2 1 3 1 , , , , ,
1 1 1 1 1 1 1 s s f f f f f + = + + = + + + +

( ) ( ) ( ) ( ) ( )( ) ( )
1
1 1 1 0 1 1 1 2 1 1 1 , , , , ,
1 1 1 1 1 1 ... 1
n n
n n n n
s s f f f f f


+ = + + = + + + +
Exa mple.
1
13.4% s = ,
2
15.51% s = . Wh a t s
1 1 ,
f ?
You h a ve t wo opt ion s :
(1) In ves t $1 a t t ime zero. Pu ll you r mon ey ou t a t t h e en d of
Yea r 2. You s h ou ld r ea p a n a n n u a l r et u r n of 15.51%.

(2) In ves t $1 a t t ime zero for on e yea r a n d pu ll you r mon ey
($1.134) ou t a t t h e en d of Yea r 1, r ea pin g 13.4% r et u r n .
Th en r ein ves t you r $1.134 du r in g Yea r 2, r ea pin g a r et u r n of
1 1 ,
f du r in g Yea r 2.

Th e a bove t wo opt ion s s h ou ld pr odu ce iden t ica l wea lt h . If n ot ,
looph oles exis t for people t o exploit .

319
http://actuary88.com
Time t 0 1 2 3

1
1+ s
1 1 ,
1+ f
( )
2
2
1 s +
( )( ) ( )
2
1 2 1 1 ,
1 1 1 s s f + + = +
( ) ( )
2 2
2
1
1 1 ,
1 1 15.51%
1 1 17.66%
1 1 13.4%
s
s
f
+ +
= = =
+ +

Sample proble ms

Proble m 1

You a r e given t h e followin g t h r ee s it u a t ion s :
#1 You ea r n ed $1,000,000 in a ga mblin g ga me a t La s
Vega s la s t week.
#2 Th e exch a n ge r a t e a t New Yor k is $2= 1
Th e exch a n ge r a t e a t Lon don is 2 =$6.
#3 Pr ogr a mmer s in a s oft wa r e compa n y wer e wor kin g
a r ou n d t h e clock t o fin is h bu ildin g a n ew s oft wa r e
pa cka ge. To boos t t h e mor a le of h is over -wor ked
pr ogr a mmer s , t h e CEO of t h e compa n y s et a n $50
per bu g in cen t ive pla n . Un der t h is pla n , a
pr ogr a mmer wa s t o be a wa r ded $50 for ea ch bu g h e
fou n d in h is codes .
Expla in wh ich s it u a t ion s r epr es en t a r bit r a gin g oppor t u n it ies .

Solut ion

#1 is NOT a n a r bit r a ge. In a n a r bit r a ge, on e ma kes mon ey wit h zer o cos t ;
s u r e pr ofit s a r e ma de wit h ou t a n y r is ks t a ken . In ga mblin g, h owever , a
ga mbler t a kes lot of r is ks , yet t h e pr ofit is n ot cer t a in .

#2 open s t o a r bit r a ge. For exa mple, you ca n ch a n ge $2 in t o 1 a t New
Yor k. Immedia t ely, you ch a n ge 1 ba ck t o $3 a t Lon don . You ca n do
t h es e t wo t r a n s a ct ion s over t h e in t er n et a n d ea r n $1 pr ofit wit h zer o cos t
a n d zer o r is k.

#3 open s t o a r bit r a ge. Th is is a n ot fa ke s t or y. A s oft wa r e compa n y
a ct u a lly did s omet h in g like t h is . Th e CEO h a d a good in t en t ion t o
320
http://actuary88.com
en cou r a ge h is pr ogr a mmer s t o qu ickly dis cover a n d fix bu gs . However ,
a ft er t h e in cen t ive policy wa s implemen t ed, s ome pr ogr a mmer s pu r pos ely
a dded bu gs in t o t h eir codes s o t h ey wou ld qu a lify for mor e r ewa r ds . Soon
t h e CEO fou n d ou t a bou t t h e looph ole a n d ca n celled t h e in cen t ive pla n .

Proble m 2

Con s ider t wo bon ds A a n d B in a fin a n cia l ma r ket . A is a on e-yea r bon d
wit h 10% a n n u a l cou pon a n d $100 pa r va lu e; A s ells for $97. B is a
t wo-yea r bon d wit h 8% a n n u a l cou pon a n d $100 pa r va lu e bon d; B s ells
for $88.

Ca lcu la t e
1
s a n d
2
s , t h e 1-yea r s pot a n d 2-yea r s pot r a t e.

Solut ion

1
s is t h e s pot r a t e for on e yea r , fr om 0 t = t o 1 t = . Th is is t h e r et u r n for
in ves t in g mon ey for on e yea r . Th e ca s h flows a t t h e en d of Yea r 1 s h ou ld
u s e
1
s a s t h e dis cou n t r a t e.

2
s is t h e s pot r a t e for t wo yea r s , fr om 0 t = t o 2 t = . Th is is t h e r et u r n for
in ves t in g mon ey for t wo yea r s . Th e ca s h flows a t t h e en d of Yea r 2 s h ou ld
u s e
2
s a s t h e dis cou n t r a t e.

We n eed t o s olve t h e followin g t wo equ a t ion s :

( )
1
2
1
2
110
97
1
8 108
88
1
1
s
s
s

= +
+
+

1 2
13.4%, 15.51% s s = =
321
http://actuary88.com
Proble m 3

3 mon t h s fr om n ow (i.e. a t 0.25 t = ), you n eed t o bor r ow $5,000 for s ix
mon t h s t o fu n d you r s ix-mon t h lon g va ca t ion . Wor r yin g t h a t t h e in t er es t
r a t e ma y go u p a lot in t h r ee mon t h s , you wa n t t o lock in a bor r owin g
r a t e r igh t n ow a t 0 t = .
You a r e given t h e followin g fa ct s :
Th e 3-mon t h s pot r a t e is 4.5% a n n u a l effect ive.
Th e 9-mon t h s pot r a t e is 5.7% a n n u a l effect ive.

Expla in h ow you ca n lock in , a t 0 t = , a $5,000 loa n for s ix mon t h s wit h a
gu a r a n t eed in t er es t r a t e. Ca lcu la t e t h e gu a r a n t eed bor r owin g r a t e.

Solut ion

Lock-in s t r a t egy:

At 0 t = , bor r ow ( )
0.25
$5, 000 1 4.5% 4, 945.28

+ = for 9-mon t h s @ 5.7% a n n u a l


effect ive. An d immedia t ely depos it t h e bor r owed $4,945.28 for 3 mon t h s
@4.5% a n n u a l effect ive.

At
3
0.25
12
t = = (3 mon t h s la t er ), you r $4,945.28 will gr ow in t o:
( )
0.25
4, 945.28 1 4.5% $5, 000 + =
You will u s e t h is $5,000 t o fu n d you r 6-mon t h lon g va ca t ion . Effect ively,
you h a ve bor r owed $5,000 a t 0.25 t = .
At
9
0.75
12
t = = (9 mon t h s la t er ), you r va ca t ion is over . You r loa n of
$4,945.28 is du e. You pa y off t h is loa n wit h a followin g pa ymen t a t
0.75 t = :
( )
0.75
4, 945.28 1 5.7% $5,196.92 + =
Th e in t er es t r a t e you locked fr om 0.25 t = t o 0.75 t = ca n be s olved a s
follows :

( ) ( ) ( )
0.25 0.5 0.75
4, 945.28 1 4.5% 1 4, 945.28 1 5.7% r + + = +
( ) ( ) ( )
0.5 0.75 0.25
1 1 5.7% 1 4.5% 1.031044 r

+ = + + = 6.3052% r =
322
http://actuary88.com
Time t 0 0.25 0.75
(year)

( )
0.25
1 4.5% + ( )
0.5
1 r +
( )
0.75
1 5.7% +
Th e r a t e you locked in , 6.3052% r = , is a 6-mon t h for wa r d r a t e @ 0.25 t = .
Proble m 4

3 mon t h s fr om n ow (i.e. a t 0.25 t = ), a n in s u r a n ce compa n y will s en d you
$5,000 ca s h . You wa n t t o in ves t $5,000 immedia t ely a ft er you r eceive it
a n d in ves t it for 6 mon t h s . Wor r yin g t h a t t h e in t er es t r a t e ma y go down a
lot in t h r ee mon t h s , you wa n t t o lock in a n in t er es t r a t e r igh t n ow a t
0 t = .
You a r e given t h e followin g fa ct s :
Th e 3-mon t h s pot r a t e is 4.5% a n n u a l effect ive.
Th e 9-mon t h s pot r a t e is 5.7% a n n u a l effect ive.

Expla in h ow you ca n lock in , a t 0 t = , a n in ves t men t oppor t u n it y wh er e
you ca n len d you r fu t u r e $5,000 for s ix mon t h s wit h a gu a r a n t eed
in t er es t r a t e. Ca lcu la t e t h e gu a r a n t eed ea r n in g r a t e.

Solut ion

Th is pr oblem is s imila r t o Pr oblem 3. Th e on ly differ en ce is t h a t t h is t ime
we wa n t t o len d mon ey a n d ea r n a gu a r a n t eed in t er es t r a t e.

Lock-in s t r a t egy:

At 0 t = , bor r ow ( )
0.25
$5, 000 1 4.5% 4, 945.28

+ = for 3 mon t h s @ 4.5% a n n u a l


effect ive. An d immedia t ely depos it t h e bor r owed $4,945.28 for 9-mon t h s
@5.7% a n n u a l effect ive.

At
3
0.25
12
t = = (3 mon t h s la t er ), you r bor r owed a mou n t of $4,945.28 will
gr ow in t o:

( )
0.25
4, 945.28 1 4.5% $5, 000 + =
323
http://actuary88.com
At 0.25 t = , you ca n pa y t h is loa n a mou n t u s in g t h e pa ymen t of $5,000
s en t t o you by t h e in s u r a n ce compa n y. As a r es u lt , you r loa n is pa id off.
At 0.75 t = (9 mon t h s la t er ), you r or igin a l depos it of $4,945.28 will gr ow
in t o:

( )
0.75
4, 945.28 1 5.7% $5,196.92 + =
Th e in t er es t r a t e you locked fr om 0.25 t = t o 0.75 t = ca n be s olved a s
follows :

( ) ( ) ( )
0.25 0.5 0.75
4, 945.28 1 4.5% 1 4, 945.28 1 5.7% r + + = +
( ) ( ) ( )
0.5 0.75 0.25
1 1 5.7% 1 4.5% 1.031044 r

+ = + + =
6.3052% r =
Time t 0 0.25 0.75
(year)

( )
0.25
1 4.5% + ( )
0.5
1 r +
( )
0.75
1 5.7% +
Th e r a t e you locked in , 6.3052% r = , is a 6-mon t h for wa r d r a t e @ 0.25 t = .
Proble m 5

You a r e given t h e followin g for ce of in t er es t :
( ) 5% 0.12% t t o = +
Ca lcu la t e

5
s , t h e 5 yea r s pot r a t e.

6
s , t h e 6 yea r s pot r a t e

5 1 ,
f , t h e on e yea r for wa r d r a t e fr om 5 t = t o 6 t =
Solut ion
324
http://actuary88.com
5
s is t h e a n n u a l r et u r n you ca n lock in for in ves t in g mon ey a t t ime zer o
for 5 yea r s .

( ) ( )
5
5
5
0
1 exp s t dt o
(
+ =
(

}
( ) ( )
2
0 0
5% 0.12% 5% 0.06%
t t
x dx x dx t t o = + = +
} }

( ) ( )
5
5
2 26.5%
5
5
0
1 exp exp 5% 0.06% 1.30343098
t
s t dt t t e o
=
(
( + = = + = =
(


}
5
5.44296% s =
Simila r ly,

( ) ( )
6
6
2 32.16%
6
0
6
1 exp exp 5% 0.06% 1.37933293
t
s t dt t t e o
=
(
( + = = + = =
(


}
6
5.50625% s =
To fin d
5 1 ,
f , we u s e t h e n o a r bit r a ge pr in ciple. Compa r e t h e followin g t wo
opt ion s :

At t ime zer o, lock in a 6 yea r in ves t men t a n d get
6
5.50625% s =
a n n u a l r et u r n .

At t ime zer o, lock in a 5 yea r in ves t men t a n d get
5
5.44296% s =
a n n u a l r et u r n . Th en r ein ves t t h e t ot a l mon ey s t a r t in g fr om t h e en d
of Yea r 5 a n d en din g a t t h e en d of Yea r 1 (i.e. du r in g Yea r 6),
r ea pin g a n a n n u a l r et u r n of
5 1 ,
f
Th es e t wo opt ion s s h ou ld gen er a t e t h e s a me a mou n t of wea lt h .

325
http://actuary88.com
Time t 0 1 2 3 4 5 6

( )
5
5
1 s +
5 , 1
1+ f
( )
6
6
1 s +
( ) ( ) ( )
5 6
5 6 5 1 ,
1 1 1 s s f + + = +
( )
( )
6
32.16%
6
5 26.5%
5
5 1 ,
1
1 1 5.823243%
1
s
e
e
s
f
+
= = =
+
Proble m 6

You a r e given t h e followin g fa ct s a bou t t h r ee s ecu r it ies A, B , a n d C :
Security Selling price
at t=0
Cash flow
at t=1
Cash flow
at t=2
Cash flow
beyond
t=2
A $7,570.03 $5,000 $4,000 $0
B $16,274.27 $10,000 $9,500 $0
C $0 $10,000 $0
Ca lcu la t e
(1) Th e pr ice of Secu r it y C a t t ime zer o a s s u min g n o a r bit r a ge.

(2) If Secu r it y C s ells for $7,200 a t t ime zer o, des ign a n a r bit r a ge
s t r a t egy a n d ca lcu la t e h ow mu ch pr ofit you ca n ma ke.

(3) If Secu r it y C s ells for $8,000 a t t ime zer o, des ign a n a r bit r a ge
s t r a t egy a n d ca lcu la t e h ow mu ch pr ofit you ca n ma ke

Solut ion

Calc ulat e t he pric e of Se c urit y C at t ime ze ro as s uming no arbit rage .

Let
1
s r epr es en t t h e on e yea r s pot r a t e.
Let
2
s r epr es en t t h e t wo yea r s pot r a t e.

326
http://actuary88.com
( )
2
1
2
5, 000 4, 000
7, 570.03
1
1
s
s
= +
+
+ ( )
2
1
2
10, 000 9, 500
16, 274.27
1
1
s
s
= +
+
+
Set
1
1
1
x
s
=
+
a n d
( )
2
2
1
1
y
s
=
+
.
7, 570.03 5, 000 4, 000 x y = + 16, 274.27 10, 000 9, 500 x y = +
1 2
1.1 , 1.15 x y

= =
Th e pr ice of Secu r it y C a t t ime zer o if n o a r bit r a ge:

( )
( )
2
2
2
10, 000
10, 000 1.15 7, 561.44
1 s

= =
+
If Se c urit y C s e lls for $ 7 , 2 0 0 at t ime ze ro, de s ign an arbit rage
s t rat e gy and c alc ulat e how muc h profit you c an make .

If Secu r it y C s ells for $7,200 a t t ime zer o, t h en t h is pr ice is below it s
ma r ket fa ir pr ice of $7,561.44, cr ea t in g a n a r bit r a ge oppor t u n it y.

To exploit t h is oppor t u n it y, we fir s t s yn t h et ica lly cr ea t e a 2-yea r zer o-
cou pon bon d wit h $10,000 pa r . Th is bon d is ( )
2
6 B-2A
3
| |
|
\ .
.
Secu r it y Sellin g pr ice
a t t =0
Ca s h flow
a t t =1
Ca s h flow
a t t =2
A $7, 570.03 $5, 000 $4, 000
B $16, 274.27 $10,000 $9, 500
B-2A $16,274.27-2($7, 570. 03)
= $1, 134. 21
$10, 000-2($5, 000)
=$0
$9, 500-2($4, 000)
=$1, 500
( )
2
6 B-2A
3
| |
|
\ .

2
6 $1,134.21 $7,561.40
3
| |
=
|
\ .

$0
2
6 $1,500 $10, 000
3
| |
=
|
\ .
Ba s ed on t h e a bove t a ble, a 2-yea r zer o-cou pon bon d wit h $10,000 pa r
ca n be cr ea t ed by bu yin g (or s ellin g s h or t )
2
6
3
u n it s of Secu r it y Bs a n d
s imu lt a n eou s ly s h or t s ellin g (or bu yin g)
2 1
6 2 13
3 3
= u n it s of Secu r it y As .
Th is s yn t h et ica lly cr ea t ed 2-yea r zer o cou pon $10,000 pa r bon d is wor t h
$7,561.40 a t t ime zer o.

327
http://actuary88.com
Ou r a r bit r a ge s t r a t egy if Secu r it y C s ells for $7,200 a t t ime zer o:

Th e a r bit r a ge s t r a t egy is a lwa ys bu y low, s ell h igh .

At t ime zer o, we s pen d $7,200 a n d bu y Secu r it y C . Th is is bu y
low.

Simu lt a n eou s ly a t t ime zer o, we s h or t s ell t h e s yn t h et ica lly cr ea t ed
s ecu r it y ( )
2
6 B-2A
3
| |
|
\ .
, wh ich h a s t h e s a me ca s h flow a s Secu r it y C
bu t is wor t h y $7,561.4, $361.4 mor e t h a n Secu r it y C . Th is is s ell
h igh .

Sh or t s ellin g ( )
2
6 B-2A
3
| |
|
\ .
mea n s s h or t s ellin g
2
6
3
B
| |
|
\ .
a n d immedia t ely
bu yin g
2
6
3
A
| |
|
\ .
fr om t h e ma r ket .

At 2 t = , we get $10,000 fr om Secu r it y C . Well u s e t h is $10,000 t o
pa y s ecu r it y ( )
2
6 B-2A
3
| |
|
\ .
s ca s h flow a t 2 t = , clos in g ou r s h or t
pos it ion .

Th e n et r es u lt : a t t ime zer o, we ea r n $7,561.40 - $7,200= $361.40
s u r e pr ofit per t r a n s a ct ion a bove. Of cou r s e, if we ca n ma ke 1,000
s u ch t r a n s a ct ion s in a da y, well ma ke $361,400 in a da y.

If Se c urit y C s e lls for $ 8 , 0 0 0 at t ime ze ro, de s ign an arbit rage
s t rat e gy and c alc ulat e how muc h profit you c an make

At t ime zer o, we s ell s h or t Secu r it y C . We ea r n $8,000 ca s h .

Simu lt a n eou s ly a t t ime zer o, we bu y t h e s yn t h et ica lly cr ea t ed
s ecu r it y ( )
2
6 B-2A
3
| |
|
\ .
, wh ich mea n s bu yin g B
2
6
3
| |
|
\ .
a n d s h or t s ellin g
( )
2
6 2A
3
| |
|
\ .
.
At 2 t = , we get $10,000 fr om t h e s yn t h et ica lly cr ea t ed s ecu r it y
( )
2
6 B-2A
3
| |
|
\ .
. Well u s e t h is $10,000 t o pa y Secu r it y C s ca s h flow a t
2 t = a n d clos e ou t ou r s h or t pos it ion .

328
http://actuary88.com
Th e n et r es u lt : a t t ime zer o, we ea r n $8,000- $7,561.40= $438.60
s u r e pr ofit per t r a n s a ct ion a bove. Of cou r s e, if we ma ke 1,000
s u ch t r a n s a ct ion s in a da y, well ma ke $438,600 in a da y.

Proble m 7

You a r e given t h e followin g s pot r a t es :

%
5%
5
t
t
s = + , wh er e 1, 2, 3 t =
Ca lcu la t e
1 1 ,
f a n d
2 1 ,
f , t h e on e yea r for wa r d r a t es a t 1 t = a n d a t 2 t = .
Solut ion

( )( ) ( )
2
1 1 1 2 ,
1 1 1 s f s + + = +
( ) ( ) ( )
2 3
2 2 1 3 ,
1 1 1 s f s + + = +
Let s look a t t h e mea n in g of t h es e t wo equ a t ion s .

Th e 1
s t
equ a t ion s a ys t h a t if you in ves t you r mon ey yea r by yea r , you
s h ou ld h a ve t h e s a me wea lt h a t 2 t = a s you in it ia lly lock in a 2-yea r
in ves t men t oppor t u n it y. Ot h er wis e, a n a r bit r a ge oppor t u n it y exit s .

Time t 0 1 2

1
1+ s
1 1 ,
1+ f
( )
2
2
1 s +
Simila r ly, t h e 2
n d
equ a t ion compa r es t wo in ves t men t opt ion s .

Opt ion 1 . At t ime zer o, in ves t men t $1 a n d lock in a t wo yea r
in ves t men t oppor t u n it y a n d a ccu mu la t e t o ( )
2
2
1 s + a t 2 t = . Next ,
immedia t ely r ein ves t you r wea lt h of ( )
2
2
1 s + in a on e yea r
in ves t men t oppor t u n it y a n d ea r n a for wa r d r a t e
2 1 ,
f du r in g Yea r
2. Opt ion 1 a ccu mu la t es a t ot a l of ( ) ( )
2
2 2 1 ,
1 1 s f + + a t 3 t = .
329
http://actuary88.com
Opt ion 2 . At t ime zer o, in ves t men t $1 in a 3 yea r in ves t men t
oppor t u n it y, lockin g in a n a n n u a l r et u r n of
3
s . Th is a ccu mu la t es a
t ot a l wea lt h of ( )
3
3
1 s + a t 3 t = .
No a r bit r a ge pr in ciple r equ ir es t h a t t h e t wo opt ion s gen er a t e a n
iden t ica l a mou n t of wea lt h a t 3 t = .
Time t 0 1 2 3

( )
2
2
1 s +
2 1 ,
1+ f
( )
3
3
1 s +
On ce you u n der s t a n d t h e mea n in g of t h es e t wo equ a t ion s , t h e r ema in in g
wor k is pu r ely a lgebr a .

%
5%
5
t
t
s = +
1
1%
5% 5.2%
5
s = + =
2
2 %
5% 5.4%
5
s = + =
2
3 %
5% 5.6%
5
s = + =
( ) ( )
2 2
2
1 1
1
,
1 1 5.4%
1 1 5.60%
1 1 5.2%
s
f
s
+ +
= = ~
+ +

( )
( )
( )
( )
3 3
3
2 1 2 2
2
,
1 1 5.6%
1 1 6.40%
1 1 5.4%
s
f
s
+ +
= = ~
+ +

Proble m 8

You a r e given t h e followin g in for ma t ion wit h r es pect t o a bon d:

pa r a mou n t : $1,000
t er m t o ma t u r it y: 3 yea r s
a n n u a l cou pon r a t e 8% pa ya ble a n n u a lly
1-yea r con t in u ou s s pot r a t e is 5%
1-yea r for wa r d r a t e @ 1 t = is 6%
1-yea r for wa r d r a t e @ 2 t = is 7%
330
http://actuary88.com
Ca lcu la t e YTM (yield t o ma t u r it y) of t h e bon d.

Solut ion

Time t 0 1 2 3

Cash flow $80 $80 $1,080

1
1+ s
1 1 ,
1+ f
2 1 ,
1+ f
Fir s t , we n eed t o ca lcu la t e t h e PV of t h e bon d. PV of t h e bon d is s u m of
ea ch ca s h flow dis cou n t ed a t a n a ppr opr ia t e s pot r a t e.

( )
( ) ( ) ( )
3
2 3
1
1
2 3
80 80 1, 080
1
1 1 1
t
t
t
PV
s
s s
CF t
s =
= = + +
+
+ + +

So we n eed t o ca lcu la t e t h e 1-yea r , 2-yea r , a n d 3-yea r s pot r a t es .



We a r e given :
0.05
1
1+s e =
Plea s e n ot e t h a t we a r e given a con t in u ou s s pot r a t e 5% o = in t h e 1
s t

yea r .

We a r e a ls o given :
1 1 ,
6% f = ,
2 1 ,
7% f =
Us in g t h e r ela t ion s h ip bet ween s pot r a t es a n d for wa r d r a t es , we h a ve:

( ) ( )( ) ( )
2
0.05
2 1 1 1 ,
1 1 1+ 1.06 s s f e + = + =
( ) ( ) ( )( ) ( ) ( )
3
0.05
3 1 1 1 2 1 , ,
1 1 1+ 1+ 1.06 1.07 s s f f e + = + =
( ) ( )( )
0.05 0.05 0.05
80 80 1, 080
1, 053.66
1.06 1.06 1.07
PV
e e e
= + + =
YTM ca n be s olved in t h e followin g equ a t ion :

( ) ( )
2 3
80 80 1, 080
1, 053.66
1
1 1
y
y y
= + +
+
+ +

331
http://actuary88.com
To s olve t h is equ a t ion , u s e BA II Plu s / BA II Plu s Pr ofes s ion a l.

En t er PMT=80, N=3, FV=1,000, PV= - 1,053.66. Let t h e ca lcu la t or s olve
for I/ Y.

We s h ou ld get : I/ Y=5.99279%

So t h e yield t o ma t u r it y is 5.99279%.

Proble m 9

Sh or t t er m, on e-yea r a n n u a l effect ive in t er es t r a t es a r e cu r r en t ly 10%;
t h ey a r e expect ed t o be 9% on e yea r fr om n ow, 8% t wo yea r s fr om n ow,
7% t h r ee yea r s fr om n ow, a n d 6% fou r yea r s fr om n ow.

Ca lcu la t e
Th e s pot yield of 1-yea r , 2-yea r , 3-yea r , 4-yea r , a n d 5-yea r zer o
cou pon bon ds r es pect ively.

Th e a n n u a l effect ive yield of a bon d r edeemed a t $100 pa r va lu e in
5 yea r s a n d pa ys 8% cou pon a n n u a lly.

Solut ion

Fir s t , let s dr a w a dia gr a m:

Time t 0 1 2 3 4 5

Cash flow $8 $8 $8 $8 $108

1
1+ s
1 1 ,
1+ f
2 1 ,
1+ f
3 1 ,
1+ f
4 1 ,
1+ f
10% 9% 8% 7% 6%
( )
2
2
1 s +
( )
3
3
1 s +
( )
4
4
1 s +
332
http://actuary88.com
( )
5
5
1 s +
Let
t
s r epr es en t t h e s pot yield over t yea r s .

1
10% s =
( )
2
2
1 1.1 1.09 s + = ,
2
9.499% s =
( )
3
3
1 1.1 1.09 1.08 s + = ,
3
8.997% s =
( )
4
4
1 1.1 1.09 1.08 1.07 s + = ,
4
8.494% s =
( )
5
5
1 1.1 1.09 1.08 1.07 1.06 s + = ,
5
7.991% s =
Next , let s ca lcu la t e t h e a n n u a l effect ive yield of a bon d r edeemed a t pa r
va lu e of $100 in 5 yea r s a n d pa ys 6% cou pon a n n u a lly.

Th e PV of t h e bon d is :

( )( )
( ) ( ) ( ) ( )
5
2 3 4 5
1
8 8 8 8 108
1
1 10%
1 9.499% 1 8.997% 1 8.494% 1 7.991%
t
t
t
CF t s

=
+ = + + + +
+
+ + + +

99.43227 =
Next , we n eed t o s olve t h e equ a t ion :

5
5
99.43227 8 100
i
a v = +
In BA II Plu s TVM, En t er PV= - 99.43227, PMT = 8, FV =100, N=5.

Pr es s CPT I/ Y. You s h ou ld get : I/ Y=8.143%

So t h e a n n u a l effect ive yield is 8.143%.

333
http://actuary88.com
Proble m 1 0 (Nove mbe r 2 0 0 5 FM #6 )
Con s ider a yield cu r ved defin ed by t h e followin g equ a t ion

2
0.09 0.002 0.001
k
i k k = +
h er e
k
i is t h e a n n u a l effect ive r a t e of r et u r n of zer o cou pon bon ds wit h
ma t u r it y of k yea r s .

Let j be t h e on e-yea r effect ive r a t e du r in g yea r 5 t h a t is implied by t h is
yield cu r ve.

Ca lcu la t e j .
Solut ion
Fir s t , let s dr a w a dia gr a m:

Time t 0 1 2 3 4 5

1
1 i +
1 1 ,
1+ f
2 1 ,
1+ f
3 1 ,
1+ f
4 1 ,
1+ f
( )
2
2
1 i +
( )
3
3
1 i +
( )
4
4
1 i +
( )
5
5
1 i +
We a r e a s ked t o fin d
4 1 ,
j f = . Plea s e n ot e t h a t Yea r 5 is fr om t =4 t o t =5,

( ) ( ) ( )
4 5
4 4 1 5 ,
1 1+ 1 i f i + = +
334
http://actuary88.com
( )
( )
( ) ( )
( ) ( )
5
2
5
5
5
4 1 4 4 4
2
4
,
1 0.09 0.002 5 0.001 5
1
1.075
1 1 1 4.74%
1.082
1
1 0.09 0.002 4 0.001 4
i
j f
i
(
+ +
+

= = = = =
+
(
+ +


Proble m 1 1

Th e on e-yea r s pot in t er es t r a t e a t 0 t = is 6% a n n u a l effect ive.

Th e a n n u a l effect ive yield of a t wo yea r bon d is s u ed a t 0 t = t h a t pa ys 4%
a n n u a l cou pon s a n d t h a t is r edeemed a t pa r va lu e of $100 is 7% a n n u a l
effect ive.

Th e is s u e pr ice a t 0 t = of a t h r ee-yea r bon d t h a t pa ys 8% cou pon s
a n n u a lly is $102 per $100 n omin a l.

Ca lcu la t e

1, 1
f , t h e on e-yea r s pot r a t e a t 1 t =

2 , 1
f , t h e on e-yea r for wa r d r a t e a t 2 t = .
Solut ion

Let s fir s t ca lcu la t e t h e PV of t h e bon d is s u ed a t 0 t = t h a t pa ys 4%
a n n u a l cou pon s a n d t h a t is r edeemed a t pa r va lu e of $100 is 7% a n n u a l
effect ive.

( )
2
2 7%
4 100 1.07 94.576 PV a

= + =
Time t 0 1 2 3

1
1 i +
1 1 ,
1+ f
2 1 ,
1+ f
1 6% +
( )
2
2
1 i +
( )
3
3
1 i +
( ) ( ) ( )
2
1 1, 1 2
4 104 4 104
94.576
1 1 6% 1 6% 1 1
i f i
= + = +
+ + + + +
335
http://actuary88.com

1, 1
8.051% f =
Th e is s u e pr ice a t 0 t = of a t h r ee-yea r bon d t h a t pa ys 8% cou pon s
a n n u a lly is $102 per $100 n omin a l:

( ) ( )
2 3
1
2 3
8 8 8 100
102
1
1 1
i
i i
+
= + +
+
+ +

( ) ( ) ( )( )( )
2 1 ,
8 8 8 100
102
1 6% 1 6% 1 8.051% 1 6% 1 8.051% 1 f
+
= + +
+ + + + + +

2 1 ,
7.805% f =
Proble m 1 2

Th e n yea r s pot r a t es a t 0 t = a r e defin ed a s follows :

0.05
1000
n
n
i = + , wh er e 1, 2, 3
Ca lcu la t e t h e implied on e-yea r for wa r d r a t e a t 1 t = a n d 2 t = .
Solut ion

We a r e a s ked t o fin d
1 1 ,
f a n d
2 1 ,
f .
Time t 0 1 2 3

1
1 i +
1 1 ,
1+ f
2 1 ,
1+ f
( )
2
2
1 i +
( )
3
3
1 i +
336
http://actuary88.com
( ) ( ) ( )
2
2 1 1 1 ,
1 1 1+ i i f + = + , ( ) ( ) ( )
3 2
3 2 2 1 ,
1 1 1+ i i f + = +
1
1
0.05 0.051
1000
i = + = ,
2
2
0.05 0.052
1000
i = + = ,
3
3
0.05 0.053
1000
i = + =
( )
2
2
2
1 1
1
,
1
1.052
1 1 5.3%
1 1.051
i
f
i
+
= = =
+
( )
( )
3
3
3
2 1 2 2
2
,
1
1.053
1 1 5.5%
1.052
1
i
f
i
+
= = =
+
Proble m 1 3
Th e followin g n -yea r s pot r a t es a r e obs er ved a t 0 t = :
1-yea r s pot r a t e is 2%
2-yea r s pot r a t e is 3%
3-yea r s pot r a t e is 4%
4-yea r s pot r a t e is 5%
5-yea r s pot r a t e is 6%
6-yea r s pot r a t e is 7%
7-yea r s pot r a t e is 8%
8-yea r s pot r a t e is 9%
9-yea r s pot r a t e is 10%

Ca lcu la t e
5 , 4
f , t h e 4-yea r for wa r d r a t e a t 5 t = .
Solut ion

( ) ( ) ( )
4
5 9
5 5 , 4 9
1 1 1 s f s + + = +
5
6% s = ,
9
10% s =
( )
( )
( )
( )
( )
9 9
4
9
5 , 4 5 5
5
1 1 10%
1
1 1 6%
s
f
s
+ +
+ = =
+ +

5 , 4
15.21% f =
337
http://actuary88.com
Proble m 1 4
Th e s pot yield cu r ve on 1/ 1/ 2006 is defin ed a s follows :

0.06
0.04 0.03
t
t
s e

= +
Ca lcu la t e
3, 2
f .
Solut ion

( ) ( ) ( )
2
3 5
3 3, 2 5
1 1 1 s f s + + = +
( ) 0.06 3
3
0.04 0.03 0.0634108 s e

= + =
( ) 0.06 5
5
0.04 0.03 0.059633 s e

= + =
( )
( )
( )
5
5
2
5
3, 2 3 3
3
1
0.0634108
1
0.059633
1
s
f
s
+
+ = =
+
3, 2
5.4% f =
Proble m 1 5
You a r e given t h e followin g for wa r d r a t es :

0 , 1
5% f = ,
1, 1
6% f = ,
2 , 1
7% f =
Ca lcu la t e t h e a n n u a l effect ive yield of a 3-yea r bon d t h a t pa ys 8% a n n u a l
cou pon wit h fa ce a mou n t of $100.

Solut ion

Time t 0 1 2 3

Cash flows $8 $8 $108

1
1 i +
1 1 ,
1+ f
2 1 ,
1+ f
( )
2
2
1 i +
338
http://actuary88.com
( )
3
3
1 i +
Th e pr ice of t h e bon d (i.e. t h e PV) is :

( ) ( )
( ) ( )( )
2 3
1
2 3
8 8 8 100 8 8 8 100
105.49378
1 1.05 1.05 1.06 1.05 1.06 1.07
1 1
i
i i
+ +
+ + = + + =
+
+ +

Next , we s olve t h e followin g equ a t ion :

3
3
105.49378 8 100
i
a v = +
Us in g BA II Plu s TVM, we s h ou ld get :

5.94675% i =
Proble m 1 6 (Sample FM #3 3 )
You a r e given t h e followin g in for ma t ion wit h r es pect t o a bon d:

Pa r a mou n t : $1,000
Ter m t o ma t u r it y: 3 yea r s
An n u a l cou pon : 6% pa ya ble a n n u a lly

Term Annual spot interest rates
1 7%
2 8%
3 9%
Ca lcu la t e t h e va lu e of t h e bon d.

Solut ion

Time t 0 1 2 3

Cash flows $60 $60 $1,060

1
1 i +
1 1 ,
1+ f
2 1 ,
1+ f
( )
2
2
1 i +
339
http://actuary88.com
( )
3
3
1 i +
Th e bon d is wor t h it s pr es en t va lu e. Th e pr es en t va lu e is :

( ) ( ) ( ) ( )
2 3 2 3
1
2 3
60 60 60 100 60 60 60 100
926.03
1 1 7%
1 1 1 8% 1 9%
i
i i
+ +
+ + = + + =
+ +
+ + + +

Proble m 1 7 (Sample FM #3 4 )

You a r e given t h e followin g in for ma t ion wit h r es pect t o a bon d:
Pa r a mou n t : $1,000
Ter m t o ma t u r it y: 3 yea r s
An n u a l cou pon : 6% pa ya ble a n n u a lly
Term Annual spot interest rates
1 7%
2 8%
3 9%
Ca lcu la t e t h e a n n u a l effect ive yield r a t e for t h e bon d if t h e bon d is s old a t
a pr ice equ a l t o it s va lu e.

Solut ion

Fr om Pr oblem 16, we kn ow t h e pr es en t va lu e of t h e bon d is 926.03.

3
3
926.03 60 1000
i
a v = +
Us in g BA II Plu s TVM, we get :

8.918% i =
340
http://actuary88.com
Chapt e r 1 6 Mac aulay durat ion,
modified durat ion, convexit y

An a s s et h a s t h e followin g ca s h flows :

Time t 0 1 k n
Cash flow ( ) 0 CF ( ) 1 CF ( ) CF k ( ) CF n
( )
( )
( ) 0 0 1
t
t
n n
t t
CF t
P CF t v
r = =
= =
+


As s et pr ice = Pr es en t va lu e of t h e fu t u r e ca s h flows
We wa n t t o fin d ou t h ow P , t h e a s s et pr ice, is s en s it ive t o t h e ch a n ge of
t h e in t er es t r a t e r , t h e effect ive in t er es t per yea r .

We defin e t h e followin g t er m:

Ma ca u la y du r a t ion = n ega t ive pr ice ela s t icit y r ela t ive t o ( ) 1 r +
( )
( )
% change in price
Asset Price Elasticity relative to 1
%change in 1
MAC
r
r
D = + =
+
Oft en t ime, Ma ca u la y du r a t ion is s imply ca lled du r a t ion . If you a r e a s ked
t o ca lcu la t e t h e du r a t ion , ju s t ca lcu la t e Ma ca u la y du r a t ion .
We ma ke t h e Ma ca u la y du r a t ion equ a l t o t h e n ega t ive pr ice ela s t icit y.
Th is wa y, t h e Ma ca u la y du r a t ion becomes a pos it ive n u mber -- we like
pos it ive n u mber s bet t er . To s ee wh y t h e Ma ca u la y du r a t ion is pos it ive,
n ot ice t h a t t h e pr es en t va lu e of a n a s s et is in ver s ely r ela t ed t o t h e
in t er es t r a t e. If r goes u p (i.e. % ch a n ge of r is pos it ive), t h en we dis cou n t
ca s h flows a t a h igh er dis cou n t r a t e, ca u s in g t h e a s s et pr ice t o go down
(i.e. % ch a n ge of t h e a s s et pr ice becomes n ega t ive). Simila r ly, if r goes
down (i.e. % ch a n ge of r is n ega t ive), t h en we dis cou n t ca s h flows a t a
lower dis cou n t r a t e, ca u s in g t h e a s s et pr ice t o go u p (i.e. % ch a n ge of t h e
341
http://actuary88.com
a s s et pr ice becomes pos it ive). By s et t in g t h e Ma ca u la y du r a t ion equ a l t o
t h e n ega t ive pr ice ela s t icit y, well get a pos it ive n u mber .

( )
( )
1
1
1
1
1
MAC
dp dp
dp p p
r
dr d r p dr
r
r
D
(
= = = +
(
+

+
+

( )
( )
( ) 0 0 1
t
t
n n
t t
CF t
P CF t v
r = =
= =
+


( ) ( ) ( ) ( ) ( )
0 0 0
1 1
1 1
1 1
n n n
t t
t
t t t
dP d
CF t r t CF t r t CF t v
dr dr r r

= = =
= + = + =
+ +


( ) ( ) ( )
0
1 1
1 1
1
1
n
t
MAC
t
dp
r r
p dr p
D t CF t v
r
=
( (
= + = +
( (


+

( ) ( )
( )
0 0
0
t
n n
t t
t t
MAC n
t
P
t CF t v t CF t v
D
CF t v
= =
=
= =

Obs er va t ion :
A zer o cou pon bon ds du r a t ion is s imply it s ma t u r it y. A zer o cou pon
bon d h a s on ly on e ca s h flow a t it s ma t u r it y n .
( )
( )
n
MAC n
n CF n
n
CF n
v
D
v
= =
We defin e t h e 2
n d
t erm:

1
Modified Duration= -
MOD
dP
p dr
D =
( ) ( )
0 0
1 1 1
- -
1 1
1 1
t
n n
t
MOD
t t
dP
p dr p p
D t CF t v t CF t v
r r
= =
| | (
= = =
| (
\ .

+


( )
0
1 1
-
1 1
1 1
t
n
MOD MAC
t
dP
p dr p
D t CF t v D
r r
=
| |
= = =
|
\ .

+ +

342
http://actuary88.com
Wa ll St r eet met h od of ca lcu la t in g t h e modified du r a t ion :

( )
( )
0
0
1
1 1
t
n
t
MAC t
MOD n
t
y
m
t CF t v
D
D
y
CF t v
m
=
=
= =
+ +

y is t h e yield t o ma t u r it y expr es s ed a s a n omin a l in t er es t r a t e


compou n din g a s oft en a s cou pon s a r e pa id; m is t h e # of cou pon s per
yea r . Con s equ en t ly,
y
m
is t h e effect ive in t er es t per cou pon per iod.
To u n der s t a n d t h e Wa ll St r eet for mu la for t h e modified du r a t ion , plea s e
n ot e t h a t wh en Wa ll St r eet t a lks a bou t a bon d yield, it u s es a n omin a l
in t er es t r a t e compou n din g a s fr equ en t ly a s cou pon s a r e pa id, n ot t h e
a n n u a l effect ive in t er es t . For exa mple, for a bon d t h a t pa ys cou pon s
s emia n n u a lly, if t h e yield t o ma t u r it y is 10.25% a n n u a l effect ive, t h en
Wa ll St r eet will qu ot e t h e bon d yield a s
( ) 2
y i = .
2
1 1 10.25% 10%
2
y
y
| |
+ = + =
|
\ .

So Wa ll St r eet qu ot es t h e yield t o ma t u r it y a s 10%.

Th en wh y does Wa ll St r eet u s e
( )
( )
1
1
1
1 1
t
n
t
MAC t
MOD n
t
y
m
t CF t v
D
D
y
CF t v
m
=
=
= =
+ +

?
Plea s e n ot e t h a t h er e we s t a r t off wit h 1 t = , n ot 0 t = . Th is is beca u s e a
bon ds fir s t cou pon does n ot s t a r t off wit h 0 t = .
Let y r epr es en t t h e yield t o ma t u r it y expr es s ed a s t h e n omin a l in t er es t
r a t e compou n din g a s fr equ en t ly a s cou pon s a r e pa id. As s u me t h a t
cou pon s a r e pa id m-t h ly. So es s en t ia lly,
( ) m
y i = a n d t h e effect ive in t er es t
r a t e 1 1
m
y
r
m
| |
= +
|
\ .
. As s u me t h e bon ds t er m t o ma t u r it y is n . Th e pr es en t
va lu e of t h e bon d is :

343
http://actuary88.com
( )
( )
( )
1 1 1
1
t
n n
m t
t t
CF t CF t
P
r y
m
= =
= =
+ | |
+
|
\ .


( )
( )
( )
1
1 1
1 1 1 1
1 1
n n
m t m t
t t
CF t t CF t
dP d
m
P dy P dy P m
y y
m m
+
= =
| |
= =
|
\ .
| | | |
+ +
|
\ . \ .

|


( ) ( )
( ) 1 1
1 1 1 1 1
1
1 1 1
1
M AC t
n n
mt
t t
t CF t CF t
D
y y y
P P
r y
m m m
m
= =
| |
= = =
|
\ . + | |
+ + +
+
|
\ .



1 1
1
M O D M AC
dP
D D
y
P dy
m
= =
+
So t h e Wa ll St r eet for mu la a n d t h e t ext book for mu la a r e differ en t :

Wa ll St r eet :
1
M O D
Wall Street
dP
D
P dy
=
(der iva t ive of pr ice r ela t ive t o n omin a l yield)
Text book:
1
M O D
textbook
dP
p dr
D =
(der iva t ive of pr ice r ela t ive t o effect ive yield)
Plea s e n ot e t h a t Bon d Wor ks h eet in BA II Plu s Pr ofes s ion a l u s es t h e Wa ll
St r eet met h od t o ca lcu la t e t h e bon d pr ice a n d t h e modified du r a t ion . If
you u s e Bon d Wor ks h eet t o ca lcu la t e t h e bon d pr ice a n d it s modified
du r a t ion , ma ke s u r e t h a t you u s e t h e n omin a l in t er es t r a t e. You ll get a
wr on g r es u lt if you en t er t h e a n n u a l effect ive yield t o ma t u r it y in t o Bon d
Wor ks h eet .

In Exa m FM, u s e t he t ext book defin it ion of t h e modified du r a t ion :

1
1
MOD MAC
D D
r
=
+
(wh er e r is t h e a n n u a l effect ive yield t o ma t u r it y)

Don t u s e t h e Wa ll St r eet defin it ion .

344
http://actuary88.com
For a bon d pa yin g cou pon s a n n u a lly, t h es e t wo met h ods pr odu ce t h e
s a me modified du r a t ion . Th is is beca u s e we h a ve 1 m = a n d
( ) 1
y i r = = .
We defin e t h e 3
r d
t er m:

( ) ( ) ( ) ( ) ( )
2
2
1 2
1 1
1 1 1
Convexity= 1 1 1
n n
t t
t t
d P d
t
p dr p dr p
t CF t r t CF t r

= =
(
= = +
(

+ +


( )
( ) ( )
2
1
1 1
Convexity 1
1
n
t
t
t
p
r
t CF t v
=
= +
+

( )
( ) ( )
2
2
1 1
1 1 1
1
n n
t t
t t
t
p p
r
t CF t v CF t v
= =
(
= +
(
+



( )
( )
2
2
1
1 1
1
n
t
t
Duration
p
r
t CF t v
=
(
= +
(
+

( )
2 2
1
1
n
t
t
v Duration
p
t CF t v
=
(
= +
(

How t h e pr ice of a n a s s et ch a n ges if t h e in t er es t ch a n ges by a s ma ll


a mou n t :

( ) ( )
( )
( ) 1 1 1
t
t
n n
t t
CF t
P r CF t v
r = =
= =
+


( )
2
2
2
1
...
2
dP d P
P r r
dr dr
A = A + A + (Ta ylor s er ies )

Divide by P :
( )
2
2
2
1 1 1
...
2
P dP d P
r r
P P dr P dr
A
= A + A +
( ) ( )
2 1
...
2
MOD
D r Convexity r = A + A +
( )( )
2 1
1
1
...
2
MAC
D
r
r Convexity r
+
= A + A +
If t h e in t er es t r a t e ch a n ge r A is s ma ll, we ca n s et ( )
2
0 r ~ A
345
http://actuary88.com
1
1
1
MOD MAC
D D
r
P dP
r r r
P P dr
~
+
A
A = A = A
Sample proble ms

Proble m 1

Bon d fa ce $100
Cou pon 4% s emia n n u a l
Ter m t o ma t u r it y 3 yea r s
Yield t o ma t u r it y 5% a n n u a l effect ive.
Us e BA II Plu s / BA II Plu s Pr ofes s ion a l Ca s h Flow Wor ks h eet , ca lcu la t e
t h e du r a t ion (i.e. Ma ca u la y du r a t ion ), modified du r a t ion , a n d con vexit y of
t h e bon d.

Solut ion

Fir s t , we dr a w a ca s h flow dia gr a m.

Unit time = 1 year
Time t 0 0.5 1 1.5 2 2.5 3
Cash flow $2 $2 $2 $2 $2 $2
$100
For bon ds , oft en it s ea s ier if we s et t h e u n it t ime = t h e pa ymen t per iod:

Unit time = 0.5 year
Time t 0 1 2 3 4 5 6
Cash flow $2 $2 $2 $2 $2 $2
$100
( )
( )
1
3
1
3
1
1.05
Unit time is
one year and
t
t
t
MAC
t
v
t CF t v
D
CF t v

=
=
=
=

(wh er e t=0.5, 1, 1. 5, 2, 2.5, 3)


346
http://actuary88.com
( )
( )
1
2
6
1
6
1
1.05
Unit time is
half a year
1
2
t
t
t
t
v
t CF t v
CF t v

=
=
=
=

(wh er e t=1, 2, 3, 4, 5, 6)
We s et u p t h e followin g t a ble (we a r e u s in g 6 mon t h s a on e u n it t ime):

t
( ) CF t ( ) t CF t
2
t ( )
2
t CF t
1 2 2 1 2
2 2 4 4 8
3 2 6 9 18
4 2 8 16 32
5 2 10 25 50
6 102 612 36 3,672
Fir s t , well fin d t h e bon ds pr ice. We en t er ( ) CF t in t o Ca s h Flow
Wor ks h eet :

t
( ) CF t
Ca s h flows Ca s h flow
fr equ en cy
1 $ 2 CF1 1
2 $ 2 CF2 1
3 $ 2 CF3 1
4 $ 2 CF4 1
5 $ 2 CF5 1
6 $ 1 0 2 CF6 1

( )
6
1
97.41125361
t
t
NPV CF t v
=
= =

In t er es t Ra t e per cou pon per iod


1.05 1 2.4695% i = =
Ca lcu la t e NPV 9 7 . 4 1 1 2 5 3 6 1
347
http://actuary88.com
Next , we ca lcu la t e
( )
6
1
t
t
t CF t v
=

. We en t er t h e followin g in t o Ca s h Flow
Wor ks h eet :

t
( ) CF t ( ) t CF t
Ca s h
flows
Ca s h flow
fr equ en cy
1 2 2 CF1 1
2 2 4 CF2 1
3 2 6 CF3 1
4 2 8 CF4 1
5 2 1 0 CF5 1
6 102 6 1 2 CF6 1
In t er es t r a t e
1.05 1 2.4695% i = =
Ca lcu la t e NPV NPV=5 5 6 . 1 1 4 4 4 3 3
( )
6
1
556.1144433
t
t
t CF t v
=
=

( )
( )
6
1
6
1
1 1 556.1144433
2.85446713
2 2 97.41125361
t
t
t
MAC
t
t CF t v
D
CF t v
=
=
| |
= = =
|
\ .

Next , well ca lcu la t e t h e modified du r a t ion .



Wa ll St r eet met h od:

1
1 effective yield per coupon period
M O D M AC
Wall Street
D D =
+
1 2.85446713
2.78567468
1.05 1.05
M AC
D = = =
Text book met h od:

1
1 annual effective yield rate
M O D M AC
Textbook
D D =
+
348
http://actuary88.com
1 2.85446713
2.71854012
1.05 1.05
M AC
D = = =
Next , well ca lcu la t e t h e con vexit y u s in g t h e followin g for mu la :

( )
( ) ( )
2
3
1
Unit time is one year
1 1
Convexity 1
1
t
t
t
p
i
t CF t v
=
= +
+

( )
( ) ( )
2
2
3 3
1 1
Unit time is one year
1 1
1 5%
1.05 1.05
t t
t t
t
p
t CF t CF t

= =
(
=
(
+
+

( )
( ) ( )
2
2
6 6
1 1
,
-1
Unit time is half a year
1
(1 ) 2.4695%
1
2
1 5%
2
t t
t t
i v i
t
p
t
CF t v CF t v
= =
= = +
(
| |
=
(
|
\ . +
(

+

( )
( ) ( )
2
2
6 6
1 1
,
-1
Unit time is half a year
1
(1 ) 2.4695%
1 1 1
4 2
1 5%
t t
t t
i v i
p
t CF t v t CF t v
= =
= = +
(
=
(
+

+

To ca lcu la t e
( )
2
6
1
t
t
t CF t v
=

, we en t er ( )
2
t CF t (bold n u mber s below) in t o
Ca s h Flow Wor ks h eet :

t
( ) CF t
2
t ( )
2
t CF t
Ca s h
flows
Ca s h flow
fr equ en cy
1 2 1 $ 2 CF1 1
2 2 4 $ 8 CF2 1
3 2 9 $ 1 8 CF3 1
4 2 16 $ 3 2 CF4 1
5 2 25 $ 5 0 CF5 1
6 102 36 $ 3 , 6 7 2 CF6 1
( )
2
6
1
3,271.595675
t
t
t CF t v
=
=

In t er es t Ra t e per cou pon per iod


1.05 1 2.4695% i = =
Ca lcu la t e NPV 3 , 2 7 1 . 5 9 5 6 7 5
349
http://actuary88.com
We a lr ea dy kn ow t h a t

( )
6
1
556.1144433
t
t
t CF t v
=
=

( )
6
1
97.41125361
t
P CF t v
=
= =

( )
( ) ( )
2
2
6 6
1 1
,
-1
Unit time is half a year
1
(1 ) 2.4695%
1 1 1
4 2
1 5%
t t
t t
i v i
Convexity
p
t CF t v t CF t v
= =
= = +
(
=
(
+

+

( )
( ) ( )
2
1 1 1 1
3,271.595675 556.1144433 10.2048222
97.41125361 4 2
1 5%
(
= =
(
+
+
Alt er n a t ively, con vexit y is :
( )
( )
( )
2 2 2
2
6
1 1
-1
Unit time is half a year
(1 ) , 2.4695%
1 1 1
1 5%
1
4
t
n
t
t t
v i i
v Duration
p p
t CF t v t CF t v duration
= =
= + =
(
(
(
(
( + =
(
+ (
(
(

+


( )
2
1 1 3,271.595675
2.85446713 10.2048222
4 97.41125361
1 5%
( | |
= =
| (
\ . +
+
Th ou gh t h e pr oblem looks complex a n d in t imida t in g, t h e s olu t ion
pr oces s is r ea lly qu ick a n d s imple.

Le t s s ummarize t he c alc ulat ion s t e ps :

If a bon d h a s
Cou pon s C pa ya ble m-t h ly per yea r
Yield t o ma t u r it y = r a n n u a l effect ive
Ter m t o ma t u r it y = n
Fa ce a mou n t = F
St eps t o ca lcu la t e t h e bon d du r a t ion , modified du r a t ion , a n d con vexit y
u s in g BA II Plu s / BA II Plu s Pr ofes s ion a l:
350
http://actuary88.com
St e p 1 Set t h e per cou pon pa ymen t per iod
1
m
a s t h e u n it t ime. Con ver t
t h e a n n u a l effect ive in t er es t r a t e in t o t h e effect ive r a t e per cou pon per iod:

( )
1
1 1
m
i r = +
St e p 2 - Set u p t h e ca s h flow ma s t er t a ble:

t ( Un it
t ime=
1
m
)
( ) CF t ( ) t CF t
2
t ( )
2
t CF t
1 C C 1 C
2 C 2C 4 4C
3 C 3C 9 9C

1 m n C
( ) 1 m n C
( )
2
1 m n ( )
2
1 m n C
m n
F+C
( ) m n F C +
( )
2
m n ( ) ( )
2
m n F C +
St e p 3 - Ma n u a lly en t er t h e ca s h flows in t o Ca s h Flow Wor ks h eet . Set
( )
1
100 100 1 1
m
I i r
(
= = +
(

. Ca lcu la t e t h e followin g 3 it ems :

( ) ( ) ( )
2
1 1 1
, ,
t
m n m n m n
t t
t t t
P CF t v t CF t v t CF t v
= = =
=


wh er e ( )
1
1
m
v r

= +
St e p 4 Ca lcu la t e t h e du r a t ion a n d con vexit y u s in g t h e followin g
for mu la s :

( )
( )
( )
1 1
1
1 1
t
m n m n
t t
t t
MAC m n
t
m m
P
t CF t v t CF t v
D
CF t v
= =
=
= =

( )
( ) ( )
2
2 2
1 1
1 1 1 1
1
t t
m n m n
t t
Convexity
p m m
r
t CF t v t CF t v
= =
(
=
(
+

+


1
1
M O D M AC
Textbook
D D
r
=
+
351
http://actuary88.com
Alt er n a t ive met h od t o ca lcu la t e con vexit y:

( )
( ) ( )
2
2 2
1 1
1 1 1 1 1
1
t t
m n m n
t t
Convexity
p m p m
r
t CF t v t CF t v
= =
(
=
(
+

+


( )
( )
2
2 2
1
1 1 1
1
t
m n
t
p m
r
t CF t v duration
=
(
=
(
+

+

Proble m 2

Bon d fa ce $100
Cou pon 4% s emia n n u a l
Ter m t o ma t u r it y 3 yea r s
Yield t o ma t u r it y 5% a n n u a l effect ive.
Us e BA II Plus Profe s s ional Bon d Wor ks h eet , ca lcu la t e t h e du r a t ion (i.e.
Ma ca u la y du r a t ion ) a n d t h e modified du r a t ion .

Solut ion

Plea s e n ot e t h a t Bon d Wor ks h eet in BA II Plu s ca n NOT dir ect ly ca lcu la t e
t h e modified du r a t ion . BA II Plu s Pr ofes s ion a l does . For t h is r ea s on , you
migh t wa n t t o bu y BA II Plu s Pr ofes s ion al.

BA II Plu s Pr ofes s iona l Bon d Wor ks h eet u s es t h e Wa ll St r eet con ven t ion
in qu ot in g a bon d by u s in g a n omin a l yield t o ma t u r it y. As a r es u lt , wh en
Bon d Wor ks h eet ca lcu la t es t h e pr ice a n d t h e modified du r a t ion , it u s es
t h e n omin a l yield t h a t compou n ds a s fr equ en t ly a s cou pon s a r e pa id.

We n eed t o fin d, y , t h e n omin a l yield compou n din g t wice a yea r
(cou pon s a r e pa id t wice a yea r ):

2
1 1 5% 4.939%
2
y
y
| |
+ = + =
|
\ .

352
http://actuary88.com
Key s t r okes in BA II Plu s Pr ofes s ion a l:
2
n d
Bon d Th is a ct iva t es Bon d Wor ks h eet .
En t er SDT=1.0100

Th is s et s SDT=1-01-2000

SDT = s et t lemen t da t e (i.e.
pu r ch a s e da t e of t h e bon d)

We a r bit r a r ily s et t h e pu r ch a s e
da t e of t h e bon d is t o 1/ 1/ 2000.
1/ 1/ 2000 is a n ea s y n u mber t o
t r a ck.
En t er CPN=4 Set cou pon = 4% of pa r .
En t er RDT=1.0103

Th is s et s RDT=1-01-2003
RDT = r edempt ion da t e (or bon ds
ma t u r it y)

We s et RDT=1-01-2003 (t h e bon d
h a s a 3 yea r ma t u r it y).
En t er RV=100 Redempt ion va lu e. Beca u s e t h e
bon d is r edeemed a t pa r a n d t h e
pa r =100, we s et RV=100.
Da y cou n t in g met h od Us e 360 cou n t in g met h od (i.e.
a s s u me a yea r h a s 360 da ys ). Don t
u s e t h e a ct u a l da y cou n t in g
met h od.
Cou pon fr equ en cy 2/ Y (i.e. t wice a yea r )
YTD=4.939015332 En t er 4.939015332, n ot
4.93901532% (i.e. don t en t er t h e
% s ign ).
CPT PRI (compu t e pr ice of t h e
bon d)
We get PRI=97.41125361. Th is is
t h e bon d pr ice.
AI=0 Accr u ed in t er es t is zer o. Don t
wor r y a bou t t h is fea t u r e. We don t
n eed it t o pa s s Exa m FM.
DUR DUR=2.78567468
Remember t h is is t h e modified
du r a t ion u n der t h e Wa ll St r eet
met h od.
2.78567468
MOD
Wall Street
D =
Next , we will con ver t t h e modified du r a t ion (Wa ll St r eet met h od) in t o
Ma ca u la y du r a t ion .

( )
1
1
MOD MAC
m
Wall Street
D D
YLD
m
=
(
(
(

+
353
http://actuary88.com
( )
1
m
MAC MOD
Wall Street
YLD
D D
m
(
= (
(

+
4.93901532%
2.78567468 1 2.85446713
2
| |
= + =
|
\ .

Fin a lly, well fin d t h e modified du r a t ion du r in g t h e t ext book defin it ion :

1
1 annual effective yield
M O D M AC
Textbook
D D =
+
1 2.85446713
2.71854012
1.05 1.05
M AC
D = = =
Plea s e n ot e t h a t Bon d Wor ks h eet ca n n ot ca lcu la t e t h e con vexit y. To
ca lcu la t e con vexit y, we h a ve t o u s e Ca s h Flow Wor ks h eet or u s e a
for mu la -dr iven a ppr oa ch .

Proble m 3

Bon d fa ce $100
Cou pon 4% s emia n n u a l
Ter m t o ma t u r it y 3 yea r s
Yield t o ma t u r it y 5% a n n u a l effect ive.
Redempt ion $110
Us e BA II Plus Profe s s ional Bon d Wor ks h eet , ca lcu la t e t h e du r a t ion (i.e.
Ma ca u la y du r a t ion ) a n d t h e modified du r a t ion .

Solut ion

Compa r ed wit h Pr oblem 2, Pr oblem 3 h a s a bon d n ot deemed a t pa r .
Wh en ca lcu la t in g t h e du r a t ion of a bon d n ot deemed a t pa r u s in g BA II
Plu s Pr ofes s ion a l Bon d Wor ks h eet , we n eed t o con ver t s u ch a bon d t o a
bon d r edeemed a t pa r . Su ch a s con ver s ion is n eeded beca u s e BA II Plu s
Pr ofes s ion a l Bon d Wor ks h eet ca n NOT ca lcu la t e t h e du r a t ion of a bon d
n ot r edeemed a t pa r .

If we don t con ver t a n on -pa r bon d t o a pa r bon d, BA II Plu s will give a
wr on g r es u lt . Let s ee.

354
http://actuary88.com
Key s t r okes in BA II Plu s Pr ofes s ion a l Bon d Wor ks h eet :
2
n d
Bon d Th is a ct iva t es Bon d Wor ks h eet .
En t er SDT=1.0100
Th is s et s SDT=1-01-2000
We a r bit r a r ily s et t h e pu r ch a s e
da t e of t h e bon d is t o 1/ 1/ 2000.
En t er CPN=4 Set cou pon = 4% of pa r .
En t er RDT=1.0103

Th is s et s RDT=1-01-2003
RDT = r edempt ion da t e

We s et RDT=1-01-2003 (t h e bon d
h a s a 3 yea r ma t u r it y).
En t er RV=110 Redempt ion va lu e.
Da y cou n t in g met h od Us e 360 cou n t in g met h od
Cou pon fr equ en cy 2/ Y (i.e. t wice a yea r )
YLD=4.939015332 Don t en t er 4.93901532%
CPT PRI (compu t e pr ice of t h e
bon d)
We get PRI=106.0496293. Th is is
t h e bon d pr ice. Th is pr ice is
cor r ect .
AI=0 Accr u ed in t er es t is zer o.
DUR DUR=2.78567468
Not ice a n yt h in g s t r a n ge h er e? Even t h ou gh t h e bon d in Pr oblem 3 is
differ en t fr om t h e bon d in Pr oblem 2, BA II Plu s gives u s t h e s a me
modified du r a t ion (Wa ll St r eet ver s ion of modified du r a t ion ). Somet h in g
mu s t be wr on g.

If we u s e t h is modified du r a t ion (Wa ll St r eet ver s ion ), t h en t h e Ma ca u la y
du r a t ion u n der t h e t ext book defin it ion is :

( )
1
1
MOD MAC
m
Wall Street
D D
YLD
m
=
(
(
(

+
( )
1
m
MAC MOD
Wall Street
YLD
D D
m
(
= (
(

+
4.93901532%
2.78567468 1 2.85446713
2
| |
= + =
|
\ .

We kn ow t h is figu r e is wr on g. Next , let s ca lcu la t e t h e bon ds r ea l
du r a t ion u s in g Ca s h Flow Wor ks h eet .

355
http://actuary88.com
Ca s h flow dia gr a m:

Unit time = 0.5 year
Time t 0 1 2 3 4 5 6
Cash flow $2 $2 $2 $2 $2 $2
$110
Th e effect ive in t er es t r a t e per cou pon per iod is 1.05 1 2.4695% i = = .
Well u s e t h e gen er ic pr ocedu r e des cr ibed in Pr oblem 2 t o fin d t h e
du r a t ion . Fir s t , we come u p wit h followin g ca s h flow t a ble (we a r e u s in g 6
mon t h s a on e u n it t ime):

t
( ) CF t ( ) t CF t
1 2 2
2 2 4
3 2 6
4 2 8
5 2 10
6 112 672
Us in g Ca s h Flow Wor ks h eet , we fin d:
( )
6
1
106.04962959
t
t
CF t v
=
=

,
( )
6
1
607.94469918
t
t
t CF t v
=
=

( )
( )
6
1
6
1
1 1 607.94469918
2.866321653
2 2 106.04962959
t
t
t
MAC
t
t CF t v
D
CF t v
=
=
| |
= = =
|
\ .

So t h e cor r ect du r a t ion is 2.866321653. Th e modified du r a t ion is :



1 2.866321653
2.72983
1 annual effective yield 1.05
M O D M AC
Textbook
D D = = =
+
We s ee t h a t BA II Plu s Pr ofes s ion a l ca lcu la t es a bon ds du r a t ion
a s s u min g a bon d is a lwa ys deemed a t pa r , n o ma t t er wh a t r edempt ion
va lu e is . For exa mple, in t h is pr oblem you ca n en t er RV=0 or a n y ot h er
n on -n ega t ive n u mber a n d you ll s t ill get DUR=2.78567468. You ca n ch eck
t h is for you r s elf.

We ca n over come t h is is s u e by con ver t in g a n on -pa r bon d in t o a pa r
bon d. In t h is pr oblem, t h e con ver s ion goes like t h is :
356
http://actuary88.com
original coupon rate par 4% 100
new coupon rate = 3.6366364%
redemption value 110

= =
Th is will give u s t h e r igh t du r a t ion .

Revis ed key s t r okes in BA II Plu s Pr ofes s ion a l Bon d Wor ks h eet :

2
n d
Bon d Th is a ct iva t es Bon d Wor ks h eet .
En t er SDT=1.0100
Th is s et s SDT=1-01-2000
We a r bit r a r ily s et t h e pu r ch a s e
da t e of t h e bon d is t o 1/ 1/ 2000.
En t er CPN=3.63636364 Revis ed cou pon r a t e
En t er RDT=1.0103

Th is s et s RDT=1-01-2003
RDT = r edempt ion da t e

We s et RDT=1-01-2003 (t h e bon d
h a s a 3 yea r ma t u r it y).
En t er RV=110 Redempt ion va lu e.
You ca n even en t er RV=0. Th is
won t a ffect t h e r es u lt .
Da y cou n t in g met h od Us e 360 cou n t in g met h od
Cou pon fr equ en cy 2/ Y (i.e. t wice a yea r )
YLD=4.939015332 Don t en t er 4.93901532%
PRI If you compu t e t h e bon ds pr ice,
you ll get a ga r ba ge n u mber
beca u s e we con ver t t h e or igin a l
bon d. So ign or e PRI.
AI=0 Accr u ed in t er es t is zer o.
DUR DUR=2.79721351 (Th is is Wa ll
St r eet s ver s ion of t h e modified
du r a t ion )
Next , well con ver t t h e Wa ll St r eet s modified du r a t ion in t o Ma ca u la y
du r a t ion :

( )
1
1
MOD MAC
m
Wall Street
D D
YLD
m
=
(
(
(

+
( )
1
m
MAC MOD
Wall Street
YLD
D D
m
(
= (
(

+
4.93901532%
2.79721351 1 2.79721351 1.05 2.86636165
2
| |
= + = =
|
\ .

357
http://actuary88.com
Th is n u mber ma t ch es t h e du r a t ion ca lcu la t ed u s in g Ca s h Flow
Wor ks h eet .

Ke y point t o re me mbe r:

If you ever u s e BA II Plu s Pr ofes s ion a l Bon d Wor ks h eet t o ca lcu la t e a
bon ds du r a t ion , wh et h er t h e bon d is r edeemed a t pa r or n ot , a lwa ys
ca lcu la t e t h e n ew cou pon r a t e
original coupon rate par
new coupon rate =
redemption value

If t h e bon d is r edeemed a t pa r , t h e n ew cou pon r a t e = or igin a l cou pon


r a t e.
En t er t h is n ew cou pon r a t e in t o Bon d Wor ks h eet . Next , con ver t t h e Wa ll
St r eet modified du r a t ion in t o Ma ca u la y du r a t ion .
Proble m 4

Bon d fa ce $100
Cou pon 4% s emia n n u a l
Ter m t o ma t u r it y 3 yea r s
Yield t o ma t u r it y 5% a n n u a l effect ive.
Redempt ion Va lu e $95
Ca lcu la t e t h e bon ds du r a t ion u s in g BA II Plu s Pr ofes s ion a l Bon d
Wor ks h eet .

Solut ion

original coupon rate par 4% 100
new coupon rate = 4.21052632%
redemption value 95

= =
358
http://actuary88.com
2
n d
Bon d Th is a ct iva t es Bon d Wor ks h eet .
En t er SDT=1.0100
Th is s et s SDT=1-01-2000
We a r bit r a r ily s et t h e pu r ch a s e
da t e of t h e bon d is t o 1/ 1/ 2000.
En t er CPN=4.21052632 Revis ed cou pon r a t e
En t er RDT=1.0103

Th is s et s RDT=1-01-2003
RDT = r edempt ion da t e

We s et RDT=1-01-2003 (t h e bon d
h a s a 3 yea r ma t u r it y).
En t er RV= a n y n on -n ega t ive
n u mber (s u ch a s zer o or 0.1)

Da y cou n t in g met h od Us e 360 cou n t in g met h od
Cou pon fr equ en cy 2/ Y (i.e. t wice a yea r )
YLD=4.939015332 Don t en t er 4.93901532%
PRI If you compu t e t h e bon ds pr ice,
you ll get a ga r ba ge n u mber
beca u s e we con ver t t h e or igin a l
bon d. So ign or e PRI.
AI=0 Accr u ed in t er es t is zer o.
DUR DUR=2.77908513 (Th is is Wa ll
St r eet s ver s ion of t h e modified
du r a t ion )
Next , well con ver t t h e Wa ll St r eet s modified du r a t ion in t o Ma ca u la y
du r a t ion :

( )
1
1
MOD MAC
m
Wall Street
D D
YLD
m
=
(
(
(

+
( )
1
m
MAC MOD
Wall Street
YLD
D D
m
(
= (
(

+
4.93901532%
2.77908513 1 2.77908513 1.05 2.84771485
2
| |
= + = =
|
\ .

Proble m 5

Bon d fa ce $100
Cou pon 4% s emia n n u a l
Ter m t o ma t u r it y 3 yea r s
Yield t o ma t u r it y 5% a n n u a l effect ive.
359
http://actuary88.com
Us e t h e for mu la dr iven a ppr oa ch , ca lcu la t e t h e du r a t ion , t h e modified
du r a t ion , a n d con vexit y.

Solut ion

Unit time = 0.5 year
Time t 0 1 2 3 4 5 6
Cash flow $2 $2 $2 $2 $2 $2
$100
Th e effect ive in t er es t r a t e per cou pon per iod is 1.05 1 2.4695% i = = .
( )
( )
6
1
6
1
1
2
t
t
t
MAC
t
t CF t v
D
CF t v
=
=
=

( )
6
6
1
6
100 @ 2.4695% 2
t
t
v i CF t v a
=
+ = =

( ) ( ) ( )
6
6
1
6
2 6 100 @ 2.4695%
t
t
v i t CF t v Ia
=
= + =

( )
( )
( ) ( )
6
6
6
1
6
1
6
6
1
2 6 100
1 2
2 100 2
t
t
t
MAC
t
v
v
t CF t v
D
CF t v
Ia
a
=
=
+
| |
= =
|
+
\ .

1 556.1144433
2.85446713
2 97.41125361
| |
= =
|
\ .

1
1 annual effective yield
M O D M AC
Textbook
D D =
+
1 2.85446713
2.71854012
1.05 1.05
M AC
D = = =
However , con vexit y is n a s t y t o ca lcu la t e.

( )
( ) ( )
2
2
6 6
1 1
1 1 1 1
4 2
1 5%
t t
t t
Convexity
p
t CF t v t CF t v
= =
(
=
(
+

+


360
http://actuary88.com
Th e t r icky pa r t is t o eva lu a t e

( ) ( )
2 2 2 6
6 6
1 1
2 6 100
t t
t t
v t CF t v t v
= =
= +


Th ou gh Kellis on gives u s a for mu la for eva lu a t in g
2
1
n
t
t
t v
=

(h is for mu la
9.23), s u ch a for mu la is u n wieldy a n d n ot wor t h memor izin g s o do NOT
memor ize it .

Beca u s e h er e we h a ve 6 n = (n ot t oo big), we s imply ca lcu la t e
2
6
1
t
t
t v
=

dir ect ly wit h ou t u s in g a n y for mu la s .



( ) ( ) ( ) ( )
1 2 3
2 2 2 2
2 2 2
6
1
1 2 1.05 2 2 1.05 3 2 1.05
t
t
t CF t v

=
| | | | | |
= + +
| | |
\ . \ . \ .

( ) ( ) ( )
4 5 6
2 2 2
2 2 2
4 2 1.05 5 2 1.05 6 102 1.05
| | | | | |
+ + +
| | |
\ . \ . \ .
( )
2
6
1
3,271.595675
t
t
t CF t v
=
=

We a lr ea dy kn ow t h a t

( )
6
1
556.1144433
t
t
t CF t v
=
=

( )
6
1
97.41125361
t
t
P CF t v
=
= =

( )
( ) ( )
2
2
6 6
1 1
1 1 1 1
4 2
1 5%
t t
t t
Convexity
p
t CF t v t CF t v
= =
(
=
(
+

+


( )
( ) ( )
2
1 1 1 1
3,271.595675 556.1144433
97.41125361 4 2
1 5%
(
=
(
+
+
10.2048222 =
361
http://actuary88.com
Alt er n a t ively,

( )
( )
2
2
1
Unit time is one year
1 1
1
n
t
t
Convexity Duration
p
r
t CF t v
=
(
(
(
= +
( +
(


( )
( )
2
2
6
1
Unit time = half a year
1 1
1
1
4
t
t
Duration
p
r
t CF t v
=
(
(
(
= +
( +
(


( )
( )
2
1 1 1
3,271.595675 2.85446713 10.2048222
97.41125361 4
1 5%
(
= =
(
+
+
I t h in k it s u n likely t h a t SOA will a s k you t o ca lcu la t e t h e con vexit y of a
cou pon bon d wit h a lon g ma t u r it y; t h e ca lcu la t ion is t oo in t en s ive.
However , SOA ca n a s k you t o ca lcu la t e t h e con vexit y of a s imple bon d
s u ch a s

a zer o bon d
1-yea r or 2-yea r bon d, wit h cou pon s pa ya ble a n n u a lly or
s emia n n u a lly

Ma ke s u r e you kn ow h ow t o ca lcu la t e.
In a ddit ion , you n eed t o be a ble t o qu ickly ca lcu la t e t h e du r a t ion of a
r egu la r bon d. Du r a t ion is a lwa ys ea s y t o ca lcu la t e.

Proble m 6

Bon d fa ce $100
Cou pon 6% s emia n n u a l
Ter m t o ma t u r it y 4 yea r s
Yield t o ma t u r it y 8% a n n u a l effect ive.
Redempt ion $105
Ca lcu la t e t h e bon ds du r a t ion u s in g t h e followin g met h od:
BA II Plu s / BA II Plu s Pr ofes s ion a l Ca s h Flow Wor ks h eet
BA II Plu s Pr ofes s ion a l Bon d Wor ks h eet
For mu la -dr iven a ppr oa ch

Ill let you s olve t h e pr oblem. Th e cor r ect a n s wer is : 3.61512007

362
http://actuary88.com
Proble m 7

Bon d fa ce $100
Cou pon 6% s emia n n u a l
Ter m t o ma t u r it y 4 yea r s
Yield t o ma t u r it y 8% a n n u a l effect ive.
Redempt ion $92
Ca lcu la t e t h e bon ds du r a t ion u s in g t h e followin g met h od:
BA II Plu s / BA II Plu s Pr ofes s ion a l Ca s h Flow Wor ks h eet
BA II Plu s Pr ofes s ion a l Bon d Wor ks h eet
For mu la -dr iven a ppr oa ch

Ill let you s olve t h e pr oblem. Th e cor r ect a n s wer is : 3.573273749
Proble m 8

Bon d fa ce $100
Cou pon 4% s emia n n u a l
Ter m t o ma t u r it y 3 yea r s
Yield t o ma t u r it y 5% a n n u a l effect ive.
Du r a t ion 2.854
Con vexit y 10.205
Bon d pr ice 97.41
Ca lcu la t e t h e bon d u pda t ed pr ice if t h e yield t o ma t u r it y ch a n ges t o 6%
a n n u a l effect ive.

Solut ion

Well ca lcu la t e t h e n ew pr ice t wice. Fir s t t ime, well u s e t h e du r a t ion a n d
con vexit y. Th e s econ d t ime, well dir ect ly ca lcu la t e t h e pr ice wit h ou t
u s in g t h e du r a t ion a n d con vexit y. Well compa r e t h e t wo r es u lt s .

Fin d t h e n ew pr ice u s in g t h e du r a t ion a n d con vexit y:

( ) ( )( )
2
2 2
2
1
1
1 1 1 1
2 2
MAC
D Convexity
r
P dP d P
r r r r
P P dr P dr
~ =
+
A
A A + A + A
We h a ve:

% 5 r = , 1% r A = , 97.41 P = , 2.854
MAC
D = , 10.205 Convexity =
363
http://actuary88.com
( )( )
2 1
1
1
2
MAC
D Convexity
r
P
r r
P
~
+
A
A + A
( ) ( ) ( )
2 1
2.854 1% 2.667%
1 5%
1
10.205 1%
2
P
P
~ =
+
A
+
( ) 2.667% P P A ~
( ) ( )
'
1 2.667% 97.41 1 2.667% 94.81 P P P P = + A ~ = =
If we ign or e t h e con vexit y, well h a ve:

( )
1 1
2.854 1% 2.718%
1 1 5%
MAC
D
r
P
r
P
~ =
+ +

A
A =
( ) ( )
'
1 2.718% 97.41 1 2.718% 94.76 P P P P = + A ~ = =
Next , we dir ect ly ca lcu la t e t h e n ew pr ice:

' 6
6
100 @ 1 6% 1 2.9563% 2 P v i a + = + = =
' 6
6
100 94.81 2 P v a + = =
We s ee t h a t for a s ma ll ch a n ge of t h e yield, t h e du r a t ion a n d con vexit y
a r e good a t pr edict in g t h e ch a n ge of t h e bon d pr ice.

Proble m 9 (SOA May 2 0 0 3 Cours e 6 #5 s implifie d)

You a r e given t h e followin g wit h r es pect t o a five-yea r bon d:
a n n u a l cou pon s of ( ) 2 % t + a r e pa ya ble a t t h e en d of ea ch yea r
pa r va lu e of $1,000
yield-t o-ma t u r it y ( ) y of 5.5%

Ca lcu la t e t h e Ma ca u la y du r a t ion .

Solut ion

( )
( )
5
1
5
1
t
t
t
MAC
t
t CF t v
D
CF t v
=
=
=

364
http://actuary88.com
Me t hod #1 Us e Cas h Flow Works he e t :

Ca s h flow t a ble:

t
( ) CF t ( ) t CF t
1 (2+1)%(1,000)=30 30
2 (2+2)%(1,000)=40 80
3 (2+3)%(1,000)=50 150
4 (2+4)%(1,000)=60 240
5 (2+5)%(1,000)+1,000=1,070 5,350
Fir s t , well fin d t h e bon ds pr ice. We en t er ( ) CF t in t o Ca s h Flow
Wor ks h eet :

t
( ) CF t
Ca s h flows Ca s h flow
fr equ en cy
1 $ 3 0 CF1 1
2 $ 4 0 CF2 1
3 $ 5 0 CF3 1
4 $ 6 0 CF4 1
5 $ 1 , 0 7 0 CF5 1
( )
5
1
=974.0815620
t
t
CF t v
=


To ca lcu la t e
( )
5
1
t
t
t CF t v
=

, we en t er ( ) t CF t (bold n u mber s below) in t o


Ca s h Flow Wor ks h eet :

t
( ) CF t ( ) t CF t
Ca s h
flows
Ca s h flow
fr equ en cy
1 (2+1)%(1,000)=30 3 0 CF1 1
2 (2+2)%(1,000)=40 8 0 CF2 1
3 (2+3)%(1,000)=50 1 5 0 CF3 1
4 (2+4)%(1,000)=60 2 4 0 CF4 1
5 (2+5)%(1,000)+1,000=1,070 5 , 3 5 0 CF5 1
In t er es t Ra t e per cou pon per iod 5.5% i =
Ca lcu la t e NPV 9 7 4 . 0 8 1 5 6 2 0
In t er es t Ra t e per cou pon per iod 5.5% i =
Ca lcu la t e NPV 4 , 5 1 5 . 2 5 5 0 7 3
365
http://actuary88.com
( )
5
1
4,515.255073
t
t
t CF t v
=
=

( )
( )
5
1
5
1
4,515.255073
4.63539733
974.0815620
t
t
t
MAC
t
t CF t v
D
CF t v
=
=
= = =

Me t hod #2 dire c t ly c alc ulat e



( )
( )
5
1
5
1
t
t
t
MAC
t
t CF t v
D
CF t v
=
=
=

( )
2 3 4 5
5
1
30 40 50 60 1070
=974.0815620
1.055 1.055 1.055 1.055 1.055
t
t
t CF t v
=
= + + + +

( )
( ) ( ) ( ) ( ) ( )
2 3 4 5
5
1
1 30 2 40 3 50 4 60 5 1070
=4,515.255073
1.055 1.055 1.055 1.055 1.055
t
t
CF t v
=
+ + + + =

( )
( )
5
1
5
1
4,515.255073
4.63539733
974.0815620
t
t
t
MAC
t
t CF t v
D
CF t v
=
=
= = =

Plea s e n ot e t h a t for t h is pr oblem we ca n NOT u s e t h e bon d wor ks h eet in


BA II Plu s Pr ofes s iona l t o ca lcu la t e t h e du r a t ion ; we don t h a ve a
s t a n da r d bon d h er e.

Proble m 1 0

Wh a t s t h e du r a t ion of a 5 yea r zer o cou pon bon d?

[A] 4 yea r s
[B] 5 yea r s
[C] 6 yea r s
[D] 7 yea r s
[E] 8 yea r s

366
http://actuary88.com
Solut ion

Th e du r a t ion of a zer o cou pon bon d is a lwa ys equ a l t o it s ma t u r it y. Th e
cor r ect a n s wer is [B]

Proble m 1 1

Wh a t s t h e modified du r a t ion of a 20 yea r zer o cou pon bon d, if t h e yield
t o ma t u r it y is 10% a n n u a l effect ive?

Solut ion

1
1 annual effective yield
M O D M AC
Textbook
D D =
+
1 20
18.18
1.1 1.1
M AC
D = = = (yea r s )

Sin ce t h e cou pon s (wh ich h a ppen t o be zer o) a r e pa id a n n u a lly, t h e
n omin a l yield t o ma t u r it y compou n din g a n n u a lly is t h e s a me a s t h e
a n n u a l effect ive yield. Con s equ en t ly, t h e modified du r a t ion u n der t h e
Wa ll St r eet met h od is t h e s a me a s t h e on e u n der t h e t ext book met h od.

1
1 annual nominal yield
M O D M AC
Wall Street
D D =
+
1 20
18.18
1.1 1.1
M AC
D = = = (yea r s )

Proble m 1 2

8 yea r s a go, Ma r k bou gh t a 15 zer o cou pon bon d. Toda y, wh ile t h e bon d
s t ill h a s 7 yea r s t o ma t u r it y, Ma r k s old it t o J oh n .

X = t h e du r a t ion of Ma r ks bon d.
Y = t h e du r a t ion of J oh n s bon d.

Ca lcu la t e X Y .
Solut ion

Th e du r a t ion of a zer o cou pon bon d is a lwa ys it ma t u r it y. So 15 X = yea r s
a n d 7 Y = yea r s . 8 X Y = .
367
http://actuary88.com
Proble m 1 3

Ca s h flows per pet u a l a n n u it y immedia t e
In t er es t r a t e 5% a n n u a l effect ive
Ca lcu la t e t h e du r a t ion of t h e ca s h flows .

Solut ion

Th e key for mu la is

( )
( )
1
1
1
1
MAC
dp
dp p
r
d r p dr
r
D
(
= = +
(
+

+

1
P
r
= (Th is is t h e pr ice of a per pet u a l a n n u it y immedia t e)

2
1 1 dP d
dr dr r r
| |
= =
|
\ .

( ) ( )
2
1 1 1 1
1 1 1
MAC
dp r
r r r
p dr r r r
D
( ( + | |
= + = + = = +
| ( (
\ .

1 1
1 1 21
5%
MAC
r
D = + = + =
Proble m 1 4

Ca lcu la t e t h e du r a t ion a n d t h e con vexit y of t h e lia bilit y, given :
Th e lia bilit y h a s a con t in u ou s pa ymen t s t r ea m of $5,000 per yea r
over t h e n ext t en yea r
Th e in t er es t r a t e is 6% a n n u a l effect ive

368
http://actuary88.com
Solut ion

For a con t in u ou s pa ymen t s t r ea m,

( )
( )
( )
0 0
0
n n
n
t t
t
t v CF t dt t v CF t dt
Duration
P
v CF t dt
= =
} }
}
( )
( ) ( )
( )
( ) ( )
2
2 2
0 0 0
1 1 1 1
1
1 1
n n n
t t t
Convexity t t CF t dr t CF t dr t CF t dr
p p
r r
v v v
(
= + = +
(
+ +

} } }
If ( ) CF t c = wh er e c is con s t a n t for a n y t , t h en
( ) ( )
0 10
10
0
n
n
n
n
t
t
c t v dt
Ia Ia
Duration
c v dt
a a
= = =
}
}
( )
10
10
10
10 a v
Ia
o

=
( )
10 10
6%
7.36008705 7.57874546
ln1.06
i
a a
o
= = =
In t h e a bove,
10
a is ca lcu la t ed u s in g BA II Plu s / BA II Plu s Pr ofes s ion a l
TVM.

( )
( )
10 10
10
10
7.57874546 10 1.06 10
34.23434140
ln1.06
a v
Ia
o


= = =
( )
10
10
34.23434140
4.51715150
7.57874546
Ia
Duration
a
= = =
Next , well ca lcu la t e t h e con vexit y.

369
http://actuary88.com
( )
( )
( ) ( )
2
2
0 0
0
1 1
1
n n
n
t t
t
Convexity t CF t dr t CF t dr
r
CF t dr
v v
v
(
(
(

= +
+
} }
}
Beca u s e t h e ca s h flow is con s t a n t , we h a ve:

( ) ( )
( )
2 2
0 0 0
2 2
0
1 1
1 1
n n n
n
n
n
t t t
t
t dr t dr t dr Ia
Convexity
r r
dr
v v v
v
a
=
+ +
=
+ +
} } }
}
2 2 2 2 2
0 0 0 0
0
1 1
n
n n n n
t t t t t
t v dt t e dt t de e dt t e
o o o o
o o

(
= = =
(

} } } }

( )
2
0 0
2 2
n
n n
t t
e dt te dt Ia
o o
= =
} }
,
2 2 2
0
n
t n n
t e n e n v
o o
( = =


( )
2
2
0
2
n
n
n
t
Ia n v
t v dt
o

=
}
( )
( )
2
2
2 10
0
10 10
2 10
2 34.23434140 10 1.06
216.7400337
ln1.06
n
t
Ia v
t v dt
o


= = =
}
( )
( )
2
0
2
1
1
n
n
n
t
t dr Ia
Convexity
r
v
a
=
+

+
}
2
29.47272758
1.06
1 216.7400337 34.23434140
7.57874546
= =
+
Alt er n a t ively,

( )
( )
2
2
0
1 1
1
n
t
Convexity t CF t dr Duration
p
r
v
(
= +
(
+

}
2
4.51715150 29.47272758
1.06
1 216.7400337
7.57874546
(
= + =
(


370
http://actuary88.com
Proble m 1 5 (#6 May 2 0 0 5 FM)

J oh n pu r ch a s ed t h r ee bon ds t o for m a por t folio a s follows :

Bon d A h a s s emi-a n n u a l cou pon s a t 4%, a du r a t ion of 21.46 yea r s ,
a n d wa s pu r ch a s ed for 980.

Bon d B is a 15-yea r bon d wit h a du r a t ion of 12.35 yea r s a n d wa s
pu r ch a s ed for 1015.

Bon d C h a s a du r a t ion of 16.67 yea r s a n d wa s pu r ch a s ed for
1000.

Ca lcu la t e t h e du r a t ion of t h e por t folio a t t h e t ime of pu r ch a s e.

(A) 16.62 yea r s
(B) 16.67 yea r s
(C) 16.72 yea r s
(D) 16.77 yea r s
(E) 16.82 yea r s

Solut ion

As a gen er a l r u le, t h e du r a t ion (or con vexit y) of a por t folio is t h e weigh t ed
a ver a ge du r a t ion (or con vexit y) of t h e a s s et s , wit h weigh t bein g t h e
pr es en t va lu e of ea ch a s s et .

k k
1
k
1
PV Duration
porfolio duration =
PV
n
k
n
k
=
=

k k
1
k
1
PV Convexity
porfolio convexity =
PV
n
k
n
k
=
=

Let s pr ove t h is . To ma ke ou r pr oof s imple, let s a s s u me t h a t a por t folio


con s is t s of t wo a s s et s , A a n d B . Th e pr oof is t h e s a me if we h a ve mor e
t h a n t wo a s s et s .

371
http://actuary88.com
Th e du r a t ion of t h e por t folio con s is t in g of t wo a s s et s is :

( )
1
1
d
D i PV
PV di
| |
= +
|
\ .
A B
PV PV PV = +
( ) ( ) ( )
1
1 1
A B
A B
A B A B
d d
PV PV
d
di di
D i PV PV i
PV PV di PV PV
+
= + + = +
+ +

( ) ( )
1 1
1 1
A A B B
A B
A B
d d
i PV PV i PV PV
PV di PV di
PV PV
| | | |
+ + +
| |
\ . \ .
=
+
Bu t ( ) ( )
1 1
1 , 1
A A B B
A B
d d
i PV D i PV D
PV di PV di
+ = + =
A A B B
A B
PV D PV D
D
PV PV
+
=
+
A B
A B
A B A B
PV PV
D D
PV PV PV PV
= +
+ +

Simila r ly, we ca n s h ow t h a t t h e con vexit y of t h e por t folio is t h e weigh t ed
a ver a ge con vexit y wit h weigh t s bein g t h e pr es en t va lu e of ea ch a s s et .

2
2
1 d
C PV
PV di
=
A B
PV PV PV = +
( )
2 2
2
2 2
2
1
A B
A B
A B A B
d d
PV PV
d
di di
C PV PV
PV PV di PV PV
+
= + =
+ +

2 2
2 2
1 1
A A B B
A B
A B
d d
PV PV PV PV
PV di PV di
PV PV
| | | |
+
| |
\ . \ .
=
+
Bu t
2 2
2 2
1 1
,
A A B B
A B
d d
PV C PV C
PV di PV di
= =
372
http://actuary88.com
A A B B
A B
PV C PV C
C
PV PV
+
=
+
A B
A B
A B A B
PV PV
C C
PV PV PV PV
= +
+ +

Come ba ck t o t h e pr oblem.

As s et PV Du r a t ion
#1 980 21.46
#2 1,015 12.35
#3 1,000 16.67
Th e weigh t ed a ver a ge du r a t ion is :

( ) ( ) ( ) 980 21.46 1,015 12.35 1,000 16.67
16.7733
980 1,015 1,000
+ +
=
+ +

So t h e a n s wer is [D]

Proble m #1 6

A por t folio con s is t s of t h e followin g t h r ee a s s et s :

As s et PV Con vexit y
#1 1,000 20
#2 1,500 32
#3 2,000 15
Ca lcu la t e t h e por t folios con vexit y.

Solut ion

Th e weigh t ed a ver a ge con vexit y is :

( ) ( ) ( ) 1, 000 20 1,500 32 2,000 15
21.78
1, 000 1,500 2,000
+ +
=
+ +

So t h e por t folios con vexit y is 21.78.

Proble m #1 7 (SOA May 2 0 0 0 EA-1 #8 )

0.07 o =
373
http://actuary88.com
Th e modified du r a t ion of a 20-yea r bon d wit h 7% a n n u a l cou pon s wit h a
ma t u r it y a n d pa r va lu e of $1,000 is y .
Ca lcu la t e y .
Solut ion

1 e i
o
= + ,
0.07
1 1 7.25% i e e
o
= = = (a n n u a l effect ive in t er es t r a t e)

1
1
MOD MAC
D D
i
=
+
Let s fir s t ca lcu la t e
MAC
D .
time t (year) 0 1 2 3 20
( ) CF t
$70 $70 $70 $70 $1,070
( )
( )
( ) ( )
( )
20
20
20
20
20
1
20
1
70 20 1, 000
70 85.67388 4,931.93928
11.22
70 1, 000 973.9385
i
t
i
t
t
MAC
t
Ia v
a v
t CF t v
D
CF t v
=
=
+
+
= = = =
+

0.07
11.22 10.46
1
1
MOD MAC
e D D
i

= = =
+
Proble m #1 8 (SOA May 2 0 0 1 EA-1 #1 3 )

Pu r ch a s e da t e of a per pet u it y: 1/ 1/ 2001

Da t e of t h e 1
s t
pa ymen t : 12/ 31/ 2001

Fr equ en cy of pa ymen t s : An n u a l

Amou n t of ea ch pa ymen t : $1

In t er es t r a t e: 6% per yea r , compou n ded a n n u a lly

Ca lcu la t e t h e a bs olu t e va lu e of t h e differ en ce bet ween t h e modified
du r a t ion of t h e per pet u it y a n d t h e pr es en t va lu e of t h e per pet u it y.

Solut ion
374
http://actuary88.com
Th e pr es en t va lu e of per pet u it y immedia t e is
1
P
i
=
Th e Ma ca u la y du r a t ion of per pet u it y immedia t e is :

( ) ( ) ( )
2
1 1 1 1
1 1 1
MAC
dP d i
D i i i i i
P di di i i i
+ | | | |
= + = + = + =
| |
\ . \ .

Th e modified du r a t ion is :

1 1 1
1 1
MAC
MOD
D i
D
i i i i
+
= = =
+ +

1 1
0
MOD
P D
i i
= =
375
http://actuary88.com
Chapt e r 1 7 Immunizat ion

The ne e d for immunizat ion
In s u r a n ce compa n ies oft en h a ve t o pa y pr es ch edu led pa ymen t s in
t h e fu t u r e.

For exa mple, s ome in s u r a n ce compa n ies offer CD like in ves t men t
pr odu ct s ca lled GIC (gu a r a n t eed in ves t men t con t r a ct s ). In GIC,
in ves t or depos it s ca s h t o t h e GIC a ccou n t , wh ich ea r n s a
gu a r a n t eed in t er es t r a t e for a s pecific per iod of t ime. For exa mple,
a n in ves t or depos it ed $10,000 t oda y. Th e GIC offer s a gu a r an t eed
in t er es t r a t e of 8% for 5 yea r s . Th en a t t h e en d of Yea r 5, t h e
in s u r a n ce compa n y mu s t pa y t h e in ves t or
$10,000(1+8%)
5
=$14,693.28.
In t h is GIC, t h e in s u r a n ce compa n y pr omis es t o pa y t h e in ves t or
8% a n n u a l effect ive. So t h e in s u r a n ce compa n y n eeds t o ea r n a t
lea s t 8% a n n u a l effect ive t o br ea k even .

How ca n t h e in s u r a n ce compa n y in ves t it s collect ed depos it of
$10,000 wis ely s o it ca n ea r n a t lea s t 8% for 5 yea r s ?

At fir s t gla n ce, we migh t s u gges t t h a t t h e in s u r a n ce compa n y
bu ys , a t t ime zer o, a 5-yea r bon d wit h 8% a n n u a l cou pon a n d
$10,000 fa ce a mou n t . As s u me t h e cu r r en t ma r ket in t er es t r a t e is
8%. Th en t h is bon d cos t s exa ct ly $10,000 a t 0 t = . An d it gen er a t es
exa ct ly $14,693.28 a t 5 t = .
Let s ca lcu la t e t h e t ot a l ca s h flow a t 5 t = if we bou gh t t h e bon d a t
0 t = . As s u me t h e ma r ket in t er es t r a t is 8%. Th en we ca n r ein ves t
ea ch cou pon of $800 a t 8%.

Th e t ot a l ca s h flow a t 5 t = :
8% 5
800 10, 000 14, 693.28 s + =
Th is a mou n t exa ct ly offs et s ou r pa ymen t of $14,693.28.

Time t 0 1 2 3 4 5
Cash flow $800 $800 $800 $800 $10,800
Aft er mor e a n a lys is , h owever , we r ea lize t h a t bu yin g t h is bon d ma y
n ot wor k. Wh a t if t h a t immedia t ely a ft er we bou gh t t h e bon d, t h e
376
http://actuary88.com
in t er es t dr ops t o 7% a n d s t a ys a t 7%? If t h is h a ppen s , ou r t ot a l
a ccu mu la t ed ca s h flow a t 5 t = is :

7% 5
800 10, 000 14, 600.59 14, 693.28 s + = <
Th e a ccu mu la t ed va lu e is lower beca u s e n ow we h a ve t o r ein ves t
t h e cou pon s a t 7% (in s t ea d of 8%).

A gen er a l qu es t ion a r is es . In ma n y occa s ion s , a n in s u r a n ce
compa n y h a s t o pa y it s pr es ch edu led pa ymen t s in t h e fu t u r e.
However , t h e r et u r n s t h e in s u r a n ce compa n y get s fr om it s
in ves t men t a r e vola t ile, depen din g on t h e cu r r en t ma r ket in t er es t
r a t e. If t h e in t er es t ra t es a r e t oo vola t ile, t h e in s u r a n ce compa n y
won t h a ve s u fficien t in ves t men t in come t o pa y it s fixed lia bilit ies .

How ca n t h e in s u r a n ce compa n y in ves t it s collect ed pr emiu ms in
s u ch a wa y t h a t n o ma t t er h ow t h e in t er es t r a t e ch a n ges , it a lwa ys
h a s en ou gh mon ey t o pa y it s pr es ch edu led lia bilit ies in t h e fu t u r e?

Th is led ma n y a ct u a r ies t o s ea r ch for a n in ves t men t s t r a t egy t h a t
will pr odu ce a gu a r a n t eed level of ca s h flows n o ma t t er wh a t
h a ppen s t o t h e in t er es t r a t e. On e s t r a t egy is immu n iza t ion .

Bas ic ide as be hind immunizat ion
Fir s t , we don t wor r y t h a t t h e in t er es t r a t e ma y ch a n ge. At t ime
zer o, we s imply h old a s s et s t h a t exa ct ly offs et lia bilit ies . We ma ke
t h is h a ppen by for cin g PV of As s et s = PV of Lia bilit ies . Th is en s u r es
t h a t we h a ve en ou gh a s s et s t o pa y ou r lia bilit ies if t h e in t er es t r a t e
does n ot ch a n ge.

Next , well con s ider t h e pos s ibilit y t h a t t h e in t er es t r a t e ma y
ch a n ge. We ca n t s t op t h e in t er es t r a t e fr om goin g u p or down , bu t
we ca n ma ke ou r a s s et s a n d lia bilit ies equ a lly s en s it ive t o t h e
ch a n ge of t h e in t er es t r a t e. If a s s et s a n d lia bilit ies ca n in cr ea s e
a n d decr ea s e by r ou gh ly t h e s a me a mou n t for a given ch a n ge in
t h e in t er es t r a t e, t h en t h e ma r ket va lu e of t h e a s s et a n d t h e
ma r ket va lu e of t h e lia bilit y will offs et ea ch ot h er . As a r es u lt , ou r
a s s et s will be en ou gh t o pa y ou r lia bilit ies .

To ma ke ou r a s s et s a n d lia bilit ies equ a lly s en s it ive t o t h e ch a n ge of
t h e in t er es t r a t e, we fir s t for ce ou r a s s et s a n d lia bilit ies t o h a ve a n
equ a l du r a t ion . Du r a t ion is t h e 1
s t
der iva t ive of t h e in t er es t r a t e.
As a r es u lt , we wa n t a s s et s a n d lia bilit ies t o ma t ch du r a t ion .

Next , we for ce ou r a s s et s a n d lia bilit ies h a ve a n equ a l s econ d
der iva t ive (con vexit y) a s t o t h e in t er es t r a t e. To be s a fe, we for ce
377
http://actuary88.com
ou r a s s et s t o h a ve a s ligh t ly bigger s econ d der iva t ive t h a n ou r
lia bilit ies does . Th is en s u r es t h a t for a given ch a n ge in t h e in t er es t
r a t e, ou r a s s et s ch a n ge by a s ligh t ly bigger a mou n t t h a n ou r
lia bilit ies .

We ca n follow t h is lin e of t h in kin g a n d for ce a s s et s a n d lia bilit ies
ma t ch 3
r d
or 4
t h
der iva t ives t o t h e in t er es t r a t e. However , ma t ch in g
a s s et s a n d lia bilit ies in t h eir 3
r d
or 4
t h
der iva t ives is mor e difficu lt
a n d mor e expen s ive t h a n ma t ch in g t h e 1
s t
a n d 2
n d
der iva t ives .

Sample proble ms and s olut ions

Proble m 1
You h a ve a lea ky r oof. Ever y t ime it r a in s , wa t er dr ips down fr om t h e
ceilin g. In t h e pa s t ever y t ime you decided t o h a ve t h e r oof fixed,
s omet h in g ca me u p; you u s ed u p a ll t h e mon ey you h a d a n d h a d n o
mon ey left t o r epa ir t h e r oof. Fin a lly, you h a d en ou gh . You vowed t o
r epa ir t h e lea ky r oof.

Th es e a r e t h e fa ct s :
Accor din g t o t h e wea t h er for eca s t , t h er e will n ot be a n y r a in fa ll in
t h e n ext five yea r s in you r t own . However , t h er e will be a big r a in
5 yea r s a n d 10 da ys fr om t oda y.

You decide t o h a ve t h e r oof fixed 5 yea r s fr om n ow (i.e. a t 5 t = ).

Th e cos t of h a vin g t h e r oof fixed a t 5 t = is $10,000.

You r on ly in ves t men t oppor t u n it y is in you r loca l ba n k, wh ich
offer s on ly t wo pr odu ct s -- s a vin g a ccou n t s a n d CD (cer t ified
depos it ). Bes ides you r loca l ba n k, t h er e a r e n o ot h er in ves t men t
oppor t u n it ies for you .

A s a vin g a ccou n t in you r loca l ba n k ea r n s a n in t er es t r a t e
a dju s t a ble on ce ever y 6 mon t h s . Ever y 6 mon t h s , t h e boa r d of
dir ect or s of you r loca l ba n k s et s a n in t er es t r a t e a ccor din g t o
t h eir wh ims . For exa mple, on J a n u a r y 1 la s t yea r , t h e boa r d of
dir ect or s felt t h e econ omy wa s good a n d decided t h a t a ll s a vin g
a ccou n t s s h ou ld ea r n a 10% a n n u a l effect ive in t er es t r a t e for t h e
n ext 6 mon t h s . However , on J u ly 1, t h e boa r d of dir ect or s felt
t h a t 10% wa s t oo h igh a n d decla r ed t h a t a ll s a vin g a ccou n t s
s h ou ld ea r n a 1% a n n u a l effect ive in t er es t r a t e per yea r for t h e
n ext 6 mon t h s .

378
http://actuary88.com
CD offer ed by you r loca l ba n k h a s a 5 yea r t er m a n d ea r n s a
gu a r a n t eed a n n u a l effect ive in t er es t r a t e of 3%.

Not t o r epea t you r pa s t fa ilu r es , you decide t o immu n ize you r cos t of
r epa ir in g t h e r oof a t 5 t = .
Expla in wh a t immu n iza t ion mea n s in t h is ca s e.
Des ign a n immu n iza t ion s t r a t egy.

Solut ion
Immu n iza t ion in t h is ca s e mea n s t h a t you in ves t mon ey s omewh er e a t
0 t = t o gen er a t e exa ct ly $10,000 a t 5 t = , n o ma t t er h ow t h e in t er es t r a t e
ch a n ges du r in g t h e n ext five yea r s . If you ca n h a ve exa ct ly $10,000 a t
5 t = n o ma t t er h ow low t h e in t er es t ca n be, you h a ve s ecu r ed you r fu t u r e
pa ymen t of $10,000 a t 5 t = .
You h a ve on ly 2 in ves t men t opt ion s in ves t in g in a s a vin g a ccou n t or in
a 5 yea r CD.

In ves t in g in a s a vin g a ccou n t will n ot wor k. Th e in t er es t r a t e you ea r n in
a s a vin g a ccou n t is u n pr edict a ble for t h e n ext 5 yea r s . Un les s you
depos it $10,000 a t 0 t = , t h er es n o wa y t o gu a r a n t ee t h a t you will h a ve
$10,000 a t 5 t = .
You ca n in ves t you r mon ey in t h e 5 yea r CD. Beca u s e t h e 5 yea r CD
offer s a gu a r a n t eed in t er es t r a t e of 3%, you r in it ia l depos it a t 0 t = s h ou ld
be: ( )
5
10, 000 1 3% 8, 626.09

+ =
If you in ves t 8,626.09 in t h e 5 yea r CD, you a r e gu a r a n t eed t o h a ve
$10,000 a t 5 t = . You h a ve immu n ized you r pa ymen t of $10,000 a t 5 t =
a ga in s t a n y a dver s e in t er es t r a t e ch a n ges . In ot h er wor ds , you h a ve
locked in a gu a r a n t eed in t er es t r a t e of 3% n o ma t t er h ow low t h e ma r ket
in t er es t r a t e ca n be.

Comme nt one :
If a s a vin g a ccou n t in you r loca l ba n k offer s a gu a r a n t eed in t er es t r a t e,
t h en you ca n in ves t in a s a vin g a ccou n t a n d immu n ize you r fu t u r e
pa ymen t a t 5 t = . For exa mple, if t h e s a vin g a ccou n t offer s a gu a r a n t eed a
2% a n n u a l effect ive in t er es t r a t e, t h en you r in it ia l depos it a t 0 t = s h ou ld
be: ( )
5
10, 000 1 2% 9, 057.31

+ =
If you in ves t $9,057.31 in a s a vin g a ccou n t t oda y, you a r e 100% cer t a in
t o get $10,000 a t 5 t = . You h a ve immu n ized you r pa ymen t of $10,000 a t
379
http://actuary88.com
5 t = . However , t h is immu n iza t ion s t r a t egy is les s fa vor a ble t h a n if you
in ves t in a 5 yea r CD.

Comme nt Two:
Immu n iza t ion t ypically mea n s in ves t in g mon ey in bon ds , n ot in a s a vin g
a ccou n t or CD, beca u s e t h e in t er es t r a t e gen er a t ed by a s a vin g a ccou n t
or CD is t oo low. In t h is pr oblem, we u s e a s a vin g a ccou n t a n d CD a s
s imple exa mples t o illu s t r a t e t h e es s en ce of immu n iza t ion .

Es s en ce of immu n iza t ion
Wh en t h e in t er es t is vola t ile, we n eed t o fin d wa ys t o lock in a
gu a r a n t eed r et u r n a n d a ccu mu la t e ju s t en ou gh mon ey t o exa ct ly pa y ou r
fu t u r e lia bilit y.
Proble m 2
You h a ve a lea ky r oof. Ever y t ime it r a in s , wa t er dr ips down fr om t h e
ceilin g. In t h e pa s t ever y t ime you decided t o h a ve t h e r oof fixed,
s omet h in g ca me u p; you u s ed u p a ll t h e mon ey you h a d a n d h a d n o
mon ey left t o r epa ir t h e r oof. Fin a lly, you h a d en ou gh . You vowed t o
r epa ir t h e lea ky r oof.

Th es e a r e t h e fa ct s :
Accor din g t o t h e wea t h er for eca s t , t h er e will n ot be a n y r a in fa ll in
t h e n ext five yea r s in you r t own . However , t h er e will be a big r a in
5 yea r s a n d 10 da ys fr om t oda y.

You decide t o h a ve t h e r oof fixed 5 yea r s fr om n ow (i.e. a t 5 t = ).

Th e cos t of h a vin g t h e r oof fixed a t 5 t = is $10,000.

Th e bon d ma r ket offer s a 5 yea r zer o-cou pon bon d wit h $10,000
fa ce a mou n t yieldin g 6% a n n u a l effect ive.

Not t o r epea t you r pa s t fa ilu r es , you decide t o immu n ize you r cos t of
r epa ir in g t h e r oof a t 5 t = . Des ign a n immu n iza t ion s t r a t egy.

Solut ion
You r goa l is t o in ves t mon ey s omewh er e t oda y t o a ccu mu la t e exa ct ly
$10,000 a t 5 t = . If you bu y t h is 5 yea r zer o bon d wit h $10,000 pa r va lu e
t oda y, you a r e gu a r a n t eed t o h a ve $10,000 a t 5 t = .
Th e ma r ket pr ice of t h e bon d is : ( )
5
10, 000 1 6% 7, 472.58

+ =
380
http://actuary88.com
To immu n ize you r fu t u r e pa ymen t of $10,000 a t 5 t = , you s h ou ld bu y
t h is 5 yea r zer o bon d a t $7,472.58 t oda y. By doin g s o, you h a ve locked in
a 6% in t er es t r a t e, n o ma t t er h ow low t h e ma r ket in t er es t ca n be.

Proble m 3
You h a ve a lea ky r oof. Ever y t ime it r a in s , wa t er dr ips down fr om t h e
ceilin g. In t h e pa s t ever y t ime you decided t o h a ve t h e r oof fixed,
s omet h in g ca me u p; you u s ed u p a ll t h e mon ey you h a d a n d h a d n o
mon ey left t o r epa ir t h e r oof. Fin a lly, you h a d en ou gh . You vowed t o
r epa ir t h e lea ky r oof.

Th es e a r e t h e fa ct s :
Accor din g t o t h e wea t h er for eca s t , t h er e will n ot be a n y r a in fa ll in
t h e n ext five yea r s in you r t own . However , t h er e will be a big r a in
5 yea r s a n d 10 da ys fr om t oda y.

You decide t o h a ve t h e r oof fixed 5 yea r s fr om n ow (i.e. a t 5 t = ).

Th e cos t of h a vin g t h e r oof fixed a t 0 t = is $10,000. Du e t o
in fla t ion a n d t h e s h or t a ge of r oof r epa ir s kills , t h e cos t of la bor
a n d ma t er ia ls in cr ea s es by 8% per yea r .

To fu n d you r r epa ir cos t , a t 0 t = you bou gh t a 5 yea r $10,000 pa r
va lu e bon d t h a t offer s a n n u a l cou pon s of 8%. Th e cu r r en t ma r ket
in t er es t r a t e is a ls o 8%.

An a lyze wh et h er you r bon d will gen er a t e s u fficien t fu n d t o pa y t h e r epa ir
cos t 5 t = u n der t h e followin g s cen a r ios :

(1) t h e ma r ket in t er es t r a t e s t a ys a t 8% for ever .
(2) immedia t ely a ft er you h a ve bou gh t t h e bon d, t h e ma r ket in t er es t r a t e
dr ops t o 7.5% a n d s t a ys a t 7.5%.
(3) immedia t ely a ft er you h a ve bou gh t t h e bon d, t h e ma r ket in t er es t r a t e
r is es t o 8.5% a n d s t a ys a t 8.5%.

Solut ion

Sc e nario 1 t he marke t int e re s t rat e s t ays at 8 %.
Let s fir s t ca lcu la t e you r r epa ir cos t a t 5 t = . If you r epa ir you r r oof n ow,
you pa y $10,000. If you r epa ir it a t 5 t = , you ll pa y
( )
5
10, 000 1 8% 14, 693.28 + =
Next , let s s ee h ow mu ch mon ey you r bon d ca n a ccu mu la t e a t 5 t = . Th e
bon d pa ys you 5 a n n u a l cou pon s of $800 ea ch . You ca n r ein ven t t h es e
381
http://actuary88.com
cou pon s a t t h e ma r ket r a t e of 8% a n d a ccu mu la t e t o
5 8%
800s a t 5 t = . In
a ddit ion , you ll get a pa ymen t of $10,000 a t 5 t = . You r t ot a l mon ey a t
5 t = is :
5 8%
800 10, 000 14, 693.28 s + =
You r bon d will a ccu mu la t e ju s t en ou gh mon ey t o pa y you r r epa ir cos t .

Sc e nario 2 imme diat e ly aft e r you bought t he bond, t he marke t
int e re s t rat e drops t o 7 . 5 % and s t ays at 7 . 5 %.
You s t ill get 5 a n n u a l cou pon s ea ch wor t h $800, bu t t h is t ime you ca n
r ein ves t t h em on ly a t 7.5%. You r a ccu mu la t ed va lu e a t 5 t = is :

5 7.5%
800 10, 000 14, 646.71 s + =
You r r epa ir cos t a t 5 t = is s t ill $14,693.28.
You r s h or t fa ll: $14,693.28 - $14,647.71 = $45.57
Wh y t h is t ime don t you h a ve en ou gh mon ey t o pa y you r r epa ir cos t a t
5 t = ? Beca u s e wh en t h e in t er es t r a t e dr ops t o 7.5%, you ca n n o lon ger
r ein ves t you r cou pon s a t 8%. However , t o come u p wit h $14,693.28, you
mu s t be a ble t o r ein ves t you r cou pon s a t lea s t a t 8%.

Sc e nario 3 t he marke t int e re s t rat e ris e s t o 8 . 5 % imme diat e ly
aft e r you bought t he bond and s t ays at 8 . 5 %.
Th is t ime, you ca n r ein ves t you r cou pon s a t 8.5%. You r a ccu mu la t ed
va lu e a t 5 t = is :

5 8.5%
800 10, 000 14, 740.30 s + =
You ll h a ve mor e t h a n en ou gh t o pa y t h e r epa ir cos t a t 5 t = . You r
s u r plu s a t 5 t = is

$14,740.30 $14,693.28 = $47.02

Moral of t his proble m:

Wh en you fir s t bou gh t you r a s s et (5 yea r 8% a n n u a l cou pon wit h pa r
$10,000), a t t h e t h en ma r ket in t er es t r a t e of 8%, you r a s s et will gen er a t e
t h e exa ct a mou n t of mon ey t o pa y you r r epa ir cos t a t 5 t = . However , if
t h e in t er es t r a t e goes u p or down a ft er you bou gh t you r a s s et , you r a s s et
will a ccu mu la t e mor e or les s mon ey t h a n you r pa ymen t a t 5 t = . Wh ile
you a r e h a ppy if you en d u p wit h mor e t h a n you n eed a t 5 t = , you ll be
s a d if you in cu r a los s a t 5 t = .
382
http://actuary88.com
Now ima gin e a ba n k or a n in s u r a n ce compa n y t h a t mu s t pa y
$10,000,000,000 five yea r s fr om n ow. If t h e compa n y does n ot diligen t ly
ma n a ge t h e vola t ilit y of t h e in t er es t r a t e, it ma y los s million s of dolla r s .
Th is is wh y immu n iza t ion is impor t a n t .
Proble m 4
You wa n t t o in ves t you r mon ey n ow t o h a ve ( )
5
10, 000 1 8% 14, 693.28 + = a t
5 t = t o fix you r lea ky r oof. Or igin a lly, you wer e t h in kin g of bu yin g a 5
yea r bon d wit h $10,000 pa r a n d 8% a n n u a l cou pon s . Th en you did s ome
ma t h (like in Pr oblem 3). You r ea lized t h a t if t h e in t er es t r a t e dr ops
immedia t ely a ft er you bou gh t t h e bon d, you won t be a ble t o h a ve
$14,693.28 a t 5 t = .
Th en a n immu n iza t ion wiza r d a dvis es you t o bu y a 6 yea r bon d wit h
$10,000 pa r a n d 8% a n n u a l cou pon s . He s a ys t h a t if you bu y t h is bon d,
even if t h e in t er es t r a t e r is es or dr ops a bit , you will a lwa ys be a ble t o
a ccu mu la t e $14,693.28 a t 5 t = .
Tes t t h e va lidit y of t h e wiza r ds a dvice u n der t h e followin g s cen a r ios :
Immedia t ely a ft er you h a ve bou gh t t h e bon d, t h e ma r ket in t er es t
r a t e dr ops t o 7.5% a n d s t a ys a t 7.5%.

Immedia t ely a ft er you h a ve bou gh t t h e bon d, t h e ma r ket in t er es t
r a t e r is es t o 8.5% a n d s t a ys a t 8.5%.

Expla in you r fin din gs .

Solut ion
Let s ca lcu la t e t h e a ccu mu la t ed va lu e a t 5 t = u n der t h e t wo s cen a r ios :

Sc e nario 1 - Imme diat e ly aft e r you have bought t he bond, t he
marke t int e re s t rat e drops t o 7 . 5 % and s t ays at 7 . 5 %.

You r mon ey a t 5 t = comes fr om t wo s ou r ces . Fir s t , you ca n r ein ves t you r
a n n u a l cou pon s a t 7.5% a n d let t h em a ccu mu la t e t o 5 t = . Secon d, you
ca n s ell you r bon d a t 5 t = . At 5 t = , you r bon d s t ill h a s 1 yea r t er m left
wit h a n in comin g ca s h flow of $10,800 a t 6 t = . You ca n s ell t h is ca s h
flow a t 7.5% in t er es t r a t e.

You r t ot a l mon ey a t 5 t = is :

5 7.5%
10, 800
800 14, 693.22 14, 693.
1.075
s + = ~ 28
383
http://actuary88.com
Sc e nario 2 - Imme diat e ly aft e r you have bought t he bond, t he
marke t int e re s t rat e drops t o 8 . 5 % and s t ays at 8 . 5 %.
You r t ot a l mon ey a t 5 t = is :

5 8.5%
10, 800
800 14, 694.22 14, 693.28
1.085
s + = ~
Let s a n a lyze wh y you ca n a lwa ys a ccu mu la t e ( )
5
10, 000 1 8% 14, 693.28 + = a t
5 t = n o ma t t er t h e in t er es t r a t e r is es or fa lls .

You bought a 6
year bond with
$10,000 par and
8% annual
coupons

The accumulated value
at 5 t = by reinvesting
coupons at the market
interest rate.
The sales price of the
bonds remaining cash flow
(a zero coupon bond with
cash flow of $10,800 one
year from now)
Total

Scenario #1

The market interest
rate stays at 8%
(this should exactly
accumulate the
needed amount)
5 8%
800 4, 693.28 s =
$10,800/1.08=$10,000

We sell the remaining zero
bond at par.
$4,693.28+$10,000
= $14,693.28
Scenario #2

Immediately after
you bought the
bond, the market
interest rate
dropped to 7.5%
and stayed at 7.5%
5 7.5%
800 4, 646.71 s =
Compare Scenario #2
and #1

Decrease
= 4,693.28-4,646.71
= 46.57
$10,800/1.075=$10,046.51

Compare Scenario #2 and #1

Increase:
=$10,0046.51 -$10,000
= $46.51

We sell the remaining zero
bond at a premium of $46.51.
$4,646.71+10,046.51
= $14,693.22 .

Loss due to investing
coupons at a lower
rate is almost exactly
offset by the gain of
discounting the
bonds remaining
cash flow at a lower
rate.
Scenario #3

Immediately after
you bought the
bond, the market
interest rate rose to
8.5% and stayed at
8.5%
5 8.5%
800 4, 740.30 s =
Compare Scenario #3
and #1

Increase
= 4,740.30-4,693.28
= 47.02
$10,800/1.085=$9,953.92

Compare Scenario #3 and #1

Decrease:
=$10,000 - $9,953.92
= $46.08

We sell the remaining zero
bond at a discount of $47.02
$4,740.30+9,953.92
= $14,694.22.

Gain due to
investing coupons at
a higher rate almost
exactly offsets the
loss of discounting
the bonds remaining
cash flow at a higher
interest rate.
384
http://actuary88.com
Moral of t his proble m:

Wh en you bu y a bon d a n d s ell it befor e it s ma t u r it y, you h a ve t wo
s ou r ces of in comes : t h e a ccu mu la t ed va lu e of cou pon s r ein ves t ed a t t h e
ma r ket in t er es t r a t e a n d t h e s a les pr ice of t h e bon ds r ema in in g ca s h
flows dis cou n t ed a t t h e ma r ket in t er es t r a t e. Th es e a r e t wo oppos in g
for ces .
If t h e ma r ket in t er es t r a t e goes down a n d s t a ys low immedia t ely a ft er you
bu y a bon d, you r ein ves t you r cou pon s a t a lower in t er es t r a t e a n d in cu r
a los s . At t h e s a me t ime, h owever , you ca n s ell t h e bon ds r ema in in g
ca s h flows dis cou n t ed a t a lower in t er es t r a t e a n d h a ve a ga in .
If t h e ma r ket in t er es t r a t e goes u p a n d s t a ys h igh immedia t ely a ft er you
bu y a bon d, you ca n r ein ves t you r cou pon s a t a h igh er in t er es t r a t e a n d
h a ve a ga in . At t h e s a me t ime, h owever , you s ell t h e bon ds r ema in in g
ca s h flows dis cou n t ed a t a h igh er in t er es t r a t e a n d in cu r a los s .
If you ca n fin d t h e r igh t bon d a n d h old it for t h e r igh t a mou n t of t ime,
you ca n ma ke t h es e t wo oppos in g for ces exa ct ly offs et ea ch ot h er ,
a ccu mu la t in g a gu a r a n t eed a mou n t of mon ey n o ma t t er t h e in t er es t r a t e
goes u p or down . Th is is h ow immu n iza t ion wor ks .
Th e key is t o fin d t h e r igh t bon d a n d h old it for t h e r igh t per iod of t ime.
How? Th is is ou r n ext pr oblem.
Proble m 5
St a t e t h e 3 r equ ir emen t s of immu n iza t ion . Ver ify t h a t t h e 3 r equ ir emen t s
a r e s a t is fied in t h e followin g immu n iza t ion a r r a n gemen t :

As s et 6 yea r bon d wit h $10,000 pa r a n d 8% a n n u a l
cou pon s
Lia bilit y
( )
5
10, 000 1 8% 14, 693.28 + = a t 5 t = .
In t er es t r a t e 8%
Solut ion

Th e 3 r equ ir emen t s of immu n iza t ion :
PV As s et s = PV Lia bilit ies
Du r a t ion of As s et s = Du r a t ion of Lia bilit ies
Con vexit y of As s et > Con vexit y of Lia bilit ies

385
http://actuary88.com
If t h es e 3 con dit ion s a r e met , t h en t h e a s s et s will gen er a t e s u fficien t ca s h
flows t o pa y t h e fu t u r e kn own lia bilit ies , even t h ou gh t h e in t er es t r a t e
migh t u n ifor mly in cr ea s e or u n ifor mly decr ea s e by a s ma ll a mou n t .

Next , let s ver ify t h a t t h e 3 immu n iza t ion r equ ir emen t s a r e met .

(1) PV As s et s = PV of 6 yea r bon d wit h $10,000 pa r a n d 8% a n n u a l
cou pon s dis cou n t ed a t 8%.

Wit h ou t doin g a n y ca lcu la t ion s (wit h or wit h ou t a ca lcu la t or ), you s h ou ld
immedia t ely kn ow t h a t PV of a 6 yea r bon d wit h $10,000 pa r a n d 8%
a n n u a l cou pon s dis cou n t ed a t 8% is $10,000. As a gen er a l r u le, for a n y
a n n u a l cou pon bon d, if t h e dis cou n t r a t e is equ a l t o t h e a n n u a l cou pon
r a t e, t h e PV of t h e bon d is t h e pa r va lu e.

PV Lia bilit ies = $10,000.

PV As s et s = PV Lia bilit ies

(2) Next , we ch eck wh et h er Du r a t ion of As s et s = Du r a t ion of Lia bilit ies .
Well u s e t h e gen er a l for mu la :

Du r a t ion =
( )
( )
1
1
n
t
n
t
t
t
t v CF t
v CF t
=
=

, wh er e ( ) CF t s t a n ds for a ca s h flow a t t .
As s et
Time t
( ) CF t
1 800
2 800
3 800
4 800
5 10,800
Du r a t ion of As s et s :

( )
( )
( ) ( ) ( ) ( ) ( )
2 3 4 5
2 3 4 5
6
1
6
1
1 800 2 800 3 800 4 800 5 10, 800
800 800 800 800 10, 800
t
t
t
t
t v CF t
v v v v v
v v v v v
v CF t
=
=
+ + + +
=
+ + + +

49,927
4.9927 5
10,000
~ = ~
386
http://actuary88.com
Du r a t ion of lia bilit y (on ly on e ca s h flow of ( )
5
10, 000 1 8% 14, 693.28 + = a t
5 t = :
Lia bilit y:
Time t
( ) CF t
5
( )
5
10, 000 1 8% +
Du r a t ion =
( )
( )
( ) ( ) ( )
( ) ( )
5 5
5 5
5
5
5 10, 000 1 8% 1 8%
5
10, 000 1 8% 1 8%
t
t
t
t
t v CF t
v CF t

=
=
+ +
= =
+ +

Gen er a lly, t h e du r a t ion of a s in gle ca s h flow (s u ch a s a zer o-cou pon


bon d) a t t ime t is t .
Du r a t ion of As s et s = Du r a t ion of Lia bilit ies .

(3) Let ch eck wh et h er
Con vexit y of As s et s > Con vexit y of Lia bilit ies

( )
( )
( )
2
2 2 1
1
1
2
1
n
n
t t
n
t
t
t
t CF t
Convexity v Duration v Duration
p
v CF t
t CF t v
=
=
=
(
(
(
( = + = +
(
(
(

Beca u s e
Liability Asset
Duration Duration = (we a lr ea dy ch ecked t h is ), t o ma ke

Liability Asset
Convexity Convexity > , we ju s t n eed t o
( )
( )
( )
( )
2 2
1 1
1 1
n m
t t
n m
t t
t t
t AssetCF t t LiabiltyCF t
v AssetCF t v LiabilityCF t
= =
= =
>



In t h e a bove equ a t ion , n is t h e t er m t o ma t u r it y of t h e a s s et a n d m is
t h e t er m t o ma t u r it y of t h e lia bilit y.

In ot h er wor ds , if a s s et s a n d lia bilit ies a lr ea dy h a ve a n equ a l du r a t ion ,
t h en by ma kin g

387
http://actuary88.com
( )
( )
( )
( )
2 2
1 1
1 1
n m
t t
n m
t t
t t
t AssetCF t t LiabiltyCF t
v AssetCF t v LiabilityCF t
= =
= =
>



Well s u r ely h a ve

Liability Asset
Convexity Convexity >
( )
( )
2
1
1
n
t
n
t
t
t CF t
v CF t
=
=

is ca lled t h e Ma ca u la y con vexit y.



Let s s u mma r ize t h e a bove dis cu s s ion .

Or igin a l con dit ion s for immu n iza t ion
PV a s s et = PV of lia bilit y
Du r a t ion of a s s et = Du r a t ion of lia bilit y
Con vexit y of a s s et > Con vexit y of lia bilit y
Revis ed con dit ion s for immu n iza t ion
PV a s s et = PV of lia bilit y
Du r a t ion of a s s et = Du r a t ion of lia bilit y
Ma ca u la y con vexit y of a s s et > Ma ca u la y con vexit y of lia bilit y
So in t h e fu t u r e, we will u s e t h e r evis ed con dit ion s t o ch eck wh et h er t h e
immu n iza t ion s t a n ds t r u e; it s ea s ier t o ca lcu la t e t h e Ma ca u la y con vexit y
t h a n t o ca lcu la t e t h e con vexit y. Let s ch eck:

For a s s et s :
( )
( )
2
6
1
6
1
t
t
t
t
Asset
t v CF t
Macaulay Convexity
v CF t
=
=
=

( ) ( ) ( ) ( ) ( )
2 2 2 2 3 2 4 2 5
2 3 4 5
1 800 2 800 3 800 4 800 5 10, 800
800 800 800 800 10,800
v v v v v
v v v v v
+ + + +
=
+ + + +

277,229.8141
27.72
10, 000
~ ~
388
http://actuary88.com
For lia bilit y:
( )
( )
2
2
5 25
t
t
Liability
t v CF t
Macaulay Convexity
v CF t
= = =

(beca u s e lia bilit y h a s on ly on e ca s h flow a t t =5)



Liability Asset
Macaulay Convexity Macaulay Convexity >
Liability Asset
Convexity Convexity >
So Con vexit y of As s et s > Con vexit y of Lia bilit ies .

As a s h or t cu t , t o ch eck wh et h er t h e con vexit y of a s s et s exceeds t h e
con vexit y of lia bilit ies , you oft en don t n eed t o ca lcu la t e t h e con vexit y of
a s s et s a n d t h e con vexit y of lia bilit ies . You ca n s imply dr a w t h e ca s h
flows of a s s et s a n d t h e ca s h flows of lia bilit ies . If you ca n vis u a lly s ee
t h a t t h e ca s h flows of a s s et s a r e mor e s pr ea d ou t t h a n t h e ca s h flows of
lia bilit ies , t h en t h e con vexit y of a s s et s will exceeds t h e con vexit y of
lia bilit ies .

389
http://actuary88.com
Ca s h flow a mou n t

0 1 2 3 4 5 Time t
As s et ca s h flows

Ca s h flow a mou n t

0 1 2 3 4 5 Time t
Lia bilit y ca s h flows

You ca n s ee t h a t a s s et ca s h flows a r e mor e s pr ea d ou t t h a n lia bilit ies
ca s h flows . As a r es u lt , t h e con vexit y of a s s et s is gr ea t er t h a n t h e
con vexit y of t h e lia bilit ies .

Wh y s o? Th e a n s wer lies in ou r r ein t er pr et a t ion of t h e mea n in g of
du r a t ion a n d con vexit y.

390
http://actuary88.com
( )
( )
( )
( )
( )
1
1
1 1
n
n
t
n n
t
t t
t
t
t t
t v CF t
v CF t
Duration t E t
v CF t v CF t
=
=
= =
(
(
( = = =
(
(



( )
( )
( )
( )
( )
2
2 2 1
1
1 1
n
n
t
n n
t
t t
t
t
t t
t v CF t
v CF t
Macaulay Convexity t E t
v CF t v CF t
=
=
= =
(
(
( = = =
(
(



( ) ( ) ( )
2 2
Spread out Varance t E t E t = =
In ot h er wor ds , du r at ion is t h e weigh t ed a ver a ge t ime of t h e ca s h flows ,
wit h weigh t s bein g t h e pr es en t va lu e of ea ch ca s h flow. Ma ca u la y
con vexit y is t h e weigh t ed a ver a ge t ime s qu a r ed of t h e ca s h flows , wit h
weigh t s bein g t h e pr es en t va lu e of ea ch ca s h flow. How ca s h flows s pr ea d
ou t ca n be mea s u r ed by va r ia n ce of t h e t ime of t h e ca s h flows .

Th en we ca n s ee t h a t if t h e du r a t ion of a s s et is equ a l t o t h e du r a t ion of
lia bilit y a n d t h e a s s et ca s h flows a r e mor e s pr ea d ou t (i.e. h a vin g a bigger
va r ia n ce) t h a n t h e lia bilit ies ca s h flows , t h e con vexit y of t h e a s s et is
gr ea t er t h a n t h e con vexit y of t h e lia bilit y.

Ma t h ema t ica lly, t h is is :

If ( ) ( )
Liabililty Asset
E t E t = ,
If
( ) ( )
Liability Asset
Varance t Varance t >
Th en
( ) ( )
2 2 Liabililty Asset
E t E t >
Beca u s e
( ) ( ) ( )
2 2
E t E t Varance t = +
Th en
Liability Asset
Convexity Convexity >
391
http://actuary88.com
Proble m 6
Expla in wh y if t h e followin g 3 con dit ion s a r e met :
PV As s et s = PV Lia bilit ies
Du r a t ion of As s et s = Du r a t ion of Lia bilit ies
Con vexit y of As s et > Con vexit y of Lia bilit ies

t h en even if t h e in t er es t r a t e u n ifor mly in cr ea s es or u n ifor mly decr ea s es
by a s ma ll a mou n t , t h e pr es en t va lu e of t h e a s s et will be gr ea t er t h a n
t h e pr es en t va lu e of t h e lia bilit y (i.e. you h a ve immu n ized a ga in s t a s ma ll
ch a n ge of t h e in t er es t r a t e).

Solut ion

On e ea s y wa y t o u n der s t a n d t h is is t o u s e Ta ylor s er ies . As s u me t h e
cu r r en t in t er es t r a t e is r . Immedia t ely a ft er you pu r ch a s e a n a s s et (s u ch
a s a bon d), t h e in t er es t r a t e ch a n ges by r A . In ot h er wor ds , t h e n ew
in t er es t r a t e is r r + A immedia t ely a ft er you pu r ch a s e t h e a s s et .

( )
2
2
2
1
2
dP d P
P r r
dr dr
A ~ A + A (Ta ylor s er ies )
( ) ( )( )
2
2 2
2
1
1
1 1 1 1
2 2
MAC
D
r
P dP d P
r r r Convexity r
P P dr P dr
~ =
+
A
A + A + A A
( ) ( )( )
2 1
1
1
2
Asset
Asset
Asset Duration
r
P
r Asset Convexity r
P
~
+
+
A
A A
( ) ( )( )
2 1
1
1
2
Liability
Liability
Liability Duration
r
P
r Liability Convexity r
P
~
+
A
A + A
if t h e 3 con dit ion s of immu n iza t ion a r e s a t is fied, we h a ve:

Asset Liab
P P =
As s et Du r a t ion = Lia bilit y Du r a t ion
As s et Con vexit y > Lia bilit y Con vexit y

Liability Asset
Asset Liability
P P
P P
>
A A

Liability Asset
P P A > A
( ) ( )
( )
( ) 1 1 1
t
t
n n
t t
CF t
P r CF t v
r = =
= =
+

392
http://actuary88.com
' ' Liability Liability Asset Asset
Asset Liability
P P P P P P + + = A > = A
So if t h e in t er es t r a t e ch a n ges , t h e PV of t h e a s s et is gr ea t er t h a n t h e PV
of t h e lia bilit y.

Proble m 7

Bes ides s a t is fyin g t h e 3 con dit ion s
PV As s et s =PV Lia bilit ies
Du r a t ion of As s et s = Du r a t ion of Lia bilit ies
Con vexit y of As s et > Con vexit y of Lia bilit ies

immu n iza t ion implicit ly a s s u mes t h a t t h e followin g s t a n da r ds a r e met :

bot h t h e t imin g a n d t h e dolla r a mou n t of ea ch lia bilit y ca s h flows
mu s t be 100% kn own in a dva n ce
t h e in t er es t r a t e ch a n ge mu s t be s ma ll
t h e in t er es t r a t e ch a n ge t a kes pla ce immedia t ely a ft er t h e a s s et is
pu r ch a s ed
t h e in t er es t r a t e ch a n ge mu s t be u n ifor m (i.e. t h e s a me in cr ea s e is
a pplied t o a n y t ime t, r es u lt in g a con s t a n t dis cou n t r a t e
0
i i + A ).

Expla in wh y immu n iza t ion implicit ly a s s u mes s o.

Solut ion

If eit h er t h e t imin g or t h e dolla r a mou n t of a lia bilit y ca s h flow is n ot
100% cer t a in a t t=0, t h en we don t kn ow wh en we n eed t o pa y ou r bill.
Hen ce, we don t h a ve a clea r goa l t o s t a r t immu n iza t ion .

For exa mple, if you don t kn ow wh en you a r e goin g t o fix you r r oof
(t imin g u n kn own ), or if you don t kn ow h ow mu ch it will cos t you t o fix
t h e r oof a t t =5 (a mou n t u n kn own ), you r ea lly don t kn ow h ow mu ch
mon ey you n eed t o in ves t n ow. Immu n iza t ion ca n t be u s ed.

All t h e ot h er implicit a s s u mpt ion s mu s t be met for t h e Ta ylor s er ies t o
s t a n d. For exa mple, if i A is n ot s ma ll (for exa mple, if i A =500%), or if i A is
n ot a con s t a n t , or if i A does n t t a ke pla ce immedia t ely a ft er 0 t = , t h e
Ta ylor s er ies will n ot s t a n d.
393
http://actuary88.com
Chapt e r 1 8 Cas h flow mat ching

Why t o mat c h liabilit y c as h flows ?
An a lt er n a t ive s t r a t egy t o immu n iza t ion

Like immu n iza t ion , ca s h flow ma t ch in g s h ields u s fr om a dver s e
in t er es t r a t e ch a n ges s o we h a ve en ou gh mon ey t o pa y bills .

If a s s et s h a ve exa ct ly t h e s a me ca s h flows a s do t h e lia bilit ies , n o
ma t t er h ow t h e in t er es t r a t e ch a n ges in t h e fu t u r e, we a r e
gu a r a n t eed t o h a ve ju s t en ou gh ca s h t o pa y ou r lia bilit ies .

If a s s et s ca s h flows per fect ly ma t ch lia bilit y ca s h flows , we a r e
100% s a fe a ga in s t a n y in t er es t r a t e ch a n ges . In con t r a s t ,
immu n iza t ion s h ields u s a ga in s t on ly a s ma ll pa r a llel in t er es t r a t e
s h ift .

However , per fect ly ma t ch in g lia bilit y ca s h flows is
(a ) impos s ible -- bon ds exceedin g 30 yea r s ma t u r it y a r e h a r d t o
fin d;
(b) or pr oh ibit ively expen s ive

Mat c hing proc e dure s (working bac kwards , s t art ing from t he final
liabilit y c as h flow t o t he e arlie s t liabilit y c as h flow)
1. Ma t ch t h e fin a l lia bilit y ca s h flow by bu yin g t h e followin g bon d

Th e bon d ma t u r es a t t h e s a me t ime wh en t h e fin a l lia bilit y ca s h
flow is du e.

Th e bon ds fin a l cou pon plu s t h e r edempt ion va lu e is equ a l t o t h e
fin a l lia bilit y ca s h flow.

2. Remove t h e ma t ch in g a s s et s ca s h flows (per iodic cou pon s plu s a
fin a l r edempt ion va lu e) fr om t h e lia bilit y ca s h flows . Th r ow a wa y
ou r fir s t ma t ch in g a s s et . It h a s don e it s s h a r e a n d won t be n eeded
a n y mor e. Now t h e # of a va ila ble a s s et s for u s t o ma t ch t h e
r ema in in g lia bilit y ca s h flows is r edu ced by on e.

3. Aft er t h e 1
s t
ma t ch in g a s s et s ca s h flows a r e r emoved, t h e fin a l
lia bilit y ca s h flow becomes zer o; t h e n ext -t o-fin a l lia bilit y ca s h flow
pops u p a n d becomes t h e fin a l lia bilit y ca s h flow. Th en , we a pply
St ep 1 a n d St ep 2 t o t h is n ew fin a l lia bilit y ca s h flow.

4. Repea t St ep 3 u n t il t h e ea r lies t lia bilit y ca s h flow is ma t ch ed.
394
http://actuary88.com
Sample proble ms and s olut ions
Proble m 1 (Sample FM #5 1 , #5 2 , and #5 3 )
Th e followin g in for ma t ion a pplies t o qu es t ion s 51 t h r ou gh 53.

J oe mu s t pa y lia bilit ies of 1,000 du e 6 mon t h s fr om n ow a n d a n ot h er
1,000 du e on e yea r fr om n ow.

Th er e a r e t wo a va ila ble a s s et s :
A 6-mon t h bon d wit h fa ce a mou n t of 1,000, 8% n omin a l a n n u a l
cou pon r a t e con ver t ible s emia n n u a lly, a n d a 6% n omin a l a n n u a l
yield r a t e con ver t ible s emia n n u a lly;

A 1-yea r bon d wit h fa ce a mou n t of 1,000, a 5% n omin a l a n n u a l
cou pon r a t e con ver t ible s emia n n u a lly, a n d a 7% n omin a l a n n u a l
yield r a t e con ver t ible s emia n n u a lly.

Q #5 1 - How mu ch of ea ch bon d s h ou ld J oe pu r ch a s e in or der t o exa ct ly
(a bs olu t ely) ma t ch t h e lia bilit ies ?

Q #5 2 - Wh a t s J oes t ot a l cos t of pu r ch a s in g t h e bon ds r equ ir ed t o
exa ct ly (a bs olu t ely) ma t ch t h e lia bilit ies ?

Q #5 3 - Wh a t s t h e a n n u a l effect ive yield r a t e for in ves t in g t h e bon ds
r equ ir ed t o exa ct ly (a bs olu t ely) ma t ch t h e lia bilit ies ?

Solut ion
Q #5 1 bu y bon ds t o ma t ch t h e lia bilit y ca s h flows .

As a lwa ys , we dr a w ca s h flow dia gr a ms for ou r lia bilit y a n d t wo bon ds .

Lia bilit y
Time t 0 0.5 1
Cash flow $1,000 $1,000
Bon d #1
Time t 0 0.5 1
Cash flow $40
$1,000
1
6%
1, 040 1 1, 009.7087
2
PV

| |
= + =
|
\ .

( )
2
6% YTM i = =
395
http://actuary88.com
Bon d #2
Time t 0 0.5 1
Cash flow $25 $25
$1,000
2
2
25 1, 000 @3.5% PV v a = +
( )
2
7% YTM i = =
981.0031 PV =
Let s combin e t h e t h r ee dia gr a ms in t o a t a ble:

Sellin g pr ice
t =0
Ca s h flow a t
t =0.5
Ca s h flow a t
t =1
Lia bilit y $1,000 $1,000
Bon d #1 $1,009.7087 $1,040 $0
Bon d #2 $981.0031 $25 $1,025
Pr ocedu r e t o ma t ch lia bilit y ca s h flows :

St ep 1 Ma t ch t h e fin a l lia bilit y ca s h flow.
Th e fin a l lia bilit y ca s h flow is $1,000 a t 1 t = . Beca u s e Bon d #1 does n t
h a ve a n y ca s h flows a t 1 t = , we ca n t u s e it t o ma t ch t h e fin a l ca s h flow.
So we a r e left wit h Bon d #2, wh ich pr odu ces $1,025 a t 1 t = .
As s u me we bu y X u n it s of Bon d #2. To ma t ch t h e fin a l lia bilit y ca s h
flow, we n eed t o h a ve:
( )
1, 000
1, 025 1, 000 0.97561
1, 025
X X = = =
Sellin g pr ice
t =0
Ca s h flow a t
t =0.5
Ca s h flow a t
t =1
Lia bilit y $1,000 $1,000
Bon d #1 $1,009.7087 $1,040 $0
X (Bon d #2)
1, 000
981.0031
1, 025
957.0762
| |
|
\ .
=
1, 000
25
1, 025
24.3902
| |
|
\ .
=
1, 000
1, 025
1, 025
1, 000
| |
|
\ .
=
396
http://actuary88.com
St ep 2 Remove t h e ca s h flows of t h e ma t ch ed a s s et fr om t h e lia bilit y
ca s h flows . Th r ow a wa y Bon d #2. Now we h a ve on ly Bon d #1 left .
Sellin g pr ice
t =0
Ca s h flow a t
t =0.5
Ca s h flow a t
t =1
Lia bilit y X(Bon d 2)
$1,000-24.3902
=975.6098
$1,000-$1,000
=0
Bon d #1 $1,009.7087 $1,040 $0
Now, t h e lia bilit y ca s h flow a t t =0.5 becomes t h e fin a l lia bilit y ca s h flow.

St ep 3 Ma t ch t h e cu r r en t fin a l ca s h flow of $975.6098 a t t =0.5.
As s u me we bu y Y u n it s of Bon d #1. Th en t o ma t ch t h e fin a l lia bilit y ca s h
flow, we n eed t o h a ve:

( ) 1,040 975.6098 0.9381 Y Y = =
Sellin g pr ice
t =0
Ca s h flow a t
t =0.5
Lia bilit y
X(Bon d 2)
-Y(Bon d 1)
$975.6098-975.6098
=$0
Y(Bon d #1)
$1,009.7087(0.9381)
=947.2077
$1,040(0.9381)
=975.6098
Aft er ma t ch in g t h e lia bilit y ca s h flow a t t =0.5, we n eed t o t h r ow a wa y
Bon d #1. Now we h a ve n o a s s et s left . For t u n a t ely, we h a ve n o lia bilit y
ca s h flows left eit h er . All of t h e lia bilit y ca s h flows a r e ma t ch ed.

To ma t ch ou r lia bilit y ca s h flows , we n eed t o bu y 0.9756 X = u n it s of Bon d
#2 a n d 0.9381 Y = u n it s of bon d #1; we a s s u me t h a t we ca n bu y a
fr a ct ion a l bon d.
Next , well fin d t h e cos t of t h e ma t ch in g a s s et s . Th is s h ou ld be ea s y.

( ) ( ) Bond #2 Bond #1 957.0762 947.2077=1,904.2839 X PV Y PV + = +
397
http://actuary88.com
Fin a lly, well ca lcu la t e a n n u a l effect ive yield for in ves t in g X u n it s of Bon d
#2 a n d Y u n it s of Bon d #1. Well dr a w a ca s h flow dia gr a m:

( ) ( ) Matching Asset Bond #2 Bond #1 X Y = +
Time t 0 0.5 1
Matching assets cash flows $1,000 $1,000
of the matching asset 1, 904.2839 PV =
We n eed t o s olve t h e followin g equ a t ion :

2
1, 904.2839 1, 000 3.33269524%
i
i a = =
Next , well con ver t 3.33269524% i = (in t er es t r a t e per 6-mon t h ) in t o a n
a n n u a l effect ive r a t e:

( ) ( )
2 2
1 1 1 3.33269524% 1 6.77645906% i + = + =
Proble m 2 (SOA Cours e 6 , #9 , May 2 0 0 1 )
You a r e given t h e followin g in for ma t ion :

Pr oject ed lia bilit y ca s h flows

Yea r 1 Yea r 2 Yea r 3 Yea r 4 Yea r 5
43 123 214 25 275
Ava ila ble a s s et s for in ves t men t :
2-yea r bon d wit h a n n u a l cou pon of 5%
3-yea r bon d wit h a n n u a l cou pon of 8%
5-yea r bon d wit h a n n u a l cou pon of 10%

Fa ce a mou n t of t h e bon d: 100
Cu r r en t ma r ket yield cu r ve: 7% for a ll du r a t ion s

Ca lcu la t e t h e in it ia l cos t t o ca s h -flow ma t ch t h e pr oject ed lia bilit y ca s h
flows u t ilizin g t h e a s s et s lis t ed a bove.

Solut ion

398
http://actuary88.com
We ca n u s e t h e s t a n da r d ba ckwa r d ma t ch in g met h od t o s olve t h is
pr oblem. We fir s t ma t ch t h e lia bilit y ca s h flow a t Yea r 5. Th en we move
on t o Yea r 4, 3, 2, a n d 1. Th is is t h e met h od u s ed in t h e SOA officia l
s olu t ion . Plea s e down loa d t h e SOA officia l s olu t ion . Ma ke s u r e you ca n
r ecr ea t e t h e s olu t ion .

However , well u s e a qu icker met h od t o s olve t h is pr oblem. Not ice t h a t
t h e t h r ee bon ds h a ve a n iden t ica l yield of 7% a t a ll du r a t ion s . As a
r es u lt , we ca n dis cou n t a ll t h e ca s h flows of ou r con s t r u ct ed ma t ch in g
a s s et (wh ich is a mixt u r e of bon d #1,#2, a n d #3) a t 7%.

Th e con s t r u ct ed ma t ch in g a s s et will h a ve t h e s a me ca s h flows a s does
t h e lia bilit y. So we r ea lly don t n eed t o kn ow h ow t o con s t r u ct t h e
ma t ch in g a s s et . No ma t t er h ow we mix t h e t h r ee bon ds , t h e r es u lt in g
ma t ch in g a s s et will s u r ely h a ve t h e followin g ca s h flows :

Yea r 1 Yea r 2 Yea r 3 Yea r 4 Yea r 5
43 123 214 25 275
As a r es u lt , t h e cos t of t h e ma t ch in g a s s et is s imply t h e a bove ca s h flows
dis cou n t ed a t 7%:

( ) ( ) ( ) ( ) ( )
1 2 3 4 5
43 1.07 123 1.07 214 1.07 25 1.07 275 1.07 537.4512

+ + + + =
We don t n eed t o ma n u a lly ca lcu la t e t h e a bove r es u lt . We ca n s imply
en t er t h e ca s h flows a n d t h e in t er es t r a t e in t o BA II Plu s / BA II Plu s
Pr ofes s ion a l Ca s h Flow Wor ks h eet . Th e ca lcu la t or will gen er a t e t h e r es u lt
for u s .

Plea s e n ot e t h a t t h is met h od wor ks on ly wh en a ll t h e a s s et s h a ve t h e
s a me yield t o ma t u r it y. Ot h er wis e, we h a ve t o u s e t h e s t a n da r d
ba ckwa r d ma t ch in g met h od.

Proble m 3 (FM #1 0 , Nov 2 0 0 5 )

A compa n y mu s t pa y lia bilit ies of 1000 a n d 2000 a t t h e en d of yea r s 1
a n d 2, r es pect ively. Th e on ly in ves t men t s a va ila ble t o t h e compa n y a r e
t h e followin g t wo zer o-cou pon bon ds :

Ma t u r it y
(yea r s )
Effect ive
a n n u a l yield
Pa r
1 10% 1000
2 12% 1000
Det er min e t h e cos t t o t h e compa n y t oda y t o ma t ch it s lia bilit ies exa ct ly.
399
http://actuary88.com
Solut ion

SOA ma de t h is pr oblem ea s y beca u s e t h er e a r en t a n y cou pon s . To
ma t ch t h e lia bilit y, you ju s t n eed t o bu y a 1-yea r zer o bon d wit h fa ce
a mou n t of $1,000 a n d bu y t wo 2-yea r zer o bon ds wit h fa ce a mou n t of
$1,000 ea ch .

You r t ot a l cos t of t h e ma t ch in g a s s et s is :

2
1, 000 2, 000
2, 503
1.1 1.12
+ ~
Ke y point t o re me mbe r

Ca s h flow ma t ch in g pr oblems a r e n ot h a r d, bu t it s ver y ea s y for
ca n dida t es t o ma ke s illy mis t a kes . To elimin a t e er r or s , u s e a s ys t ema t ic
a ppr oa ch .
400
http://actuary88.com
Va lu e of t h is PDF s t u dy ma n u a l

1. Don t pa y t h e s h ippin g fee (ca n cos t $5 t o $10 for U.S. s h ippin g
a n d over $30 for in t er n a t ion a l s h ippin g). Big s a vin g for Ca n a dia n
ca n dida t es a n d ot h er in t er n a t ion a l exa m t a ker s .

2. Don t wa it a week for t h e ma n u a l t o a r r ive. You down loa d t h e
s t u dy ma n u a l in s t a n t ly fr om t h e web a n d begin s t u dyin g r igh t
a wa y.

3. Loa d t h e PDF in you r la pt op. St u dy a s you go. Or if you pr efer a
pr in t ed copy, you ca n pr in t t h e ma n u a l you r s elf.

4. Us e t h e s t u dy ma n u a l a s fla s h ca r ds . Click on bookma r ks t o
ch oos e a ch a pt er a n d qu iz you r s elf.

5. Sea r ch a n y t opic by keywor ds . Fr om t h e Adobe Acr oba t r ea der
t oolba r , click Edit ->Sea r ch or Edit ->Fin d. Th en t ype in a key
wor d.

401
http://actuary88.com


About the author

Yufeng Guo was born in central China. After receiving his Bachelors
degree in physics at Zhengzhou University, he attended Beijing Law
School and received his Masters of law. He was an attorney and law
school lecturer in China before immigrating to the United States. He
received his Masters of accounting at Indiana University. He has
pursued a life actuarial career and passed exams 1, 2, 3, 4, 5, 6, and 7
in rapid succession after discovering a successful study strategy.

Mr. Guos exam records are as follows:
Fall 2002 Passed Course 1
Spring 2003 Passed Courses 2, 3
Fall 2003 Passed Course 4
Spring 2004 Passed Course 6
Fall 2004 Passed Course 5
Spring 2005 Passed Course 7

Mr. Guo currently teaches an online course for Exam P, FM, MLC, and
MFE. For more information, visit http://actuary88.com.

If you have any comments or suggestions, you can contact Mr. Guo at
yufeng.guo.actuary@gmail.com.





















402
Deeper Understanding Exam FM Part II: Derivatives Markets
Yufeng Guo
Derivatives Markets Chapter 0 c Yufeng Guo www.actuary88.com
FM: Derivatives Markets c Yufeng Guo 2
Contents
0 Introduction 5
0.1 Recommended study method . . . . . . . . . . . . . . . . . . . . . . . . . . . 5
0.2 Types of questions to be tested . . . . . . . . . . . . . . . . . . . . . . . . . . 5
0.3 How to use this study manual . . . . . . . . . . . . . . . . . . . . . . . . . . . 6
1 Introduction to derivatives 7
1.1 What is a derivative? . . . . . . . . . . . . . . . . . . . . . . . . . . . . . . . . 7
1.1.1 Denition of derivatives . . . . . . . . . . . . . . . . . . . . . . . . . . 8
1.1.2 Major types of derivatives . . . . . . . . . . . . . . . . . . . . . . . . . 8
1.1.3 Basic vocabulary . . . . . . . . . . . . . . . . . . . . . . . . . . . . . . 9
1.1.4 Uses of Derivatives . . . . . . . . . . . . . . . . . . . . . . . . . . . . . 10
1.1.5 Perspectives on Derivatives . . . . . . . . . . . . . . . . . . . . . . . . 10
1.1.6 Financial Engineering and Security Design . . . . . . . . . . . . . . . . 11
1.2 The role of nancial markets . . . . . . . . . . . . . . . . . . . . . . . . . . . 11
1.2.1 Financial Markets and Averages . . . . . . . . . . . . . . . . . . . . . 11
1.2.2 Risk-sharing . . . . . . . . . . . . . . . . . . . . . . . . . . . . . . . . 11
1.3 Derivatives in practice . . . . . . . . . . . . . . . . . . . . . . . . . . . . . . . 12
1.3.1 Growth in derivatives trading . . . . . . . . . . . . . . . . . . . . . . . 12
1.3.2 How are derivatives used? . . . . . . . . . . . . . . . . . . . . . . . . . 12
1.4 Buying and short-selling nancial assets . . . . . . . . . . . . . . . . . . . . . 12
1.4.1 The lease rate of an asset . . . . . . . . . . . . . . . . . . . . . . . . . 12
1.4.2 Risk and scarcity in short-selling . . . . . . . . . . . . . . . . . . . . . 13
1.5 Chapter summary . . . . . . . . . . . . . . . . . . . . . . . . . . . . . . . . . 13
1.6 Review questions . . . . . . . . . . . . . . . . . . . . . . . . . . . . . . . . . . 13
2 An Introduction to Forwards and Options 15
2.1 Forward contracts . . . . . . . . . . . . . . . . . . . . . . . . . . . . . . . . . 15
2.1.1 Denition . . . . . . . . . . . . . . . . . . . . . . . . . . . . . . . . . . 15
2.1.2 The payo on a forward contract . . . . . . . . . . . . . . . . . . . . . 15
2.1.3 Graphing the payo on a forward contract . . . . . . . . . . . . . . . . 16
2.1.4 Comparing a forward and outright purchase . . . . . . . . . . . . . . . 16
2.1.5 Zero-coupon bonds in payo and prot diagrams . . . . . . . . . . . . 17
2.1.6 Cash settlement vs. delivery . . . . . . . . . . . . . . . . . . . . . . . . 17
2.1.7 Credit risk . . . . . . . . . . . . . . . . . . . . . . . . . . . . . . . . . 17
2.2 Call options . . . . . . . . . . . . . . . . . . . . . . . . . . . . . . . . . . . . . 17
2.2.1 Option terminology . . . . . . . . . . . . . . . . . . . . . . . . . . . . 17
3
Derivatives Markets Chapter 0 c Yufeng Guo www.actuary88.com
2.2.2 Payo and prot for a purchased call option . . . . . . . . . . . . . . . 18
2.2.3 Payo and prot for a written call option . . . . . . . . . . . . . . . . 18
2.3 Put options . . . . . . . . . . . . . . . . . . . . . . . . . . . . . . . . . . . . . 18
2.3.1 Payo and prot for a purchased put option . . . . . . . . . . . . . . . 18
2.3.2 Payo and prot for a written put option . . . . . . . . . . . . . . . . 18
2.4 Summary of forward and option positions . . . . . . . . . . . . . . . . . . . . 18
2.5 Options are insurance . . . . . . . . . . . . . . . . . . . . . . . . . . . . . . . 19
2.6 Example: equity-linked CD . . . . . . . . . . . . . . . . . . . . . . . . . . . . 19
3 Insurance, Collars, and Other Strategies 21
3.1 Basic insurance strategies . . . . . . . . . . . . . . . . . . . . . . . . . . . . . 21
3.1.1 Insuring a long position: oors . . . . . . . . . . . . . . . . . . . . . . 21
3.1.2 Insuring a short position: caps . . . . . . . . . . . . . . . . . . . . . . 22
3.1.3 Selling insurance . . . . . . . . . . . . . . . . . . . . . . . . . . . . . . 22
3.2 Synthetic forwards . . . . . . . . . . . . . . . . . . . . . . . . . . . . . . . . . 23
3.2.1 Put-call parity . . . . . . . . . . . . . . . . . . . . . . . . . . . . . . . 23
3.3 Spreads and collars . . . . . . . . . . . . . . . . . . . . . . . . . . . . . . . . . 24
3.3.1 Bull and bear spreads . . . . . . . . . . . . . . . . . . . . . . . . . . . 24
3.3.2 Box spreads . . . . . . . . . . . . . . . . . . . . . . . . . . . . . . . . . 25
3.3.3 Ratio spreads . . . . . . . . . . . . . . . . . . . . . . . . . . . . . . . . 25
3.3.4 Collars . . . . . . . . . . . . . . . . . . . . . . . . . . . . . . . . . . . . 25
3.4 Speculating on volatility . . . . . . . . . . . . . . . . . . . . . . . . . . . . . . 26
3.4.1 Straddle . . . . . . . . . . . . . . . . . . . . . . . . . . . . . . . . . . . 26
3.4.2 Buttery spread . . . . . . . . . . . . . . . . . . . . . . . . . . . . . . 26
3.4.3 Asymmetric buttery spreads . . . . . . . . . . . . . . . . . . . . . . . 26
3.5 Example: another equity-linked note . . . . . . . . . . . . . . . . . . . . . . . 27
4 Introduction to risk management 29
4.1 Basic risk management: the producers perspective . . . . . . . . . . . . . . . 29
4.1.1 Hedging with a forward contract . . . . . . . . . . . . . . . . . . . . . 29
4.1.2 Insurance: guaranteeing a minimum price with a put option . . . . . . 29
4.1.3 Insuring by selling a call . . . . . . . . . . . . . . . . . . . . . . . . . . 29
4.1.4 Adjusting the amount of insurance . . . . . . . . . . . . . . . . . . . . 30
4.2 Basic risk management: the buyers perspective . . . . . . . . . . . . . . . . . 30
4.2.1 Hedging with a forward contract . . . . . . . . . . . . . . . . . . . . . 30
4.2.2 Insurance: guaranteeing a maximum price with a call option . . . . . 30
4.3 Why do rms manage risk? . . . . . . . . . . . . . . . . . . . . . . . . . . . . 30
4.3.1 An example where hedging adds value . . . . . . . . . . . . . . . . . . 30
4.3.2 Reasons to hedge . . . . . . . . . . . . . . . . . . . . . . . . . . . . . . 31
4.3.3 Reasons not to hedge . . . . . . . . . . . . . . . . . . . . . . . . . . . 31
4.3.4 Empirical evidence on hedging . . . . . . . . . . . . . . . . . . . . . . 32
4.4 Golddiggers revisited . . . . . . . . . . . . . . . . . . . . . . . . . . . . . . . . 32
FM: Derivatives Markets c Yufeng Guo 4
Derivatives Markets Chapter -1 c Yufeng Guo www.actuary88.com
5 Financial forwards and futures 33
5.1 Alternative ways to buy a stock . . . . . . . . . . . . . . . . . . . . . . . . . . 33
5.2 Tailing . . . . . . . . . . . . . . . . . . . . . . . . . . . . . . . . . . . . . . . . 33
5.3 Pricing a forward contract . . . . . . . . . . . . . . . . . . . . . . . . . . . . . 34
5.4 Forward premium . . . . . . . . . . . . . . . . . . . . . . . . . . . . . . . . . . 35
5.5 Creating a synthetic forward contract . . . . . . . . . . . . . . . . . . . . . . 36
5.5.1 Synthetic forwards in market-making and arbitrage . . . . . . . . . . . 37
5.5.2 No-arbitrage bounds with transaction costs . . . . . . . . . . . . . . . 38
5.5.3 Quasi-arbitrage . . . . . . . . . . . . . . . . . . . . . . . . . . . . . . . 39
5.5.4 Does the forward price predict the future spot price? . . . . . . . . . . 40
5.6 Futures contracts . . . . . . . . . . . . . . . . . . . . . . . . . . . . . . . . . . 40
5.6.1 Role of the clearing house . . . . . . . . . . . . . . . . . . . . . . . . . 40
5.6.2 S&P 500 futures contract . . . . . . . . . . . . . . . . . . . . . . . . . 40
5.6.3 Dierence between a forward contract and a futures contract . . . . . 41
5.6.4 Margins and markings to market . . . . . . . . . . . . . . . . . . . . . 41
5.6.5 Comparing futures and forward prices . . . . . . . . . . . . . . . . . . 44
5.6.6 Arbitrage in practice: S&P 500 index arbitrage . . . . . . . . . . . . . 45
5.6.7 Appendix 5.B: Equating forwards and futures . . . . . . . . . . . . . . 46
8 Swaps 53
8.1 An example of a commodity swap . . . . . . . . . . . . . . . . . . . . . . . . 53
8.1.1 Physical versus nancial settlement . . . . . . . . . . . . . . . . . . . . 54
8.1.2 Pricing swaps . . . . . . . . . . . . . . . . . . . . . . . . . . . . . . . . 55
8.2 Interest rate swap . . . . . . . . . . . . . . . . . . . . . . . . . . . . . . . . . 56
8.2.1 Key features of an interest rate swap . . . . . . . . . . . . . . . . . . . 56
8.2.2 Example of a plain vanilla interest rate swap . . . . . . . . . . . . . . 57
8.2.3 Motivations for an interest swap . . . . . . . . . . . . . . . . . . . . . 58
8.2.4 How to price an interest rate swap . . . . . . . . . . . . . . . . . . . . 63
8.2.5 The swap curve . . . . . . . . . . . . . . . . . . . . . . . . . . . . . . . 69
8.2.6 Swaps implicit loan balance . . . . . . . . . . . . . . . . . . . . . . . . 70
8.2.7 Deferred swaps . . . . . . . . . . . . . . . . . . . . . . . . . . . . . . . 70
8.2.8 Why swap interest rates? . . . . . . . . . . . . . . . . . . . . . . . . . 70
8.2.9 Amortizing and accrediting swaps . . . . . . . . . . . . . . . . . . . . 71
FM: Derivatives Markets c Yufeng Guo 5
Derivatives Markets Chapter -1 c Yufeng Guo www.actuary88.com
FM: Derivatives Markets c Yufeng Guo 6
Chapter 0
Introduction
0.1 Recommended study method
The following are my recommendations on how to prepare for Derivatives Markets:
1. Download the FM syllabus from the SOA website. Know precisely what chapters of
Derivatives Markets are the required readings for Exam FM.
2. Buy Derivatives Markets (2nd Edition). Derivatives Markets is expensive, but youd
better buy it. No study guides can thoroughly explain every detail of the textbook. Or
if it does, itll be more expensive than the textbook. Besides, if you buy Derivatives
Markets, you can use it for Exam MF, MFE, and C.
3. Read all of the required readings of Derivatives Markets. Work through all of the
examples in Derivatives Markets.
4. Work through all of the problems in Derivatives Markets at the end of the required
chapter. Use the solution manual to check your answers.
5. Download load the sample problems for Derivatives Markets from the SOA website.
Work through these problems.
0.2 Types of questions to be tested
SOA can write two types of questions on Derivatives Markets:
1. Numerical calculations. For example, SOA can give you some facts about an interest
rate swap and asks you to calculate the xed swap rate R. Many students nd that
solving a numerical question is easier because they can use a formula to calculate the
answer.
2. Essay-type multiple choice questions. This is an example: Which of the following
statements is true about the interest rate swap (or Eurodollars futures contracts)? This
is another example: All the followings are dierences between a forward contract and a
futures contract EXCEPT... Essay-type questions are generally harder than numerical
calculations because you dont have a formula to produce the answer. To answer an
7
Derivatives Markets Chapter 0 c Yufeng Guo www.actuary88.com
essay type multiple choice question correctly, youll probably need to memorize lot of
facts.
When reading Derivatives Markets, be prepared to learn formulas and memorize facts.
Memorizing facts is no fun, but youll probably have to do it to answer essay-type questions.
Many people nd that reading the textbook multiple times and using ash cards help
them memorize facts. You can try these two methods.
0.3 How to use this study manual
This manual is written under the following philosophy: If the textbook explains it well,
Ill skip it or explain it briey; if the textbook doesnt explain it well, Ill try
to explain it well. Its best that you read this manual together with the textbook. This
manual is not a replacement of Derivatives Markets.
FM: Derivatives Markets c Yufeng Guo 8
Chapter 1
Introduction to derivatives
1.1 What is a derivative?
Today, a U.S. company (called Mr. US) signed a purchase contract with a manufacturer in
Britain (called Mr. UK). According to the contract, Mr. US will buy an expensive machine
from Mr. UK. The price of the machine is 1,000,000 pounds. Mr. UK will deliver the machine
to Mr. US in exactly 6 months from today. Upon receiving the machine, Mr. US will pay
Mr. UK exactly 1,000,000 pounds.
Mr. US faces the currency risk. The exchange rate between US dollars and British
pounds uctuates. If 6 months from today the value of British pounds goes up and the value
of American dollars goes down, Mr. US will need to spend more dollars to convert to 1
million British pound, incurring a loss.
For example, the exchange rate today is 1 British Pound=1.898 US Dollars. If Mr. US
pays Mr. UK today, Mr. US will spend $1,898,000 to buy 1 million pounds. If 6 months
from today the exchange rate is 1 British Pound=2 US Dollars, Mr. US will need to spend
$2,000,000 to buy 1 million pounds.
How can Mr. US reduce its currency risk?
One simple approach is for Mr. US to buy 1 million pound today at the price of
1,898,000 US dollars. This eliminates the currency risk, but it requires Mr. US to
spend $1,898,000 today.
Another approach is for Mr. US to pre-order 1 million pounds at a xed exchange rate
from a third party. All Mr. US has to do is to nd a third party willing to sell, in
6 months, 1 million pounds at a xed exchange rate. When the pre-order contract is
signed, no money exchanges hands. Mr. US is not required to take out any capital.
After 6 months has passed, Mr. US will buy 1 millon British pound from the third
party at the pre-agreed exchange rate (say 1 British pound=1.9 US dollars), regardless
of the market exchange rate between pounds and US dollars 6 months from today.
The pre-order contract here is called the forward contract. A forward contract is an
agreement today to transact in the future. Under the currency forward contract, Mr. US
(buyer) agrees to buy a specic amount of goods or services (1 million British pound) from
the seller (the third party) at a pre-agreed price (1 British pound=1.9 US dollars) in a
specic future date (6 months from today). And the seller (the third party) agrees to deliver
9
Derivatives Markets Chapter 1 c Yufeng Guo www.actuary88.com
the specic amount of goods or services (1 million pounds) at a pre-agreed price (1 British
pound=1.9 US dollars) in a specic future date (6 months from today). When the forward
is signed, no money or goods exchanges hands between Mr. US and Mr. UK. When the
specied future date (6 months from today) arrives, Mr. US and Mr. UK each fulll their
promises. Then the forward contract terminates.
The currency forward contract is just one example of nancial derivatives.
1.1.1 Denition of derivatives
Derivatives are nancial contracts designed to create pure price exposure to an underlying
commodity, asset, rate, index, or event. In general they do not involve the exchange or transfer
of principal or title. Rather, their purpose is to capture, in the form of price changes, some
underlying price change or event.
1
You can think of derivatives as children and the underlying assets as parents. Parents
give birth to children; the underlying gives birth to derivatives.
Two key characteristics of a derivative:
1. A derivative is a contract between two parties. One party is a seller and the other
buyer.
2. A derivative is a conditional asset whose value depends on something else (called the
underlying asset or the underlying for short). In the currency forward contract between
Mr. US and Mr. UK, the forward contract by itself doesnt have any value. Its value
depends on the currency exchange rate; the currency exchange rate is the underlying
asset. If, 6 months from today, the British pound becomes more valuable thus more
US dollars are required to buy one British pound, Mr. US has a gain in the currency
forward and Mr. UK has a loss. If on the other hand the British pound becomes less
valuable (thus fewer US dollars are required to buy one British pound), Mr. US has a
loss and Mr. UK has a gain.
1.1.2 Major types of derivatives
There are 2 major types of derivatives:
Option type derivatives. An option is a contract between a buyer (i.e. the owner of
the option) and seller (one who sold the option) that gives the option owner the right,
not the obligation, to buy or sell an asset at a pre-determined price in a specied future
date.
Two major types of options:
Call. A call option gives the option owner the right to buy an asset for a specied
price (the exercise price or strike price) on or before a specied expiration date.
Put. A put option gives the option owner the right to sell an asset for a specied
price (the exercise price or strike price) on or before a specied expiration date.
1
Denition by http://www.financialpolicy.org/dscprimer.htm
FM: Derivatives Markets c Yufeng Guo 10
Derivatives Markets Chapter 1 c Yufeng Guo www.actuary88.com
Forward type derivatives. In forward-type derivatives, buyers and sellers agree to
do business in a specied future date at a pre-determined price. The major use of
forward type derivatives is pricing-locking. Forward type derivatives include:
Forward. A forward contract is a commitment to buy/sell in a future date. The
dierence between a forward and an option is that an option represents the right,
not obligation, to buy or sell in the future (hence the word option) but a forward
contract represents the obligation to buy or sell in the future.
Futures. A futures contract is also a commitment between a buyer and a seller
to transact in a future date at a price agreed upon today. One key dierence
between a forward contract and a futures contract is that a futures contract is a
standardized contract traded over the exchange whereas a forward contract is a
privately negotiated contract traded over the counter.
Swap. In a swap, two parties agree to exchange cash ows in the future. One
common swap is an interest rate swap, where one party pays oating (ie. variable)
interest rate and receives xed interest rate and the other party pays a xed
interest rate and receives a oating interest rate.
2
1.1.3 Basic vocabulary
The following terms are used throughout the textbook. They look odd to newcomers. How-
ever, after a while, these terms will become your second nature.
1. Non-arbitrage. Financial derivatives are generally priced using the non-arbitrage
principle. The non-arbitrage principle means that theres no free lunch. If two things
have identical cash ows, they must sell at the same price. Otherwise, anyone can
become an instant millionaire by buy low, sell high.
2. Long and short. Long means to own or buy something. Short means to sell something.
If you are long in a stock, then you have bought a stock or you own a stock. If you
are short on a stock, then you have sold a stock. Like credits and debits in accounting,
long and short were invented by professionals probably to scare novice.
3. Short selling. If you borrow something from someone else and sell you what you have
borrowed, you are selling short.
4. Spot price, futures price, and forward price. For virtually every commodity,
there are two prices: spot price and futures price (or forward price). Spot price is the
price you pay to get something immediately (spot=immediate delivery). Futures price
or forward price is the price you pay in the future date in order to get something in
the same future date (futures and forward mean future delivery). Most of us pay the
spot price (like shopping in a department store). Only a small portion of people pay
forward prices or futures prices.
5. Spot market and futures (or forward) market. The market associated with the
spot price is the spot market. The market associated with the futures (or forward)
2
For more information, visit http://www.answers.com/topic/interest-rate-swap
FM: Derivatives Markets c Yufeng Guo 11
Derivatives Markets Chapter 1 c Yufeng Guo www.actuary88.com
price is the futures (forward) market. Most of us trade in the spot market. The spot
market is the largest. The forward market is the smallest. The futures market is in
between.
6. In the money, at the money, out of the money. Suppose you own a call or put
option.
If exercising your call or put option brings you some prot, then you are in the
money or the option is in the money.
If exercising your call or put option brings you a loss, then you are out of the
money or the option is out of the money.
If after the option is exercised your wealth stays unchanged, then you are at the
money or your option is at the money.
For example, you own a call option on one IBM stock. The call option allows you to
buy one share of IBM stock for $80.
If IBM stocks sell for $85 per share, then by exercising your call option youll make
$5 prot. Then you are in the money.
If IBM stocks sell for $75 per share, then exercising the call option brings you $5
loss. Then you are out of the money.
If IBM stocks sell for $80 per share, then you gain zero prot if exercising the call
option. You are at the money.
1.1.4 Uses of Derivatives
The textbook lists four major uses of derivatives:
Risk management. For example, companies can use forwards and futures to lock
in xed prices of raw materials and exchange rates and protect themselves against
uctuations of raw materials and foreign currency exchange rates.
Speculation. You can use derivatives to earn some prots (or incur losses).
Reducing transaction costs. For example, by entering an interest rate swap, a rm
can reduce its borrowing cost.
Regulatory arbitrage. For example, you can use derivatives to produce temporary
losses to lower your tax.
These are common sense uses of derivatives and should be pretty easy to remember.
1.1.5 Perspectives on Derivatives
End-user perspective.
Market-maker perspective.
Economic observer
Note: There isnt much meat in this section. Feel free to skip it.
FM: Derivatives Markets c Yufeng Guo 12
Derivatives Markets Chapter 1 c Yufeng Guo www.actuary88.com
1.1.6 Financial Engineering and Security Design
First, lets think about the term nancial engineering. Engineering means to build. Me-
chanical engineering is about building mechanical devices (engines for example). Electrical
engineering is about building electrical devices. Similarly, nancial engineering is about
building nancial devices (ie. nancial products).
Lets look at an example of a new nancial product called index linked CD (certied
deposit). While a conventional CD guarantees a xed interest rate, a market index linked
CD oers a variable interest rate linked to performance of a market index (such as S&P 500).
If the market index goes up, the CD owner earns a higher interest rate. If the market index
goes down, however, the CD owner is guaranteed a minimal interest rate.
However, theres a dierence between nancial engineering and mechanical engineering.
While a new model of cars can be built better than the old model, newly built nancial
products are not necessarily better than an older product. A newly built product might give
an investor a higher returns but it also exposes him to higher risks.
3
Next, the textbook says that nancial engineering has four implications:
1. The fact that new nancial products with desired payos can be built using basic
ingredients (such as CDs, stocks, bonds, calls, puts, forwards, futures, swaps) makes
it feasible for banks and other nancial institutions to produce new nancial products
and hedge the risks in the newly built nancial products.
2. Sellers can build a special product for a special customer.
3. Sellers can rene their process for building new nancial products (since building a new
nancial product is no longer a mysterious process any more).
4. A company can evade tax or circumvent regulations by building a new nancial product
that behaves the same ways as a security thats taxed or regulated (so the company
can say it doesnt directly own the security and thus is not subject to the tax code or
regulations otherwise applicable).
1.2 The role of nancial markets
1.2.1 Financial Markets and Averages
Many nancial risks are split and parceled out to others. Risk sharing is an important
function of nancial markets.
1.2.2 Risk-sharing
Risk is an inevitable; individuals and companies all face risks. Naturally, we want to set up
risk-sharing mechanism where the lucky shares with the unlucky (similar to life insurance
where the healthy help pay the death benets of the unhealthy).
Some risks are diversiable; others are non-diversiable. Risk-sharing benets everyone.
3
For drawbacks of the market index linked CD, visit http://www.usatoday.com/money/perfi/columnist/
block/2006-05-01-cds_x.htm
FM: Derivatives Markets c Yufeng Guo 13
Derivatives Markets Chapter 1 c Yufeng Guo www.actuary88.com
1.3 Derivatives in practice
The use and types of derivatives have grown a lot in the past 30 years.
Derivatives based on government statistics, industrial production, retail sales, and con-
sumer price indexes were invented.
1.3.1 Growth in derivatives trading
When the price of an asset uctuates a lot, derivatives on this asset are often invented to
help manage the price risk.
Millions of contracts are traded annually at the Chicago Board of Trade (CBT), Chicago
Mercantile Exchange (CME), and the New York Mercantile Exchange (NYMEX), the three
largest exchanges in the U.S..
The use of futures contracts has grown signicantly over the last 30 years.
1.3.2 How are derivatives used?
Little is known about how companies use derivatives to manage risks.
1.4 Buying and short-selling nancial assets
When calculating the cost of buying an asset, remember commission and the bid-ask spread.
How to memorize bid-ask spread
Bid price=dealers Buying price=what you get if you sell your security to a dealer
aSk price=dealers Selling price=what you pay if you buy a security from a dealer
Short Selling. Short selling XYZ stocks means that you rst borrow XYZ stocks and then
selling them.
3 reasons to short sell:
To speculate.
To borrow money.
To hedge a risk.
1.4.1 The lease rate of an asset
If you borrow an asset from the asset owner, you may have to pay a fee to the owner. This
fee is called the least rate of an asset.
If you short sell a stock and the stock pays dividend before you return the stock to the
lender, you need to pay the dividend to the lender.
FM: Derivatives Markets c Yufeng Guo 14
Derivatives Markets Chapter 1 c Yufeng Guo www.actuary88.com
1.4.2 Risk and scarcity in short-selling
Credit risk is the risk that the short-seller wont return the borrowed asset. To address
the credit risk, the lender keeps as collateral the proceeds generated from the short sale.
Since the lender keeps the proceeds from the short sale, the lender should give the
proceeds back to the short seller after the short seller returns the borrowed asset. In
addition, the lender should pay the short seller interest for temporarily holding on the
proceeds. The amount of interest on collateral depends on supply and demand. If lot
of people want to short sell an item, the lender may pay a small interest. The interest
rate paid on collateral is called the repo rate in bond market and the short rebate in the
stock market. The dierence between this rate and the market interest rate is another
cost of borrowing.
1.5 Chapter summary
Derivatives are conditional assets whose values depend on something else. The three major
derivatives are forwards (or futures), options, and swaps.
1.6 Review questions
Problem 1
You are the owner of an airline company. Fuel cost is a signicant portion of your annual
operating expense. You heard the news that the rising fuel cost brought several airlines to
bankruptcy. Outline ways you can manage you fuel cost using nancial derivatives.
Solution
One way to lock in fuel cost is for you to enter an energy swap with a fuel supplier. In
this swap, you pay xed cost each year to the supplier over a number of years. In return, the
supplier gives you certain amount of fuel each year while the swap is eective. Now your fuel
cost is xed during the duration of the energy swap. A swap generally doesnt require any
initial cash outlay.
You can also buy a certain number of call options on fuel price. If fuel price goes up, then
you can exercise your call option and buy fuel at the predetermined strike price. However,
unlike an energy swap, buying call options requires you to pay premiums upfront.
You can also enter a futures or forward contract to order certain amount of fuel to be
delivered to you at specied future dates with prices determined today.
Problem 2
Why is it reasonable for us to assume that nancial markets are free of arbitrage?
Solution
FM: Derivatives Markets c Yufeng Guo 15
Derivatives Markets Chapter 1 c Yufeng Guo www.actuary88.com
There are well-informed buyers and sellers in the market. If there are arbitrage opportu-
nities, intelligent buyers and sellers will rush to the opportunities and soon the opportunities
will vanish.

FM: Derivatives Markets c Yufeng Guo 16


Chapter 2
An Introduction to Forwards and
Options
2.1 Forward contracts
2.1.1 Denition
Buyer of a forward contract
Has an obligation to buy an asset (the underlying)
At a specic future date (maturity/expiration date)
In an amount (contract size)
At a priced agreed upon today (the forward price)
Seller of a forward contract
Has an obligation to deliver an asset (the underlying)
At a specic future date (maturity/expiration date)
In an amount (contract size)
At a priced agreed upon today (the forward price)
A prepaid forward contract is a forward contract except that the payment is made at t = 0
when the contract is signed.
2.1.2 The payo on a forward contract
If you buy something, you have a long position (long=having more of something).
If you sell something, you have a short position (short=having less of something).
In a forward contract, the buyers payo=AB, where
17
Derivatives Markets Chapter 2 c Yufeng Guo www.actuary88.com
A=what the buyer has to pay to get the asset at T assuming he doesnt have a forward
contract
B= what the buyer has to pay to get the asset at T if he has a forward contract
Clearly, A=the spot price at T (without a forward contract, the buyer has to buy the
asset from the market at T); B=the forward price at T.
So the buyers payo at T=Spot price at T - forward price at T
Since the buyer of a forward contract has a long position in the forward contract, so
payo to long forward at T=Spot price at T - forward price at T
The sellers payo=C-D
1. C=What the seller has to sell at T because he has a forward contract
2. D=What the seller can sell at T if he doesnt have a forward contract
Clearly, C=forward price at T; D=Spot price at T
Since the seller in a forward contract has a short position, payo to short forward at
T=Forward price at T - Spot price at T
payo to long forward + payo to short forward=0
A forward is a zero sum game. If one party gains, the other party must lose.
2.1.3 Graphing the payo on a forward contract
If you transfer the data from Table 2.1 to a graph (putting S&R Index in 6 months as X
and Payo as Y), youll get Figure 2.2. Dont make it overly-complex. Just draw dots and
connect the dots.
To draw the payo diagram for the long forward position, draw the following points:
(900, 120), (950, 70), (1000, 20), (1020, 0), (1050, 30), (1100, 80). Then connect these dots.
To draw the payo diagram for the short forward position, draw the following points:
(900, 120), (950, 70), (1000, 20), (1020, 0), (1050, 30), (1100, 80). Then connect these dots.
2.1.4 Comparing a forward and outright purchase
Whats the dierence between buying an asset at t = 0 as opposed to entering into a forward
contract and getting the same thing at T?
If you buy an asset outright at t = 0:
Whats good: You get what you need right away. You dont need to enter into a forward
contract. It takes time to nd a seller who wants to sign a forward contract with you.
You may have to hire an attorney to review the complex clauses of a forward contract.
Finally, buying an asset outright eliminates the risk that the seller might not deliver
the asset at T.
FM: Derivatives Markets c Yufeng Guo 18
Derivatives Markets Chapter 2 c Yufeng Guo www.actuary88.com
Whats bad: Outright purchase ties up your capital. In addition, if you dont need the
asset right away, you have to store the asset. Storing the asset may cost you money.
If you get the asset at T through a forward contract
Whats good: Just in time delivery saves you storage cost. In addition, you dont have
to pay anything at t = 0.
Whats bad: You need to take the time to nd a willing seller who has what you need
and is willing to enter a forward contract with you. In addition, the seller in a forward
contract may break his promise (credit risk).
Make sure you can reproduce Figure 2.3.
Make sure you memorize the conclusion of this section. Since it costs zero to store an
index (index is not a physical asset such as wheat or corn), assuming theres zero credit risk
(assuming both parties in a forward contract keep their promises), then a forward contract
and a cash index are equivalent investments, diering only in the timing of the cash ows.
Neither form of investing has an advantage over the other.
Make sure you understand the dierence between payo and prot; between a payo
diagram and a prot diagram.
2.1.5 Zero-coupon bonds in payo and prot diagrams
The textbook says that buying the physical index is like entering into the forward contract
and simultaneously investing $1000 in a zero-coupon bond. Please note that investing
$1000 in a zero-coupon bond simply means that you put $1000 in a savings account. This
way, at the expiration date T, your initial $1000 will grow into 1000 1.02 = 1200, which is
exactly what you need to pay the seller to buy the index.
2.1.6 Cash settlement vs. delivery
The idea is simple. If you owe me an orange (worth $2) and I owe you an apple (worth $5),
one way to settle out debts is that you give me an orange and I give you an apple. This is
called the physical delivery.
A simpler approach is that I give you $3. This is called the cash settlement.
2.1.7 Credit risk
This is the risk that one party in a contract breaks his promise and doesnt do what he is
supposed to do according to the contract.
2.2 Call options
2.2.1 Option terminology
Make sure you understand the following terms: strike price, exercise, expiration, exercise
style (American and European). The textbook explains well. Just follow the textbook.
FM: Derivatives Markets c Yufeng Guo 19
Derivatives Markets Chapter 2 c Yufeng Guo www.actuary88.com
2.2.2 Payo and prot for a purchased call option
Just memorize the key formulas:
Purchased call payo=max(0, S
T
K)
Purchased call prot=Purchased call payo - FV of Option Premium
Purchased call prot=max(0, S
T
K)- FV of Option Premium
where S
T
is the stock price at expiration date T and K is the strike price.
2.2.3 Payo and prot for a written call option
Written call payo + Purchased call payo =0 (zero sum game)
Written call payo =max(0, S
T
K)
Written call prot + Purchased call prot =0 (zero sum game)
Written call prot =FV of Option Premium max(0, S
T
K)
2.3 Put options
2.3.1 Payo and prot for a purchased put option
Purchased put payo=max(0, K S
T
)
Purchased put prot=Purchased put payo - FV of Option Premium
Purchased put prot=max(0, K S
T
) - FV of Option Premium
2.3.2 Payo and prot for a written put option
Written put payo + Purchased put payo =0 (zero sum game)
Written put payo = max(0, K S
T
)
Written put prot + Purchased put prot =0 (zero sum game)
Written put prot = FV of Option Premium max(0, K S
T
)
2.4 Summary of forward and option positions
Make sure you understand Table 2.4. Make sure you are comfortable with the following
terms:
1. Long Positions
(a) Long forward.
(b) Purchased call.
(c) Written call.
FM: Derivatives Markets c Yufeng Guo 20
Derivatives Markets Chapter 2 c Yufeng Guo www.actuary88.com
2. (a) Short forward.
(b) Written call.
(c) Purchased call.
2.5 Options are insurance
This is the basic idea. First, options are insurance. This shouldnt surprise you. A call
option is an insurance policy against the risk that the price of a stock may go up in the
future. Similarly, a put option is an insurance policy against the risk that the price of a stock
may go down in the future.
Next, a homeowner insurance policy is a put option. This is a simple idea too. If you
have a put option on a stock, you are guaranteed to sell your stock at a xed price no matter
how low the stock price has become. Similarly, if you have a homeowner insurance policy,
no matter how low your house has become, you are guaranteed to sell your house to the
insurance company at a xed price (i.e. the insurance company will give you a xed amount
of money regardless of how low your house has become).
Make sure you can recreate Figure 2.13. To recreate Figure 2.13, just draw a few critical
points and then connect these points. There are 2 critical points:
(House price, Prot)=(0, 160, 000), (200, 000, 15, 000).
This is how to get the rst critical point (0, 160, 000). If the house is blown away by
big wind, the house price is zero. Then the insurer pays the full replacement value of the
house $200,000 subject to a deductible of $25,000. So the insurance company will pay you
200, 000 25, 000 = 175, 000. However, you pay premium $15,000 at t = 0. If we ignore the
lost interest of your premium (had you put your premium in a savings account, you would
have earned some interest), then your prot is 175, 000 15, 000 = 160, 000.
This is how to get the second critical point (200, 000, 15, 000). If after you bought the
insurance policy, nothing bad happens to your house, then the payo of your insurance is
zero. You lost your premium $15,000; you pay $15,000 for nothing. Your loss is $15,000.
This gives us the critical point (200, 000, 15, 000).
Your can verify for yourself that if nothing bad happens to your house, even if the value
of your house goes up, youll always lose $15,000 from the insurance policy. Now you should
get Figure 2.13.
2.6 Example: equity-linked CD
SOA can easily test Formula (2.11). Memorize it.
V
T
= V
0

1 +k max

0,
S
T
S
0
1

V
T
is the CD value at time T
V
0
is the CD value at time 0
S
T
is the index value at time T
S
0
is the index value at time 0
FM: Derivatives Markets c Yufeng Guo 21
Derivatives Markets Chapter 2 c Yufeng Guo www.actuary88.com
k is the participation rate
Make sure you can recreate Figure 2.14 and Table 2.5.

FM: Derivatives Markets c Yufeng Guo 22


Chapter 3
Insurance, Collars, and Other
Strategies
Unfortunately, this is a messy chapter with lot of new phrases such as bull spread and collars.
Just follow the text and understand these new phrases.
The most important topic in this chapter is the put-call parity. Most likely SOA will test
this topic. Make sure you understand the put-call parity.
Ill highlight the key points in this chapter.
3.1 Basic insurance strategies
3.1.1 Insuring a long position: oors
A put option gives you a price oor. If you have a put option on an asset, then you dont
need to worry that the asset price drops below the strike price. If indeed the assets price
drops below the strike price, you can exercise the put and sell the asset at the strike price.
Make sure you can recreate Table 3.1 and Figure 3.1
The 6-month interest rate is 2%. A put with 1000-strike price with 6-month to expiration
sells for $74.201. A call with 1000-strike price with 6-month to expiration sells for $93.809.
Payo at expiration T of Put + Stock is max(S
T
, K). If you have a put and a stock, then
the value of Put+ Stock at the expiration date T is the greater of the strike price K and the
stock price S
T
. If, at T, S
T
< K, then you can exercise your put and sell your stock for K.
If S
T
K, then you just let your put expire worthless and your payo is S
T
.
So the payo of Put + Stock at expiration=max(S
T
, K).
When recreating Table 3.1, please note that Prot = Payo - (Cost + Interest)
To recreate Figure 3.1 (a), (b), (c), and (d), just plug in the data from Table 3.1. For
example, to recreate Figure 3.1(d), enter the following points of (S&R index, Prot):
(900, 95.68), (950, 95.68), (1000, 95.68), (1050, 45.68), (1100, 4.32), (1150, 54.32), and
(1200, 104.32)
Connect these points and you should get Figure 3.1(d).
Make sure you can recreate Figure 3.2. Figure 3.2 is continuation of Figure 2.13. This is how
to draw the prot diagram for the uninsured house. If the house is washed away by ood (ie.
the price of the house becomes zero), then you loose $200,000. If the house appreciates and
23
Derivatives Markets Chapter 3 c Yufeng Guo www.actuary88.com
the new price is $250,000, then you gain 250, 000 200, 000 = 50, 000. So you have two data
points:
(Price of the house, Prot)=(0, 200, 000), (250, 000, 50, 000).
Connecting these two points, youll get the prot line for the uninsured house.
This is how to draw the diagram for the insured house. The prot is 40, 000 when the
house price is from 0 to 175,000. If the damage to your house is equal to or greater than the
deductible $25,000 (ie. the house price is 200, 000 25, 000 = 175, 000 or less), youll lose
both the deductible and the premium. So your total loss is 25, 000 + 15, 000 = 40, 000. This
is a straight line from (0, 40, 000) to (175, 000, 40, 000).
If after you bought the insurance, nothing bad happens to your house and the price of
your house goes up to say $250,000, then you gain $50,000 in the value of your house but
you lose your premium. So your prot is 50, 000 15, 000 = 35, 000. This gives us the point
(250, 000, 35, 000).
If you connect (175, 000, 40, 000), (200, 000, 15, 000), and (250, 000, 35, 000), youll get
the prot diagram when the house price is 175,000 or greater.
The conclusion drawn from Figure 3.2 is that an insured house has a prot diagram that
looks like a call option.
3.1.2 Insuring a short position: caps
A call option gives you a price ceiling (or cap). If you have a call option on an asset, then
you dont need to worry that the asset price may go up. If indeed the price of the asset goes
above the strike price, you can exercise the call and buy the asset at the strike price.
Make sure you can reproduce Table 3.2 and Figure 3.3.
3.1.3 Selling insurance
If you sell a call on a stock, youd better have a stock on hand. This way, if the stock price
goes up and the call is exercised, you already have a stock to deliver to the owner of the call.
If you buy a stock and simultaneously sell a call option, you are selling a covered call. A
covered call limits your loss in case the stock price goes up (because you already have a stock
on hand).
If you sell a call option but dont have a stock on hand, you are selling a naked call.
Writing a naked call is a big risk if the stock price goes up, in which case the call writer has
to buy a stock from the market.
Covered put. If you sell a put, you have two ways to cover it. One is to short sell a
stock: when you sell a put, you simultaneously short sell a stock. If the stock happens to be
worthless and the buyer of the put sells the worthless stock to you at the guaranteed price
(i.e. strike price), you turn around and give the worthless stock to the broker who lent you
a stock for short selling.
Another way to cover your put is to set aside the present value of the strike price at t = 0.
If the owner of the put exercises the put, you already have the strike price in your pocket to
pay the owner of the put.
If you dont cover your put and the stock happens to be worthless, then you may suddenly
nd that you dont have the money to pay the strike price.
FM: Derivatives Markets c Yufeng Guo 24
Derivatives Markets Chapter 3 c Yufeng Guo www.actuary88.com
3.2 Synthetic forwards
Buying a call and selling a put on the same underlying with each option having the same
strike price and time to expiration produces a synthetic forward.
Dierence between a synthetic long forward and an actual forward:
1. The actual forward contract has zero premium, while a synthetic forward requires a net
option premium.
2. With the forward contract, we pay the forward price; with the synthetic forward, we
pay the strike price.
3.2.1 Put-call parity
The net cost of buying the index using options must equal the net cost of buying the index
using a forward contract.
Call(K, T) Put(K, T) = PV (F
0,T
K)
To prove the parity equation, lets rewrite it as Call(K, T) + PV(K) = Put(K, T) +
PV(F
0,T
).
Proof. Assume that at t = 0, you hold two portfolios A and B. Portfolio A consists of
two assets: (1) a call option with strike price K and maturity T, and (2) the present value of
the strike price PV(K). Please note that at time T, PV(K) will become K and youll have
K in your pocket.
Portfolio B consists of two assets: (1) a put option with strike price K and maturity T,
and (2) the present value of the forward price PV(F
0,T
). Please note that at time T, PV(F
0,T
)
will accumulate to (F
0,T
), which is the price you pay to buy one stock from the seller in the
forward contract.
Lets consider 3 situations.
1. If at T, S
T
> K. For Portfolio A, you exercise the call option and buy one share of
the stock at price K; Portfolio A is worth S
T
. For Portfolio B, you let your put option
expire worthless. Portfolio B is also worth S
T
.
2. If at T, S
T
< K. For Portfolio A, you let your call option expire worthless. Portfolio
A is worth K. For Portfolio B, you exercise the put option, sell your stock, and get K.
So Portfolio B is also worth K.
3. If at T, S
T
= K. Both the call option and the put option expire worthless. Portfolio A
and B are both worth S
T
= K.
Under any situation, Portfolio A and B have equal values. Hence
Call(K, T) +PV (K) = Put(K, T) +PV (F
0,T
)
=Call(K, T) Put(K, T) = PV (F
0,T
K)
FM: Derivatives Markets c Yufeng Guo 25
Derivatives Markets Chapter 3 c Yufeng Guo www.actuary88.com
3.3 Spreads and collars
3.3.1 Bull and bear spreads
Suppose you expect that a stock price will go up. To make some money, you can do one of
the following three things today
buy a share (or a forward contract)
buy a call
sell a put
Buying a stock is expensive and has the risk that the stock price may drop in the future.
Buying a call requires paying upfront premium. Selling a put receives money upfront but has
a large downside risk that the stock price may go down.
Instead of the above 3 strategies, you can use a bull spread.
Bull spread. Two ways to build a bull spread.
Buy one call at a lower strike price K
1
and sell another call at a higher strike price
K
2
(> K
1
), with two calls on the same stock and having the same maturity. This
strategy costs you a some money upfront. You get some premiums for selling another
call at a higher strike price K
2
(> K
1
), but by doing so you give up some upside
potentials. If the stock price exceeds K
2
, your selling price is capped at K
2
.
Buy one put with a lower strike price K
1
and sell another put with a higher strike
price K
2
(> K
1
), with two puts on the same stock and having the same maturity. This
strategy gives you positive cash ows upfront but has zero or negative payo.
No matter you use calls or puts, in a bull spread, you always buy low and sell high.
The word spread means buying and selling the same type of option. The word bull
means that you can possibly make money if the stock price goes up.
Bear spread. Two ways to build a bear spread.
Buy one call at a higher strike price and sell another call at a lower strike price, with
two calls on the same stock and having the same maturity. (This strategy gives you
positive cash ows upfront)
Buy one put with a higher strike price and sell another put with a lower strike price,
with two puts on the same stock and having the same maturity. (This strategy costs
you some money upfront)
The word bear means that you can possibly make money if the stock price goes down.
No matter you use calls or puts, in a bear spread, you always buy high and sell low.
FM: Derivatives Markets c Yufeng Guo 26
Derivatives Markets Chapter 3 c Yufeng Guo www.actuary88.com
3.3.2 Box spreads
A box spread is accomplished by using options to create a synthetic long forward at one price
and a synthetic short forward at another price.
A box spread is a means of borrowing or lending money: it has no stock price risk.
Consider the textbook example. You can do the following two transactions at the same
time:
1. Buy a $40-strike call and sell a $40-strike put.
2. Sell a $45-strike call and buy a $45-strike put.
If you buy a $40-strike call and sell a $40-strike put, you are really buying a stock for
$40. If you sell a $45-strike call and buy a $45-strike put, you are really selling a stock for
$45. By doing these two things, you buy a stock for $40 and sell it for $45. This produces a
guaranteed prot of $5.
Of course, theres no free lunch. The $5 prot to be earned at t = 0.25 (T = 90 days)
must be discounted by the risk free interest rate. So the box spread must sell at t = 0 for
5 1.0833
0.25
= 4.9
As the textbook points out, $4.9 is exactly the net premium of the 4 options in the box
spread.
3.3.3 Ratio spreads
Ratio spread. An options strategy in which an investor simultaneously holds an unequal
number of long and short positions. A commonly used ratio is two short options for every
option purchased.
For example, a ratio spread can be achieved by buying one call option with a strike
price of $45 and selling two call options with a strike price of $50. This allows the investor
to capture a gain on a small upward move in the underlying stocks price. However, any
move past the higher strike price ($50) of the sold options will cause this position to lose
value. Theoretically, an extremely large increase in the underlying stocks price can cause an
unlimited loss to the investor due to the extra short call.
3.3.4 Collars
Collars. A spread involves either a call or a put but not both. However, in a collar, you use
both a call and a put.
A collar is to buy a put with one strike price K
1
and sell a call with a higher strike
price K
2
> K
1
, with both options having the same underlying asset and having the same
expiration date.
The collar width is K
2
K
1
.
Say you have lot of stocks on Google. Say Google stocks are currently selling $200 per
share. You are worrying that Google stocks may go down. You can put a lower bound on
Google stock price by buying a put with strike price $180. This way, if Googles stock price
goes to $100, you can sell your Google stocks for $180.
FM: Derivatives Markets c Yufeng Guo 27
Derivatives Markets Chapter 3 c Yufeng Guo www.actuary88.com
However, buying a put option requires money up front. To raise money to buy a put, you
can sell a call option with strike price equal to $220. If the money you get from selling a call
osets the cost of buying a put, you have a zero-cost collar.
Selling a call limits the stocks upside potential. If Googles stock price goes above $220,
you wont realize the gain because your stock will be called away at $220.
3.4 Speculating on volatility
Here the investor thinks that the stock price may change by a substantial amount but is
unsure about the direction of the change (i.e. not sure whether the price will go up or down).
The investors goal is to make some money whether the stock price will go up or down as
long as it is within a range. The investor is speculating on the stocks volatility.
3.4.1 Straddle
Straddle. In a straddle, you buy both a call and a put on the same stock with the same
strike price and the same expiration date.
Suppose that a court will issue its ruling on a company. The ruling may make or break the
company so the companys stock price may double or be cut in half. Unsure of the outcome,
you can buy both a call and a put. If the stock price goes up, you exercise the call; if the
stock price goes down, you exercise the put.
The disadvantage of a straddle is the high premium cost. The investor has to pay both
the call premium and the put premium.
Strangle. A strangle is similar to a straddle except that now you buy an out-of-money call
and an out-of-money put. A strangle is cheaper than a straddle.
Written straddle v.s. purchased straddle. If you think the stock volatility is higher
than the markets assessment (i.e. you think that the option is underpriced), you buy a
straddle. If you think that the stock volatility is lower than expected (i.e. you think the
option is over priced), you write (i.e. sell) a straddle.
3.4.2 Buttery spread
A buttery spread insures against large losses on a straddle.
1
3.4.3 Asymmetric buttery spreads
Pay attention to the following formulas
=
K
3
K
2
K
3
K
1
K
2
= K
1
+ (1 )K
3
1
Refer http://www.bigtrends.com/showQuestion.do?q_id=1735
FM: Derivatives Markets c Yufeng Guo 28
Derivatives Markets Chapter 3 c Yufeng Guo www.actuary88.com
3.5 Example: another equity-linked note
The main point of this example is that debt can be designed like an option. Marshall & Ilsley
Corp paid its debt by making annual coupons 6.5% before maturity and by paying shares of
its stocks at maturity. The number of shares paid by Marshall & Ilsley Corp at the maturity
of the debt was calculated using the formula in Table 3.7.
According to Exam FM syllabus, candidates need to know denitions of key terms of
nancial economics at an introductory level. This section seems to be beyond the introductory
level. So dont spend lot of time on convertible bond.
FM: Derivatives Markets c Yufeng Guo 29
Derivatives Markets Chapter 3 c Yufeng Guo www.actuary88.com
FM: Derivatives Markets c Yufeng Guo 30
Chapter 4
Introduction to risk management
Firms convert inputs into goods and services. A rm is protable if the cost of what it
produces exceeds the cost of the inputs.
A rm that actively uses derivatives and other techniques to alter its risk and protect its
protability is engaging in risk management.
4.1 Basic risk management: the producers perspective
This is a case study on Golddiggers. Golddiggers has to pay its xed cost. As long as the
gold price is higher than the variable cost, Golddiggers should continue its production.
Golddiggers sells gold and has an inherent long position in gold. It needs to manage the
risk that the price of gold may go down.
To lock in price of gold, Golddiggers can sell forwards, buy puts, and buy collars.
4.1.1 Hedging with a forward contract
A producer can use a short forward contract to lock in a price for his output. Golddiggers
can enter into a short forward contract, agreeing to sell gold at a price of $420/oz. in 1 year.
Make sure you can reproduce Table 4.2 and Figure 4.1.
4.1.2 Insurance: guaranteeing a minimum price with a put option
A downside of locking-in gold price with a short forward contract is that gold price may go
up. If gold price goes up, a rm cant prot from the rise of the price because it has locked
in xed price during the life of the forward contract.
To gain from the rise of the product price and still have a oor on the price, a producer
can buy a put.
Make sure you understand Table 4.3 and Figure 4.2. Sometimes a short forward gives a
producer more prot; other times buying a put yield more prot.
No hedging strategy always outperforms all other strategies.
4.1.3 Insuring by selling a call
Golddiggers can sell a call. Selling a call earns premiums. However, if the gold price goes up
beyond the call strike price, Golddiggers has to sell gold at the strike price.
31
Derivatives Markets Chapter 4 c Yufeng Guo www.actuary88.com
Make sure you understand Figure 4.4.
4.1.4 Adjusting the amount of insurance
Buying a put is like buying insurance. Insurance is expensive.
There are at least 2 ways to reduce insurance premium:
Reduce the insured amount by lowering the strike price of the put. This permits some
additional losses.
Sell some of the gain and put a cap on the potential gain.
4.2 Basic risk management: the buyers perspective
A buyer faces price risk on an input and has an inherent short position in the commodity.
If the price of the input goes up, the buyers prot goes down.
4.2.1 Hedging with a forward contract
A long forward contract lets a buyer lock in a price for his input. For example, Auric can
lock in gold price at $420 per ounce.
Make sure you understand Table 4.4 and Figure 4.6.
4.2.2 Insurance: guaranteeing a maximum price with a call option
Auric might want to put a cap on gold price but pay the market price if the price of gold
falls. Auric can buy a call option.
Make sure you understand Table 4.5.
4.3 Why do rms manage risk?
In a world with fairly priced derivatives, no transaction costs, and no other market imper-
fections such as taxes, derivatives dont increase the value of cash ow; they change the
distribution of cash ows.
Using derivatives, Golddiggers shifts dollars from high gold prices states to low gold price
states. Hedging is benecial for a rm when an extra dollar of income received in times of
high prots is worth less than an extra dollar of income received in times of low prots.
4.3.1 An example where hedging adds value
Make sure you can reproduce the calculation that the expected prot before tax is $0.1.
Make sure you know why the expected prot after tax is -$0.14. On the after tax basis,
theres 50% chance that the prot is $0.72 and 50% chance that the prot is -$1. So the
expected after-tax prot is:
0.5 (0.72 1) = 0.14
Make sure you can reproduce Figure 4.8. Point A and B are based on Table 4.6. Point
C is based on the fact that if the unit price is $10.1, then the after-tax prot is $0.06. The
FM: Derivatives Markets c Yufeng Guo 32
Derivatives Markets Chapter 4 c Yufeng Guo www.actuary88.com
product cost is $10. So the price $10.1 gives us $0.1 prot before tax, which is $0.06 after
tax.
Point D is the expected after-tax prot of -$0.14 if the unit price is either $9 or $11.2
with equal probability. Its calculated as follows: 0.5 (0.72 1) = 0.14
Line ACB is concave. A concave is an upside down U shape. When prots are concave,
the expected value of prots is increased by reducing uncertainty.
4.3.2 Reasons to hedge
Concave prot patterns can arise from:
Tax. If you can fully deduct your loss as it occurs, then you might not need to hedge
every loss; you can just incur loss and deduct your loss from your gain to reduce your
taxable income. However, if you cant fully deduct your loss or if you can deduct this
year loss the next year, then losses are no good. You might want to use derivatives to
hedge against potential losses.
Bankruptcy and distress costs. A large loss can threaten the survival of a rm. A
rm may be unable to meet its xed obligations (such as debt payments and wages).
Customers may be less willing to purchase goods of a rm in distress. Hedging allows
a rm to reduce the probability of bankruptcy or nancial distress.
Costly external nancing. If you pay losses, you have less money for money-making
projects. You might have to borrow money to fund your projects. Raising funds
externally can be costly. There are explicit costs (such as bank and underwriting fees)
and implicit costs due to asymmetric information. So you might want to use derivatives
to hedge against losses. Costly external nancing can lead a rm to forego investment
projects it would have taken had cash been available to use for nancing. Hedging can
safeguard cash reserves and reduce the need of raising funds externally.
Increase debt capacity. The amount that a rm can borrow is its debt capacity.
When raising funds, a rm may prefer debt to equity because interest expense is tax-
deductible. However, lenders may be unwilling to lend to a rm that has a high level
of debt; high-debt companies have higher probability of bankruptcy. Hedging allows a
rm to credibly reduce the riskiness of its cash ows, and thus increase its debt capacity.
Managerial risk aversion. Managers have incentives to reduce uncertainty through
hedging.
Nonnancial risk management. If you start a new business, you may need to decide on
where to set up the plan and choose between leasing and buying equipment. You need
to think through various risks associated with your decision and nd ways to manage
your risk. Risk management is not a simple matter of hedging or not hedging using
nancial derivatives, but rather a series of decisions that start when the business is rst
conceived.
4.3.3 Reasons not to hedge
Reasons why rms may elect not to hedge:
FM: Derivatives Markets c Yufeng Guo 33
Derivatives Markets Chapter 4 c Yufeng Guo www.actuary88.com
High transaction cost in using derivatives (commissions and the bid-ask spread)
A rm must assess costs and benets of a given strategy; this requires costly expertise.
(You need to hire experts to do hedging for you. Experts arent cheap!)
The rm must set up control procedures to monitor transactions and prevent unautho-
rized trading.
If you use derivatives to hedge, be prepared for headaches in doing accounting and ling
tax returns. Accounting and tax for derivatives are complex.
A rm can face collateral requirements if their derivatives lose money.
4.3.4 Empirical evidence on hedging
About half of nonnancial rms report using derivatives
Big rms are more likely to use derivatives than small rms.
Among rms that do use derivatives, less than 25% of perceived risk is hedged
Firms are more likely to hedge short term risks than long term risks.
Firms with more investment opportunities are more likely to hedge.
Firms that use derivatives have a higher market value and more leverages.
4.4 Golddiggers revisited
This section discusses additional hedging strategies for Golddiggers. The textbook has clear
explanations. Follow the textbok.

FM: Derivatives Markets c Yufeng Guo 34


Chapter 5
Financial forwards and futures
5.1 Alternative ways to buy a stock
Make sure you understand Table 5.1 from the textbook.
Outright purchase. This is like buying things from Wal-Mart. After paying the cashier
at the checkout, you immediately own the items bought.
Fully leveraged purchase. This is like buying a car with a car loan. You go to a bank
and borrow the full price of the car. Then you pay the car dealer and bring the car
home right away.
Prepaid forward contract. This is pre-ordering an item. You pay the full price at t = 0.
The product is delivered to you at time T > 0.
Forward contract. This is another way of pre-ordering an item. You dont pay anything
at t = 0. The product is delivered to you at time T > 0. Youll pay the full price at T.
5.2 Tailing
Next, lets learn a phrase called tailing. Tailing answers the following questions:
My goal is to possess one share of stock at time T. How many shares of stock do I need to
buy at t = 0 so Ill have one share of stock at time T?
If the stock doesnt pay dividend, youll need to buy one share of stock at t = 0 in order
to have one share at time T. However, if the stock pays dividend at a continuous rate of ,
you can reinvest your dividend continuously and keep buying additional fractional shares of
stocks. This way, you need to buy only a fractional share of stock at t = 0 to have one share
of stock at time T.
The textbook shows you that if you buy one share at t = 0, after dividends are contin-
uously reinvested, your initial one share will become e
T
shares at time T. Consequently,
youll need to buy e
T
share of stock at t = 0 to have one share of stock at T.
35
Derivatives Markets Chapter 5 c Yufeng Guo www.actuary88.com
t =0 Time T
1 share
T
e

shares

t =0 Time T
T
e

share 1 share

Tailing Diagram: e
T
share at t = 0 becomes one share at time T. is the continuously
compounded dividend rate.
5.3 Pricing a forward contract
Price of a prepaid forward contract is:
F
P
0,T
= S
0
- PV of future dividends (if dividends are discrete)
F
P
0,T
= S
0
e
T
(if dividends are continuous)
Price of a forward contract is:
F
0,T
= S
0
e
rt
- FV of dividends at time T (if dividends are discrete)
F
0,T
= S
0
e
(r)T
(if dividends are continuous)
The textbook derives these formulas using 3 approaches: by analogy, by discounting cash
ows, and by the no-arbitrage principle. The easiest way to derive and memorize these
formulas, however, is to use the cost-of-carry concept.
Cost of carry: The forward price F
0,T
must equal the spot price at t = 0 plus the
cost of carrying the spot price asset from t = 0 to T for delivery.
Suppose you and I sign a forward contract at t = 0. This contract requires me to deliver
a stock at time T to you and requires you to pay me F
0,T
at time T. The essence of this
forward contract is that I need to sell a stock to you at time T for a preset price F
0,T
.
Lets calculate how much it costs me to have a stock ready for delivery at time T. If the
stock pays continuous dividend at the rate of , to have one share of stock at time T, I just
need to buy e
T
share at t = 0. Then the cost of having a stock ready for delivery at T is:
Spot price at time zero + Cost-of-carry during [0, T]
Spot price at time zero = Cost of buying e
T
share of stock at time zero=S
0
e
T
FM: Derivatives Markets c Yufeng Guo 36
Derivatives Markets Chapter 5 c Yufeng Guo www.actuary88.com
Cost-of-carry during [0, T]=Foregone interest of tying my capital during [0, T]
The foregone interest of tying my capital during [0, T] is S
0
e
T
(e
rT
1), where r is
the continuously compounded risk-free interest rate. Had I put S
0
e
T
in a bank account
(instead of buying stocks) at t = 0, my fund balance at T would be S
0
e
T
e
rT
and I would
have earned S
0
e
T
e
rT
S
0
e
T
= S
0
e
T
(e
rT
1) interest during [0, T].
So the cost of having a stock ready for delivery at T is:
S
0
e
T
+S
0
e
T
(e
rT
1) = S
0
e
T
e
rT
= S
0
e
(r)T
The forward price is: F
0,T
= S
0
e
T
e
rT
= S
0
e
(r)T
If the stock doesnt pay any dividend, to nd the forward price, just set = 0: F
0,T
= S
0
e
rT
What if the stock pays discrete dividends? Easy. To have a stock ready for delivery at T, I
can buy one share of stock at time zero and pay S
0
. The foregone interest of tying up my
capital S
0
during [0, T] is S
0
e
rT
S
0
. The dividend I receive will reduce my cost of carry.
Then the forward price is:
S
0
+S
0
e
rT
S
0
- FV of all the dividends received=S
0
e
rT
- FV of all the dividends received
Next, lets apply the cost-of-carry concept to a prepaid forward contract. With a prepaid
forward contract, the buyer pays me right away and I dont tie up my capital any more; my
foregone interest is zero. The PV of the dividends I will receive during [0, T] will reduce of
the cost of carrying the asset from time zero to T. Now we have:
F
P
0,T
=Spot price + Cost of carry =S
0
- PV of future dividends (If dividends are discrete)
However, if the dividend is continuous, to have one stock ready for delivery at T, I just need
to buy e
T
share of stock at time zero. So the prepaid forward price is:
F
P
0,T
= S
0
e
T
Now you shouldnt have trouble memorizing the pricing formulas for a prepaid forward and
a typical (post-paid) forward contract.
5.4 Forward premium
The forward premium is the ratio of the forward price to the spot price:
Forward Premium=
F
0,T
S
0
The annualized forward premium (AFP) is dened as:
S
0
e
AFPT
= F
0,T
=AFP =
1
T
ln

F
0,T
S
0

FM: Derivatives Markets c Yufeng Guo 37


Derivatives Markets Chapter 5 c Yufeng Guo www.actuary88.com
5.5 Creating a synthetic forward contract
The seller in a forward contract faces the risk that the securitys spot price at time T (that
is S
T
) may exceed the forward price F
0,T
by a large amount. For example, you and I sign a
forward contract at t = 0. Under the contract, I need to give you one share of ABC stock for
$20 at time T. If one share of ABC stock will sell for $100 at T, I will lose $80. (If I sell one
share of ABC stock in the market at T, I can earn $100. However, I have to sell one share of
ABC stock to you at for $20. So I lose $80 and you gain $80.)
At the expiration date T:
The payo earned by the buyer in the forward contract is S
T
F
0,T
The payo earned by the seller in the forward contract is F
0,T
S
T
The payo earned by the seller + the payo earned by the buyer=0
To hedge the risk that the spot price S
T
may rise, the seller in a forward contract can buy
e
T
share of stock at t = 0 by paying a xed cost of S
0
e
T
and hold it to T for delivery.
This way, hell already have one share of the stock and doesnt have to worry how high S
T
may become. (If he doesnt have one share of stock ready at T for delivery and S
T
happens
to be very high, the seller is forced to buy the stock at in the open market and will suer a
huge loss).
To avoid tying up his own money, the seller can go to a bank and borrow S
0
e
T
. When
the delivery time T arrives, the seller just needs to pay back the principal and the interest
to the bank. The sum of the principal and the interest due at T is S
0
e
T
e
rT
. This strategy
costs the seller nothing to set up but gives the seller S
T
S
0
e
T
e
rT
at time T.
Please note that S
T
S
0
e
T
e
rT
is exactly the net payment at T by the seller to the
buyer in a fairly priced forward contract. A fairly priced forward contract on one share of
stock to be delivered at T is priced at F
0,T
= S
0
e
T
e
rT
. Then at time T, the buyer needs to
pay the seller F
0,T
= S
0
e
T
e
rT
; the seller needs to deliver a stock worth S
T
. Instead of the
buyer writing the seller a check in the amount of F
0,T
= S
0
e
T
e
rT
and the seller delivering
a stock worth S
T
to the buyer, the forward contract can be settled by having the seller send
the buyer a check in the amount of S
T
S
0
e
T
e
rT
.
These transactions are summarized in Table 5.3.
The textbook gives you the formula Forward = Stock - Zero-coupon bond. To under-
stand what this means, notice that borrowing money from a bank is the same as issuing a
zero coupon bond; lending money to someone is just like buying a zero-coupon bond.
Say you borrow $100 from a bank at time zero. You plan to pay o your loan at time
one. The interest rate is 8%. Your total loan payment (interest plus principal) to the bank
at time one is 100 1.08 = 108. Conceptually, this is the same as you issuing (ie. selling) a
1-year zero coupon bond with $100 par value and 8% annual coupon to the bank. At time
zero, you collect $100 from a bank. At time one, you pay the bank face amount plus coupon
in the total amount of 100 + 8=108.
So conceptually, borrowing from a bank is selling a zero coupon bond to the bank. Lending
your money to bank (ie. putting money in savings account) is the same as buying a zero
coupon bond.
FM: Derivatives Markets c Yufeng Guo 38
Derivatives Markets Chapter 5 c Yufeng Guo www.actuary88.com
Forward = Stock - Zero-coupon bond means that borrowing money and buying some
stock can synthetically create a forward contract. As explained earlier, if at time zero you
borrow S
0
e
T
and buy e
T
share of stock, then at time T youll have S
T
S
0
e
T
e
rT
,
which is exactly the payo earned by the buyer in the forward contract.
To help memorize the equation Forward = Stock - Zero-coupon bond, notice that
negative zero-coupon bond means borrowing money (ie issuing a zero coupon bond).
Make sure you understand Table 5.4 and Table 5.5. Table 5.4 has two transaction. First, you
enter a forward contract to buy one share of stock at T (i.e. having a long position in a forward
contract). To make sure you indeed can pay the forward price F
0,T
= S
0
e
T
e
rT
at time T,
at time zero you deposit the present value of this forward price F
0,T
e
rt
= S
0
e
T
e
rT
e
rt
=
S
0
e
T
to earn risk free interest rate r. Then at time T, youll have exactly F
0,T
= S
0
e
T
e
rT
,
which you give to the seller and receive one share of stock. So your initial cost at time zero
is S
0
e
T
. Your wealth at time T is S
T
(ie. one share of stock).
The formula Stock=Forward + Zero-coupon bond means that buying a forward contract
and a zero coupon bond is the same as owning a stock. Please note a positive sign before
zero-coupon bond means that you are lending more (ie buying a zero-coupon bond).
Table 5.5 shows how to synthetically create a savings account. At time zero, you sell one
forward contract and simultaneously buy e
T
share of stock. Then at T, you have exactly
one share of stock, which you deliver to the buyer and get F
0,T
. So your initial cost at time
zero is S
0
e
T
; your wealth at time T is F
0,T
. This is like you putting S
0
e
T
at time zero into
a savings account and getting F
0,T
at time T.
The formula Zero-coupon bond = Stock - Forward means that buying a stock and
selling is the same as lending more (ie. buying a zero-coupon bond).
5.5.1 Synthetic forwards in market-making and arbitrage
The seller in a forward contract can hedge his risk. The sellers biggest risk is this: If at
time T the stock price is S
T
= and he doesnt already have one share of stock ready for
delivery at T, then hes forced to buy a share of stock from the open market at time T and
pay S
T
= ; hell be bankrupt.
To hedge this risk, the seller needs to have one share of stock ready ahead of time. The
sellers hedging transactions are listed at Table 5.6.
The buyer in a forward contract can also hedge his risk. The buyers biggest risk is this: if at
time T the stock price is S
T
= 0, he has to pay F
0,T
and buy a worthless stock. To hedge this
risk, the buyer should somehow get rid of this worthless stock immediately after receiving it.
This can be achieved by shorting selling e
T
share of stock at time zero (then at time T one
share of stock needs to be returned to the broker who facilitates the short selling). These
hedging transactions are listed in Table 5.7.
How to arbitrage if the forward price is too high or too low. If the forward price is too high,
you can use cash-and-carry to arbitrage: you buy stocks at time zero and carry it forward to
T for delivery. Cash-and-carry transactions are listed in Table 5.6.
If, on the other hand, the forward price is too low, you use reverse cash-and-carry: at
time zero, you short sell stocks and simultaneously buy a forward. Reverse cash-and-carry
FM: Derivatives Markets c Yufeng Guo 39
Derivatives Markets Chapter 5 c Yufeng Guo www.actuary88.com
transactions are listed in Table 5.7.
5.5.2 No-arbitrage bounds with transaction costs
How to derive the no-arbitrage upper bond. If the forward price F
0,T
is too high, this is how
to make some money:
(1) Buy low: At t = 0 pay ask price and buy one stock. Incur transaction cost k. Total
cash outgo: S
a
0
+k.
(2) Sell high: At t = 0 sell a forward contract. Incur transaction cost k. Total cash
outgo: k.
(3) At t = 0, borrow S
a
0
+ 2k to pay for (1) and (2)
(4) At time T, deliver the stock to the buyer. Get F
0,T
.
(5) At time T, pay (S
a
0
+ 2k)e
r
b
T
Your initial cash outgo after (1) through (5) is zero. Your payo at time T is
F
0,T
(S
a
0
+ 2k)e
r
b
T
Arbitrage is possible if:
F
0,T
(S
a
0
+ 2k)e
r
b
T
> 0
F
0,T
> F
+
= (S
a
0
+ 2k)e
r
b
T
How to derive the no-arbitrage lower bond. If the forward price F
0,T
is too low, this is how
to make some money:
(1) Buy low: At t = 0 enter a forward contract to buy one stock. Incur transaction
cost k. Total cash outgo: k.
(2) Sell high: At t = 0 sell a stock short and get bid price S
b
0
. Incur transaction cost k.
Total cash inow: S
b
0
k.
(3) After (1) and (2), the net cash inow is S
b
0
2k. At time zero, lend S
b
0
2k at
the lending rate r
l
(i.e. depositing S
b
0
2k into a savings account). At time T, your
account should be (S
b
0
2k)e
r
l
T
.
(4) At time T, pay F
0,T
and buy one stock from the seller in the forward contract.
Return the stock to the broker who facilitated the short sale.
(5) At time T, take out (S
b
0
2k)e
r
l
T
from the savings account.
Your initial cash outgo after (1) through (5) is zero. Your payo at time T is
(S
b
0
2k)e
r
l
T
F
0,T
FM: Derivatives Markets c Yufeng Guo 40
Derivatives Markets Chapter 5 c Yufeng Guo www.actuary88.com
Arbitrage is possible if:
(S
b
0
2k)e
r
l
T
F
0,T
> 0
F
0
T
< F

= (S
b
0
2k)e
r
l
T
To avoid arbitrage, we need to have:
(S
b
0
2k)e
r
l
T
= F

F
0,T
F
+
= (S
a
0
+ 2k)e
r
b
T
How to memorize
(S
b
0
2k)e
r
l
T
= F

F
0,T
F
+
= (S
a
0
+ 2k)e
r
b
T
Please note that S
b
0
< S
a
0
due to the bid-ask spread.
In addition, r
l
< r
b
; when you put money in a savings account (i.e. lending money to a
bank), you get a lower interest rate; when you borrow money from a bank, you pay a higher
interest rate.
Your lower bound should be small; your higher bound should be big. So it makes sense
to have
(S
b
0
2k)e
r
l
T
= F

F
0,T
F
+
= (S
a
0
+ 2k)e
r
b
T
Please note that there may be complex structure for the transaction cost where the above
non-arbitrage formulas cant be blindly used. Under a complex transaction cost structure,
youll want to derive the non-arbitrage bound using the approach presented above. Please
refer to my solution to the textbook problem 5.15 on how to handle complex transaction cost.
5.5.3 Quasi-arbitrage
In a full arbitrage, you either borrow money from a bank to buy a stock (if its priced low
relative to a forward) or short sell a stock (if it is priced high relative to a forward). If you
dont need to borrow money or short sell a stock yet you can still earn free money, you are
doing a quasi-arbitrage.
Quasi-arbitrage Example 1 (textbook example)
The corporation borrows at 8.5% and lends at 7.5%. Knowing the formula Zero-coupon
bond = Stock - Forward, it can build (synthetically create) a zero coupon bond using stocks
and forward. In this zero coupon bond, the corporation lends money at time zero and gets
principal and interest paid at time T. If it earns more than 7.5% (such as 8%) in this
synthetically created zero-coupon bond, the corporate can stop lending at 7.5% and keep
building lots of zero-coupon bonds to earn an 8% interest rate.
This is a quasi-arbitrage. Here the corporation doesnt need to borrow money from a bank
to make free money (because it already has borrowed money at 8.5% and its now trying to
pay o its debt). If the corporation doesnt already have a 8.5% debt to begin with, it will be
silly for the corporation to borrow at 8.5%, buy a bunch of stocks, and sell bunch of forwards
so it can use the formula Zero-coupon bond = Stock - Forward to earn an 8% interest rate.
Quasi-arbitrage Example 2
Suppose the forward price on a stock is too low and you already own a stock. To arbitrage,
you can sell your stock (instead of selling a stock short) and buy a forward contract. You
FM: Derivatives Markets c Yufeng Guo 41
Derivatives Markets Chapter 5 c Yufeng Guo www.actuary88.com
can deposit the sales proceeds in a savings account and earn interest. Then on the expiration
date T of the forward contract, you pay a low forward price F
0,T
and buy a stock. The net
eect is that you temporarily give up your ownership of a stock during [0, T] but regain the
ownership of a stock at T. Such a transaction will give you some free money if F
0,T
is too
low.
5.5.4 Does the forward price predict the future spot price?
Though the author says that the forward price F
0,T
doesnt reect the future spot price S
T
,
this is a controversial topic. Experts disagree on this issue. Some argue that F
0,T
reects S
T
.
For example, if you think S
T
will be low, you dont want to pay a high forward price F
0,T
.
However, since SOA chose Derivatives Markets as the textbook, you might want to accept
the view that F
0,T
doesnt reect the expected future spot price S
T
at least when you are
taking the exam.
5.6 Futures contracts
5.6.1 Role of the clearing house
The clearinghouse guarantees the performance of every buyer and every seller of a futures
contract. It stands as a buyer to every seller and a seller to every buyer. The buyer and the
seller dont have to worry that the counterparty may default.
Instead of having the seller and the buyer in a forward contract deal with each other
directly, the clearinghouse sits between them serving as a counterparty to each of them. To
the buyer, the clearinghouse is the seller; the clearinghouse is obligated to deliver the asset
to the buyer at the agreed upon price. To the seller, the clearinghouse is the buyer; the
clearinghouse is obligated to buy the asset at the expiration date with the agreed upon price.
Money Money

Asset Asset

Buyer
Clearing
House Seller
With a clearinghouse as the counterparty, the buyer/seller in a futures contract can easily
liquidate his position prior to the expiration date. If the buyer or seller wants to back out
from a futures contract before the contract matures, he can do a reverse trade and close out
his position.
5.6.2 S&P 500 futures contract
The most important concept for the purpose of passing the exam is the value of one S&P 500
futures contract. The value of one S&P 500 futures contract is calculated by multiplying the
FM: Derivatives Markets c Yufeng Guo 42
Derivatives Markets Chapter 5 c Yufeng Guo www.actuary88.com
futures price by 250:
One S&P 500 futures contract value = 250 futures price
For example, if the S&P 500 futures price is 1000, then one S&P 500 futures contract is
worth 250, 000 = 250, 000.
5.6.3 Dierence between a forward contract and a futures contract
This is a very important topic. The textbook has a nice comparison of forwards and futures.
Make sure you memorize the dierences.
Forward contracts are settled at expiration; futures contracts are settled daily through
marking to market.
Forward contracts are less liquid; futures contracts are more liquid.
Forward contracts are customized to t the special needs of the buyer and the seller
(hence less liquid); futures contracts are standardized (hence more liquid).
Forward contracts have credit risk (any party may break its promises); futures contracts
have minimal credit risks.
Futures markets have daily price limits while forward contracts dont.
Please note that these are general dierence. Some forward contracts (such as in foreign
exchanges) are standardized and liquid; some futures contracts give the buyer and the seller
greater exibility in contractual obligations.
Final point. Forward contracts are typically simpler than futures contracts. Forwards
are settled at the expiration date; futures contracts are settled daily and have margins re-
quirement. As a result, forward contracts require less management control and are good for
unsophisticated and infrequent users.
5.6.4 Margins and markings to market
Open interest. Open interest is the total number of outstanding contracts held by market
participants at the end of each day. For each seller of a futures contract there is a buyer. A
seller and a buyer together are counted as only one contract. (The clearinghouse is excluded
in the calculation of the open interest since its net position is zero).
To determine the total open interest for any given market we need only to know the totals
from one side or the other, buyers or sellers, not the sum of both.
If two parties have just signed a futures contract, the open interest will increase by one
contract. If the buyer and the seller have closed out a futures contract, the open interest will
decrease by one contract. If one party passes o his position to a new party (one old buyer
sells to one new buyer), then the open interest will not change.
Initial margin, Maintenance margin, Margin call. In the futures market, margin is the
initial good faith deposit made into an account in order to enter into a futures contract.
When you open a futures contract, the futures exchange will ask you to deposit a minimum
amount of money (cash or cash equivalent such as T-bills) into your account. This original
FM: Derivatives Markets c Yufeng Guo 43
Derivatives Markets Chapter 5 c Yufeng Guo www.actuary88.com
deposit is called the initial margin. At initial execution of a trade, both the seller and the
buyer must establish a margin account.
When your position is closed out, you will be refunded the initial margin plus or minus
any gains or losses that occur over the span of the futures contract.
The initial margin is the minimum deposit one needs to put in the margin account before
he can enter into a new futures contract. The maintenance margin is the lowest amount an
account can reach before needing to be replenished. If the margin account drops to a certain
level due to daily losses, the buyer or the seller will get a margin call asking him to deposit
more money into the account to bring the margin back up.
Marking to market. On any date when futures contracts are traded, futures prices may go
up or down. Instead of waiting until the maturity date for the buyer and the seller to realize
all gains and losses, the clearinghouse requires that the buyer and the seller recognize their
gain and loss at the end of each day.
The best way to explain marking to market is through an example.
Example 1. Assume the current futures price for gold for delivery 6 days form today is $495
per ounce. Contract size is 100 ounces. The initial margin is 5% of the contract value. The
maintenance margin is $2,000.
You are given the following prices of gold futures.
Day Futures Price
0 (Today) $495
1 $496
2 $487
3 $493
4 $480
5 $490
6 (Delivery Date) $485
Calculate the daily account value of the buyers margin account. Determine when the buyer
gets the margin call.
Solution
Day Futures price price change Gain/loss Margin Bal Margin call
0 $495 $2,475
1 $496 $1 $100 $2,575 $0
2 $487 -$9 -$900 $1,675 $325
3 $493 $6 $600 $2,600 $0
4 $480 -$13 -$1,300 $1,300 $700
5 $490 $10 $1,000 $3,000 $0
6 $485 -$5 -$500 $2,500 $0
Total -$1,000
Explanations:
FM: Derivatives Markets c Yufeng Guo 44
Derivatives Markets Chapter 5 c Yufeng Guo www.actuary88.com
The futures prices $495, $496, $487, $493, $480, $490, and $485 all have Day 6 as the
delivery time. The Day 6 futures price $485 is the spot price on Day 6.
The initial margin is 2475 = 1004955%. Both the buyer and the seller must deposit
at least this amount to their respective margin account to initiate a futures contract.
2, 575 = 2, 475 + 100. Each day the clearinghouse updates the delivery price using the
current days futures price. So on Day 1, the clearinghouse (who acts as the seller to
the buyer) assumes that it will delivery 100 ounces of gold to the buyer at the delivery
price of $496 per ounce. The buyer gain $1 per ounce (it locked in the $495 delivery
price, while the current delivery price is $496). $100 gains are immediately added to
the buyers margin account.
1, 675 = 2, 575 900. The buyer loses $9 per ounce. Yesterday, the clearinghouse could
delivery gold at the price of $496 per ounce. However, today the clearinghouse can
deliver it at a cheaper price of $487 per ounce. The total loss $900 was immediately
deducted from the buyers margin account.
325 = 2, 000 1, 675. The margin balance $1,675 is below the maintenance margin of
$2,000. So the buyer must deposit $325 to bring his margin account to the maintenance
margin level.
2, 600 = 1, 675 + 325 + 600. The buyer gains $6 per ounce.
The sellers margin account:
Day Futures price price change Gain/loss Margin Bal Margin call
0 $495 $2,475
1 $496 $1 -$100 $2,375 $0
2 $487 -$9 $900 $3,275 $0
3 $493 $6 -$600 $2,675 $0
4 $480 -$13 $1,300 $3,975 $0
5 $490 $10 -$1,000 $2,975 $0
6 $485 -$5 $500 $3,475 $0
Total $1,000
Example 2 Reproduce Table 5.8 in Derivatives Markets.
Solution
Facts:
The notional value of one contract = 250 1, 100
The number of contracts is 8
Initial margin = 10%
Margin account earns 6% continuously compounding
FM: Derivatives Markets c Yufeng Guo 45
Derivatives Markets Chapter 5 c Yufeng Guo www.actuary88.com
week multiplier Futures Price price change interest earned Loss Margin balance
0 2,000.00 $1,100.00 $0.00 $220,000.00
1 2,000.00 $1,027.99 -$72.01 $253.99 -$144,020 $76,233.99
2 2,000.00 $1,037.88 $9.89 $88.01 $19,780 $96,102.01
3 2,000.00 $1,073.23 $35.35 $110.95 $70,700 $166,912.96
4 2,000.00 $1,048.78 -$24.45 $192.70 -$48,900 $118,205.66
5 2,000.00 $1,090.32 $41.54 $136.47 $83,080 $201,422.13
6 2,000.00 $1,106.94 $16.62 $232.54 $33,240 $234,894.67
7 2,000.00 $1,110.98 $4.04 $271.19 $8,080 $243,245.86
8 2,000.00 $1,024.74 -$86.24 $280.83 -$172,480 $71,046.69
9 2,000.00 $1,007.30 -$17.44 $82.02 -$34,880 $36,248.72
10 2,000.00 $1,011.65 $4.35 $41.85 $8,700 $44,990.57
Explanations
The margin account balance at the end of Week 0 is
220, 000 = 8 250 1, 100 10%
The interest earned in Week 1 is 253.99 = 220, 000(e
6%
52
1)
The loss incurred in Week 1 is 144, 020 = 8 250 (72.01)
The margin account balance at the end of Week 1 is 76, 233.99 = 220, 000 + 253.99
144, 020
The interest earned in Week 2 is 88.01 = 76, 233.99(e
6%
52
1)
5.6.5 Comparing futures and forward prices
At expiration T, forward price and futures price are both equal to the spot price S
T
.
At T 1 (one day before expiration), a futures contracts will be marked to market the
very next day T and a forward contract will also be marked to market at the very next day T
(a forward contract is marked to market only at T). As a result, one day prior to expiration,
a forward contracts and a futures contracts have identical cash ows and should have the
same price.
The price of a futures contract two or more days before expiration is more complex.
However, the following general conclusions can be drawn:
If interest rates are positively correlated with futures prices (i.e. interest rates go up
if futures prices go up), then the investor holding a long position (i.e. the buyer) will
prefer a futures contract to a forward contract. When futures prices go up, the buyer
realizes his gain through daily marking-to-market. This gain earns a higher interest
rate. On the other hand, if futures prices decline, interest rates also decline. The buyer
suer losses through marking to market but losses are incurred when the opportunity
cost is lower (because interest rates are lower). On average, a long futures contract will
outperform a long forward contract.
FM: Derivatives Markets c Yufeng Guo 46
Derivatives Markets Chapter 5 c Yufeng Guo www.actuary88.com
If interest rates are negatively correlated with futures prices (i.e. interest rates go down
if futures prices go up), then the investor holding a long position (i.e. the buyer) will
prefer a forward contract to a futures contract. When futures prices go up, the buyers
gain through marking to market is reinvested at a lower interest rate. When future
prices go down, the buyers losses are incurred at a higher interest rate. On average, a
long futures contract will perform worse than a long forward contract.
5.6.6 Arbitrage in practice: S&P 500 index arbitrage
Multiple theoretically correct prices may exist for futures contracts. Reasons include:
Future dividends are uncertain.
Transaction costs are not zero and hence a range of fair prices may exist.
Assumptions (such as margin requirement, credit risk, daily settlement, clearing house
guarantee, etc) in futures pricing may be dierent, producing dierence prices for the
same futures contract.
Arbitrageurs usually buy a subsect of 500-stock index. As such, futures contracts and
the osetting position in stock may not move together.
FM: Derivatives Markets c Yufeng Guo 47
Derivatives Markets Chapter 5 c Yufeng Guo www.actuary88.com
5.6.7 Appendix 5.B: Equating forwards and futures
The most important thing about Appendix 5.B is Table 5.13. Before taking Exam FM, make
sure can reproduce Table 5.13.
To understand Table 5.13, lets look at three dierent positions.
Position 1 You enter into 8 long forward contracts on S&P 500 Index at time zero. The
delivery date is 10 weeks from today. The forward price is 1100. Your initial margin for 8
long forwards is 217,727.21. The margin account earns a continuously compounded annual
interest rate 6%. Whats your prot from your long forward position?
Additional information is:
Week Forward price
0 $1,100.00
1 $1,027.99
2 $1,037.88
3 $1,073.23
4 $1,048.78
5 $1,090.32
6 $1,106.94
7 $1,110.98
8 $1,024.74
9 $1,007.30
10 $1,011.65
In the above Table, $1,100.00 is the forward price youll pay at Week 10. $1,027.99 is the
new forward price one week later (i.e if you enter into a long S&P 500 forward at Week
1 to receive the index at Week 10, youll pay $1,027.99 at Week 10 to receive the index).
Similarly,$1,037.88 is the forward price at Week 2 to receive the index at Week 10. So on
and so forth. The nal price $1,011.65 is the spot price at Week 10.
Since a forward contract is only marked to market at the expiration date, the forward
prices at Week 1, 2, 3, ..., and 9 are not needed for calculating the prot. Only the forward
price at Week 1 and the forward price at Week 10 matter. Please note one S&P 500 forward
contract is worth 250 times the forward price. Since you long 8 contracts, your the prot is
217, 727.21e
0.06
10
52
+ 250 8 (1, 011.65 1, 100) = 43, 554.997 = 43, 554.00
Your prot is:
43, 554.00 217, 727.21e
0.06
10
52
= 176, 700
Alternative calculation of the prot:
250 8 (1, 011.65 1, 100) = 176, 700
FM: Derivatives Markets c Yufeng Guo 48
Derivatives Markets Chapter 5 c Yufeng Guo www.actuary88.com
Position 2 You enter into 8 long futures contracts on S&P 500 Index at time zero. The
delivery date is 10 weeks from today. The futures price is 1100. Your initial margin for 8 long
futures is 217,727.21. The margin account earns a continuously compounded annual interest
rate 6%. Assuming weekly marking to market. Whats your prot from your long futures
position?
Additional information is:
Week Futures price
0 $1,100.00
1 $1,027.99
2 $1,037.88
3 $1,073.23
4 $1,048.78
5 $1,090.32
6 $1,106.94
7 $1,110.98
8 $1,024.74
9 $1,007.30
10 $1,011.65
The solution is
Week Multiplier Futures Price Price Change Margin Balance
0 2,000 $1,100.00 $217,727.21
1 2,000 $1,027.99 -$72.01 $73,958.58
2 2,000 $1,037.88 $9.89 $93,823.96
3 2,000 $1,073.23 $35.35 $164,632.29
4 2,000 $1,048.78 -$24.45 $115,922.36
5 2,000 $1,090.32 $41.54 $199,136.19
6 2,000 $1,106.94 $16.62 $232,606.09
7 2,000 $1,110.98 $4.04 $240,954.64
8 2,000 $1,024.74 -$86.24 $68,752.83
9 2,000 $1,007.30 -$17.44 $33,952.20
10 2,000 $1,011.65 $4.35 $42,691.40
Explanation of the above table.
250 8 = 2, 000
217, 727.21e
0.06
1
52
+ 2, 000 (72.01) = 73, 958.58
73, 958.58e
0.06
1
52
+ 2, 000 9.89 = 93, 823.96
Your prot is:
42, 691.40 217, 727.21e
0.06
10
52
= 177, 562.60
If you compare Position 1 with Position 2, youll see that your prot on 8 long futures
contracts is lower than your prot on 8 long forward contracts. This is due to futures marking
to market.
FM: Derivatives Markets c Yufeng Guo 49
Derivatives Markets Chapter 5 c Yufeng Guo www.actuary88.com
Position 3 Your enter into long futures contracts S&P 500 Index at time zero. Your goal
is for Position 3 to produce exactly the same prot as in Position 1. The futures prices are
as follows:
Week Futures price
0 $1,100.00
1 $1,027.99
2 $1,037.88
3 $1,073.23
4 $1,048.78
5 $1,090.32
6 $1,106.94
7 $1,110.98
8 $1,024.74
9 $1,007.30
10 $1,011.65
How can you make Position 3 and Position 1 have the same prot?
Solution
FM: Derivatives Markets c Yufeng Guo 50
Derivatives Markets Chapter 5 c Yufeng Guo www.actuary88.com
Week Multiplier Futures Price Price Change Margin Balance
0 1,979.34 $1,100.00 $217,727.21
1 1,981.62 $1,027.99 -$72.01 $75,446.43
2 1,983.91 $1,037.88 $9.89 $95,131.79
3 1,986.20 $1,073.23 $35.35 $165,372.88
4 1,988.49 $1,048.78 -$24.45 $117,001.18
5 1,990.79 $1,090.32 $41.54 $199,738.33
6 1,993.09 $1,106.94 $16.62 $233,055.87
7 1,995.39 $1,110.98 $4.04 $241,377.01
8 1,997.69 $1,024.74 -$86.24 $69,573.26
9 2,000.00 $1,007.30 -$17.44 $34,813.80
10 $1,011.65 $4.35 $43,554.00
Explanation. Since marking to market magnies gains and losses, youll need to hold fewer
than 8 long futures contracts at t = 0. To determine how many contracts to hold at t = 0,
notice that if Week 1 futures price is one dollar more than Week 0 futures price, youll realize
one dollar gain. This one dollar will be deposited into your margin account at the end of
Week 1 and accumulate with interest to the end of Week 10 (i.e. earning 9-month interest).
To undo the eect of marking to market, you should hold 8e
0.06
9
52
= 7.91735295 contracts.
The contract size is 250 7.91735295 = 1, 979.34, which is less than 2,000.
What if the Week 1 futures price is one dollar less than Week 0 futures price? In this
case, youll realize one dollar loss. This one dollar will be deducted from your margin account
at the end of Week 1, reducing the future value of your margin account. Once again, you
need to hold 8e
0.06
9
52
= 7.91735295 contracts.
Similarly, you need to hold 8e
0.06
8
52
= 1, 981.62 contracts at the end of Week 1 (every
dollar gain recognized at the end of Week 2 will accumulate 8 months interest to Week 10).
And you need to hold 8e
0.06
7
52
= 7.93564486 at the end of Week 3. So on and so forth.
After marking to market at the end of Week 9, the futures contract becomes a forward
contract (since both contracts will be marked to market at the nal week). At the end of
Week 9, you should hold exactly 8 futures contracts.
Explanations of other calculations.
217, 727.21e
0.06
52
+ 1, 979.34 (72.01) = 75, 446.43
75, 446.43e
0.06
52
+ 1, 981.62 (9.89) = 95, 131.79
You ending margin account value at the end of Week 10 is
34, 813.80e
0.06
52
+ 2, 000 (4.35) = 43, 554.00
Your prot is:
43, 554.00 217, 727.21e
0.06
10
52
= 176, 700.00
Now you see that Position 1 and Position 3 have the same ending margin account and
the same prot.
FM: Derivatives Markets c Yufeng Guo 51
Derivatives Markets Chapter 5 c Yufeng Guo www.actuary88.com
Practice problem You enter into 8 long futures contracts on S&P 500 Index at time
zero. The delivery date is 10 weeks from today. The initial margin for the 8 contracts is
300,000. The margin account earns a continuously compounded annual interest rate 6%.
Calculate your prot from the long position under the following 3 scenarios:
1. Marking to market is done only at the expiration date.
2. Marking to market is done weekly and you always hold 8 contracts.
3. Marking to market is done weekly. You re-balance your holdings weekly to produce the
same prot as Scenario 1.
Futures prices are:
Week Futures Price
0 $1,000
1 $1,020
2 $1,015
3 $1,026
4 $1,110
5 $1,065
6 $1,105
7 $1,085
8 $1,093
9 $1,008
10 $1,004
Solution
Scenario 1
The ending balance of the margin account is
300, 000e
0.06
10
52
+ 8 250 (1, 004 1, 000) = 311, 481.59
The prot is:
311, 481.59 300, 000e
0.06
10
52
The prot is:
8 250 (1, 004 1, 000) = 8, 000
FM: Derivatives Markets c Yufeng Guo 52
Derivatives Markets Chapter 5 c Yufeng Guo www.actuary88.com
Scenario 2
Week Multiplier Futures Price Price Change Margin Balance
0 2,000 $1,000 $300,000.00
1 2,000 $1,020 $20 $340,346.35
2 2,000 $1,015 -$5 $330,739.29
3 2,000 $1,026 $11 $353,121.13
4 2,000 $1,110 $84 $521,528.81
5 2,000 $1,065 -$45 $432,130.92
6 2,000 $1,105 $40 $512,629.82
7 2,000 $1,085 -$20 $473,221.66
8 2,000 $1,093 $8 $489,768.00
9 2,000 $1,008 -$85 $320,333.44
10 $1,004 -$4 $312,703.27
The ending balance at the end of Week 10 is $312,703.27. The prot is: 312, 703.27
300, 000e
0.06
10
52
= 9, 221.69
Sample calculations:
340, 346.35 = 300, 000e
0.06
1
52
+ 2000 20
330, 739.29 = 340, 346.35e
0.06
1
52
+ 2000 (5)
FM: Derivatives Markets c Yufeng Guo 53
Derivatives Markets Chapter 5 c Yufeng Guo www.actuary88.com
Scenario 3
Week Multiplier Futures Price Price Change Margin Balance
0 1,979.34 $1,000 $300,000.00
1 1,981.62 $1,020 $20 $339,933.12
2 1,983.91 $1,015 -$5 $330,417.46
3 1,986.20 $1,026 $11 $352,621.95
4 1,988.49 $1,110 $84 $519,870.00
5 1,990.79 $1,065 -$45 $430,987.93
6 1,993.09 $1,105 $40 $511,117.13
7 1,995.39 $1,085 -$20 $471,845.44
8 1,997.69 $1,093 $8 $488,353.32
9 2,000.00 $1,008 -$85 $319,113.17
10 $1,004 -$4 $311,481.59
The ending balance at the end of Week 10 is 311,481.59. The prot is 311, 481.59300, 000e
0.06
10
52
=
8, 000
Sample calculation:
1, 979.34 = 8 250e
0.06
9
52
1, 981.62 = 8 250e
0.06
8
52
1, 997.69 = 8 250e
0.06
1
52
2, 000 = 8 250e
0.06
0
52
339, 933.12 = 300, 000e
0.06
1
52
+ 1, 979.34 20
330, 417.46 = 339, 933.12e
0.06
1
52
+ 1, 981.62 (5)

FM: Derivatives Markets c Yufeng Guo 54


Chapter 8
Swaps
Chapter 8 of Derivative Markets is a dicult but important chapter. The most important
concept is pricing of swaps. SOA can easily come up questions on this topic. Make sure you
know how to calculate the price of a swap.
8.1 An example of a commodity swap
First, lets understand what a swap is. Lets not bother memorizing the textbook denition
of a swap but focus on the essence of a swap.
A swap is multiple forward contracts combined into one big contract. For example, you
are an owner of an airline company. Your two biggest operating expenses will be labor and
fuel. Labor cost can be pretty much controlled by adding or reducing sta. However, the
fuel cost is subject to a wild uctuation. From time to time, we all complain about how
expensive gas is, but wait till you own multiple airplanes and see how quickly your gas bill
goes to the roof.
One way to control the fuel cost is to use a series of forward contracts. For example, at
t = 0, you sign up 3 separate forward contracts with the same supplier to lock in the fuel
cost for the next 3 years.
The rst forward contract requires the supplier to deliver 100 barrels of oil at t = 1 (end
of Year 1) at $20 per barrel; the second contract requires the supplier to deliver 100 barrels
of oil at t = 2 at $25 per barrel; and the third contract requires the supplier to deliver 100
barrels of oil at $30 per barrel.
Accordingly, youll make 3 separate payments to the supplier. Your rst payment is
100 20 = 2, 000 at t = 1; your second payment is 100 25 = 2, 500 at t = 2; and your third
payment is 100 30 = 3, 000 at t = 3.
As a busy CEO, you quickly realize that signing up 3 separate contracts and keeping
track of 3 separate payments is a bit of work. Why dont I sign one big contract and make
3 level payments to the supplier? you ask. Indeed, combing 3 separate forward contracts
into one and making level payments is a good idea.
In the combined contract, the supplier is required to deliver 100 barrels of oil at $20 per
barrel at t = 1, 100 barrels of oil at $25 per barrel at t = 2, and 100 barrels of oil at $30 per
barrel at t = 3. Accordingly, you make payments of X at = 1, 2, 3 respectively.
Next, lets calculate the level payment X.
55
Derivatives Markets Chapter 8 c Yufeng Guo www.actuary88.com
Your payments BEFORE the 3 forward contracts are combined:
t 0 1 2 3
Payment $2,000 $2,500 $3,000
Your payments AFTER the 3 forward contracts are combined:
t 0 1 2 3
Payment X X X X
Assume that the interest rate is 10% per year. The present value of your total payments
should be the same whether you pay 3 forward contracts separately or you pay level payments
in a combined contract.
2000v + 2500v
2
+ 3000v
3
= X(v +v
2
+v
3
)
v = 1.1
1
X = 2468.28
What if the interest rate is not level? Lets say that the interest rate is 10% per year
during t [0, 1], 12% per year during t [0, 2], and 14% per year during t [0, 3]. No
problem. The present value of your total payments should be the same whether you pay 3
forward contracts separately or you pay level payments in a combined contract.
2000
1.1
+
2500
1.12
2
+
3000
1.14
3
= X

1
1.1
+
1
1.12
2
+
1
1.14
3

X = 24650.84
The combined contract is called a swap. A swap is a series of exchanges between two
parties that takes place in multiple dates in the future. Typically, one party of the swap pays
level cash ows and receives variable cash ows (or receives something of variable values);
the other party receives level cash ows and pays variable cash ows (or delivers something
of variable values). In our example, you (owner of an airline) pay the supplier X at t = 1, 2, 3
respectively; the supplier gives you 100 barrels at t = 1, 2, 3 respectively.
8.1.1 Physical versus nancial settlement
Next, lets talk about how a swap is settled. Use our example and assume the interest rate
is 10% for all years. The swap can be settled if you pay the supplier $2,468.28 at t = 1, 2, 3
respectively and the supplier has you come to its warehouse to pick up 100 barrels of oil at
t = 1, 2, 3 respectively. This is physical settlement.
However, the swap can be settled in a simpler way called nancial settlement. You still
write a check of $2,468.28 at t = 1, 2, 3 respectively. However, instead of giving you 100
barrels of oil at t = 1, 2, 3, the supplier simply gives you the market price of 100 barrels at
FM: Derivatives Markets c Yufeng Guo 56
Derivatives Markets Chapter 8 c Yufeng Guo www.actuary88.com
t = 1, 2, 3. You just go to the market and buy 100 barrels using the money the supplier gives
you. For example, if the market price is $20 per barrel at t = 1, $25 per barrel at t = 2, and
$30 per barrel at t = 3, the supplier gives you a check of $2,000 at t = 1, another check of
$2,500 at t = 2, and the third check of $3,000 at t = 3. You can go to the market place and
purchase 100 barrels of oil at t = 1, 2, 3 yourself.
The settlement can be simplied further. Instead of you writing the supplier a check of
$2,468.28 at t = 1, 2, 3 respectively and the supplier writing you a check of $2,000 at t = 1,
$2,500 at t = 2, and $3,000 at t = 3, you and the supplier can focus on the net payment.
The swap can be settled this way:
You write the supplier a check of 2, 468.28 2, 000 = 468.28 at t = 1
The supplier writes you a check of 2, 500 2, 468.28 = 31.72 at t = 2
The supplier writes you a check of 3, 000 2, 468.28 = 531.72 at t = 3
Typically in a swap the two parties dont write checks to each other back and forth. At each
settlement date, the net payment is made from the party who owes more obligation to the
other party who owes less.
8.1.2 Pricing swaps
Now lets think about how we calculated the price of the level payment from the xed-payer
to the oating-payer. The equation used for solving the xed level payments is:
PV of xed payments = PV of oating payments
This equation says that when the swap contract is signed at t = 0, two parties promise to
exchange equal value of cash ows; nobody wins and nobody loses. The market value of the
swap is zero. The swap is a fair game.
Remember this equation. Whenever you need to calculate the xed payment, use this
equation.
Though the market value of a swap is zero at t = 0, as time passes, the market value of
the swap is generally not zero. Reasons include:
The supply and demand may change. For example, one day after the swap contract
is signed, a war breaks out in a major oil-producing country. Suddenly, theres a
shortage of oil worldwide. The oil price goes up. If this happens, the suppliers cost
of delivering 100 barrels of oil to you at t = 1, 2, 3 will go up. However, your payment
to the supplier at t = 1, 2, 3 is xed at X, which was based the old market condition.
You get a good deal from the swap. If, on the other hand, one day after the swap
contract is signed, theres an over-abundance of oil supply in the market and the oil
price plummets. No matter how low the oil price turns out to be at t = 1, 2, 3, you still
have to pay the supplier the pre-set price X at t = 1, 2, 3. You will suer a loss.
FM: Derivatives Markets c Yufeng Guo 57
Derivatives Markets Chapter 8 c Yufeng Guo www.actuary88.com
The interest rate may change. The xed payment X is calculated according to the
expected yield curve at t = 0. If, after the swap contract is signed, the actual yield
curve is dierent from the yield curve expected at t = 0, the present value of the xed
payments will be dierent from the present value of the oating payments.
Even if the supply/demand and interest rate wont change, the value of the
swap is zero only before the rst swap payment X is made. Once the rst
payment X is made, the present value of the x-payers cash ows for the remaining
duration of the swap is no longer equal to the present value of the oat-payers cash
ows for the remaining duration of the swap.
For example, if the interest rate is 10%, you need to pay the level amount of $2,468.28
at t = 1, 2, 3. If you pay as you go (instead of paying level payments $2,468.28 at
t = 1, 2, 3), you will pay $2,000 at t = 1, $2,500 at t = 2, and $3,000 at t = 3.
Compared with the level payment $2,468.28 at t = 1, 2, 3, you overpay $468.28 at t = 1,
underpay 2, 500 2, 468.28 = 31.72 at t = 2, and underpay 3, 000 2, 468.28 = 531.72
at t = 3. The present value of your overpayment and under-payments should be zero:
468.28v 31.72v
2
531.72v
3
= 0
where v = 1.1
1
If the two parties want to back out from the swap after t = 1, the oating-payer needs
to refund the xed-payer $468.28. If the two parties want to back out from the swap
after t = 2, the xed-payer needs to needs to pay the oating-payer 531.72v = 483.38.
8.2 Interest rate swap
The most common interest rate swap is xed-for-oating swap, commonly referred to as a
plain vanilla swap.
8.2.1 Key features of an interest rate swap
The notional principal is xed at t = 0.
The notional principal is never exchanged. Its a scaling factor to calculate the interest
rate payments.
One party agrees to pay a xed interest rate applied to the notional principal regularly
during the life of the swap. The life of a swap is called the swap term or tenor.
The other party agrees to pay a oating (ie. variable) interest rate applied to the
notional principal regularly during the life of a swap. The oating rate is typically
based on a benchmark rate such as LIBOR.
The oating rate is set in advance, paid in arrears. The oating rate is determined
at the beginning of a settlement period but is paid at the end of the settlement period.
FM: Derivatives Markets c Yufeng Guo 58
Derivatives Markets Chapter 8 c Yufeng Guo www.actuary88.com
8.2.2 Example of a plain vanilla interest rate swap
Consider a hypothetical 5-year swap initiated on 3/1/2006 between Microsoft and Intel.
The details of the swap are as follows:
Swap initiation date: March 1, 2006
Swap tenor: 3 years
Settlement dates: September 1 and March 1
Notional principal: $100 million
Fixed rate payer: Microsoft pays 6.04% per year
Floating rate payer: Intel pays 6-month LIBOR
Cash ows received by Microsoft (in one million dollars):
Time Dates 6-month Intel to Microsoft Microsoft to Intel Net $ to Microsoft
0 3/1/2006 LIBOR
0
= 5.80%
0.5 9/1/2006 LIBOR
0.5
=? $2.9 $3.02 ($0.12)
1 3/1/2007 LIBOR
1
=? ? $3.02
1.5 9/1/2007 LIBOR
1.5
=? ? $3.02
2 3/1/2008 LIBOR
2
=? ? $3.02
2.5 9/1/2008 LIBOR
2.5
=? ? $3.02
3 3/1/2009 ? $3.02
Explanations:
2.9 = 5.8%
180
360
100. This is the 6-month interest on $100 million principal with
LIBOR rate. The 6-month LIBOR rates are simple annual rates. Here we assume 1
year=360 days and 6 months =180 days. In reality, the two parties in the swap can
determine the day counting method for calculating the interest payment. For example,
they can use
ActualDays
365
to calculate the interest payment.
3.02 = 6.04%
180
360
100. This is the 6-month interest on $100 million principal with
6.04% rate. The 6% rate is a simple annual interest rate.
When the swap contract is signed on 3/1/2006, only the LIBOR for the next 6 months
is known. The LIBOR rates beyond 9/1/2006 are unknown.
LIBOR is set in advance and paid in arrears. For example, on 3/1/2006, the LIBOR rate
5.8% is used for calculating Intels payment to Microsoft on 9/1/2006. On 9/1/2006,
the next 6-month LIBOR rate is used for calculating the interest payment on 3/1/2007.
LIBOR rates beyond 3/1/2009 are irrelevant to the swap. The oating interest payment
on the nal settlement date 3/1/2009 is based on LIBOR from 9/1/2008 to 3/1/2009.
FM: Derivatives Markets c Yufeng Guo 59
Derivatives Markets Chapter 8 c Yufeng Guo www.actuary88.com
Suppose the LIBROR rates are known. Then the cash ow diagrams are as follows:
Time Dates 6-month Intel to Microsoft Microsoft to Intel Net $ to Microsoft
0 3/1/2006 5.80%
0.5 9/1/2006 5.90% $2.90 ($3.02) ($0.12)
1 3/1/2007 6.00% $2.95 ($3.02) ($0.07)
1.5 9/1/2007 6.10% $3.00 ($3.02) ($0.02)
2 3/1/2008 6.20% $3.05 ($3.02) $0.03
2.5 9/1/2008 6.30% $3.10 ($3.02) $0.08
3 3/1/2009 $3.15 ($3.02) $0.13
8.2.3 Motivations for an interest swap
Example 1 Interest rate swap changes oating liabilities to xed liabilities or vice versa.
Consider a bank that receives deposits from small investors like you and me and lends
out money in mortgage loans. Here the bank has a mismatch of interest rates. On the one
hand, it receives xed interest rate on mortgages; most new homeowners avoid variable rate
loans and want to lock in a xed interest on their mortgages for the next 15 or 30 years. On
the other hand, the bank pays oating interest rate to individual depositors. We want our
banks to gives us interest rate that reects the market interest rate. If the market interest
is high but we get a low interest rate from a bank, we can deposit our money elsewhere and
earn a higher interest rate.
Suppose the bank pays deposits at a oating rate of LIBOR - 1% but receives mortgage
payments at xed 6%. If LIBOR=7%, the bank will break even. If LIBOR> 7%, the bank
will lose money. To hedge the risk that LIBOR may go up, the bank can enter a swap and
pay xed, and get oat. In the swap, the bank pays xed 6% and receives LIBOR. The
counterparty receives xed 6% and pays LIBOR.
Before the swap, the bank faces the risk that LIBOR may go up and exceed 7%.
After the swap, the bank receives 6% mortgage payments from new homeowners and
passes the payments to the counterparty. The counterparty receives 6% xed and sends
LIBOR to the bank. The bank pays its depositors LIBOR - 1%, earning 1% prot no matter
how high LIBOR may be.
FM: Derivatives Markets c Yufeng Guo 60
Derivatives Markets Chapter 8 c Yufeng Guo www.actuary88.com
LIBOR 1%

Fixed 6%

LIBOR

6%

Depositors
Bank
Counter
Party
New
Homeowners
Similarly, a rm can use interest rate swaps to change its oating assets to xed assets
or vice versa. Consider an rm which holds a oating rate note (FRN). An FRN is a bond
except that the coupon rate is a variable rate (such as LIBOR + 1%) instead of a xed rate
(such as 8%). The rm originally bought an FRN instead of a xed coupon bond because it
forecast that the interest rate would rise. Suppose two years later after purchasing the FRN,
the rm forecast that the interest rate would fall. The rm can use an interest rate swap to
change its oating interest rate payment received from the FRN issuer to a set of xed cash
ows.
Example 2 Interest rate swap reduces a rms borrowing cost (comparative advantage)
First, a few words on comparative advantage. The idea behind comparative advantage is
Do what you the very best, no even your second best. A classic example is that the best
attorney in town happens to be the best typist in down. Since the attorney earns more
income by doing litigation rather than typing, he should work exclusively on litigation and
give up typing. He can hire a secretary to type for him.
Suppose there are two companies. One is a well known company, which can borrow money
at lower rate. The other is a new startup and doesnt have a track record. It has to borrow
money at a higher rate.
Their borrowing rates are as follows:
FM: Derivatives Markets c Yufeng Guo 61
Derivatives Markets Chapter 8 c Yufeng Guo www.actuary88.com
Company Fixed borrowing rate Floating borrowing rate
Mr. Established 10% LIBOR+0.3%
Mr. StartUp 11.20% LIBOR+1%
Mr. StartUp minus Mr. Established xed = 1.2% oat = 0.7%
Comparative advantage xed - oat=0.5%
Mr. Established has absolute advantage in borrowing xed and oating rate debt. Mr.
StartUp, on the other hand, has comparative advantage in borrowing oating rate; it pays
only 0.7% more than Mr. Established in borrowing oating rate debt yet pays 1.2% more
than Mr. Established in borrowing at a xed rate.
Suppose Mr. Established wants to borrow some money at LIBOR and Mr. StartUp wants
to borrow the same amount of money at a xed rate.
Total borrowing cost BEFORE using an interest rate swap: Mr. Established wants
to borrow at a oating rate. So it goes out, nds a lender, and borrows $1 million at a
oating rate of LIBOR+0.3% per year.
Mr. StartUp wants to borrow at a xed rate. So it goes out, nds a lender, and borrows
at 11.2%.
Total borrowing cost: LIBOR + 0.3% + 11.2% = LIBOR + 11.5%
Total borrowing cost AFTER using an interest rate swap: Mr. Established goes out,
nds a lender, and borrow $1 million at a xed rate of 10%. Mr. StartUp goes out, nds a
lender, and borrows $1 million at a oating rate of LIBOR+1%. Then Mr. Established and
Mr. StartUp enter an interest rate swap where Mr. Established pays xed and gets oat and
Mr. StartUp pays oat and gets xed.
Total borrowing cost: 10% +LIBOR + 1% = LIBOR + 11%
By entering a swap, Mr. Established and Mr. StartUp achieve 0.5% saving. Lets assume
that Mr. Established and Mr. StartUp share the 0.5% saving equally (i.e each gains 0.25%
saving).
Mr. Establisheds cash ows are as follows:
Go to a lender and borrow $1M at 10%. Pay 10% interest rate to this lender. So the
annual payment to the lender is 1 10% = 0.1M
Pay Mr. StartUp LIBOR on $1M principal. For example, if LIBRO happens to be 9%
in Year 1, Mr. Established will pay Mr. StartUp 9% 1M=0.09M at the end of Year
1.
Receive 9.95% interest payment on $1M principal (i.e. receive $0.0995M) annual pay-
ment from Mr. StartUp.
Mr. StartUPs cash ows are as follows:
Go to a lender and borrow $1 M at LIBOR+1%.
Pay Mr. Established 9.95% annual interest on $1 M principal (ie. Pay $0.0995M)
Receive LIBOR interest payment on $1M principal from Mr. Established.
FM: Derivatives Markets c Yufeng Guo 62
Derivatives Markets Chapter 8 c Yufeng Guo www.actuary88.com
DIRECT BORROWING

Pay Pay
LIBOR+0.3% $1M 11.2% $1M
on $1M on $1M
principal principal
annually annually

Mr. Established
wants to borrow
$1M and pay
floating interest
rate
Mr. StartUp
wants to borrow
$1M and pay
fixed interest
rate
Mr.
Establishers
Lender
Mr.
StartUps
Lender
The net result of direct borrowing:
Mr. Established gets $1M loan and pays LIBOR+0.3%.
Mr. StartUp gets $1M loan and pays 11.2%.
Total borrowing cost: LIBOR+0.3% + 11.2% = LIBOR+11.5%
INTEREST SWAP WITHOUT A SWAP DEALER Assume Mr. Established and Mr.
StartUp directly negotiate a swap without a swap dealer.
FM: Derivatives Markets c Yufeng Guo 63
Derivatives Markets Chapter 8 c Yufeng Guo www.actuary88.com
pay LIBOR interest on
$1M principal annually

pay 9.95% interest on
$1M principal annually

Pay 10% Pay
interest $1M LIBOR + 1% $1M
on $1M on $1M
principal principal
annually annually

Mr.
Establishers
Lender
Mr.
StartUps
Lender
Mr. Established
wants to borrow
$1M and pay
floating interest
rate
Mr. StartUp
wants to borrow
$1M and pay
fixed interest
rate
Result after the swap:
Mr. Established gets $1M loan. Its annual interest rate for loan repayment on $1M
principal is 10%9.95%+LIBOR = LIBOR+0.05%. Mr. Established borrows $1M
and pays LIBOR + 0.05%. This is 0.25% less direct borrowing rate of LIBOR +0.3%
without a swap.
Mr. StartUp gets $1M loan. Its annual interest rate for loan repayment on $1M
principal is LIBOR+1% + 9.95% - LIBOR =10.95%. This is 0.25% less than direct
borrowing rate of 11.2%.
Total borrowing cost: LIBOR+0.05% + 10.95% = LIBOR+11%
INTEREST SWAP WITH A DEALER Suppose that the dealer receives 9.97% from Mr.
StartUp but passes on only 9.93% to Mr. Established (swap bid/ask spread). The dealer
takes away 9.97% - 9.93%= 0.04% as a compensation for setting up the swap.
FM: Derivatives Markets c Yufeng Guo 64
Derivatives Markets Chapter 8 c Yufeng Guo www.actuary88.com
9.93% 9.97%

LIBOR LIBOR

Pay 10% Pay
interest $1M LIBOR + 1% $1M
on $1M on $1M
principal principal
annually annually

Mr.
Establishers
Lender
Mr.
StartUps
Lender
Mr. Established
wants to borrow
$1M and pay
floating interest
rate
Mr. StartUp
wants to borrow
$1M and pay
fixed interest
rate
Swap
Dealer
The net result after the swap using a swap dealer:
Mr. Established gets $1M loan. Its annual interest rate for loan repayment on $1M
principal is 10% - 9.93% + LIBOR = LIBOR + 0.07%. Mr. Established borrows $1M
and pays LIBOR+0.07%. This is 0.23% less than the direct borrowing rate of LIBOR
+0.3% without a swap.
Mr. StartUp gets $1M loan. Its annual interest rate for loan repayment on $1M
principal is LIBOR+1% + 9.97% - LIBOR =10.97%. This is 0.23% less than the direct
borrowing rate of 11.2%.
Total borrowing cost: LIBOR+0.07% + 10.97% = LIBOR+11.04%
8.2.4 How to price an interest rate swap
Lets walk through the example on Derivatives Markets page 255 Pricing and the swap
counterparty. This example relies on Table 7.1. Make sure you understand Table 7.1.
Excerpt of Table 7.1 from Derivatives Markets
Yrs to maturity Zero-coupon bond yield Zero-coupon bond price 1-Yr implied forward rate
1 6.00% P
0
(0, 1) = 0.943396 r(0, 1) = 0.06
2 6.50% P
0
(0, 2) = 0.881659 r(1, 2) = 0.0700236
3 7.00% P
0
(0, 3) = 0.816298 r(2, 3) = 0.0800705
FM: Derivatives Markets c Yufeng Guo 65
Derivatives Markets Chapter 8 c Yufeng Guo www.actuary88.com
Here the zero-coupon bond prices with maturity of 1 year, 2 years, and 3 years are bond
selling prices in the market. They are the known values. The remaining values are based on
bond price.
Yrs to maturity Zero-coupon bond yield Zero-coupon bond price 1-Yr implied forward rate
1 ? P
0
(0, 1) = 0.943396 r(0, 1) =?
2 ? P
0
(0, 2) = 0.881659 r(1, 2) =?
3 ? P
0
(0, 3) = 0.816298 r(2, 3) =?
Zero-coupon bond yields are calculated as follows. Consider a 1-year zero coupon bond. For
each dollar to be received at t = 1, the buyer has to pay 0.943396 at t = 0. So the bond yield
is calculated as follows:
0.943396(1 +i) = 1
i 6%
Consider a 2-year zero coupon bond. For each dollar to be received at t = 2, the buyer
has to pay 0.881659 at t = 0. So the bond yield is calculated as follows:
0.881659(1 +i)
2
= 1
i 6.5%
Consider a 3-year zero coupon bond. For each dollar to be received at t = 3, the buyer
has to pay 0.816298 at t = 0. So the bond yield is calculated as follows:
0.816298(1 +i)
3
= 1
i 7%
The 1-year implied forward rates are calculated as follows.
Time t 0 1 2 3

( ) 0,1 r ( ) 1, 2 r ( ) 2, 3 r
6%
6.5% per Yr
7% per Yr
FM: Derivatives Markets c Yufeng Guo 66
Derivatives Markets Chapter 8 c Yufeng Guo www.actuary88.com
At t = 0, the 1-year implied forward rate during t [0, 1] is obviously 6%. So r(0, 1) = 6%
At t = 1, the 1-year implied forward rate during t [1, 2] is r(1, 2).
(1 + 6%)[1 +r(1, 2)] = (1 + 6.5%)
2
r(1, 2) = 7.00236%
At t = 2, the 1-year implied forward rate during t [2, 3] is r(2, 3).
(1 + 6.5%)
2
[1 +r(2, 3)] = (1 + 7%)
3
r(2, 3) = 8.00705%
Time t 0 1 2 3

( ) 0,1 r ( ) 1, 2 r ( ) 2, 3 r
6% 7.0024% 8.0071%
6.5% per Yr
7% per Yr
Next, lets determine the price of the swap (ie. nding the constant rate R by which the
swap dealer gets paid by the counterparty). Remember the swap dealer gets xed and pays
oat. And the counterparty (the investor) gets oat and pays xed.
The annual payment by the swap dealer to the counterparty is: R Notional Principal
The annual payment by counterparty to the swap dealer is: LIRBOR Notional Principal
As explained before, when two parties enter an interest rate swap, only the LIBOR rate
during the 1st settlement period is known. The LIBOR rates beyond the rst settlement is
not known. Then how can we determine the oating payments if we know only the LIBOR
rate for the rst settlement period?
When pricing a swap, we can create various LIBOR interest rates using forward interest
rate agreements. For the non-arbitrage principal to hold, the LIBOR rates must be close to
the 1-year forward interest rate extracted from the forward interest rate agreements.
FM: Derivatives Markets c Yufeng Guo 67
Derivatives Markets Chapter 8 c Yufeng Guo www.actuary88.com
Time t 0 1 2 3

( ) 0,1 r ( ) 1, 2 r ( ) 2, 3 r
6% 7.0024% 8.0071%
6.5% per Yr
7% per Yr
Payment to the R R R
swap dealer

Pay by the dealer 6% 7.0024% 8.0071%
to the counterparty

Assuming the notional principal is $1.
Set up the equation: PV of xed payments = PV of oating payments. Set the notional
principal to $1. We have:
R

1
1.06
+
1
1.065
2
+
1
1.07
3

=
6%
1.06
+
7.0024%
1.065
2
+
8.0071%
1.07
3
R =
6%
1.06
+
7.0024%
1.065
2
+
8.0071%
1.07
3
1
1.06
+
1
1.065
2
+
1
1.07
3
R =
0.943396 6% + 0.881659 7.0024% + 0.816298 8.0071%
0.943396 + 0.881659 + 0.816298
R =
P
0
(0, 1)r(0, 1) +P
0
(0, 2)r(1, 2) +P
0
(0, 3)r(2, 3)
P
0
(0, 1) +P
0
(0, 2) +P
0
(0, 3)
=

n=3
i=1
P
0
(0, t
i
)r(t
i1
, t
i
)

n=3
i=1
P
0
(0, t
i
)
= 6.9543%
The general formula is:
R =

n
i=1
P
0
(0, t
i
)r(t
i1
, t
i
)

n
i=1
P
0
(0, t
i
)
Here n is the total number of settlements (ie. the number of times when cash ows change
hand between two parties in a swap); t
i
represents the time the i-th settlement takes place.
P
0
(0, t
i
) is the present value of $1 at t
i
discounted to t = 0. It is also the price of a
zero-coupon bond maturing on the date t
i
. r(t
i1
, t
i
) is the implied forward rate during
t [t
i1
, t
i
].
Next, lets use a shortcut to calculate the present value of the oating payments. The
oating payments in an interest rate swap are very similar to the cash ows in a oating rate
FM: Derivatives Markets c Yufeng Guo 68
Derivatives Markets Chapter 8 c Yufeng Guo www.actuary88.com
bond. In a oating rate bond, the coupon rates are oating (instead of being xed like in a
typical bond). The oating coupon rates are set in advance and paid in arrears.
Consider a oating rate bond. Its annual coupon rates for Year 1, Year 2, and Year 3 are
6%, 7.0024%, and 8.0071% respectively. Suppose the face amount of this oating rate bond
is $1. Then the bond holder will get $0.06 at t = 1, $0.070024 at t = 2, and $1.080071 at
t = 3.
Time t 0 1 2 3

( ) 0,1 r ( ) 1, 2 r ( ) 2, 3 r
6% 7.0024% 8.0071%
6.5% per Yr
7% per Yr
Cash flow received $6% $7.0024% $8.0071%+$1
by the bondholder =$0.06 =$0.070024 =$1.080071

Assuming the face amount of the floating rate note is $1.

Whats the present value of this oating rate bond? Surprisingly, the present value is the
face amount $1. To see why, notice that if you discount $1.080071 from t = 3 to t = 2, youll
get $1:
1.080071
1 + 8.0071%
= 1
This $1 is combined with the second oating payment $0.070024, becoming $1.070024. If
we discount this cash ow from t = 2 to t = 1, once again we get $1:
1.070024
1 + 7.0024%
= 1
This $1 is combined with the 1st oating payment $0.06, becoming $1.06. If we discount
this cash ow from t = 1 to t = 0, once again we get $1:
1.06
1 + 6%
= 1
Here is another way to see why the present value of the oating note is its face amount.
The present value of our oating note is:
6%
1 + 6%
+
7.0024%
(1 + 6.5%)
2
+
1 + 8.0071%
(1 + 7%)
3
Notice that
(1 + 7%)
3
= (1 + 6%) (1 + 7.0024%) (1 + 8.0071%)
FM: Derivatives Markets c Yufeng Guo 69
Derivatives Markets Chapter 8 c Yufeng Guo www.actuary88.com
(1 + 6.5%)
2
= (1 + 6%) (1 + 7.0024%)
Then you can verify for yourself that
6%
1 + 6%
+
7.0024%
(1 + 6.5%)
2
+
1 + 8.0071%
(1 + 7%)
3
= 1
Key point to remember: The present value of a oating rate bond is its face
amount.
Next, were going to quickly calculate the present value of the oating payments using the
present value formula for a oating rate bond. At the nal settlement time t = 3, suppose
the oating-payer (the swap dealer in this case) gives the xed-payer $1 and immediately
gets $1 back from the xed payer. The cash ow diagram is:
Time t 0 1 2 3

( ) 0,1 r ( ) 1, 2 r ( ) 2, 3 r
6% 7.0024% 8.0071%
6.5% per Yr
7% per Yr
Payment to the R R R
swap dealer

Pay by the dealer $6% $7.0024% $(1+8.0071%) - $1
to the counterparty

Assuming the notional principal is $1.
The equation to solve for R is: PV xed payments=PV oating payments.
R

1
1.06
+
1
1.065
2
+
1
1.07
3

=
6%
1.06
+
7.0024%
1.065
2
+
1 + 8.0071%
1.07
3

1
1.07
3
The present value of a oating rate bond is its face amount:
6%
1.06
+
7.0024%
1.065
2
+
1 + 8.0071%
1.07
3
= 1
This gives us:
R

1
1.06
+
1
1.065
2
+
1
1.07
3

= 1
1
1.07
3
R =
1
1
1.07
3
1
1.06
+
1
1.065
2
+
1
1.07
3
=
1 P
0
(0, 3)
P
0
(0, 1) +P
0
(0, 2) +P
0
(0, 3)
=
1 P
0
(0, 3)

n=3
i=1
P
0
(0, t
i
)
The general formula is:
FM: Derivatives Markets c Yufeng Guo 70
Derivatives Markets Chapter 8 c Yufeng Guo www.actuary88.com
R =
1 P
0
(0, t
n
)

n
i=1
P
0
(0, t
n
)
=

n
i=1
P
0
(0, t
i
)r(t
i1
, t
i
)

n
i=1
P
0
(0, t
i
)
Make sure you understand the symbols in this formula. If you nd it hard to memorize this
formula, dont worry. You can just use the following procedure to calculate the xed rate R
in an interest rate swap:
1. Identify the LIBOR yield curve.
2. Identify xed and oating payments.
3. Set up the equation that PV of xed payments =PV of oating payments
8.2.5 The swap curve
To understand this section, youll need to read Section 5.7. The most important part of
Section 5.7 is Equation 5.19. Though Section 5.7 is excluded from the syllabus, I recommend
that you read it any way to help you understand the swap curve.
In addition, youll need to know whats Eurodollars. Here is some background on Eurodol-
lars. Eurodollars is not the joint European currency, the Euro. Eurodollars are simply US
dollars deposited in commercial banks outside the United States, and thus are not under
the jurisdiction of the Federal Reserve. As a result, such deposits are subject to much less
regulation than similar deposits within the United States, allowing for higher margins.
Historically, such deposits were held mostly by European banks and nancial institutions,
and hence were called Eurodollars. Such deposits are now available in many countries
worldwide. However, they continue to be called as Eurodollars regardless of the location.
Eurodollar interest rate is the interest rate earned on Eurodollars deposited by one bank
with another bank. Three-month Eurodollar futures contracts are futures contracts on three-
month Eurodollar interest rate.
Make sure you know how to read the quote of the Eurodollar futures contract. If F is
a Eurodollars futures quote, then (100 F)% is the Eurodollars futures interest rate for a
3-month period. This formula is explained in Derivatives Markets Section 5.7, which says
that the Eurodollars futures price at expiration of the contract is:
100 Annualized 3-month LIBOR
Now you see that if the annualized 3-month LIBOR rate goes up, the quote F goes down;
if the annualized 3-month LIBOR rate goes down, the quote F goes up.
For example, a Eurodollars futures price 95.53 corresponds to (100 95.53)% = 4.47%
annualized three-month interest rate or
4.47%
4
= 1.1175% interest rate for a 3-month period
on $1,000,000 loan. The interest due on $1,000,000 loan during the 3-month borrowing period
is 1.1175%$1, 000, 000 = $11, 175.
Now you shouldnt have trouble understanding Equation 5.19:
r
91
= (100 F)
1
100

1
4

91
90
FM: Derivatives Markets c Yufeng Guo 71
Derivatives Markets Chapter 8 c Yufeng Guo www.actuary88.com
This formula can be rewritten as:
r
91
=
(100 F)%
4

91
90
Here
(100F)%
4
is the un-annualized or the actual interest rate per 3-month period. The
factor
91
90
scales the 90-day actual interest rate into a 91-day actual interest rate.
Now you should be able to follow the textbook and understand Table 8.4. For example,
the 0.0037% interest rate is calculated as follows:
r
91
=
(100 F)%
4

91
90
=
(100 98.555)%
4

91
90
= 0.37%
One not obvious point to know is that the 3-month LIBOR rate starts when the Eurodol-
lars futures contract expires. In other words, you enter into a Eurodollars futures contract
to lock in the next 3-month LIBOR rate. This is why in Table 8.4 the maturity date is t
i
(Column 1 in Table 5.4) and the implied quarterly rate is r(t
i
, r
i+1
).
The swap spread represents the credit risk in the swap relative to the corresponding risk-
free Treasury yield. It is the price tag on the risk that one of the parties to the swap will fail
to make a payment.
8.2.6 Swaps implicit loan balance
At its inception, an interest rate swap has zero value to both parties. However, as time
passes, the market value of the swap may no longer be zero.
8.2.7 Deferred swaps
The equation for solving the xed rate R in a deferred swap is still
PV of xed payments = PV of oating payments.
The formula is:
R =
P
0
(0, t
k1
) P
0
(0, t
n
)

n
i=k
P
0
(0, t
i
)
=

T
i=k
P
0
(0, t
i
)r(t
i1
, t
i
)

n
i=1
P
0
(0, t
i
)
8.2.8 Why swap interest rates?
The main idea of this section is that by using an interest rate swap a rm can lower its
borrowing cost.
Firms like to borrow short-term loans; borrowing a loan for a short period of time is
generally less risky and cheaper (i.e paying lower interest rate) than borrowing the same
amount of money for a longer period. However, its hard for rms to borrow lot of money on
the short-term basis. Lenders still worry that the borrower wont pay back the loan, even if
the loan is a short term loan.
In an interest rate swap, the notional principal doesnt change hands and theres no risk
that any party to the swap may default the principal. As a result, an interest rate swap has
lower credit risk than a short term loan. By entering an interest rate swap, a rm can borrow
at the short term interest rate.
FM: Derivatives Markets c Yufeng Guo 72
Derivatives Markets Chapter 8 c Yufeng Guo www.actuary88.com
8.2.9 Amortizing and accrediting swaps
If the notional principal in an interest rate swap decreases over time, the swap is called an
amortizing swap. If the notional principal increases over time, the interest rate swap is called
an accrediting swap.
When the notional principal is not a constant, then the equation for solving the xed rate
R becomes:
R =

n
i=1
Q
t
i
P
0
(0, t
i
)r(t
i1
, t
i
)

n
i=1
Q
t
i
P
0
(0, t
i
)
where Q
t
i
is notional principal at time t
i
.

FM: Derivatives Markets c Yufeng Guo 73


Solution to Derivatives Markets: for Exam FM
Yufeng Guo
www.actuary88.com c _Yufeng Guo ii
Contents
Introduction vii
1 Introduction to derivatives 1
2 Introduction to forwards and options 7
3 Insurance, collars, and other strategies 29
4 Introduction to risk management 79
5 Financial forwards and futures 129
8 Swaps 141
iii
CONTENTS CONTENTS
www.actuary88.com c _Yufeng Guo iv
Preface
This is Guos solution to Derivatives Markets (2nd edition ISBN 0-321-28030-
X) for Exam FM. Unlike the ocial solution manual published by Addison-
Wesley, this solution manual provides solutions to both the even-numbered and
odd-numbered problems for the chapters that are on the Exam FM syllabus.
Problems that are out of the scope of the FM syllabus are excluded.
Please report any errors to yufeng.guo.actuary@gmail.com.
This book is the exclusive property of Yufeng Guo. Redistribution of this
book in any form is prohibited.
v
PREFACE PREFACE
www.actuary88.com c _Yufeng Guo vi
Introduction
Recommendations on using this solution manual:
1. Obviously, youll need to buy Derivatives Markets (2nd edition) to see the
problems.
2. Make sure you download the textbook errata from http://www.kellogg.
northwestern.edu/faculty/mcdonald/htm/typos2e_01.html
vii
CHAPTER 0. INTRODUCTION
www.actuary88.com c _Yufeng Guo viii
Chapter 1
Introduction to derivatives
Problem 1.1.
Derivatives on weather are not as farfetched as it might appear. Visit http:
//www.cme.com/trading/ and youll nd more than a dozen weather derivatives
currently traded at CME such as "CME U.S. Monthly Weather Heating Degree
Day Futures" and "CME U.S. Monthly Weather Cooling Degree Day Futures."
a. Soft drink sales greatly depend on weather. Generally, warm weather
boosts soft drink sales and cold weather reduces sales. A soft drink producer
can use weather futures contracts to reduce the revenue swing caused by weather
and smooth its earnings. Shareholders of a company generally want the earnings
to be steady. They dont want the management to use weather as an excuse for
poor earnings or wild uctuations of earnings.
b. The recreational skiing industry greatly dependents on weather. A ski
resort can lose money due to warm temperatures, bitterly cold temperatures, no
snow, too little snow, or too much snow. A resort can use weather derivatives
to reduce its revenue risk.
c. Extremely hot or cold weather will result in greater demand for electricity.
An electric utility company faces the risk that it may have to buy electricity at
a higher spot price.
d. Fewer people will visit an amusement park under extreme weather con-
ditions. An amusement park can use weather derivatives to manage its revenue
risk.
How can we buy or sell weather? No one can accurately predict weather. No
one can deliver weather. For people to trade on weather derivatives, weather
indexes need to be invented and agreed upon. Once we have weather indexes,
we can link the payo of a weather derivative to a weather index. For more
information on weather derivatives, visit:
http://hometown.aol.com/gml1000/wrms.htm
http://www.investopedia.com
1
CHAPTER 1. INTRODUCTION TO DERIVATIVES
Problem 1.2.
Anyone (such as speculators and investors) who wants to earn a prot can
enter weather futures. If you can better predict a weather index than does
the market maker, you can enter weather futures and make a prot. Of
course, its hard to predict a weather index and hence loss may occur.
Two companies with opposite risks may enter weather futures as counter
parties. For example, a soft drink company and a ski-resort operator
have opposite hedging needs and can enter a futures contract. The soft
drink company can have a positive payo if the weather is too cold and
a negative payo if warm. This way, when the weather is too cold, the
soft drink company can use the gain from the weather futures to oset its
loss in sales. Since the soft drink company makes good money when the
weather is warm, it doesnt mind a negative payo when the weather is
cold. On the other hand, the ski resort can have a negative payo if the
weather is too cold and a positive payo if too warm. The ski resort can
use the gain from the futures to oset its loss in sales.
Problem 1.3.
a. 100 41:05 + 20 = 4125
b. 100 40:95 20 = 4075
c. For each stock, you buy at $41:05 and sell it an instant later for $40:95.
The total loss due to the ask-bid spread: 100 (41:05 40:95) = 10. In addition,
you pay $20 twice. Your total transaction cost is 100 (41:05 40:95) +2 (20) =
50
Problem 1.4.
a. 100 41:05 + 100 41:05 0:003 = 4117: 315
b. 100 40:95 100 40:95 0:003 = 4082: 715
c. For each stock, you buy at $41:05 and sell it an instant later for $40:95.
The total loss due to the ask-bid spread: 100 (41:05 40:95) = 10. In addition,
your pay commission 100 41:05 0:003 + 100 40:95 0:003 = 24: 6. Your
total transaction cost is 10 + 24: 6 = 34: 6
Problem 1.5.
The market maker buys a security for $100 and sells it for $100:12. If the
market maker buys 100 securities and immediately sells them, his prot is
100 (100:12 100) = 12
Problem 1.6.
www.actuary88.com c _Yufeng Guo 2
CHAPTER 1. INTRODUCTION TO DERIVATIVES
Your sales proceeds: 300 (30:19) 300 (30:19) (0:005) = 9011: 715
Your cost of buying 300 shares from the market to close your short position
is:
300 (29:87) + 300 (29:87) (0:005) = 9005: 805
Your prot: 9011: 715 9005: 805 = 5: 91
Problem 1.7.
a. Consider the bid-ask spread but ignore commission and interest.
Your sales proceeds: 400 (25:12) = 10048
Your cost of buying back: 400 (23:06) = 9224
Your prot: 10048 9224 = 824
b. If the bid-ask spread and 0.3% commission
Your sales proceeds: 400 (25:12) 400 (25:12) (0:003) = 10017: 856
Your cost of buying back: 400 (23:06) + 400 (23:06) (0:003) = 9251: 672
Your prot: 10017: 856 9251: 672 = 766: 184
Prot drops by: 824 766: 184 = 57: 816
c. Your sales proceeds stay in your margin account, serving as a collateral.
Since you earn zero interest on the collateral, your lost interest is
If ignore commission: 10048 (0:03) = 301: 44
If consider commission: 10017: 856 (0:03) = 300: 54
Problem 1.8.
By signing the agreement, you allow your broker to act as a bank, who lends
your stocks to someone else and possibly earns interest on the lent stocks.
Short sellers typically leave the short sale proceeds on deposit with the bro-
ker, along with additional capital called a haircut. The short sale proceeds and
the haircut serve as a collateral. The short seller earns interest on this collateral.
This interest is called the short rebate in the stock market.
The rebate rate is often equal to the prevailing market interest rate. How-
ever, if a stock is scarce, the broker will pay far less than the prevailing interest
rate, in which case the broker earns the dierence between the short rate and
the prevailing interest rate.
This arrangement makes short selling easy. Also short selling can be used to
hedge nancial risks, which is good for the economy. By the way, you are not
hurt in any way by allowing your broker to lend your shares to short sellers.
Problem 1.9.
According to http://www.investorwords.com, the ex-dividend date was
created to allow all pending transactions to be completed before the record date.
If an investor does not own the stock before the ex-dividend date, he or she will
www.actuary88.com c _Yufeng Guo 3
CHAPTER 1. INTRODUCTION TO DERIVATIVES
be ineligible for the dividend payout. Further, for all pending transactions that
have not been completed by the ex-dividend date, the exchanges automatically
reduce the price of the stock by the amount of the dividend. This is done
because a dividend payout automatically reduces the value of the company (it
comes from the companys cash reserves), and the investor would have to absorb
that reduction in value (because neither the buyer nor the seller are eligible for
the dividend).
If you borrow stock to make a short sale, youll need to pay the lender the
dividend distributed while you maintain your short position. According to the
IRS, you can deduct these payments on your tax return only if you hold the
short sale open for a minimum period (such as 46 days) and you itemize your
deductions.
In a perfect market, if a stock pays $5 dividend, after the ex-dividend date,
the stock price will be reduced by $5. Then you could buy back stocks from
the market at a reduced price to close your short position. So you dont need
to worry whether the dividend is $3 or $5.
However, in the real world, a big increase in the dividend is a sign that
a company is doing better than expected. If a company pays a $5 dividend
instead of the expected $3 dividend, the companys stock price may go up after
the announcement that more dividend will be paid. If the stock price goes up,
you have to buy back stocks at a higher price to close your short position. So
an unexpected increase of the dividend may hurt you.
In addition, if a higher dividend is distributed, you need to pay the lender
the dividend while you maintain your short position. This requires you to have
more capital on hand.
In the real world, as a short seller, you need to watch out for unexpected
increases of dividend payout.
Problem 1.10.
http://www.investopedia.com/articles/01/082201.asp oers a good ex-
planation of short interest:
Short Interest
Short interest is the total number of shares of a particular stock that have
been sold short by investors but have not yet been covered or closed out. This
can be expressed as a number or as a percentage.
When expressed as a percentage, short interest is the number of shorted
shares divided by the number of shares outstanding. For example, a stock with
1.5 million shares sold short and 10 million shares outstanding has a short in-
terest of 15% (1.5 million/10 million = 15%).
Most stock exchanges track the short interest in each stock and issue reports
at months end. These reports are great because by showing what short sellers
are doing, they allow investors to gauge overall market sentiment surrounding
a particular stock. Or alternatively, most exchanges provide an online tool to
calculate short interest for a particular security.
www.actuary88.com c _Yufeng Guo 4
CHAPTER 1. INTRODUCTION TO DERIVATIVES
Reading Short Interest
A large increase or decrease in a stocks short interest from the previous
month can be a very telling indicator of investor sentiment. Lets say that
Microsofts (MSFT) short interest increased by 10% in one month. This means
that there was a 10% increase in the amount of people who believe the stock will
decrease. Such a signicant shift provides good cause for us to nd out more.
We would need to check the current research and any recent news reports to
see what is happening with the company and why more investors are selling its
stock.
A high short-interest stock should be approached for buying with extreme
caution but not necessarily avoided at all costs. Short sellers (like all investors)
arent perfect and have been known to be wrong from time to time.
In fact, many contrarian investors use short interest as a tool to determine
the direction of the market. The rationale is that if everyone is selling, then
the stock is already at its low and can only move up. Thus, contrarians feel
that a high short-interest ratio (which we will discuss below) is bullish - because
eventually there will be signicant upward pressure on the stocks price as short
sellers cover their short positions (i.e. buy back the stocks they borrowed to
return to the lender).
The more likely that investors can speculate on the stock, the higher the
demand for the stock and the higher the short interest.
A broker can short sell more than his existing inventory. For example, if a
broker has 500 shares of IBM stocks, he can short sell 600 shares of IBM stocks
as long as he knows where to nd the additional 100 shares of IBM stocks. If
all the brokers simultaneously lend out more than what they have in their stock
inventories, then the number of stocks sold short might exceed the total number
of the stocks outstanding.
NASDAQ short interest is available by issue for a rolling twelve months
and is based on a mid-month settlement date. For more information, visit
http://www.nasdaqtrader.com/asp/short_interest.asp.
Problem 1.11.
You go to a bank. The bank uses its customers deposits and lends you an
asset worth $100. Then 90 days later you buy back the asset at $102 from the
open market (i.e. you come up with $102 from whatever sources) and return
$102 to the bank. Now your short position is closed.
Problem 1.12.
We need to borrow an asset called money from a bank (the asset owner) to
pay for a new house. The asset owner faces credit risk (the risk that we may
not be able to repay the loan). To protect itself, the bank needs collateral.
The house is collateral. If we dont pay back our loan, the bank can foreclose
the house.
www.actuary88.com c _Yufeng Guo 5
CHAPTER 1. INTRODUCTION TO DERIVATIVES
To protect against the credit risk, the bank requires a haircut (i.e. requires
that the collateral is greater than the loan). Typically, a bank lends only 80%
of the purchase price of the house, requiring the borrower to pay a 20% down
payment.
www.actuary88.com c _Yufeng Guo 6
Chapter 2
Introduction to forwards
and options
Problem 2.1.
Long a stock=Own a stock (or buy a stock).
If you own a stock, your payo at any time is the stocks market price because
you can sell it any time at the market price. Let S represent the stock price at
T = 1.
Your payo at T = 1 is S.
Your prot at T = 1 is:
Payo - FV(initial investment)= S 50 (1:1) = S 55.
You can see that the prot is zero when the stock price S = 55. Alternatively,
set S 55 = 0 S = 55.
7
CHAPTER 2. INTRODUCTION TO FORWARDS AND OPTIONS
Payo=S Prot=S 55
10 20 30 40 50 60 70 80
-40
-20
0
20
40
60
80
Stock Price
Payoff
Profit
Payo and prot: Long one stock
www.actuary88.com c _Yufeng Guo 8
CHAPTER 2. INTRODUCTION TO FORWARDS AND OPTIONS
Problem 2.2.
Short a stock=Short sell a stock. If you short sell a stock, your payo at
any time after the short sale is the negative of the stocks market price. This
is because to close your short position youll need to buy the stock back at the
market price and return it to the broker. Your payo at T = 1 is S. Your
prot at T = 1 is: Payo - FV(initial investment)= S + 50 (1:1) = 55 S
You can see that the prot is zero when the stock price S = 55. Alternatively,
set 55 S = 0 S = 55.
10 20 30 40 50 60
-60
-50
-40
-30
-20
-10
0
10
20
30
40
50
Stock Price
Payoff or Profit
Profit
Payoff
Payo and prot: Short one stock
www.actuary88.com c _Yufeng Guo 9
CHAPTER 2. INTRODUCTION TO FORWARDS AND OPTIONS
Problem 2.3.
The opposite of a purchased call is written call (or sold call).
The opposite of a purchased put is written put (or sold put).
The main idea of this problem is:
The opposite of a purchased call ,= a purchased put
The opposite of a purchased put ,= a purchased call
Problem 2.4.
a. Long forward means being a buyer in a forward contract.
Payo of a buyer in a forward at T is
Payoff = S
T
F = S
T
50
S
T
Payoff = S
T
50
40 10
45 5
50 0
55 5
60 10
b. Payo of a long call (i.e. owning a call) at expiration T is:
Payoff = max (0; S
T
K) = max (0; S
T
50)
S
T
Payoff = max (0; S
T
50)
40 0
45 0
50 0
55 5
60 10
c. A call option is a privilege. You exercise a call and buy the stock only if
your payo is positive.
In contrast, a forward is an obligation. You need to buy the stock even if
your payo is negative.
A privilege is better than an obligation.
Consequently, a long call is more expensive than a long forward on the same
underlying stock with the same time to expiration.
www.actuary88.com c _Yufeng Guo 10
CHAPTER 2. INTRODUCTION TO FORWARDS AND OPTIONS
Problem 2.5.
a. Short forward = Enter into a forward as a seller
Payo of a seller in a forward at T is
Payoff = F S
T
= 50 S
T
S
T
Payoff = 50 S
T
40 10
45 5
50 0
55 5
60 10
b. Payo of a long put (i.e. owning a put) at expiration T is:
Payoff = max (0; K S
T
) = max (0; 50 S
T
)
S
T
Payoff = max (0; 50 S
T
)
40 10
45 5
50 0
55 0
60 0
c. A put option is a privilege. You exercise a put and sell the stock only if
your payo is positive.
In contrast, a forward is an obligation. You need to sell the stock even if
your payo is negative.
A privilege is better than an obligation.
Consequently, a long put is more expensive than a short forward on the same
underlying stock with the same time to expiration.
www.actuary88.com c _Yufeng Guo 11
CHAPTER 2. INTRODUCTION TO FORWARDS AND OPTIONS
Problem 2.6.
91 (1 +r) = 100 r = 0:0989
The eective annual interest rate is 9.89%.
If you buy the bond at t = 0, your payo at t = 1 is 100
Your prot at t = 1 is 100 91 (1 + 0:0989) = 0 regardless of the stock price
at t = 1.
If you buy a bond, you just earn the risk-free interest rate. Beyond this,
your prot is zero.
0 10 20 30 40 50 60
99.0
99.5
100.0
100.5
101.0
Stock Price
Payoff
Payo: Long a bond
www.actuary88.com c _Yufeng Guo 12
CHAPTER 2. INTRODUCTION TO FORWARDS AND OPTIONS
10 20 30 40 50 60 70 80 90 100
-1.0
-0.8
-0.6
-0.4
-0.2
0.0
0.2
0.4
0.6
0.8
1.0
Stock Price
Profit
Prot of longing a bond is zero.
www.actuary88.com c _Yufeng Guo 13
CHAPTER 2. INTRODUCTION TO FORWARDS AND OPTIONS
If you sell the bond at t = 0, your payo at t = 1 is 100 (you need to pay
the bond holder 100).
Your prot at t = 1 is 91 (1 + 0:0989) 100 = 0 regardless of the stock price
at t = 1
If you sell a bond, you just earn the risk-free interest rate. Beyond this, your
prot is zero.
0 10 20 30 40 50 60
-101.0
-100.5
-100.0
-99.5
-99.0
Stock Price
Payoff
Payo: Shorting a bond
www.actuary88.com c _Yufeng Guo 14
CHAPTER 2. INTRODUCTION TO FORWARDS AND OPTIONS
10 20 30 40 50 60 70 80 90 100
-1.0
-0.8
-0.6
-0.4
-0.2
0.0
0.2
0.4
0.6
0.8
1.0
Stock Price
Profit
Prot of shorting a bond is zero.
www.actuary88.com c _Yufeng Guo 15
CHAPTER 2. INTRODUCTION TO FORWARDS AND OPTIONS
Problem 2.7.
a. It costs nothing for one to enter a forward contract. Hence the payo of
a forward is equal to the prot.
Suppose we long a forward (i.e. we are the buyer in the forward). Our payo
and prot at expiration is:
S
T
F = S
T
55
10 20 30 40 50 60 70 80 90 100
-50
-40
-30
-20
-10
0
10
20
30
40
Stock Price
Payoff (Profit)
Payo (and prot) of a long forward
www.actuary88.com c _Yufeng Guo 16
CHAPTER 2. INTRODUCTION TO FORWARDS AND OPTIONS
Suppose we short a forward (i.e. we are the seller in the forward), our payo
and prot at expiration is:
F S
T
= 55 S
T
10 20 30 40 50 60 70 80 90 100
-40
-30
-20
-10
0
10
20
30
40
50
Stock Price
Payoff (Profit)
Payo (and prot) of a short forward
www.actuary88.com c _Yufeng Guo 17
CHAPTER 2. INTRODUCTION TO FORWARDS AND OPTIONS
b. If the stock doesnt pay dividend, buying a stock outright at t = 0 and
getting a stock at T = 1 through a forward are identical. Theres no benet to
owning a stock early.
c. Suppose the stock pays dividend before the forward expiration date T = 1.
Please note that if you own a stock prior to the dividend date, you will receive
the dividend. In contrast, if you are a buyer in a forward contract, at T = 1,
youll get a stock but you wont receive any dividend.
If the stock is expected to pay dividend, then the stock price is expected
to drop after the dividend is paid. The forward price agreed upon at t = 0
already considers that a dividend is paid during (0; T); the dividend will
reduce the forward rate. Theres no advantage to buying a stock outright
over buying a stock through a forward. Otherwise, there will be arbitrage
opportunities.
If the stock is not expected to pay dividend but actually pays dividend
(a surprise dividend), then the forward price F agreed upon at t = 0 was
set without knowing the surprise dividend. So F is the forward price on
a non-dividend paying stock. Since dividend reduces the value of a stock,
F is higher than the forward price on an otherwise identical but dividend-
paying stock. If you own a stock at t = 0, youll receive the windfall
dividend. If you buy a stock through a forward, youll pay F, which
is higher than the forward price on an otherwise identical but dividend-
paying stock. Hence owning a stock outright is more benecial than buying
a stock through a forward.
Problem 2.8.
r =risk free interest rate
Under the no-arbitrage principle, you get the same prot whether you buy
a stock outright or through a forward.
Prot at T = 1 if you buy a stock at t = 0 is: S
T
50 (1 +r)
Prot at T = 1 if you buy a stock through a forward: S
T
53
S
T
50 (1 +r) = S
T
53 50 (1 +r) = 53 r = 0:06
www.actuary88.com c _Yufeng Guo 18
CHAPTER 2. INTRODUCTION TO FORWARDS AND OPTIONS
Problem 2.9.
a. Price of an index forward contract expiring in one year is:
F
Index
= 1000 (1:1) = 1100
To see why: If the seller borrows 1000 at t = 0, buys an index, and holds it
for one year, then hell have one stock to deliver at T = 1. The sellers cost is
1000 (1:1) = 1100. To avoid arbitrage, the forward price must be 1100.
Prot at T = 1 of owning a forward on an index:
S
T
F
Index
= S
T
1100
If you buy an index at t = 0, your prot at T = 1 is S
T
1000 (1:1) =
S
T
1100
So you get the same prot whether you buy the index outright or buy the
index through a forward. This should make sense. If owning a stock outright
and buying it through a forward have dierent prots, arbitrage opportunities
exist.
b. The forward price 1200 is greater than the fair forward price 1100. No
rational person will want to enter such an unfair forward contract. Thus the
seller needs to pay the buyer an up-front premium to incite the buyer. The
buyer in the forward needs to receive
1200 1100
1:1
= 90: 91 at t = 0 to make
the forward contract fair. Of course, the buyer needs to pay the forward price
1200 at T = 1.
c. Now the forward price 1000 is lower than the fair forward price 1100.
You can imagine thousands of bargain hunters are waiting in line to enter this
forward contract. If you want to enter the forward contract, you have to pay the
seller a premium in the amount of
1100 1000
1:1
= 90: 91 at t = 0. In addition,
youll need to pay the forward price 1000 at T = 1:
Problem 2.10.
a. Prot=max (0; S
T
1000) 95:68
Set prot to zero:
max (0; S
T
1000) 95:68 = 0
S
T
1000 95:68 = 0 S
T
= 1000 + 95:68 = 1095: 68
b. The prot of a long forward (i.e. being a buyer in a forward): S
T
1020
S
T
1020 = max (0; S
T
1000) 95:68
If S
T
> 1000, theres no solution
If S
T
_ 1000:
S
T
1020 = 0 95:68 S
T
= 1020 95:68 = 924: 32
www.actuary88.com c _Yufeng Guo 19
CHAPTER 2. INTRODUCTION TO FORWARDS AND OPTIONS
Problem 2.11.
a. Prot of a long (i.e. owning) put is max (0; 1000 S
T
) 75:68
max (0; 1000 S
T
) 75:68 = 0
1000 S
T
75:68 = 0 S
T
= 924: 32
b. Prot of a short forward (i.e. being a seller in a forward) is 1020 S
T
max (0; 1000 S
T
) 75:68 = 1020 S
T
If 1000 _ S
T
1000 S 75:68 = 1020 S no solution
If 1000 _ S
T
75:68 = 1020 S
T
S
T
= 1020 + 75:68 = 1095: 68
www.actuary88.com c _Yufeng Guo 20
CHAPTER 2. INTRODUCTION TO FORWARDS AND OPTIONS
Problem 2.12.
Table 2.4 is:
Position Maximum Loss Maximum Gain
Long forward (buyer in forward) -Forward price Unlimited
Short forward (seller in forward) Unlimited Forward Price
Long call (own a call) -FV (premium) Unlimited
Short call (sell a call) Unlimited FV(premium)
Long put (own a put) -FV(premium) Strike Price - FV(premium)
Short put (sell a put) PV(premium)-Strike Price FV(premium)
If you are a buyer in a forward, the worst that can happen to you is S
T
= 0
(i.e. stock price at T is zero). If this happens, you still have to pay the
forward price F at T to buy the stock which is worth zero. Youll lose F.
Your best case is S
T
= , where you have an unlimited gain.
If you are a seller in a forward, the worst case is that S
T
= ; youll incur
unlimited loss. Your best case is that S
T
= 0, in which case you sell a
worthless asset for the forward price F.
If you buy a call, your worst case is S
T
< K, where K is the strike price. If
this happens, you just let the call expire worthless. Youll lose the future
value of your premium (if you didnt buy the call and deposit your money
in a bank account, you could earn the future value of your deposit). Your
best case is that S
T
= , where youll have an unlimited gain.
If you sell a call, your worst case is S
T
= , in which case youll incur an
unlimited loss. Your best case is S
T
< K, in which case the call expires
worthless; the call holder wastes his premium and your prot is the future
value of the premium you received from the buyer.
If you buy a put, your worst case is that S
T
_ K, in which case youll
let your put expire worthless and youll lose the future value of the put
premium. Your best case is S
T
= 0, in which case you sell a worthless
stock for K. Your prot is K FV (premium).
If you sell a put, your worst case is S
T
= 0, in which case the put holder
sells you a worthless stock for K; your prot is FV (premium) K. Your
best case is S
T
_ K, where the written put expires worthless and your
prot is FV (premium).
www.actuary88.com c _Yufeng Guo 21
CHAPTER 2. INTRODUCTION TO FORWARDS AND OPTIONS
Problem 2.13.
Let S represent the stock price at the option expiration date.
Ill draw a separate diagram for the payo and a separate diagram for the
prot.
a. Suppose you long a call (i.e. buy a call).
(i) Payo at expiration is max (0; S 35) =
_
0 if S < 35
S 35 if S _ 35
Your prot at expiration= Payo - FV (premium)
= max (0; S 35) 9:12 (1:08) = max (0; S 35) 9: 849 6
=
_
0 if S < 35
S 35 if S _ 35
9: 849 6 =
_
9: 849 6 if S < 35
S 44: 849 6 if 35 _ S
10 20 30 40 50 60 70 80 90 100
-10
0
10
20
30
40
50
60
Stock Price
Payoff
Profit
Payo and Prot: Long a 35 strike call
www.actuary88.com c _Yufeng Guo 22
CHAPTER 2. INTRODUCTION TO FORWARDS AND OPTIONS
(ii) Payo at expiration is max (0; S 40) =
_
0 if S < 40
S 40 if S _ 40
Your prot at expiration= Payo - FV (premium)
= max (0; S 40) 6:22 (1:08) = max (0; S 40) 6: 717 6
=
_
0 if S < 40
S 40 if S _ 40
6: 717 6 =
_
6: 717 6 if S < 40
S 46: 717 6 if S _ 40
10 20 30 40 50 60 70 80 90 100
0
10
20
30
40
50
60
Stock price at expiration
Payoff
Profit
Payo and Prot: Long a 40 strike call
www.actuary88.com c _Yufeng Guo 23
CHAPTER 2. INTRODUCTION TO FORWARDS AND OPTIONS
(iii) Payo at expiration is max (0; S 45) =
_
0 if S < 45
S 45 if S _ 45
Your prot at expiration= Payo - FV (premium)
= max (0; S 45) 4:08 (1:08) = max (0; S 45) 4: 406 4
=
_
0 if S < 45
S 45 if S _ 45
4: 406 4 =
_
4: 406 4 if S < 45
S 49: 406 4 if S _ 45
10 20 30 40 50 60 70 80 90 100
0
10
20
30
40
50
Stock price at expiration
Payoff
Profit
Payo and Prot: Long a 45 strike call
b. The payo of a long call is max (0; S K). As K increases, the payo
gets worse and the option becomes less valuable. Everything else equal, the
higher the strike price, the lower the call premium.
www.actuary88.com c _Yufeng Guo 24
CHAPTER 2. INTRODUCTION TO FORWARDS AND OPTIONS
Problem 2.14.
Suppose we own a put (i.e. long put).
a. Payo at expiration is max (0; 35 S) =
_
35 S if S _ 35
0 if S > 35
Your prot at expiration = Payo - FV (premium)
= max (0; 35 S) 1:53 (1:08) = max (0; 35 S) 1: 652 4
=
_
35 S if S _ 35
0 if S > 35
1: 652 4 =
_
33: 347 6 S if S _ 35
1: 652 4 if S > 35
10 20 30 40 50 60 70 80 90 100
0
10
20
30
Stock price at expiration
Payo and prot: Long a 35 strike put
The blue line is the payo. The black line is the prot.
www.actuary88.com c _Yufeng Guo 25
CHAPTER 2. INTRODUCTION TO FORWARDS AND OPTIONS
b. Payo at expiration is max (0; 40 S) =
_
40 S if S _ 40
0 if S > 40
Your prot at expiration = Payo - FV (premium)
= max (0; 40 S) 3:26 (1:08) = max (0; 40 S) 3: 520 8
=
_
40 S if S _ 40
0 if S > 40
3: 520 8 =
_
36: 479 2 S if S _ 40
3: 520 8 if S > 40
10 20 30 40 50 60 70 80 90 100
0
10
20
30
40
Stock price at expiration
Payo and prot: Long a 40 strike put
The blue line is the payo. The black line is the prot.
www.actuary88.com c _Yufeng Guo 26
CHAPTER 2. INTRODUCTION TO FORWARDS AND OPTIONS
c. Payo at expiration is max (0; 45 S) =
_
45 S if S _ 45
0 if S > 45
Your prot at expiration = Payo - FV (premium)
= max (0; 45 S) 5:75 (1:08) = max (0; 45 S) 6: 21
=
_
45 S if S _ 45
0 if S > 45
6: 21 =
_
38: 79 S if S _ 45
6: 21 if S > 45
10 20 30 40 50 60 70 80 90 100
0
10
20
30
40
Stock price at expiration
Payo and prot: Long a 45 strike put
The blue line is the payo. The black line is the prot.
As the strike price increases, the payo of a put goes up and the more
valuable a put is. Everything else equal, the higher the strike price, the more
expensive a put is.
www.actuary88.com c _Yufeng Guo 27
CHAPTER 2. INTRODUCTION TO FORWARDS AND OPTIONS
Problem 2.15.
If you borrow money from a bank to buy a $1000 S&R index, your borrowing
cost is known at the time of borrowing. Suppose the annual eective risk free
interest rate is r. If you borrow $1000 at t = 0, then at T you just pay the bank
1000 (1 +r)
T
. You can sleep well knowing that your borrowing cost is xed in
advance.
In contrast, if you short-sell n number of IBM stocks and use the short sale
proceeds to buy a $1000 S&R index, you own the brokerage rm n number of
IBM stocks. If you want to close your short position at time T, you need to
buy n stocks at T. The cost of n stocks at T is nS
T
, where S
T
is the price of
IBM stocks per share at T. Since S
T
is not known in advance, if you use short
selling to nance your purchase of a $1000 S&R index, your borrowing cost nS
T
cannot be known in advance. This brings additional risk to your position. As
such, you cant determine your prot.
Problem 2.16.
Skip this problem. SOA is unlikely to ask you to design a spreadsheet on
the exam.
www.actuary88.com c _Yufeng Guo 28
Chapter 3
Insurance, collars, and
other strategies
Problem 3.1.
The put premium is 74:201. At t = 0, you
spend 1000 to buy an S&R index
spend 74:201 to buy a 1000-strike put
borrow 980:39
take out (1000 + 74:201) 980:39 = 93: 811 out of your own pocket.
So your total borrowing is 980:39 + 93: 811 = 1074: 20.
The future value is 1074: 20 (1:02) = 1095: 68
S&R index S&R Put Payo -(Cost+Interest) Prot
900 100 1000 1095: 68 1000 1095: 68 = 95: 68
950 50 1000 1095: 68 1000 1095: 68 = 95: 68
1000 0 1000 1095: 68 1000 1095: 68 = 95: 68
1050 0 1050 1095: 68 1050 1095: 68 = 45: 68
1100 0 1100 1095: 68 1100 1095: 68 = 4: 32
1150 0 1150 1095: 68 1150 1095: 68 = 54: 32
1200 0 1200 1095: 68 1200 1095: 68 = 104: 32
29
CHAPTER 3. INSURANCE, COLLARS, AND OTHER STRATEGIES
Payo. The payo of owning an index is S, where S is the price of the index
at the put expiration
The payo of owning a put is max (0; 1000 S) at expiration.
Total payo:
S+max (0; 1000 S) = S+
_
1000 S if S _ 1000
0 if S > 1000
=
_
1000 if S _ 1000
S if S > 1000
Prot is:
_
1000 if S _ 1000
S if S > 1000
1095: 68 =
_
95: 68 if S _ 1000
S 1095: 68 if S > 1000
200 400 600 800 1000 1200 1400 1600 1800 2000
0
500
1000
1500
2000
Stock price at expiration
Payoff (Profit)
Payoff
Profit
Payo and Prot: index + put
www.actuary88.com c _Yufeng Guo 30
CHAPTER 3. INSURANCE, COLLARS, AND OTHER STRATEGIES
Problem 3.2.
At t = 0 you
short sell one S&R index, receiving $1000
sell a 1000-strike put, receiving $74:201
deposit 1000 + 74:201 = 1074: 201 in a savings account. This grows into
1074: 201 (1:02) = 1095: 68 at T = 1
The payo of the index sold short is S
The payo of a sold put: max (0; 1000 S)
The total payo at expiration is:
Smax (0; 1000 S) = S
_
1000 S if S _ 1000
0 if S > 1000
=
_
1000 if S _ 1000
S if S > 1000
The prot at expiration is:
_
1000 if S _ 1000
S if S > 1000
+ 1095: 68 =
_
95: 68 if S _ 1000
1095: 68 S if S > 1000
S&R index S&R Put Payo -(Cost+Interest) Prot
900 100 1000 1095: 68 1000 + 1095: 68 = 95: 68
950 50 1000 1095: 68 1000 + 1095: 68 = 95: 68
1000 0 1000 1095: 68 1000 + 1095: 68 = 95: 68
1050 0 1050 1095: 68 1050 + 1095: 68 = 45: 68
1100 0 1100 1095: 68 1100 + 1095: 68 = 4: 32
1150 0 1150 1095: 68 1150 + 1095: 68 = 54: 32
1200 0 1200 1095: 68 1200 + 1095: 68 = 104: 32
www.actuary88.com c _Yufeng Guo 31
CHAPTER 3. INSURANCE, COLLARS, AND OTHER STRATEGIES
Payo=
_
1000 if S _ 1000
S if S > 1000
Prot=
_
1000 if S _ 1000
S if S > 1000
+ 1095: 68
200 400 600 800 1000 1200 1400 1600 1800 2000
-2000
-1500
-1000
-500
0
Index Price
Profit
Payoff
short index +sell put
You can verify that the prot diagram above matches the textbook Figure
3.5 (d).
www.actuary88.com c _Yufeng Guo 32
CHAPTER 3. INSURANCE, COLLARS, AND OTHER STRATEGIES
Problem 3.3.
Option 1: Buy S&R index for 1000 and buy a 950-strike put
Option 2: Invest 931:37 in a zero-coupon bond and buy a 950-strike call.
Verify that Option 1 and 2 have the same payo and the same prot.
Option 1:
If you own an index, your payo at any time is the spot price of the index
S. The payo of owning a 950-strike put is max (0; 950 S). Your total payo
at the put expiration is
S+max (0; 950 S) = S+
_
950 S if S _ 950
0 if S > 950
=
_
950 if S _ 950
S if S > 950
To calculate the prot, we need to know the initial investment. At t = 0,
we spend 1000 to buy an index and 51:777 to buy the 950-strike put. The total
investment is 1000 + 51:777 = 1051: 777. The future value of the investment is
1051: 777 (1:02) = 1072: 81
So the prot is:
_
950 if S _ 950
S if S > 950
1072: 81 =
_
122: 81 if S _ 950
S 1072: 81 if S > 950
www.actuary88.com c _Yufeng Guo 33
CHAPTER 3. INSURANCE, COLLARS, AND OTHER STRATEGIES
Payo=
_
950 if S _ 950
S if S > 950
Prot=
_
950 if S _ 950
S if S > 950
1072: 81
200 400 600 800 1000 1200 1400 1600 1800 2000
0
500
1000
1500
2000
Index
Payoff
Profit
index + put
www.actuary88.com c _Yufeng Guo 34
CHAPTER 3. INSURANCE, COLLARS, AND OTHER STRATEGIES
Option 2:
Payo of the zero-coupon bond at T = 0:5 year is: 931:37 (1:02) = 950
Payo of owning a 950-strike call: max (0; S 950)
Total payo:
950+max (0; S 950) = 950+
_
0 if S _ 950
S 950 if S > 950
=
_
950 if S _ 950
S if S > 950
To calculate the prot, we need to know the initial investment. We spend
931:37 to buy a bond and 120:405 to buy a 950-strike call. The future value of
the investment is (931:37 + 120:405) 1:02 = 1072: 81. The prot is:
_
950 if S _ 950
S if S > 950
1072: 81 =
_
122: 81 if S _ 950
S 1072: 81 if S > 950
Option 1 and 2 have the same payo and the same prot. But why? Its
because the put-call parity:
C (K; T) +PV (K) = P (K; T) +S
0
Option 1 consists of buying S&R index and a 950-strike put
Option 2 consists of investing PV (K) = 950
_
1:02
1
_
= 931: 37 and buying
a 950-strike call. Due to the put-call parity, Option 1 and 2 have the same
payo and the same prot.
www.actuary88.com c _Yufeng Guo 35
CHAPTER 3. INSURANCE, COLLARS, AND OTHER STRATEGIES
Problem 3.4.
Option 1: Short sell S&R index for 1000 and buy a 950-strike call
Option 2: Borrow 931:37 and buy a 950-strike put
Verify that Option 1 and 2 have the same payo and the same prot.
Option 1: At t = 0:5, your payo from the short sale of an index is S,
where S is the index price at T = 0:5. At T = 0:5, your payo from owning a
call is max (0; S 950) =
_
0 if S < 950
S 950 if S _ 950
.
Your total payo is
S +
_
0 if S < 950
S 950 if S _ 950
=
_
S if S < 950
950 if S _ 950
Please note that when calculating the payo, well ignore the sales price of
the index $1; 000 and the call purchase price 120: 405. These two numbers aect
your prot, but they dont aect your payo. Your payo is the same no matter
whether you sold your index for $1 or $1000, and no matter whether you buy
the 950-strike call for $10 or $120: 41.
Next, lets nd the prot at T = 0:5. At t = 0, you sell an index for 1000.
Of 1000 you get, you spend 120: 405 t 120: 41 to buy a 950-strike call. You
have 1000 120: 41 = 879: 59 left. This will grow into 879: 59 1:02 = 897: 18
at T = 0:5 At T = 0:5, your prot is 897: 18 plus the payo:
Prot =897: 18 +
_
S if S < 950
950 if S _ 950
=
_
897: 18 S if S < 950
52: 82 if S _ 950
www.actuary88.com c _Yufeng Guo 36
CHAPTER 3. INSURANCE, COLLARS, AND OTHER STRATEGIES
Payo=
_
S if S < 950
950 if S _ 950
Prot =897: 18+
_
S if S < 950
950 if S _ 950
200 400 600 800 1000 1200 1400 1600 1800 2000
-800
-600
-400
-200
0
200
400
600
800
Index Price
Profit
Payoff
Payo and Prot: Short index + Long call
www.actuary88.com c _Yufeng Guo 37
CHAPTER 3. INSURANCE, COLLARS, AND OTHER STRATEGIES
Option 2 payo. At t = 0:5, you need to pay the lender 931:371:02 = 950,
so, a payo of 950 (youll write the lender a check of 950). At T = 0:5, the pay-
o from buying a 950-strike put is max (0; 950 S) =
_
950 S if S < 950
0 if S _ 950
.
Your total payo at T = 0:5 is:
950 +
_
950 S if S < 950
0 if S _ 950
=
_
S if S < 950
950 if S _ 950
.
This is the same as the payo in Option 1.
Option 2 prot. There are two ways to calculate the prot.
Method 1. The total prot is the sum of prot earned from borrowing 931:37
and the prot earned by buying a 950-strike put. The prot from borrowing
931:37 is zero; you borrow 931:37 at t = 0. This grows into 931:37 1:02 = 950
at T = 0:5 in your savings account. Then at T = 0:5, you take out 950 from
your savings account and pay the lender. Now your savings account is zero. So
the prot earned from borrowing 931:37 is zero.
Next, lets calculate the prot from buying the put. The put premium is
$51.78. So your prot earned from buying the put option is
51:78 1:02 + max (0; 950 S) = 52: 82 + max (0; 950 S)
= 52: 82 +
_
950 S if S < 950
0 if S _ 950
=
_
897: 18 S if S < 950
52: 82 if S _ 950
Method 2. We already know the payo is
_
S if S < 950
950 if S _ 950
. We just
need to deduct the future value of the initial investment. At t = 0, you receive
931:37 from the lender and pay 51:78 to buy the put. So your total cash is
931:37 51:78 = 879: 59, which grows into 879: 59 1:02 = 897: 18 at t = 0:5.
Hence, your prot is:
_
S if S < 950
950 if S _ 950
+ 897: 18 =
_
897: 18 S if S < 950
52: 82 if S _ 950
No matter whether you use Method 1 or Method 2, the Option 2 prot is
the same as the Option 1 prot.
You might wonder why Option 1 and Option 2 have the same payo and the
same prot. The parity formula is
Call (K; T) Put (K; T) = PV (F
0;T
K) = PV (F
0;T
) PV (K)
Rearranging this equation, we get:
www.actuary88.com c _Yufeng Guo 38
CHAPTER 3. INSURANCE, COLLARS, AND OTHER STRATEGIES
Call (K; T) +PV (K) = Put (K; T) +PV (F
0;T
)
Since PV (F
0;T
) = S
0
, now we have:
Call (K; T)
. .
own a call
+ PV (K)
. .
own PV of strike price
= Put (K; T)
. .
own a put
+ S
0
..
own one index
The above equation can also be read as;
Call (K; T)
. .
buy a call
+ PV (K)
. .
invest PV of strike price
= Put (K; T)
. .
buy a put
+ S
0
..
buy one index
Rearranging the above formula, we get:
Call (K; T)
. .
own a call
+ S
0
..
sell one index
= Put (K; T)
. .
own a put
+ PV (K)
. .
borrow PV of strike price
The above equation can also be read as:
Call (K; T)
. .
buy a call
+ S
0
..
sell one index
= Put (K; T)
. .
buy a put
+ PV (K)
. .
borrow PV of strike price
According to the parity equation, Option 1 and Option 2 are identical port-
folios and should have the same payo and the same prot. In this problem,
Option 1 consists of shorting an S&R index and buying a 950-strike call. Op-
tion 2 consists of borrowing PV (K) = 950
_
1:02
1
_
= 931: 37 and buying a
950-strike put. As a result, Option 1 and 2 have the same payo and the same
prot.
www.actuary88.com c _Yufeng Guo 39
CHAPTER 3. INSURANCE, COLLARS, AND OTHER STRATEGIES
Problem 3.5.
Option 1: Short sell index for 1000 and buy 1050-strike call
Option 2: Borrow 1029.41 and buy a 1050-strike put.
Verify that Option 1 and 2 have the same payo and the same prot.
Option 1:
Payo:
S+max (0; S 1050) = S+
_
0 if S < 1050
S 1050 if S _ 1050
=
_
S if S < 1050
1050 if S _ 1050
Prot:
Your receive 1000 from the short sale and spend 71.802 to buy the 1050-strike
call.
The future value is: (1000 71:802) 1:02 = 946: 76
So the prot is:
_
S if S < 1050
1050 if S _ 1050
+ 946: 76 =
_
946: 76 S if S < 1050
103: 24 if S _ 1050
www.actuary88.com c _Yufeng Guo 40
CHAPTER 3. INSURANCE, COLLARS, AND OTHER STRATEGIES
Payo=
_
S if S < 1050
1050 if S _ 1050
Prot=
_
S if S < 1050
1050 if S _ 1050
+ 946: 76
200 400 600 800 1000 1200 1400 1600 1800 2000
-1000
-800
-600
-400
-200
0
200
400
600
800
Index Price
Profit
Payoff
Payo and Prot: Short index + Long call
www.actuary88.com c _Yufeng Guo 41
CHAPTER 3. INSURANCE, COLLARS, AND OTHER STRATEGIES
Option 2:
Payo:
If you borrow 1029:41, youll need to pay 1029:41 (1:02) = 1050 at T = 0:5
So the payo of borrowing 1029:41 is 1050.
Payo of the purchased put is max (0; 1050 S)
Total payo is:
= 1050 +
_
1050 S if S < 1050
0 if S _ 1050
=
_
S if S < 1050
1050 if S _ 1050
Initially, you receive 1029:41 from a bank and spend 101:214 to buy a 950-
strike put. So your net receipt at t = 0 is 1029:41 101:214 = 928: 196. Its
future value is 928: 196 (1:02) = 946: 76. Your prot is:
_
S if S < 1050
1050 if S _ 1050
+ 946: 76 =
_
946: 76 S if S < 1050
103: 24 if S _ 1050
Option 1 and 2 have the same payo and the same prot. This is because
the put-call parity:
Call (K; T)
. .
buy a call
+ S
0
..
sell one index
= Put (K; T)
. .
buy a put
+ PV (K)
. .
borrow PV of strike price
www.actuary88.com c _Yufeng Guo 42
CHAPTER 3. INSURANCE, COLLARS, AND OTHER STRATEGIES
Problem 3.6.
(a) buy an index for 1000
(b) buy a 950-strike call, sell a 950-strike put, and lend 931:37
Verify that (a) and (b) have the same payo and the same prot.
(a)s payo is S. Prot is S 1000 (1:02) = S 1020
(b)s payo:
Payo Initial receipt
buy a 950-strike call max (0; S 950) 120:405
sell a 950-strike put max (0; 950 S) 51:777
lend 931.37 931:37 (1:02) = 950 931:37
Total 120:405 + 51:777 931:37 = 1000
Total payo:
max (0; S 950) max (0; 950 S) + 950
=
_
0 if S < 950
S 950 if S _ 950

_
950 S if S < 950
0 if S _ 950
+ 950
=
_
(950 S) + 950 if S < 950
S 950 + 950 if S _ 950
= S
Total prot: S 1000 (1:02) = S 1020
(a) and (b) have the same payo and the same prot. Why?
Call (K; T)
. .
buy a call
+ S
0
..
sell one index
= Put (K; T)
. .
buy a put
+ PV (K)
. .
borrow PV of strike price
S
0
..
buy one index
= Call (K; T)
. .
buy a call
+ Put (K; T)
. .
sell a put
+ PV (K)
. .
lend PV of strike price
www.actuary88.com c _Yufeng Guo 43
CHAPTER 3. INSURANCE, COLLARS, AND OTHER STRATEGIES
(a) and (b) have the following common payo and prot.
Payo=S Prot=S 1020
200 400 600 800 1000 1200 1400 1600 1800 2000
-1000
0
1000
2000
Index Price
Payoff
Profit
www.actuary88.com c _Yufeng Guo 44
CHAPTER 3. INSURANCE, COLLARS, AND OTHER STRATEGIES
Problem 3.7.
(a) short index for 1000
(b) sell 1050-strike call, buy a 1050-strike put, and borrow 1029.41
Verify that (a) and (b) have the same payo and the same prot.
(a) Payo is S. Prot is S + 1000 (1:02) = 1020 S
(b) Payo:
max (0; 1050 S) max (0; S 1050) 1029:41 (1:02)
=
_
1050 S if S < 1050
0 if S _ 1050

_
0 if S < 1050
S 1050 if S _ 1050
1050
=
_
(1050 S) 1050 if S < 1050
(S 1050) 1050 if S _ 1050
=
_
S if S < 1050
S if S _ 1050
= S
We need to calculate the initial investment of (b).
At t = 0, we
Receive 71:802 from selling a 1050-strike call
Pay 101:214 to buy a 1050-strike put
Receive 1029:41 from a lender
Our net receipt is 71:802 101:214 + 1029:41 = 1000.
The future value is 1000 (1:02) = 1020
So the prot at T = 0:5 is S + 1020 = 1020 S:
You see that (a) and (b) have the same payo and the same prot. Why?
From the put-call parity, we have:
Call (K; T)
. .
buy a call
+ S
0
..
sell one index
= Put (K; T)
. .
buy a put
+ PV (K)
. .
borrow PV of strike price
S
0
..
sell one index
= Call (K; T)
. .
sell a call
+ Put (K; T)
. .
buy a put
+ PV (K)
. .
borrow PV of strike price
www.actuary88.com c _Yufeng Guo 45
CHAPTER 3. INSURANCE, COLLARS, AND OTHER STRATEGIES
Payo= S. Prot= 1020 S
200 400 600 800 1000 1200 1400 1600 1800 2000
-2000
-1000
0
1000
Index Price
Profit
Payoff
www.actuary88.com c _Yufeng Guo 46
CHAPTER 3. INSURANCE, COLLARS, AND OTHER STRATEGIES
Problem 3.8.
Put-call parity: Call (K; T)
. .
buy a call
+ PV (K)
. .
invest PV of strike price
= Put (K; T)
. .
buy a put
+ S
0
..
buy one index
109:2 +PV (K) = 60:18 + 1000
PV (K) = 60:18 + 1000 109:2 = 950: 98
PV (K) =
K
1:02
K = 950: 98 (1:02) = 970
Problem 3.9.
Buy a call (put) a lower strike + Sell an otherwise identical call (put) with
a higher strike=Bull call (put) spread
Option 1: buy 950-strike call and sell 1000-strike call
Option 2: buy 950-strike put and sell 1000-strike put.
Verify that option 1 and 2 have the same prot.
Option 1:
Payo=max (0; S 950) max (0; S 1000)
=
_
0 if S < 950
S 950 if S _ 950

_
0 if S < 1000
S 1000 if S _ 1000
=
_
_
_
0 if S < 950
S 950 if 1000 > S _ 950
S 950 if S _ 1000

_
_
_
0 if S < 950
0 if 1000 > S _ 950
S 1000 if S _ 1000
=
_
_
_
0 if S < 950
S 950 if 1000 > S _ 950
S 950 (S 1000) if S _ 1000
=
_
_
_
0 if S < 950
S 950 if 1000 > S _ 950
50 if S _ 1000
Initial cost:
Spend 120:405 to buy 950-strike call
Sell 1000-strike call receiving 93:809
Total initial investment: 120:405 93:809 = 26: 596
The future value is 26: 596 (1:02) = 27: 127 92
Prot is:
=
_
_
_
0 if S < 950
S 950 if 1000 > S _ 950
50 if S _ 1000
27:13 =
_
_
_
27:13 if S < 950
S 950 27:13 if 1000 > S _ 950
50 27:13 if S _ 1000
=
_
_
_
27:13 if S < 950
S 977: 13 if 1000 > S _ 950
22: 87 if S _ 1000
www.actuary88.com c _Yufeng Guo 47
CHAPTER 3. INSURANCE, COLLARS, AND OTHER STRATEGIES
Payo=
_
_
_
0 if S < 950
S 950 if 1000 > S _ 950
50 if S _ 1000
Prot=
_
_
_
0 if S < 950
S 950 if 1000 > S _ 950
50 if S _ 1000
27:13
200 400 600 800 1000 1200 1400 1600 1800 2000
-20
-10
0
10
20
30
40
50
Index Price
Payoff
Profit
www.actuary88.com c _Yufeng Guo 48
CHAPTER 3. INSURANCE, COLLARS, AND OTHER STRATEGIES
Option 2: buy 950-strike put and sell 1000-strike put.
The payo:
max (0; 950 S) max (0; 1000 S)
=
_
950 S if S < 950
0 if S _ 950

_
1000 S if S < 1000
0 if S _ 1000
=
_
_
_
950 S if S < 950
0 if 1000 > S _ 950
0 if S _ 1000

_
_
_
1000 S if S < 950
1000 S if 1000 > S _ 950
0 if S _ 1000
=
_
_
_
(950 S) (1000 S) if S < 950
(1000 S) if 1000 > S _ 950
0 S _ 1000
=
_
_
_
50 if S < 950
S 1000 if 1000 > S _ 950
0 if S _ 1000
Initial cost:
Buy 950-strike put. Pay 51:777
Sell 1000-strike put. Receive 74:201
Net receipt: 74:201 51:777 = 22: 424
Future value: 22: 424 (1:02) = 22: 872 48
The prot is:
_
_
_
50 if S < 950
S 1000 if 1000 > S _ 950
0 if S _ 1000
+ 22: 87
=
_
_
_
50 + 22: 87 if S < 950
S 1000 + 22: 87 if 1000 > S _ 950
22: 87 if S _ 1000
=
_
_
_
27: 13 if S < 950
S 977: 13 if 1000 > S _ 950
22: 87 if S _ 1000
www.actuary88.com c _Yufeng Guo 49
CHAPTER 3. INSURANCE, COLLARS, AND OTHER STRATEGIES
Payo=
_
_
_
50 if S < 950
S 1000 if 1000 > S _ 950
0 if S _ 1000
Prot=
_
_
_
50 if S < 950
S 1000 if 1000 > S _ 950
0 if S _ 1000
+ 22: 87
200 400 600 800 1000 1200 1400 1600 1800 2000
-50
-40
-30
-20
-10
0
10
20
Index Price
Profit
Payoff
www.actuary88.com c _Yufeng Guo 50
CHAPTER 3. INSURANCE, COLLARS, AND OTHER STRATEGIES
The payo of the rst option is $50 greater than the payo of the second
option. However, at t = 0, we pay 26: 596 to set up option 1; we pay 22: 424
(i.e. we receive 22: 424) to set up option 2. It costs us 26: 596 (22: 424) =
49: 02 more initially to set up option 1 than option 2. The future value of this
initial set up cost is 49: 02 (1:02) = 50. As a result, option 1 and 2 have the
same prot at T = 0:5.
This should make sense in a world of no arbitrage. Consider two port-
folios A and B. If for any stock price Payoff (A) = Payoff (B) + c, then
InitialCost (A) = InitialCost (B) +PV (c) to avoid arbitrage.
Profit (A) = Payoff (A) FV [InitialCost (A)]
= Payoff (B) +c FV [InitialCost (B)] c
= Payoff (B) FV [InitialCost (B)] = Profit (B)
Similarly, if InitialCost (A) = InitialCost (B) +PV (c)
Payoff (A) = Payoff (B) +c Profit (A) = Profit (B)
Finally, lets see why option 1 is always $50 higher than option 2 in terms
of the payo and the initial set up cost. The put-call parity is:
Call (K; T)
. .
buy a call
+ PV (K)
. .
invest PV of strike price
= Put (K; T)
. .
buy a put
+ S
0
..
buy one index
The timing of the put-call parity is at t = 0. The above equation means
Call (K; T)
. .
Cost of buying a call at t=0
+ PV (K)
. .
Cost of investing PV of strike price at t=0
= Put (K; T)
. .
Cost of buying a put at t=0
+ S
0
..
Cost of buying an index at t=0
If we are interested in the payo at expiration date T, then the put-call
parity is:
Call (K; T)
. .
Payo a call at T
+ K
..
strike price at T
= Put (K; T)
. .
Payo of a put at T
+ S
..
Index price at T
Now we set up the initial cost parity for two strike prices K
1
< K
2
Call (K
1
; T)
. .
cost of buying a call
+ PV (K
1
)
. .
invest PV of K1
= Put (K
1
; T)
. .
cost of buying a put
+ S
0
..
cost of buying one index at t=0
Call (K
2
; T)
. .
cost of buying a call
+ PV (K
2
)
. .
invest PV of K2
= Put (K
2
; T)
. .
cost of buying a put
+ S
0
..
cost of buying one index at t=0

_ Call (K
1
; T)
. .
cost of buying a call
Call (K
2
; T)
. .
cost of buying a call
_

_ + [PV (K
1
) PV (K
2
)]
www.actuary88.com c _Yufeng Guo 51
CHAPTER 3. INSURANCE, COLLARS, AND OTHER STRATEGIES
=
_

_ Put (K
1
; T)
. .
cost of buying a put
Put (K
2
; T)
. .
cost of buying a put
_

_ Call (K
1
; T)
. .
cost of buying a call
Call (K
2
; T)
. .
cost of buying a call
_

_
. .
Call spread
=
_

_ Put (K
1
; T)
. .
cost of buying a put
Put (K
2
; T)
. .
cost of buying a put
_

_
. .
Put spread
+ [PV (K
2
) PV (K
1
)]

_ Call (K
1
; T)
. .
cost of buying a call
+ Call (K
2
; T)
. .
cost of selling a call
_

_
. .
Call spread
=
_

_ Put (K
1
; T)
. .
cost of buying a put
+ Put (K
2
; T)
. .
cost of selling a put
_

_
. .
Put spread
+ [PV (K
2
) PV (K
1
)]
So the initial cost of setting up a call bull spread always exceeds the initial
set up cost of a bull put spread by a uniform amount PV (K
2
) PV (K
1
).
In this problem, PV (K
2
) PV (K
1
) = (1000 950) 1:02
1
= 49: 02
Set up the payo parity at T:
Call (K
1
; T)
. .
Payo a call at T
+ K
1
..
strike price at T
= Put (K
1
; T)
. .
Payo of a put at T
+ S
..
Index price at T
Call (K
2
; T)
. .
Payo a call at T
+ K
1
..
strike price at T
= Put (K
2
; T)
. .
Payo of a put at T
+ S
..
Index price at T

_ Call (K
1
; T)
. .
Payo a long call at T
Call (K
2
; T)
. .
Payo a long call at T
_

_
. .
Call bull payo at T
www.actuary88.com c _Yufeng Guo 52
CHAPTER 3. INSURANCE, COLLARS, AND OTHER STRATEGIES
=
_

_ Put (K
2
; T)
. .
Payo of a long put at T
Put (K
1
; T)
. .
Payo of a long put at T
_

_
. .
Put spread payo at T
+K
2
K
1

_ Call (K
1
; T)
. .
Payo a long call at T
+ Call (K
2
; T)
. .
Payo short call at T
_

_
. .
Call spread payo at T
=
_

_ Put (K
2
; T)
. .
Payo of a long put at T
+ Put (K
1
; T)
. .
Payo of a short put at T
_

_
. .
Put spread payo at T
+K
2
K
1
In this problem, K
2
K
1
= 1000 950 = 50
So the payo of a call bull spread at T = 0:5 always exceeds the payo of a
put bull spread by a uniform amount 50.
www.actuary88.com c _Yufeng Guo 53
CHAPTER 3. INSURANCE, COLLARS, AND OTHER STRATEGIES
Problem 3.10.
Buy a call (put) a higher strike + Sell an otherwise identical call (put) with
lower strike=Bear call (put) spread.
In this problem, K
1
= 1050, K
2
= 950 (a bear spread)
PV (K
2
) PV (K
1
) = (950 1050) 1:02
1
= (100) 1:02
1
= 98: 04

_ Call (K
1
; T)
. .
cost of buying a call
+ Call (K
2
; T)
. .
cost of selling a call
_

_
. .
Call spread
=
_

_ Put (K
1
; T)
. .
cost of buying a put
+ Put (K
2
; T)
. .
cost of selling a put
_

_
. .
Put spread
+ [PV (K
2
) PV (K
1
)]
=
_

_ Put (K
1
; T)
. .
cost of buying a put
+ Put (K
2
; T)
. .
cost of selling a put
_

_
. .
Put spread
98: 04

_ Call (K
1
; T)
. .
Payo a long call at T
+ Call (K
2
; T)
. .
Payo short call at T
_

_
. .
Call spread payo at T
=
_

_ Put (K
2
; T)
. .
Payo of a long put at T
+ Put (K
1
; T)
. .
Payo of a short put at T
_

_
. .
Put spread payo at T
+K
2
K
1
=
_

_ Put (K
2
; T)
. .
Payo of a long put at T
+ Put (K
1
; T)
. .
Payo of a short put at T
_

_
. .
Put spread payo at T
100
For any index price at expiration, the payo of the call bear spread is always
100 less than the payo of the put bear spread. Consequently, as we have seen,
to avoid arbitrage, the initial set-up cost of the call bear spread is less than
the initial set-up cost of the put bear spread by the amount of present value of
the 100. The call bear spread and the put bear spread have the same prot at
expiration.
www.actuary88.com c _Yufeng Guo 54
CHAPTER 3. INSURANCE, COLLARS, AND OTHER STRATEGIES
Next, lets draw the payo and prot diagram for each spread.
Payo of the call bear spread:
Payo=max (0; S 1050) max (0; S 950)
=
S < 950 950 _ S < 1050 1050 _ S
Buy 1050-strike call 0 0 S 1050
Sell 950-strike call 0 950 S 950 S
Total 0 950 S 100
=
_
_
_
0 if S < 950
950 S if 950 _ S < 1050
100 if S _ 1050
The initial set-up cost of the call bear spread:
Buy 1050-strike call. Pay 71:802
Sell 950-strike call. Receive 120:405
Net receipt: 120:405 71:802 = 48: 603
Future value: 48: 603 (1:02) = 49: 575 06
So the prot of the call bear spread at expiration is
=
_
_
_
0 if S < 950
950 S if 950 _ S < 1050
100 if S _ 1050
+49: 58 =
_
_
_
49: 58 if S < 950
999: 58 S if 950 _ S < 1050
50: 42 if S _ 1050
www.actuary88.com c _Yufeng Guo 55
CHAPTER 3. INSURANCE, COLLARS, AND OTHER STRATEGIES
Payo=
_
_
_
0 if S < 950
950 S if 950 _ S < 1050
100 if S _ 1050
Prot=
_
_
_
0 if S < 950
950 S if 950 _ S < 1050
100 if S _ 1050
+ 49: 58
200 400 600 800 1000 1200 1400 1600 1800 2000
-100
-80
-60
-40
-20
0
20
40
Index Price
Profit
Payoff
Payo and Prot: Call bear spread
www.actuary88.com c _Yufeng Guo 56
CHAPTER 3. INSURANCE, COLLARS, AND OTHER STRATEGIES
Payo of the put bear spread:
Payo=max (0; 1050 S) max (0; 950 S)
=
S < 950 950 _ S < 1050 1050 _ S
Buy 1050-strike put 1050 S 1050 S 0
Sell 950-strike put S 950 0 0
Total 100 1050 S 0
=
_
_
_
100 if S < 950
1050 S if 950 _ S < 1050
0 if S _ 1050
0 200 400 600 800 1000 1200 1400 1600 1800 2000
0
10
20
30
40
50
60
70
80
90
100
Index Price
Payoff
Payo of the put bear spread
www.actuary88.com c _Yufeng Guo 57
CHAPTER 3. INSURANCE, COLLARS, AND OTHER STRATEGIES
The initial set-up cost:
Buy 1050-strike put. Pay 101:214
Sell 950-strike put. Receive 51:777
Net cost: 101:214 51:777 = 49: 437
Future value: 49: 437 (1:02) = 50: 42
The prot at expiration is:
=
_
_
_
100 if S < 950
1050 S if 950 _ S < 1050
0 if S _ 1050
50: 42 =
_
_
_
49: 58 if S < 950
999: 58 S if 950 _ S < 1050
50: 42 if S _ 1050
We see that the call bear spread and the put bear spread have the same
prot.
www.actuary88.com c _Yufeng Guo 58
CHAPTER 3. INSURANCE, COLLARS, AND OTHER STRATEGIES
Problem 3.11.
Initial Cost Payo
Buy S&R index 1000 S
Buy 950-strike put 51:777 max(0; 950 S)
Sell 1050-strike call 71:802 max(0; S 1050)
Total 1000 + 51:777 71:802 = 979: 975
FV (initial cost) 979: 975 (1:02) = 999: 574 5
The net option premium is: 51:777 71:802 = 20: 025. So we receive 20:
025 if we enter this collar.
Payo
S < 950 950 _ S < 1050 1050 _ S
Buy S&R index S S S
Buy 950-strike put 950 S 0 0
Sell 1050-strike call 0 0 1050 S
Total 950 S 1050
The payo at expiration is:
_
_
_
950 if S < 950
S if 950 _ S < 1050
1050 if S _ 1050
The prot at expiration is:
_
_
_
950 if S < 950
S if 950 _ S < 1050
1050 if S _ 1050
999: 57 =
_
_
_
950 999: 57 if S < 950
S 999: 57 if 950 _ S < 1050
1050 999: 57 if S _ 1050
=
_
_
_
49: 57 if S < 950
S 999: 57 if 950 _ S < 1050
50: 43 if S _ 1050
www.actuary88.com c _Yufeng Guo 59
CHAPTER 3. INSURANCE, COLLARS, AND OTHER STRATEGIES
Prot=
_
_
_
49: 57 if S < 950
S 999: 57 if 950 _ S < 1050
50: 43 if S _ 1050
200 400 600 800 1000 1200 1400 1600 1800 2000
-50
-40
-30
-20
-10
0
10
20
30
40
50
Index Price
Profit
Prot: long index, long 950-strike put, short 1050-strike call
The net option premium is 20: 025. So we receive 20: 025 if we enter this
collar. To construct a zero-cost collar and keep 950-strike put, we need to
increase the strike price of the call such that the call premium is equal to the
put premium of 51:777.
www.actuary88.com c _Yufeng Guo 60
CHAPTER 3. INSURANCE, COLLARS, AND OTHER STRATEGIES
Problem 3.12.
Initial Cost Payo
Buy S&R index 1000 S
Buy 950-strike put 51:777 max(0; 950 S)
Sell 1107-strike call 51:873 max(0; S 1050)
Total 1000 + 51:777 51:873 = 999: 904
FV (initial cost) 999: 904 (1:02) = 1019: 902 08
The net option premium is: 51:777 51:873 = 0:096 . So we receive 0:096
if we enter this collar. This is very close to a zero-cost collar, where the net
premium is zero.
Payo
S < 950 950 _ S < 1107 1107 _ S
Buy S&R index S S S
Buy 950-strike put 950 S 0 0
Sell 1050-strike call 0 0 1107 S
Total 950 S 1107
The prot is:
_
_
_
950 if S < 950
S if 950 _ S < 1107
1107 if S _ 1107
1019: 90 =
_
_
_
69: 9 if S < 950
S 1019: 90 if 950 _ S < 1107
87: 1 if S _ 1107
www.actuary88.com c _Yufeng Guo 61
CHAPTER 3. INSURANCE, COLLARS, AND OTHER STRATEGIES
Prot=
_
_
_
69: 9 if S < 950
S 1019: 90 if 950 _ S < 1107
87: 1 if S _ 1107
200 400 600 800 1000 1200 1400 1600 1800 2000
-60
-40
-20
0
20
40
60
80
Index Price
Profit
Prot: long index, long 950-strike put, short 1107-strike call
www.actuary88.com c _Yufeng Guo 62
CHAPTER 3. INSURANCE, COLLARS, AND OTHER STRATEGIES
Problem 3.13.
a. 1050-strike S&R straddle
Straddle = buy a call and put with the same strike price and time to expi-
ration.
Initial Cost Payo
Buy1050-strike call 71:802 max(0; S 1050)
Buy 1050-strike put 101:214 max(0; 1050 S)
Total 71:802 + 101:214 = 173: 016
FV (initial cost) 173: 016 (1:02) = 176: 476 32
Payo
S < 1050 S _ 1050
Buy1050-strike call 0 S 1050
Buy 1050-strike put 1050 S 0
Total 1050 S S 1050
The prot is:
_
1050 S if S < 1050
S 1050 if S _ 1050
176: 48 =
_
873: 52 S if S < 1050
S 1226: 48 if S _ 1050
200 400 600 800 1000 1200 1400 1600 1800 2000 2200
-100
0
100
200
300
400
500
600
700
800
900
Index Price
Profit
Prot: long 1050-strike call and long 1050-strike put
www.actuary88.com c _Yufeng Guo 63
CHAPTER 3. INSURANCE, COLLARS, AND OTHER STRATEGIES
b. written 950-strike S&R straddle
Initial revenue Payo
short 950-strike call 120:405 max(0; S 950)
short 950-strike put 51:777 max(0; 950 S)
Total 120:405 + 51:777 = 172: 182
FV (initial cost) 172: 182 (1:02) = 175: 625 64
Payo
S < 950 S _ 950
sell 950-strike call 0 950 S
sell 950-strike put S 950 0
Total S 950 950 S
The prot is:
_
S 950 if S < 950
950 S if S _ 950
+ 175: 66 =
_
S 774: 34 if S < 950
1125: 66 S if S _ 950
200 400 600 800 1000 1200 1400 1600 1800 2000 2200
-1000
-900
-800
-700
-600
-500
-400
-300
-200
-100
0
100
Index Price
Profit
Prot: short 950-strike call and short 950-strike put
www.actuary88.com c _Yufeng Guo 64
CHAPTER 3. INSURANCE, COLLARS, AND OTHER STRATEGIES
c. simultaneous purchase of 1050-straddle and sale of 950-straddle
Prot = Prot of purchase of 1050-straddle + Prot of Sale of 950-straddle
Prot
_
873: 52 S if S < 1050
S 1226: 48 if S _ 1050
+
_
S 774: 34 if S < 950
1125: 66 S if S _ 950
_
_
_
873: 52 S if S < 950
873: 52 S if 950 _ S < 1050
S 1226: 48 if S _ 1050
+
_
_
_
S 774: 34 if S < 950
1125: 66 S if 950 _ S < 1050
1125: 66 S if S _ 1050
=
_
_
_
873: 52 S + (S 774: 34) if S < 950
873: 52 S + (1125: 66 S) if 950 _ S < 1050
S 1226: 48 + (1125: 66 S) if S _ 1050
=
_
_
_
99: 18 if S < 950
1999: 18 2S if 950 _ S < 1050
100: 82 if S _ 1050
950 1000 1050 1100 1150 1200
-100
-80
-60
-40
-20
0
20
40
60
80
100
Index Price
Profit
Prot: long 1050-strike straddle and short 950 straddle
www.actuary88.com c _Yufeng Guo 65
CHAPTER 3. INSURANCE, COLLARS, AND OTHER STRATEGIES
Problem 3.14.
The put-call parity is:
Call (K; T)
. .
buy a call
+ PV (K)
. .
invest PV of strike price
= Put (K; T)
. .
buy a put
+ S
0
..
buy one index
Call (K
1
; T)
. .
buy a call
+ PV (K
1
)
. .
invest PV of strike price
= Put (K
1
; T)
. .
buy a put
+ S
0
..
buy one index
Call (K
2
; T)
. .
buy a call
+ PV (K
2
)
. .
invest PV of strike price
= Put (K
2
; T)
. .
buy a put
+ S
0
..
buy one index

_Call (K
1
; T)
. .
buy a call
Call (K
2
; T)
. .
buy a call
_

_+PV (K
1
K
2
) =
_

_Put (K
1
; T)
. .
buy a put
Put (K
2
; T)
. .
buy a put
_

_Call (K
1
; T)
. .
buy a call
+Call (K
2
; T)
. .
sell a call
_

_+PV (K
1
K
2
) =
_

_Put (K
1
; T)
. .
buy a put
+Put (K
2
; T)
. .
sell a put
_

_Call (K
1
; T)
. .
buy a call
+Call (K
2
; T)
. .
sell a call
_

_
_

_Put (K
1
; T)
. .
buy a put
+Put (K
2
; T)
. .
sell a put
_

_ = PV (K
2
K
1
)

_Call (K
1
; T)
. .
buy a call
+Call (K
2
; T)
. .
sell a call
_

_+
_

_Put (K
1
; T)
. .
sell a put
+Put (K
2
; T)
. .
buy a put
_

_ = PV (K
2
K
1
)
The initial cost is PV (K
2
K
1
) at t = 0. The payo at expiration T = 0:5
is K
2
K
1
. The transaction is equivalent to investing PV (K
2
K
1
) in a savings
account at t = 0 and receiving K
2
K
1
at T, regardless of the S&R price at
expiration. So the transaction doesnt have any S&R price risk. We just earn
the risk free interest rate over the 6- month period.
In this problem, K
1
= 950 and K
2
= 1000
_

_Call (K
1
; T)
. .
buy a call
+Call (K
2
; T)
. .
sell a call
_

_ +
_

_Put (K
1
; T)
. .
sell a put
+Put (K
2
; T)
. .
buy a put
_

_
= PV (1000 950) = PV (50) = 50
_
1:02
1
_
= 49: 02
So the total initial cost is 49:02. The payo is 49:02 (1:02) = 50. The prot
is 0. We earn a 2% risk-free interest rate over the 6-month period.
www.actuary88.com c _Yufeng Guo 66
CHAPTER 3. INSURANCE, COLLARS, AND OTHER STRATEGIES
Problem 3.15.
a. Buy a 950-strike call and sell two 1050-strike calls
Initial cost Payo
buy a 950-strike call 120:405 max(0; S 950)
sell two 1050-strike calls 2 (71:802) = 143: 604 2 max (0; S 1050)
Total 120:405 143: 604 = 23: 199
FV (initial cost) 23: 199 (1:02) = 23: 662 98
Payo
S < 950 950 _ S < 1050 S _ 1050
buy a 950-strike call 0 S 950 S 950
sell two1050-strike calls 0 0 2 (S 1050)
Total 0 S 950 S 950 2 (S 1050) = 1150 S
The prot is:
_
_
_
0 if S < 950
S 950 if 950 _ S < 1050
1150 S if S _ 1050
+23: 66 =
_
_
_
23: 66 if S < 950
S 950 + 23: 66 if 950 _ S < 1050
1150 S + 23: 66 if S _ 1050
=
_
_
_
23: 66 if S < 950
S 926: 34 if 950 _ S < 1050
1173: 66 S if S _ 1050
850 900 950 1000 1050 1100 1150 1200
-20
0
20
40
60
80
100
120
Index Price
Profit
long 950-strike call and short two1050-strike calls
www.actuary88.com c _Yufeng Guo 67
CHAPTER 3. INSURANCE, COLLARS, AND OTHER STRATEGIES
b. Buy two 950-strike calls and sell three 1050-strike calls
Initial cost Payo
buy two 950-strike calls 2 (120:405) = 240: 81 2 max (0; S 950)
sell three 1050-strike calls 3 (71:802) = 215: 406 3 max (0; S 1050)
Total 240: 81 215: 406 = 25: 404
FV (initial cost) 25: 404 (1:02) = 25: 912 08
Payo
S < 950 950 _ S < 1050 S _ 1050
buy two 950-strike calls 0 2 (S 950) 2 (S 950)
sell three 1050-strike calls 0 0 3 (S 1050)
Total 0 2 (S 950) 2 (S 950) 3 (S 1050) = 1250 S
Prot: _
_
_
0 if S < 950
2 (S 950) if 950 _ S < 1050
1250 S if S _ 1050
25: 91 =
_
_
_
25: 91 if S < 950
2 (S 950) 25: 91 if 950 _ S < 1050
1250 S 25: 91 if S _ 1050
=
_
_
_
25: 91 if S < 950
2S 1925: 91 if 950 _ S < 1050
1224: 09 S if S _ 1050
950 1000 1050 1100 1150 1200 1250
-20
0
20
40
60
80
100
120
140
160
Index Price
Profit
long two 950-strike calls and short three1050-strike calls
www.actuary88.com c _Yufeng Guo 68
CHAPTER 3. INSURANCE, COLLARS, AND OTHER STRATEGIES
c. Buy n 950-strike calls and short m 1050-strike calls such that the initial
premium is zero.
120:405n = 71:802m
n
m
=
71:802
120:405
= 0:596 3
Problem 3.16.
A spread consists of buying one option at one strike price and selling an
otherwise identical option with a dierent strike price.
A bull spread consists of buying one option at one strike and selling an
otherwise identical option but at a higher strike price.
A bear spread consists of buying one option at one strike and selling an
otherwise identical option but at a lower strike price.
A bull spread and a bear spread will never have zero premium because the
two options dont have the same premium.
A buttery spread might have a zero net premium.
Problem 3.17.
According to http://www.daytradeteam.com/dtt/butterfly-options-trading.
asp , a buttery spread combines a bull and a bear spread. It uses three strike
prices. The lower two strike prices are used in the bull spread, and the higher
strike price in the bear spread. Both puts and calls can be used. A very large
prot is made if the stock is at or very near the middle strike price on expiration
day.
When you enter a buttery spread, you are entering 3 options orders at once.
If the stock remains or moves into a dened range, you prot, and if the stock
moves out of the desired range, you lose. The closer the stock is to the middle
strike price on expiration day, the larger your prot.
For the strike price K
1
< K
2
< K
3
K
2
= K
1
+ (1 ) K
3
So for each K
2
-strike option sold, there needs to be units of K
1
-strike
options bought and (1 ) units of K
3
-strike options bought. In this problem,
K
1
= 950, K
2
= 1020, K
3
= 1050
1020 = 950 + (1 ) 1050 = 0:3
For every ten 1020-strike calls written, there needs to be three 950-strike calls
purchased and seven 1050-strike calls purchased (so we buy three 950 1020
bull spreads and seven 1020 1050 bear spreads).
Initial cost Payo
sell ten 1020-strike calls 10 (84:47) = 844: 7 10 max (0; S 1020)
three 950-strike calls 3 (120:405) = 361: 215 3 max (0; S 950)
seven 1050-strike calls 7 (71:802) = 502: 614 7 max (0; S 1050)
Total 844: 7 + 361: 215 + 502: 614 = 19: 129
FV (initial cost) 19: 129 (1:02) = 19: 511 58
www.actuary88.com c _Yufeng Guo 69
CHAPTER 3. INSURANCE, COLLARS, AND OTHER STRATEGIES
Payo
S < 950 950 _ S < 1020 1020 _ S < 1050 1050 _ S
950-strike calls 0 3 (S 950) 3 (S 950) 3 (S 950)
1020-strike calls 0 0 10 (S 1020) 10 (S 1020)
1050-strike calls 0 0 0 7 (S 1050)
Total 0 3 (S 950) 7350 7S 0
3 (S 950) 10 (S 1020) = 7350 7S
3 (S 950) 10 (S 1020) + 7 (S 1050) = 0
The payo =
_

_
0 if S < 950
3 (S 950) if 950 _ S < 1020
7350 7S if 1020 _ S < 1050
0 if 1050 _ S
A key point to remember is that for a buttery spread K
1
< K
2
< K
3
, the
payo is zero if S _ K
1
or S _ K
3
.
The prot is:
_

_
0 if S < 950
3 (S 950) if 950 _ S < 1020
7350 7S if 1020 _ S < 1050
0 if 1050 _ S
19: 51 =
_

_
19: 51 if S < 950
3 (S 950) 19: 51 if 950 _ S < 1020
7350 7S 19: 51 if 1020 _ S < 1050
19: 51 if 1050 _ S
=
_

_
19: 51 if S < 950
3S 2869: 51 if 950 _ S < 1020
7330: 49 7S if 1020 _ S < 1050
19: 51 if 1050 _ S
www.actuary88.com c _Yufeng Guo 70
CHAPTER 3. INSURANCE, COLLARS, AND OTHER STRATEGIES
Payo =
_

_
0 if S < 950
3 (S 950) if 950 _ S < 1020
7350 7S if 1020 _ S < 1050
0 if 1050 _ S
Prot=
_

_
0 if S < 950
3 (S 950) if 950 _ S < 1020
7350 7S if 1020 _ S < 1050
0 if 1050 _ S
19: 51
850 900 950 1000 1050 1100 1150 1200
-20
0
20
40
60
80
100
120
140
160
180
200
Stock Price
Buttery spread K
1
= 950, K
2
= 1020, K
3
= 1050
The black line is the prot line. The blue line is the payo line.
www.actuary88.com c _Yufeng Guo 71
CHAPTER 3. INSURANCE, COLLARS, AND OTHER STRATEGIES
Problem 3.18.
The option price table is:
Strike Call premium Put premium
35 6:13 0:44
40 2:78 1:99
45 0:97 5:08
Time to expiration is T = 91=365 - 0:25
The annual eective rate is 8:33%
The quarterly eective rate is
4
_
1:0833 1 = 2 02%
a. Buy 35strike call, sell two 40-strike calls, and buy 45-strike call. Lets
reproduce the textbook Figure 3.14.
Initial cost
buy a 35-strike call 6:13
sell two 40-strike calls 2 (2:78) = 5: 56
buy a 45-strike call 0:97
Total 6:13 5: 56 + 0:97 = 1: 54
FV (initial cost) 1: 54 (1:0202) = 1: 571 108
Payo
S < 35 35 _ S < 40 40 _ S < 45 45 _ S
35-strike call 0 S 35 S 35 S 35
40-strike calls 0 0 2 (S 40) 2 (S 40)
45-strike call 0 0 0 S 45
Total 0 S 35 45 S 0
S 35 2 (S 40) = 45 S
S 35 2 (S 40) +S 45 = 0
The payo is:
_

_
0 if S < 35
S 35 if 35 _ S < 40
45 S if 40 _ S < 45
0 if 45 _ S
The prot is:
_

_
0 if S < 35
S 35 if 35 _ S < 40
45 S if 40 _ S < 45
0 if 45 _ S
1: 57 =
_

_
1: 57 if S < 35
S 36: 57 if 35 _ S < 40
43: 43 S if 40 _ S < 45
1: 57 if 45 _ S
www.actuary88.com c _Yufeng Guo 72
CHAPTER 3. INSURANCE, COLLARS, AND OTHER STRATEGIES
Payo=
_

_
0 if S < 35
S 35 if 35 _ S < 40
45 S if 40 _ S < 45
0 if 45 _ S
Prot=
_

_
0 if S < 35
S 35 if 35 _ S < 40
45 S if 40 _ S < 45
0 if 45 _ S
1: 57
25 30 35 40 45 50 55 60
-1
0
1
2
3
4
5
Stock Price
Black line is Payoff
Blue line is Profit
Buttery spread K
1
= 35, K
2
= 40, K
3
= 45
www.actuary88.com c _Yufeng Guo 73
CHAPTER 3. INSURANCE, COLLARS, AND OTHER STRATEGIES
b. Buy a 35strike put, sell two 40-strike puts, and buy a 45-strike put.
Lets reproduce the textbook Figure 3.14.
Initial cost
buy a 35-strike put 0:44
sell two 40-strike puts 2 (1:99) = 3: 98
buy a 45-strike put 5:08
Total 0:44 3: 98 + 5:08 = 1: 54
FV (initial cost) 1: 54 (1:0202) = 1: 571 108
Payo
S < 35 35 _ S < 40 40 _ S < 45 45 _ S
35-strike put 35 S 0 0 0
40-strike puts 2 (40 S) 2 (40 S) 0 0
45-strike put 45 S 45 S 45 S 0
Total 0 S 35 45 S 0
35 S 2 (40 S) + 45 S = 0
2 (40 S) + 45 S = S 35
The payo =
_

_
0 if S < 35
S 35 if 35 _ S < 40
45 S if 40 _ S < 45
0 if 45 _ S
The prot =
_

_
0 if S < 35
S 35 if 35 _ S < 40
45 S if 40 _ S < 45
0 if 45 _ S
1: 57 =
_

_
1: 57 if S < 35
S 36: 57 if 35 _ S < 40
43: 43 S if 40 _ S < 45
1: 57 if 45 _ S
www.actuary88.com c _Yufeng Guo 74
CHAPTER 3. INSURANCE, COLLARS, AND OTHER STRATEGIES
Payo =
_

_
0 if S < 35
S 35 if 35 _ S < 40
45 S if 40 _ S < 45
0 if 45 _ S
Prot =
_

_
0 if S < 35
S 35 if 35 _ S < 40
45 S if 40 _ S < 45
0 if 45 _ S
1: 57
25 30 35 40 45 50 55 60
-1
0
1
2
3
4
5
Stock Price
Buttery spread K
1
= 35, K
2
= 40, K
3
= 45
The black line is the payo; the blue line is the prot.
www.actuary88.com c _Yufeng Guo 75
CHAPTER 3. INSURANCE, COLLARS, AND OTHER STRATEGIES
c. Buy one stock, buy a 35 put, sell two 40 calls, and buy a 45 call.
The put-call parity is:
Call (K; T)
. .
buy a call
+ PV (K)
. .
invest PV of strike price
= Put (K; T)
. .
buy a put
+ S
0
..
buy one stock
Buy stock + buy 35 put=buy 35 call + PV(35)
Buy one stock, buy a 35 put, sell two 40 calls, and buy a 45 call is the same
as:
buy a 35 call , sell two 40 calls, and buy a 45 call, and deposit PV(35) in a
savings account.
We already know from Part a. that "buy a 35 call , sell two 40 calls, and
buy a 45 call" reproduces the textbook prot diagram Figure 3.14.
Depositing PV(35) = 35
_
1:0202
1
_
= 34: 31 wont change the prot because
any deposit in a savings account has zero prot.
Hence the prot diagram of "Buy one stock, buy a 35 put, sell two 40 calls,
and buy a 45 call" is Figure 3.14.
www.actuary88.com c _Yufeng Guo 76
CHAPTER 3. INSURANCE, COLLARS, AND OTHER STRATEGIES
Problem 3.19.
a. The parity is:
Call (K; T) Put (K; T) = PV (F
0;T
K)
We are told that Call (K; T) Put (K; T) = 0
PV (F
0;T
K) = 0 PV (F
0;T
) = PV (K)
Since PV (F
0;T
) = S
0
PV (K) = S
0
b. Buying a call and selling an otherwise identical put creates a synthetic
long forward.
c. We buy the call at the ask price and sell the put at the bid price. So we
have to pay the dealer a little more than the fair price of the call when we buy
the call from the dealer; well get less than the fair price of put when we sell
a put to the dealer. To ensure that the call premium equals the put premium
given theres a bid-ask spread, we need to make the call less valuable and the
put more valuable. To make the call less valuable and the put more valuable,
we can increase the strike price. In other words, if theres no bid-ask spread,
then K = F
0;T
. If theres bid-ask spread, K > F
0;T
.
d. A synthetic short stock position means "buy put and sell call." To have
zero net premium after the bid-ask spread, we need to make the call more
valuable and the put less valuable. To achieve this, we can decrease the strike
price. In other words, if theres no bid-ask spread, then K = F
0;T
. If theres
bid-ask spread, K < F
0;T
.
e. Transaction fees is not really a wash because theres a bid-ask spread. We
pay more if we buy an option and we get less if we sell an option.
Problem 3.20.
This problem is about building a spreadsheet. You wont be asked to build
a spreadsheet in the exam. Skip this problem.
www.actuary88.com c _Yufeng Guo 77
CHAPTER 3. INSURANCE, COLLARS, AND OTHER STRATEGIES
www.actuary88.com c _Yufeng Guo 78
Chapter 4
Introduction to risk
management
Problem 4.1.
Let
S= the price of copper per pound at T = 1
P
BH
=Prot per pound of copper at T = 1 before hedging
P
AH
=Prot per pound of copper at T = 1 after hedging
For each pound of copper produced, XYZ incurs $0.5 xed cost and $0.4
variable cost.
P
BH
= S (0:5 + 0:4) = S 0:9
XYZ sells a forward. The prot of the forward at T = 1 is:
F
0;T
S = 1 S
P
AH
= (S 0:9) + (F
0;T
S) = F
0;T
0:9
We are told that F
0;T
= 1
P
AH
= 1 0:9 = 0:1
79
CHAPTER 4. INTRODUCTION TO RISK MANAGEMENT
P
BH
= S 0:9 P
AH
= 0:1
0.7 0.8 0.9 1.0 1.1 1.2
-0.3
-0.2
-0.1
0.0
0.1
0.2
0.3
Copper price
Profit
B
e
f
o
r
e

h
e
d
g
i
n
g
After hedging
Prot: before hedging and after hedging
www.actuary88.com c _Yufeng Guo 80
CHAPTER 4. INTRODUCTION TO RISK MANAGEMENT
Problem 4.2.
P
AH
= F
0;T
0:9
If F
0;T
= 0:8 P
AH
= 0:8 0:9 = 0:1
If XYZ shuts down its production, its prot at T = 1 is 0:5 (it still has to
pay the xed cost)
If XYZ continues its production, its after hedging prot at T = 1 is 0:1
0:1 > 0:5
XYZ should continue its production
If F
0;T
= 0:45 P
AH
= 0:45 0:9 = 0:45
If XYZ shuts down its production, its prot at T = 1 is 0:5 (it still has to
pay the xed cost)
If XYZ continues its production, its after hedging prot at T = 1 is 0:45
0:45 > 0:5
XYZ should continue its production
Problem 4.3.
The prot of a long K-strike put at T = 1:
max (0; K S)FV (Premium) =
_
K S if S < K
0 if S _ K
FV (Premium)
P
AH
= P
BH
+
_
K S if S < K
0 if S _ K
FV (Premium)
= S 0:9 +
_
K S if S < K
0 if S _ K
FV (Premium)
=
_
K if S < K
S if S _ K
0:9 FV (Premium)
www.actuary88.com c _Yufeng Guo 81
CHAPTER 4. INTRODUCTION TO RISK MANAGEMENT
K = 0:95 FV (Premium) = 0:0178 (1:06) = 0:02
P
AH
=
_
0:95 if S < 0:95
S if S _ 0:95
0:92
0.6 0.7 0.8 0.9 1.0 1.1 1.2
0.04
0.06
0.08
0.10
0.12
0.14
0.16
0.18
0.20
0.22
0.24
0.26
0.28
Copper Price
Profit
Long 0.95-strike put
www.actuary88.com c _Yufeng Guo 82
CHAPTER 4. INTRODUCTION TO RISK MANAGEMENT
K = 1 FV (Premium) = 0:0376 (1:06) = 0:04
P
AH
=
_
1 if S < 1
S if S _ 1
0:9 0:04 =
_
0:06 if S < 1
S 0:94 if S _ 1
0.6 0.7 0.8 0.9 1.0 1.1 1.2
0.06
0.08
0.10
0.12
0.14
0.16
0.18
0.20
0.22
0.24
0.26
Copper Price
Profit
Long 1-strike put
www.actuary88.com c _Yufeng Guo 83
CHAPTER 4. INTRODUCTION TO RISK MANAGEMENT
K = 1:05 FV (Premium) = 0:0665 (1:06) = 0:07
P
AH
=
_
1:05 if S < 1:05
S if S _ 1:05
0:90:07 =
_
0:08 if S < 1: 05
S 0:97 if S _ 1:05
0.6 0.7 0.8 0.9 1.0 1.1 1.2
0.08
0.10
0.12
0.14
0.16
0.18
0.20
0.22
Copper Price
Profit
Long 1.05-strike put
www.actuary88.com c _Yufeng Guo 84
CHAPTER 4. INTRODUCTION TO RISK MANAGEMENT
Problem 4.4.
The prot of a short K-strike call at T = 1:
max (0; S K)+FV (Premium) =
_
0 if S < K
S K if S _ K
+FV (Premium)
P
AH
= P
BH

_
0 if S < K
S K if S _ K
+FV (Premium)
= S 0:9
_
0 if S < K
S K if S _ K
+FV (Premium)
=
_
S if S < K
K if S _ K
0:9 +FV (Premium)
www.actuary88.com c _Yufeng Guo 85
CHAPTER 4. INTRODUCTION TO RISK MANAGEMENT
K = 0:95 FV (Premium) = 0:0649 (1:06) = 0:07
P
AH
=
_
S if S < 0:95
0:95 if S _ 0:95
0:9+0:07 =
_
S 0:83 if S < 0:95
0:12 if 0:95 _ S
0.7 0.8 0.9 1.0 1.1 1.2
-0.2
-0.1
0.0
0.1
Copper Price
Profit
Short 0.95-strike call
www.actuary88.com c _Yufeng Guo 86
CHAPTER 4. INTRODUCTION TO RISK MANAGEMENT
K = 1 FV (Premium) = 0:0376 (1:06) = 0:04
P
AH
=
_
S if S < 1
1 if S _ 1
0:9 + 0:04 =
_
S 0:86 if S < 1
0:14 if 1 _ S
0.7 0.8 0.9 1.0 1.1 1.2
-0.2
-0.1
0.0
0.1
Copper Price
Profit
Short 1-strike call
www.actuary88.com c _Yufeng Guo 87
CHAPTER 4. INTRODUCTION TO RISK MANAGEMENT
K = 1:05 FV (Premium) = 0:0194 (1:06) = 0:02
P
AH
=
_
S if S < 1:05
1:05 if S _ 1:05
0:9+0:02 =
_
S 0:88 if S < 1: 05
0:17 if 1: 05 _ S
0.7 0.8 0.9 1.0 1.1 1.2
-0.2
-0.1
0.0
0.1
Copper Price
Profit
Short 1.05-strike call
www.actuary88.com c _Yufeng Guo 88
CHAPTER 4. INTRODUCTION TO RISK MANAGEMENT
Problem 4.5.
P
AH
= P
BH
+P
Collar
P
BH
= S 0:9
Suppose XYZ buy a K
1
-strike put and sells a K
2
-strike call.
The prot of the collar is:
P
Collar
= Payoff FV (Net Initial Premium)
= [max (0; K
1
S) max (0; S K
2
)] FV (Put Premium) + FV (Call
Premium)
a. Buy 0.95-strike put and sell 1-strike call
The payo is max (0; 0:95 S) max (0; S 1)
S < 0:95 0:95 _ S < 1 S _ 1
long 0.95-strike put 0:95 S 0 0
short 1-strike call 0 0 1 S
Total 0:95 S 0 1 S
FV (Put Premium) +FV (Call Premium)
= (0:0178 + 0:0376) 1:06 = 0:02
P
Collar
=
_
_
_
0:95 S if S < 0:95
0 if 0:95 _ S < 1
1 S if S _ 1
+ 0:02
P
AH
= P
BH
+P
Collar
= S 0:9 +
_
_
_
0:95 S if S < 0:95
0 if 0:95 _ S < 1
1 S if S _ 1
+ 0:02
=
_
_
_
0:07 if S < 0:95
S 0:88 if 0:95 _ S < 1
0:12 if 1 _ S
www.actuary88.com c _Yufeng Guo 89
CHAPTER 4. INTRODUCTION TO RISK MANAGEMENT
P
AH
=
_
_
_
0:07 if S < 0:95
S 0:88 if 0:95 _ S < 1
0:12 if 1 _ S
0.6 0.7 0.8 0.9 1.0 1.1 1.2
0.07
0.08
0.09
0.10
0.11
0.12
Copper Price
Profit
Long 0.95-strike put and short 1-strike call
www.actuary88.com c _Yufeng Guo 90
CHAPTER 4. INTRODUCTION TO RISK MANAGEMENT
b. Buy 0.975-strike put and sell 1.025-strike call
The payo is max (0; 0:975 S) max (0; S 1:025)
S < 0:975 0:975 _ S < 1:025 S _ 1:025
long 0.975-strike put 0:975 S 0 0
short 1-strike call 0 0 1:025 S
Total 0:975 S 0 1:025 S
FV (Put Premium) +FV (Call Premium)
= (0:0265 + 0:0274) 1:06 = 0:000 954 = 0:001
P
Collar
=
_
_
_
0:975 S if S < 0:975
0 if 0:975 _ S < 1:025
1:025 S if S _ 1:025
+ 0:001
P
AH
= P
BH
+P
Collar
= S 0:9 +
_
_
_
0:975 S if S < 0:975
0 if 0:975 _ S < 1:025
1:025 S if S _ 1:025
+ 0:001
=
_
_
_
0:076 if S < 0:975
S 0:899 if 0:975 _ S < 1:025
0:126 if 1: 025 _ S
www.actuary88.com c _Yufeng Guo 91
CHAPTER 4. INTRODUCTION TO RISK MANAGEMENT
P
AH
=
_
_
_
0:076 if S < 0:975
S 0:899 if 0:975 _ S < 1:025
0:126 if 1: 025 _ S
0.6 0.7 0.8 0.9 1.0 1.1 1.2
0.08
0.09
0.10
0.11
0.12
Copper Price
Profit
Long 0.975-strike put and short 1.025-strike call
www.actuary88.com c _Yufeng Guo 92
CHAPTER 4. INTRODUCTION TO RISK MANAGEMENT
c. Buy 1.05-strike put and sell 1.05-strike call
The payo is max (0; 1:05 S) max (0; S 1:05)
S < 1:05 S _ 1:05
long 1.05-strike put 1:05 S 0
short 1.05-strike call 0 1:05 S
Total 1:05 S 1:05 S
FV (Put Premium) +FV (Call Premium)
= (0:0665 + 0:0194) 1:06 = 0:05
P
Collar
= (1:05 S) 0:05 = 1 S
P
AH
= P
BH
+P
Collar
= S 0:9 + 1 S = 0:1
www.actuary88.com c _Yufeng Guo 93
CHAPTER 4. INTRODUCTION TO RISK MANAGEMENT
P
AH
= 0:1
0.7 0.8 0.9 1.0 1.1 1.2
-0.8
-0.6
-0.4
-0.2
0.0
0.2
0.4
0.6
0.8
1.0
Copper Price
Profit
Long 1.05-strike put and short 1.05-strike call
www.actuary88.com c _Yufeng Guo 94
CHAPTER 4. INTRODUCTION TO RISK MANAGEMENT
Problem 4.6.
a. Sell one 1.025-strike put and buy two 0.975-strike puts
The payo is 2 max (0; 0:975 S) max (0; 1:025 S)
S < 0:975 0:975 _ S < 1:025 S _ 1:025
long two 0.975-strike puts 2 (0:975 S) 0 0
short 1.025-strike put S 1:025 S 1:025 0
Total 0:925 S S 1:025 0
Initial net premium paid=2 (0:0265) 0:0509 = 0:002 1
Future value: 0:002 1 (1:06) = 0:002 2
P
Paylater
=
_
_
_
0:925 S if S < 0:975
S 1:025 if 0:975 _ S < 1:025
0 if S _ 1:025
0:002 2
P
AH
= P
BH
+P
Paylater
= S 0:9 +
_
_
_
0:925 S if S < 0:975
S 1:025 if 0:975 _ S < 1:025
0 if S _ 1:025
0:002 2
=
_
_
_
0:022 8 if S < 0:975
2S 1: 927 2 if 0:975 _ S < 1:025
S 0:902 2 if 1: 025 _ S
www.actuary88.com c _Yufeng Guo 95
CHAPTER 4. INTRODUCTION TO RISK MANAGEMENT
P
AH
=
_
_
_
0:022 8 if S < 0:975
2S 1: 927 2 if 0:975 _ S < 1:025
S 0:902 2 if 1: 025 _ S
0.7 0.8 0.9 1.0 1.1 1.2
-0.3
-0.2
-0.1
0.0
0.1
0.2
0.3
Copper Price
Profit
Hedged Profit
Unhedged Profit
www.actuary88.com c _Yufeng Guo 96
CHAPTER 4. INTRODUCTION TO RISK MANAGEMENT
b. Sell two 1.034-strike puts and buy three 1-strike puts
The payo is 3 max (0; 1 S) 2 max (0; 1:034 S)
S < 1 1 _ S < 1:034 S _ 1:034
long three 1-strike puts 3 (1 S) 0 0
short two1.034-strike puts 2 (S 1:034) 2 (S 1:034) 0
Total 3 (1 S) + 2 (S 1:034) = 0:932 S 2 (S 1:034) 0
Initial premium paid: 3 (0:0376) 2 (0:0563) = 0:000 2
Future value: 0:000 2 (1:06) = 0:000 212
Prot:
_
_
_
0:932 S if S < 1
2 (S 1:034) if 1 _ S < 1:034
0 if 1:034 _ S
0:000 2
P
AH
= P
BH
+P
Paylater
= S 0:9 +
_
_
_
0:932 S if S < 1
2 (S 1:034) if 1 _ S < 1:034
0 if 1:034 _ S
0:000 2
=
_
_
_
0:031 8 if S < 1
3S 2: 968 2 if 1 _ S < 1:034
S 0:900 2 if 1: 034 _ S
www.actuary88.com c _Yufeng Guo 97
CHAPTER 4. INTRODUCTION TO RISK MANAGEMENT
P
AH
=
_
_
_
0:031 8 if S < 1
3S 2: 968 2 if 1 _ S < 1:034
S 0:900 2 if 1: 034 _ S
0.7 0.8 0.9 1.0 1.1 1.2
-0.3
-0.2
-0.1
0.0
0.1
0.2
0.3
Copper Price
Profit
Hedged Profit
Unhedged Profit
www.actuary88.com c _Yufeng Guo 98
CHAPTER 4. INTRODUCTION TO RISK MANAGEMENT
Problem 4.7.
Telco buys copper wires from Wirco. Telcos purchase price per unit of wire
is $5 plus the price of copper. Telco collects $6.2 per unit of wire used.
Telcos unhedged prot is:
P
BH
= 6:2 (5 +S) = 1: 2 S
If Telco buys a copper forward, the prot from the forward at T = 1 is
S F
0;T
The prot after hedging is
P
AH
= P
BH
+S F
0;T
= 1: 2 S +S F
0;T
= 1: 2 F
0;T
F
0;T
= 1 P
AH
= 1: 2 1 = 0:2
www.actuary88.com c _Yufeng Guo 99
CHAPTER 4. INTRODUCTION TO RISK MANAGEMENT
P
AH
= 1: 2 F
0;T
P
AH
= 1: 2 1 = 0:2
0.6 0.7 0.8 0.9 1.0 1.1 1.2
0.0
0.1
0.2
0.3
0.4
0.5
0.6
Copper Price
Profit
Hedged Profit
U
n
h
e
d
g
e
d

P
r
o
f
i
t
www.actuary88.com c _Yufeng Guo 100
CHAPTER 4. INTRODUCTION TO RISK MANAGEMENT
Problem 4.8.
P
BH
= 1: 2 S
Telco buys a K-strike call. The prot from the long call is:
max (0; S K) FV (Premium)
P
AH
= 1: 2 S + max (0; S K) FV (Premium)
= 1: 2 S +
_
0 if S < K
S K if S _ K
FV (Premium)
P
AH
= 1: 2 FV (Premium)
_
S if S < K
K if S _ K
www.actuary88.com c _Yufeng Guo 101
CHAPTER 4. INTRODUCTION TO RISK MANAGEMENT
a. K = 0:95 FV (Premium) = 0:0649 (1:06) = 0:069
P
AH
= 1: 20:069
_
S if S < 0:95
0:95 if S _ 0:95
=
_
1: 131 S if S < 0:95
0:181 if S _ 0:95
0.6 0.7 0.8 0.9 1.0 1.1 1.2 1.3 1.4
0.2
0.3
0.4
0.5
Copper Price
Profit
www.actuary88.com c _Yufeng Guo 102
CHAPTER 4. INTRODUCTION TO RISK MANAGEMENT
b. K = 1 PV (Premium) = 0:0376 (1:06) = 0:039 856
P
AH
= 1: 2 0:04
_
S if S < 1
1 if S _ 1
=
_
1: 16 S if S < 1
0:16 if 1 _ S
0.6 0.7 0.8 0.9 1.0 1.1 1.2 1.3 1.4
0.2
0.3
0.4
0.5
Copper Price
Profit
www.actuary88.com c _Yufeng Guo 103
CHAPTER 4. INTRODUCTION TO RISK MANAGEMENT
c. K = 1:05 FV (Premium) = 0:0194 (1:06) = 0:020 564
P
AH
= 1: 20:02
_
S if S < 1:05
1:05 if S _ 1:05
=
_
1: 18 S if S < 1: 05
0:13 if 1: 05 _ S
0.6 0.7 0.8 0.9 1.0 1.1 1.2 1.3 1.4
0.2
0.3
0.4
0.5
Copper Price
Profit
www.actuary88.com c _Yufeng Guo 104
CHAPTER 4. INTRODUCTION TO RISK MANAGEMENT
Problem 4.9.
P
BH
= 1: 2 S
Telco sells a put option with strike price K:
The prot from the written put is:
max (0; K S) +FV (Premium)
P
AH
= 1: 2 S max (0; K S) +FV (Premium)
= 1: 2 S
_
K S if S < K
0 if S _ K
+FV (Premium)
=
_
K if S < K
S if K _ S
+ 1: 2 +FV (Premium)
P
AH
=
_
K if S < K
S if K _ S
+ 1: 2 +FV (Premium)
www.actuary88.com c _Yufeng Guo 105
CHAPTER 4. INTRODUCTION TO RISK MANAGEMENT
a. K = 0:95 FV (Premium) = 0:0178 (1:06) = 0:018 868
P
AH
=
_
0:95 if S < 0:95
S if 0:95 _ S
+1: 2+0:019 =
_
0:269 if S < 0:95
1: 219 S if 0:95 _ S
0.7 0.8 0.9 1.0 1.1 1.2 1.3 1.4
-0.1
0.0
0.1
0.2
Copper Price
Profit
www.actuary88.com c _Yufeng Guo 106
CHAPTER 4. INTRODUCTION TO RISK MANAGEMENT
b. K = 1 FV (Premium) = 0:0376 (1:06) = 0:039 856
P
AH
=
_
1 if S < 1
S if 1 _ S
+ 1: 2 + 0:04 =
_
0:24 if S < 1
1: 24 S if 1 _ S
0.7 0.8 0.9 1.0 1.1 1.2 1.3 1.4
-0.1
0.0
0.1
0.2
Copper Price
Profit
www.actuary88.com c _Yufeng Guo 107
CHAPTER 4. INTRODUCTION TO RISK MANAGEMENT
c. K = 1:05 FV (Premium) = 0:0665 (1:06) = 0:070 49
P
AH
=
_
1:05 if S < 1:05
S if 1:05 _ S
+1: 2+0:07 =
_
0:22 if S < 1: 05
1: 27 S if 1: 05 _ S
0.7 0.8 0.9 1.0 1.1 1.2 1.3 1.4
-0.1
0.0
0.1
0.2
Copper Price
Profit
www.actuary88.com c _Yufeng Guo 108
CHAPTER 4. INTRODUCTION TO RISK MANAGEMENT
Problem 4.10.
Suppose Telco sells a K
1
-strike put and buys a K
2
-strike call.
The prot of the collar is:
P
Collar
= Payoff FV (Net Initial Premium)
= [max (0; K
1
S) + max (0; S K
2
)] + FV (Put Premium) FV (Call
Premium)
a. Sell 0.95-strike put and buy 1-strike call
The payo is max (0; 0:95 S) + max (0; S 1)
S < 0:95 0:95 _ S < 1 S _ 1
short 0.95-strike put (0:95 S) 0 0
long 1-strike call 0 0 (1 S)
Total (0:95 S) 0 (1 S)
FV (Put Premium)FV (Call Premium) = (0:0178 0:0376) 1:06 = 0:02
P
Collar
=
_
_
_
S 0:95 if S < 0:95
0 if 0:95 _ S < 1
S 1 if S _ 1
0:02
P
AH
= P
BH
+P
Collar
= 1: 2S+
_
_
_
S 0:95 if S < 0:95
0 if 0:95 _ S < 1
S 1 if S _ 1
0:02 =
_
_
_
0:23 if S < 0:95
1: 18 S if 0:95 _ S < 1
0:18 if 1 _ S
www.actuary88.com c _Yufeng Guo 109
CHAPTER 4. INTRODUCTION TO RISK MANAGEMENT
P
AH
=
_
_
_
0:23 if S < 0:95
1: 18 S if 0:95 _ S < 1
0:18 if 1 _ S
0.7 0.8 0.9 1.0 1.1 1.2 1.3 1.4
0.18
0.19
0.20
0.21
0.22
0.23
Copper Price
Profit
Short 0.95-strike put and long 1-strike call
www.actuary88.com c _Yufeng Guo 110
CHAPTER 4. INTRODUCTION TO RISK MANAGEMENT
b. Sell 0.975-strike put and buy 1.025-strike call
The payo is max (0; 0:975 S) max (0; S 1:025)
S < 0:975 0:975 _ S < 1:025 S _ 1:025
short 0.975-strike put (0:975 S) 0 0
long 1-strike call 0 0 (1:025 S)
Total (0:975 S) 0 (1:025 S)
FV (Put Premium)FV (Call Premium) = (0:0265 0:0274) 1:06 = 0:001
P
Collar
=
_
_
_
S 0:975 if S < 0:975
0 if 0:975 _ S < 1:025
S 1:025 if S _ 1:025
0:001
P
AH
= P
BH
+P
Collar
= 1: 2 S +
_
_
_
S 0:975 if S < 0:975
0 if 0:975 _ S < 1:025
S 1:025 if S _ 1:025
0:001
=
_
_
_
0:224 if S < 0:975
S + 1: 199 if 0:975 _ S < 1:025
0:174 if 1: 025 _ S
www.actuary88.com c _Yufeng Guo 111
CHAPTER 4. INTRODUCTION TO RISK MANAGEMENT
P
AH
=
_
_
_
0:224 if S < 0:975
S + 1: 199 if 0:975 _ S < 1:025
0:174 if 1: 025 _ S
0.7 0.8 0.9 1.0 1.1 1.2 1.3 1.4
0.18
0.19
0.20
0.21
0.22
Copper Price
Profit
Short 0.95-strike put and long 1-strike call
www.actuary88.com c _Yufeng Guo 112
CHAPTER 4. INTRODUCTION TO RISK MANAGEMENT
c. Sell 0.95-strike put and buy 0.95-strike call
The payo is max (0; 0:95 S) + max (0; S 0:95)
S < 0:95 S _ 0:95
short 0.95-strike put (0:95 S) 0
long 0.95-strike call 0 S 0:95
Total S 0:95 S 0:95
FV (Put Premium)FV (Call Premium) = (0:0178 0:0649) 1:06 = 0:05
P
Collar
= (S 0:95) 0:05 = S 1
P
AH
= P
BH
+P
Collar
= 1: 2 S +S 1 = 0:2
www.actuary88.com c _Yufeng Guo 113
CHAPTER 4. INTRODUCTION TO RISK MANAGEMENT
P
AH
= 0:2
0.7 0.8 0.9 1.0 1.1 1.2
-0.8
-0.6
-0.4
-0.2
0.0
0.2
0.4
0.6
0.8
1.0
1.2
Copper Price
Profit
Short 0.95-strike put and long 0.95-strike call
www.actuary88.com c _Yufeng Guo 114
CHAPTER 4. INTRODUCTION TO RISK MANAGEMENT
Problem 4.11.
a. sell 0:975-strike call and buy two 1:034-strike calls
The payo is 2 max (0; S 1:034) max (0; S 0:975)
S < 0:975 0:975 _ S < 1:034 S _ 1:034
short 0.975-strike call 0 (S 0:975) (S 0:975)
long 1.034-strike calls 0 0 2 (S 1:034)
Total 0 (S 0:975) S 1: 093
(S 0:975) + 2 (S 1:034) = S 1: 093
The payo=
_
_
_
0 if S < 0:975
0:975 S if 0:975 _ S < 1:034
S 1: 093 if S _ 1:034
Initial net premium paid=2 (0:0243) 0:05 = 0:001 4
Future value: 0:001 4 (1:06) = 0:001 484 - 0:001
P
Paylater
=
_
_
_
0 if S < 0:975
0:975 S if 0:975 _ S < 1:034
S 1: 093 if S _ 1:034
+ 0:001
=
_
_
_
0:001 if S < 0:975
0:976 S if 0:975 _ S < 1:034
S 1: 092 if 1: 034 _ S
P
AH
= P
BH
+P
Paylater
= 1: 2 S +
_
_
_
0:001 if S < 0:975
0:976 S if 0:975 _ S < 1:034
S 1: 092 if 1: 034 _ S
=
_
_
_
1: 201 S if S < 0:975
2: 176 2S if 0:975 _ S < 1:034
0:108 if 1: 034 _ S
www.actuary88.com c _Yufeng Guo 115
CHAPTER 4. INTRODUCTION TO RISK MANAGEMENT
P
AH
=
_
_
_
1: 201 S if S < 0:975
2: 176 2S if 0:975 _ S < 1:034
0:108 if 1: 034 _ S
0.8 0.9 1.0 1.1 1.2 1.3 1.4
-0.2
-0.1
0.0
0.1
0.2
0.3
0.4
0.5
Copper Price
Profit
Hedged Profit
U
n
h
e
d
g
e
d

P
r
o
f
i
t
www.actuary88.com c _Yufeng Guo 116
CHAPTER 4. INTRODUCTION TO RISK MANAGEMENT
b. sell two 1-strike calls and buy three 1:034-strike calls
The payo is 2 max (0; S 1) + 3 max (0; S 1:034)
S < 1 1 _ S < 1:034 S _ 1:034
sell two 1-strike calls 0 2 (S 1) 2 (S 1)
buy three 1:034-strike calls 0 0 3 (S 1:034)
Total 0 2 (S 1) S 1: 102
2 (S 1) + 3 (S 1:034) = S 1: 102
The payo=
_
_
_
0 if S < 1
2 (S 1) if 1 _ S < 1:034
S 1: 102 if S _ 1:034
Initial net premium paid=3 (0:0243) 2 (0:0376) = 0:002 3
Future value: 0:002 3 (1:06) = 0:002 438 - 0:002 4
P
Paylater
=
_
_
_
0 if S < 1
2 (S 1) if 1 _ S < 1:034
S 1: 102 if S _ 1:034
+ 0:002 4
P
AH
= P
BH
+P
Paylater
= 1: 2 S +
_
_
_
0 if S < 1
2 (S 1) if 1 _ S < 1:034
S 1: 102 if S _ 1:034
+ 0:002 4
=
_
_
_
1: 202 4 S if S < 1
3: 202 4 3S if 1 _ S < 1:034
0:100 4 if 1: 034 _ S
www.actuary88.com c _Yufeng Guo 117
CHAPTER 4. INTRODUCTION TO RISK MANAGEMENT
P
AH
=
_
_
_
1: 202 4 S if S < 1
3: 202 4 3S if 1 _ S < 1:034
0:100 4 if 1: 034 _ S
0.8 0.9 1.0 1.1 1.2 1.3 1.4
-0.2
-0.1
0.0
0.1
0.2
0.3
0.4
0.5
Copper Price
Profit
Hedged Profit
U
n
h
e
d
g
e
d

P
r
o
f
i
t
www.actuary88.com c _Yufeng Guo 118
CHAPTER 4. INTRODUCTION TO RISK MANAGEMENT
Problem 4.12.
Wircos total prot per unit of wire produced:
Revenue. S + 5, where S is the price of copper
Copper cost is S
Fixed cost is 3
Variable cost 1.5
Prot before hedging: P
BH
= S + 5 (S + 3 + 1:5) = 0:5
The prot is xed regardless of copper price.
If Wirco buys a copper forward, this will introduce the copper price risk (i.e.
Wircos prot will now depend on the copper price).
Wirco buys a copper forward. The prot from the forward is S F
0;T
The prot after hedging is
P
AH
= P
BH
+S F
0;T
= 0:5 +S F
0;T
If F
0;T
= 1 P
AH
= 0:5 +S 1 = S 0:5
Now you the prot depends on S. If S goes down, the prot goes down.
Problem 4.13.
The unhedged prot is P
BH
= 0:5. This doesnt depend on the copper price
at T = 1. However, if Wirco uses any derivatives (call, put, forward, etc.), this
will make the hedged prot as a function of the copper price at T = 1. Using
any derivatives will make the hedged prot uctuate with the copper price,
increasing the variability of the prot.
Problem 4.14.
The question "Did the rm earn $10 in prot (relative to accounting break-
even) or lose $30 in prot (relative to the prot that could be obtained by
hedging?" portrays derivatives a way to make prot. However, most companies
use derivatives to manage their risks, not to seek additional prot. If they have
idle money, they would rather invest in their core business than buy call or
put options to make money on stocks. This is all you need to know about this
question.
Problem 4.15.
Price=$9 Price=$11:20
Pre-tax operating income $1 $1:2
Tax at 40% 1 (0:4) = 0:4 1:2 (0:4) = 0:48
After tax income 1 (0:4) = 0:6 1:2 0:48 = 0:72
Because losses are fully tax deductible, we pay 0:4 tax (i.e. IRS will send
us a check of 0:4)
Expected prot is: 0:5 (0:6 + 0:72) = 0:06
www.actuary88.com c _Yufeng Guo 119
CHAPTER 4. INTRODUCTION TO RISK MANAGEMENT
Problem 4.16.
a. Expected pre-tax prot:
Firm A: 0:5 (1000 600) = 200
Firm B: 0:5 (300 + 100) = 200
b. Expected after-tax prot:
Firm A
Good State Bad State
Pre-tax income 1000 600
Tax at 40% 1000 (0:4) = 400 600 (0:4) = 240
After tax income 1000 400 = 600 600 (240) = 360
Expected after-tax prot: 0:5 (600 360) = 120
Firm B
Good State Bad State
Pre-tax income 300 100
Tax at 40% 300 (0:4) = 120 100 (0:4) = 40
After tax income 300 120 = 180 100 40 = 60
Expected after-tax prot: 0:5 (180 + 60) = 120
Problem 4.17.
a. Expected pre-tax prot:
Firm A: 0:5 (1000 600) = 200
Firm B: 0:5 (300 + 100) = 200
b. Expected after-tax prot:
Firm A
Good State Bad State
Pre-tax income 1000 600
Tax 1000 (0:4) = 400 0
After tax income 1000 400 = 600 600
Expected after-tax prot: 0:5 (600 600) = 0
Firm B
Good State Bad State
Pre-tax income 300 100
Tax at 40% 300 (0:4) = 120 100 (0:4) = 40
After tax income 300 120 = 180 100 40 = 60
Expected after-tax prot: 0:5 (180 + 60) = 120
c. This question is vague. Im not sure to whom A or B might pay. This is
what I guess the author wants us to answer:
www.actuary88.com c _Yufeng Guo 120
CHAPTER 4. INTRODUCTION TO RISK MANAGEMENT
The expected cash ow Company A receives depends on the taw law.
E
_
Profit
A
_
=
_
120 if loss is tax deductible
0 if loss is not tax deductible
Suppose A is unsure about the IRSs tax policy (i.e. not sure whether the
loss is deductible or not next year). Then the present value of the dierence of
the tax law is
120=1:1 = 109: 09. So the eect of the tax law has a present value 109.09.
Company B doesnt have any loss. Its after tax prot doesnt depend on
whether a loss is tax deductible. So the eect of the tax law has a present value
of zero.
Problem 4.18.
We are given: r = = 4:879% T = 1
F
0;T
= S
0
e
(r)T
= 420 S
0
= 420
Call strike price K
C
= 440; put strike price K
P
= 400
First, nd the call and put premiums. Install the CD contained in the
textbook Derivatives Markets in your computer. Open the spreadsheet titled
"optbasic2." Enter:
Inputs
Stock Price 420
Exercise Price 440
Volatility 5:500%
Risk-free interest rate 4:879%
Time to Expiration (years) 1
Dividend Yield 4:879%
You should get the call premium: C = 2:4944
Inputs
Stock Price 420
Exercise Price 400
Volatility 5:500%
Risk-free interest rate 4:879%
Time to Expiration (years) 1
Dividend Yield 4:879%
You should get the put premium: P = 2:2072
a. Buy 440-strike call and sell a 440-put
Let S represent the gold price at the option expiration date T = 1.
www.actuary88.com c _Yufeng Guo 121
CHAPTER 4. INTRODUCTION TO RISK MANAGEMENT
The payo is:
S < 400 400 _ S < 440 S _ 440
Buy 440-strike call 0 0 S 440
Sell 400-strike put (400 S) 0 0
Total S 400 0 S 440
Payo=
_
_
_
S 400 if S < 400
0 if 400 _ S < 440
S 440 if S _ 440
The initial cost of the collar is:
Premium = 2:4944 2:2072 = 0:287 2
FV (Premium) = 0:287 2e
0:04879(1)
= 0:30
So the prot from the collar is:
P
Collar
=
_
_
_
S 400 if S < 400
0 if 400 _ S < 440
S 440 if S _ 440
0:30
The prot before hedging:
Auric sells each widget for $800
It has xed cost: $340
Input (gold) cost: S
Prot before hedging: P
BH
= 800 (340 +S) = 460 S
www.actuary88.com c _Yufeng Guo 122
CHAPTER 4. INTRODUCTION TO RISK MANAGEMENT
So Aurics prot after hedging:
P
AH
= P
BH
+P
Collar
= 460S+
_
_
_
S 400 if S < 400
0 if 400 _ S < 440
S 440 if S _ 440
0:30 =
_
_
_
59: 7 if S < 400
459: 7 S if 400 _ S < 440
19: 7 if 440 _ S
360 370 380 390 400 410 420 430 440 450 460 470 480 490 500
20
30
40
50
60
Gold Price
Profit
www.actuary88.com c _Yufeng Guo 123
CHAPTER 4. INTRODUCTION TO RISK MANAGEMENT
b. We need to nd
_
C = P
K
C
K
P
= 30
Using the spreadsheet "optbasic2," after trial-and-error, we nd that:
K
C
= 435:52 K
P
= 405:52
C = 3:4264 P = 3:4234 C - P
Let K
C
and K
P
represent the strike price for the call and the put.
Collar width 30 K
C
K
P
= 30
Let C and P represent the call and put premium calculated by the Black-
Scholes formula
When we buy the call, we pay C
0
= C + 0:25
When we sell the put, we get P
0
= P 0:25
Zero collarC
0
= P
0
C + 0:25 = P 0:25 C = P 0:5
So we need to nd C and P such
_
C = P 0:5
K
C
K
P
= 30
This is all the concepts you need to know about this problem.
Using the spreadsheet "optbasic2," after trial-and-error, we nd that:
K
C
= 436:53 K
P
= 406:53
C = 3:1938 P = 3:6938
Problem 4.19.
a. Sell 440-strike call and buy two K-strike calls such the net premium is
zero.
The 440-strike call premium is: C
440
= 2:4944
We need to nd K such that C
440
2C
K
= 0
2:4944 2C
K
= 0 C
K
= 2:4944=2 = 1: 247 2
We know that K > 440 (otherwise C
K
_ C
440
)
Using the spreadsheet "optbasic2," after trial-and-error, we nd that:
K = 448:93 C
K
= 1:2469 - 1: 247 2
b. Prot before hedging: P
BH
= 800 (340 +S) = 460 S
Zero cost collar Prot = Payo
The payo is:
S < 440 440 _ S < 448:93 S _ 448:93
sell 440-strike call 0 (S 440) (S 440)
buy two 448:93-strike calls 0 0 2 (S 448:93)
Total 0 440 S S 457: 86
www.actuary88.com c _Yufeng Guo 124
CHAPTER 4. INTRODUCTION TO RISK MANAGEMENT
(S 440) + 2 (S 448:93) = S 457: 86
P
Collar
=
_
_
_
0 if S < 440
440 S if 440 _ S < 448:93
S 457: 86 if S _ 448:93
So Aurics prot after hedging:
P
AH
= P
BH
+P
Collar
= 460S+
_
_
_
0 if S < 440
440 S if 440 _ S < 448:93
S 457: 86 if S _ 448:93
=
_
_
_
460 S if S < 440
900 2S if 440 _ S < 448:93
2: 14 if 448: 93 _ S
www.actuary88.com c _Yufeng Guo 125
CHAPTER 4. INTRODUCTION TO RISK MANAGEMENT
P
AH
=
_
_
_
460 S if S < 440
900 2S if 440 _ S < 448:93
2: 14 if 448: 93 _ S
P
BH
= 460 S
410 420 430 440 450 460 470 480 490 500
-40
-30
-20
-10
0
10
20
30
40
50
60
Gold Price
Profit
Hedged profit
Unhedged Profit
www.actuary88.com c _Yufeng Guo 126
CHAPTER 4. INTRODUCTION TO RISK MANAGEMENT
Problem 4.20.
Ignore. Not on the FM syllabus.
Problem 4.21.
Ignore. Not on the FM syllabus.
Problem 4.22.
Ignore. Not on the FM syllabus.
Problem 4.23.
Ignore. Not on the FM syllabus.
Problem 4.24.
Ignore. Not on the FM syllabus.
Problem 4.25.
Ignore. Not on the FM syllabus.
www.actuary88.com c _Yufeng Guo 127
CHAPTER 4. INTRODUCTION TO RISK MANAGEMENT
www.actuary88.com c _Yufeng Guo 128
Chapter 5
Financial forwards and
futures
Problem 5.1.
Description Get paid at time Deliver asset at time payment
Sell asset outright 0 0 S
0
at t = 0
Sell asset through loan T 0 S
0
e
rT
at T
Short prepaid forward 0 T S
0
at t = 0
Short forward T T S
0
e
rT
at T
Problem 5.2.
a. F
P
0;T
= S
0
e
T
PV (Div)
= 50e
0(1)

_
e
0:06(3=12)
+e
0:06(6=12)
+e
0:06(9=12)
+e
0:06(12=12)

= 46: 146 7
b. F
0;T
= F
P
0;T
e
rT
= 46: 146 7e
0:06(1)
= 49: 000 3
Problem 5.3.
a. F
P
0;T
= S
0
e
T
= 50e
0:08(1)
= 46: 155 8
b. F
0;T
= F
P
0;T
e
rT
= 46: 155 8e
0:06(1)
= 49: 009 9
Problem 5.4.
129
CHAPTER 5. FINANCIAL FORWARDS AND FUTURES
a. F
0;T
= S
0
e
(r)T
= 35e
(0:050)0:5
= 35: 886 0
b.
1
T
ln
F
0;T
S
0
=
1
0:5
ln
35:5
35
= 0:02837
c. F
0;T
= S
0
e
(r)T
35:5 = 35e
(0:05)0:5
= 2: 163%
www.actuary88.com c _Yufeng Guo 130
CHAPTER 5. FINANCIAL FORWARDS AND FUTURES
Problem 5.5.
a. F
0;T
= S
0
e
(r)T
= 1100e
(0:050)9=12
= 1142: 033 2
b. As a buyer in a forward contract, we face the risk that the index may be
worth zero at T (i.e. S
T
= 0), yet we still have to pay F
0;T
to buy it. This is
how to hedge our risk:
Transactions t = 0 T
long a forward (i.e. be a buyer in forward) 0 S
T
F
0;T
short sell an index S
0
S
T
deposit S
0
in savings account S
0
S
0
e
rT
Total 0 S
0
e
rT
F
0;T
For this problem:
Transactions t = 0 T
buy a forward 0 S
T
1142: 03
short sell an index 1100 S
T
deposit S
0
in savings account 1100 1100e
(0:05)9=12
= 1142: 03
Total 0 0
After hedging, our prot is zero.
c. As a seller in the forward contract, we face the risk that S
T
= . If
S
T
= and we dont already have an index on hand for delivery at T, we have
to pay S
T
= and buy an index from the open market. Well be bankrupt.
This is how to hedge:
Transactions t = 0 T
sell a forward (i.e. be a seller in forward) 0 F
0;T
S
T
buy an index S
0
S
T
borrow S
0
S
0
S
0
e
rT
Total 0 F
0;T
S
0
e
rT
For this problem:
Transactions t = 0 T
sell a forward (i.e. be a seller in forward) 0 1142: 03 S
T
buy an index 1100 S
T
borrow S
0
1100 1100e
(0:05)9=12
= 1142: 03
Total 0 0
www.actuary88.com c _Yufeng Guo 131
CHAPTER 5. FINANCIAL FORWARDS AND FUTURES
Problem 5.6.
a. F
0;T
= S
0
e
(r)T
= 1100e
(0:050:015)9=12
= 1129: 26
b.
Transactions t = 0 T
long a forward (i.e. be a buyer in forward) 0 S
T
F
0;T
short sell e
T
index S
0
e
T
S
T
deposit S
0
e
T
in savings account S
0
e
T
S
0
e
(r)T
Total 0 S
0
e
(r)T
F
0;T
For this problem:
Transactions t = 0 T
buy a forward 0 S
T
1129: 26
short sell e
T
index 1100e
(0:015)9=12
= 1087: 69 S
T
deposit S
0
e
T
in savings 1087: 69 1087: 69e
(0:05)9=12
= 1129: 26
Total 0 0
c.
Transactions t = 0 T
short a forward (i.e. be a seller in forward) 0 F
0;T
S
T
buy e
T
index S
0
e
T
S
T
borrow S
0
e
T
S
0
e
T
S
0
e
(r)T
Total 0 S
0
e
(r)T
F
0;T
For this problem:
Transactions t = 0 T
short a forward 0 1129: 26 S
T
buy e
T
index 1100e
(0:015)9=12
= 1087: 69 S
T
borrow S
0
e
T
1087: 69 1087: 69e
(0:05)9=12
= 1129: 26
Total 0 0
www.actuary88.com c _Yufeng Guo 132
CHAPTER 5. FINANCIAL FORWARDS AND FUTURES
Problem 5.7.
F
0;T
= S
0
e
rT
= 1100e
(0:05)0:5
= 1127: 85
a. The 6-month forward price in the market is 1135, which is greater than
the fair forward price 1127: 85. So we have two identical forwards (one in the
open market and one that can be synthetically built) selling at dierent prices.
To arbitrage, always buy low and sell high.
Transactions t = 0 T = 0:5
sell expensive forward from market 0 1135 S
T
build cheap forward
buy an index 1100 S
T
borrow 1100 1100 1100e
(0:05)0:5
= 1127: 85
Total prot 0 1135 1127: 85 = 7: 15
We didnt pay anything at t = 0, but we have a prot 7: 15 at T = 0:5.
b. The 6-month forward price in the market is 1115, which is cheaper than
the fair forward price 1127: 85. So we have two identical forwards (one in the
open market and one that can be synthetically built) selling at dierent prices.
To arbitrage, always buy low and sell high.
Transactions t = 0 T = 0:5
buy cheap forward from market 0 S
T
1115
build expensive forward for sale
short sell an index 1100 S
T
lend 1100 1100 1100e
(0:05)0:5
= 1127: 85
Total prot 0 1127: 85 1115 = 12: 85
We didnt pay anything at t = 0, but we have a prot 12: 85 at T = 0:5.
www.actuary88.com c _Yufeng Guo 133
CHAPTER 5. FINANCIAL FORWARDS AND FUTURES
Problem 5.8.
F
0;T
= S
0
e
(r)T
= 1100e
(0:050:02)0:5
= 1116: 62
a. The 6-month forward price in the market is 1120, which is greater than
the fair forward price 1116: 62. So we have two identical forwards (one in the
open market and one that can be synthetically built) selling at dierent prices.
To arbitrage, always buy low and sell high.
Transactions t = 0 T = 0:5
sell expensive forward 0 1120 S
T
build cheap forward
buy e
T
index 1100e
(0:02)0:5
= 1089: 05 S
T
borrow S
0
e
T
1100e
(0:02)0:5
= 1089: 05 1089: 05e
(0:05)0:5
= 1116: 62
Total prot 0 1120 1116: 62 = 3: 38
We didnt pay anything at t = 0, but we have a prot 3: 38 at T = 0:5.
b.
The 6-month forward price in the market is 1110, which is cheaper than the
fair forward price 1116: 62. So we have two identical forwards (one in the open
market and one that can be synthetically built) selling at dierent prices. To
arbitrage, always buy low and sell high.
Transactions t = 0 T = 0:5
buy cheap forward from market 0 S
T
1110
build expensive forward for sale
short sell e
T
index 1100e
(0:02)0:5
= 1089: 05 S
T
lend S
0
e
T
1089: 05 1089: 05e
(0:05)0:5
= 1116: 62
Total prot 0 1116: 62 1110 = 6: 62
We didnt pay anything at t = 0, but we have a prot 6: 62 at T = 0:5.
Problem 5.9.
This is a poorly designed problem, more amusing than useful for passing the
exam. Dont waste any time on this. Skip.
www.actuary88.com c _Yufeng Guo 134
CHAPTER 5. FINANCIAL FORWARDS AND FUTURES
Problem 5.10.
a. F
0;T
= S
0
e
(r)T
1129:257 = 1100e
(0:05)0:75
e
(0:05)0:75
=
1129:257
1100
0:05 =
1
0:75
ln
1129:257
1100
= 3: 5%
= 1:5%
b. If you believe that the true dividend yield is 0:5%, then the fair forward
price is:
F
0;T
= 1100e
(0:050:005)0:75
= 1137: 759
The market forward price is 1129:257, which is cheaper than the fair price.
To arbitrage, buy low and sell high.
Transactions t = 0 T = 0:5
buy cheap forward from market 0 S
T
1129:257
build expensive forward for sale
short sell e
T
index 1100e
(0:005)0:75
= 1095: 883 S
T
lend S
0
e
T
1095: 883 1095: 883e
(0:05)0:75
= 1137: 75 9
Total prot 0 1137: 75 9 1129:257 = 8: 502
c. If you believe that the true dividend yield is 3%, then the fair forward
price is:
F
0;T
= 1100e
(0:050:03)0:75
= 1116: 624
The market forward price is 1129:257, which is higher than the fair price.
To arbitrage, buy low and sell high.
Transactions t = 0 T = 0:5
sell expensive forward 0 1129:257 S
T
build cheap forward
buy e
T
index 1100e
(0:03)0:75
= 1075: 526 S
T
borrow S
0
e
T
1075: 526 1075: 526e
(0:05)0:75
= 1116: 624
Total prot 0 1129:257 1116: 624 = 12: 633
www.actuary88.com c _Yufeng Guo 135
CHAPTER 5. FINANCIAL FORWARDS AND FUTURES
Problem 5.11.
a. One contract is worth 1200 points. Each point is worth $250. The
notional value of 4 S&P futures is:
4 1200 250 = $1; 200 ; 000
b. The value of the initial margin: $1; 200 ; 000 0:1 = $120; 000
Problem 5.12.
a. Notional value of 10 S&P futures:
10 950 250 = $2; 375 ; 000
The initial margin: $2375 000 0:1 = $237; 500
b. The maintenance margin: 237; 500 (0:8) = 190; 000
At the end of Week 1, our initial margin grows to:
237500e
0:06(1=52)
= 237774: 20
Suppose the futures price at the end of Week 1 is X. The futures price at
t = 0 is 950. After marking-to-market, we gain (X 950) points per contract.
The notional gain of the 4 futures after marking-to-market is:
(X 950) (10) (250)
After marking-to-market, our margin account balance is
237774: 20 + (X 950) (10) (250) = 2500X 2137225: 8
We get a margin call if
2500X 2137225: 8 < 190000 X < 930: 890 32
For example, X = 930: 89 will lead to a margin call.
Problem 5.13.
a.
Transactions t = 0 T
buy forward 0 S
T
F
0;T
= S
T
S
0
e
rT
lend S
0
S
0
S
0
e
rT
Total S
0
S
T
b.
Transactions t = 0 T
buy forward 0 S
T
F
0;T
= S
T
S
0
e
rT
+FV (Div)
lend S
0
PV (Div) S
0
+PV (Div) S
0
e
rT
FV (Div)
Total S
0
+PV (Div) S
T
c.
Transactions t = 0 T
buy forward 0 S
T
F
0;T
= S
T
S
0
e
(r)T
lend S
0
e
T
S
0
e
T
S
0
e
(r)T
Total S
0
e
T
S
T
www.actuary88.com c _Yufeng Guo 136
CHAPTER 5. FINANCIAL FORWARDS AND FUTURES
Problem 5.14.
If the forward price F
0;T
is too low, this is how to make some free money.
1. Buy low. At t = 0, enter a forward to buy one stock. Incur transaction
cost k.
2. Sell high. At t = 0, sell one stock short and receive S
b
0
k
3. The net cash ow after 1 and 2 is S
b
0
2k. Lend S
b
0
2k and receive
_
S
b
0
2k
_
e
r
l
T
at T
4. At T, pay F
0;T
and receive one stock. Return the stock to the broker.
The net cash ow after 1 through 4 is zero. Your prot at T is:
_
S
b
0
2k
_
e
r
l
T
F
0;T
Arbitrage is possible if:
_
S
b
0
2k
_
e
r
l
T
F
0;T
> 0 F
0;T
<
_
S
b
0
2k
_
e
r
l
T
To avoid arbitrage, we need to have:
F
0;T
_ F

=
_
S
b
0
2k
_
e
r
l
T
To avoid arbitrage, we need to have:
_
S
b
0
2k
_
e
r
l
T
= F

_ F
0;T
_ F
+
= (S
a
0
+ 2k) e
r
b
T
Problem 5.15.
a. k = 0 and theres no bid-ask spread (so S
a
0
= S
b
0
= 800)
So the non-arbitrage bound is:
800e
(0:05)1
= F

_ F
0;T
_ F
+
= 800e
(0:055)1
841: 02 = F

_ F
0;T
_ F
+
= 845: 23
Hence arbitrage is not protable if 841: 02 _ F
0;T
_ 845: 23
b. k = 1 and theres no bid-ask spread (so S
a
0
= S
b
0
= 800)
Please note that you cant blindly copy the formula:
_
S
b
0
2k
_
e
r
l
T
= F

_ F
0;T
_ F
+
= (S
a
0
+ 2k) e
r
b
T
This is because the problem states that k is incurred for longing or shorting
a forward and that k is not incurred for buying or selling an index. Given k is
incurred only once, the non-arbitrage bound is:
_
S
b
0
k
_
e
r
l
T
= F

_ F
0;T
_ F
+
= (S
a
0
+k) e
r
b
T
(800 1) e
(0:05)1
= F

_ F
0;T
_ F
+
= (800 + 1) e
(0:055)1
839: 97 = F

_ F
0;T
_ F
+
= 846: 29
c. Once again, you cant blindly use the formula
_
S
b
0
2k
_
e
r
l
T
= F

_ F
0;T
_ F
+
= (S
a
0
+ 2k) e
r
b
T
The problem states that k
1
= 1 is incurred for longing or shorting a forward
and k
2
= 2:4 is incurred for buying or selling an index. The non-arbitrage
formula becomes:
www.actuary88.com c _Yufeng Guo 137
CHAPTER 5. FINANCIAL FORWARDS AND FUTURES
_
S
b
0
k
1
k
2
_
e
r
l
T
= F

_ F
0;T
_ F
+
= (S
a
0
+k
1
+k
2
) e
r
b
T
(800 1 2:4) e
(0:05)1
= F

_ F
0;T
_ F
+
= (800 + 1 + 2:4) e
(0:055)1
837: 44 = F

_ F
0;T
_ F
+
= 848: 82
d. Once again, you cant blindly use the formula
_
S
b
0
2k
_
e
r
l
T
= F

_ F
0;T
_ F
+
= (S
a
0
+ 2k) e
r
b
T
The problem states that k
1
= 1 is incurred for longing or shorting a forward;
k
2
= 2:4 is incurred twice, for buying or selling an index, once at t = 0 and the
other at T. The non-arbitrage formula becomes:
_
S
b
0
k
1
k
2
_
e
r
l
T
k
2
= F

_ F
0;T
_ F
+
= (S
a
0
+k
1
+k
2
) e
r
b
T
+k
2
(800 1 2:4) e
(0:05)1
2:4 = F

_ F
0;T
_ F
+
= (800 + 1 + 2:4) e
(0:055)1
+
2:4
837: 44 2:4 = F

_ F
0;T
_ F
+
= 848: 82 + 2:4
835: 04 = F

_ F
0;T
_ F
+
= 851: 22
e. The non-arbitrage higher bound can be calculated as follows:
1. At t = 0 sell a forward contract. Incur cost k
1
= 1.
2. At t = 0 buy 1:003 index. This is why we need to buy 1.003 index. We
pay 0.3% of the index value to the broker. So if we buy one index, this
index becomes 1-0.3%=0.997 index after the fee. To have one index, we
need to have
1
0:997
=
1
1 0:3%
- 1 + 0:3% = 1:003 (remember we need
to deliver one index at T to the buyer in the forward). To verify, if we
have
1
0:997
index, this will become
1
0:997
(1 0:3%) = 1 index after the
fee is deducted. Notice we use the Taylor series
1
1 x
- 1 + x + x
2
+ :::
for a small x
3. At t = 0 borrow 1:003S
0
+ k
1
= 1:003 (800) + 1. Repay this loan with
(1:003S
0
+k
1
) e
r
b
T
at T
4. At T deliver the index to the buyer and receive F
0;T
. Pay the settlement
fee 0:3%S
0
Your initial cost for doing 1 through 4 is zero. Your prot at T is:
F
0;T
(1:003S
0
+k
1
) e
r
b
T
0:3%S
0
To avoid arbitrage, we need to have
F
0;T
(1:003S
0
+k
1
) e
r
b
T
0:3%S
0
_ 0
F
0;T
_ (1:003S
0
+k
1
) e
r
b
T
+ 0:3%S
0
= (1:003 800 + 1) e
(0:055)1
+ 0:003 800 = 851: 22
The lower bound price can also be calculated as follows:
www.actuary88.com c _Yufeng Guo 138
CHAPTER 5. FINANCIAL FORWARDS AND FUTURES
1. Buy low. At t = 0, enter a forward to buy 1:003 index (why 1.003 index
will be explained later). Incur transaction cost k
1
= 1.
2. Sell high. At t = 0, sell 0:997 index short. Receive 0:997 (800) 1. This is
why we need to short sell 0.997 index. If we short sell one index, the broker
charges us 0.3% of the index value and well owe the broker 1+0.3%=1.003
index. In order to owe the broker exactly one index, we need to borrow
1
1:003
- 1 0:003 = 0:997 index from the broker.
3. Lend 0:997 (800) 1 and receive (0:997 (800) 1) e
r
l
T
at T
4. At T, pay 1:003F
0;T
and receive 1:003 index. Pay settlement fee 0:3%(1:003).
After the settlement fee, we have (1:003) (1 0:3%) - 1 index left. We
return this index to the broker.
The net initial cash ow after 1 through 4 is zero. Your prot at T is:
(0:997 (800) 1) e
r
l
T
1:003F
0;T
To avoid arbitrage,
(0:997 (800) 1) e
r
l
T
1:003F
0;T
_ 0
F
0;T
_
(0:997 (800) 1) e
r
l
T
1:003
=
(0:997 800 1) e
(0:05)1
1:003
= 834: 94
The non-arbitrage bound is:
834: 94 _ F
0;T
_ 851: 22
Make sure you understand part e, which provides a framework for nd-
ing the non-arbitrage bound for complex problems. Once you understand this
framework, you dont need to memorize non-arbitrage bound formulas.
Problem 5.16.
Not on the syllabus. Ignore.
Problem 5.17.
Not on the syllabus. Ignore.
Problem 5.18.
Not on the syllabus. Ignore.
Problem 5.19.
Not on the syllabus. Ignore.
Problem 5.20.
a. r
91
= (100 93:23)
1
100

1
4

91
90
= 1: 711 3%
b. $10 (1 + 0:017113) = $10: 171 13 (million)
www.actuary88.com c _Yufeng Guo 139
CHAPTER 5. FINANCIAL FORWARDS AND FUTURES
www.actuary88.com c _Yufeng Guo 140
Chapter 8
Swaps
Problem 8.1.
time t 0 1 2
annual interest during [0; t] 6% 6:5%
xed payment R R
oating payments 22 23
PV xed payments=PV oating payments
22
1:06
+
23
1:065
2
=
R
1:06
+
R
1:065
2
, R = 22: 483
1
Problem 8.2.
a.
time t 0 1 2 3
annual interest during [0; t] 6% 6:5% 7%
xed payment R R R
oating payments 20 21 22
PV xed payments=PV oating payments
20
1:06
+
21
1:065
2
+
22
1:07
3
=
R
1:06
+
R
1:065
2
+
R
1:07
3
, R = 20: 952
b. We are now standing at t = 1
1
I recommend that initially you dont memorize the complex formula R =
P
P(0;t
i
)f
0
(t
i
)
P
P(0;t
i
)
.
Draw a cash ow diagram and set up the equation PV xed payments = PV oating payments.
Once you are familiar with the concept, you can use the memorized formula.
141
CHAPTER 8. SWAPS
time t 1 2 3
annual interest during [1; t] 6:5% 7%
xed payment R R
oating payments 21 22
PV xed payments=PV oating payments
21
1:065
+
22
1:07
2
=
R
1:065
+
R
1:07
2
, R = 21: 482
Problem 8.3.
The dealer pays xed and gets oating. His risk is that oils spot price may
drop signicantly. For example, if the spot price at t = 2 is $18 per barrel
(as opposed to the expected $21 per barrel) and at t = 3 is $19 per barrel (as
opposed to the expected $22 per barrel), the dealer has overpaid the swap. This
is because the xed swap rate R = 20: 952 is calculated under the assumption
that the oil price is $21 per barrel at t = 2 and $22 per barrel at t = 3.
To hedge his risk, the dealer can enter 3 separate forward contracts, agreeing
at t = 0 to deliver oil to a buyer at $20 per barrel at t = 1, at $21 per barrel at
t = 2 , and at $22 per barrel at t = 3:
Next, lets verify that the PV of the dealers locked-in net cash ow is zero.
time t 0 1 2 3
annual interest during [0; t] 6% 6:5% 7%
xed payment 20: 952 20: 952 20: 952
oating payments 20 21 22
net cash ow 20: 952 20 = 0:952 0:048 1: 048
PV(net cash ows)=
0:952
1:06
+
0:048
1:065
2
+
1: 048
1:07
3
= 0
Problem 8.4.
The xed payer overpaid 0:952 at t = 1. The implied interest rate in Year 2
(from t = 1 to t = 2) is
1:065
2
1:06
1 = 0:07 002 4. So the overpayment 0:952 at t = 1
will grow into 0:952 (1 + 0:07 002 4) = 1: 018 7 at t = 2. Then at t = 2, the xed
payer underpays 0:048 and his net overpayment is 1: 018 7 0:048 = 0:970 7.
The implied interest rate in Year 3 (from t = 2 to t = 3) is
1:07
3
1:065
2
1 = 0:0
8007 1. So the xed payers net overpayment 0:970 7 at t = 2 will grow into
0:970 7 (1 + 0:08007 1) = 1: 048 4, which exactly osets his underpayment 1: 048
at = 3. now the accumulative net payment after the 3rd payment is zero.
www.actuary88.com c _Yufeng Guo 142
CHAPTER 8. SWAPS
Problem 8.5.
5 basis points=5%% = 0:5% = 0:0005
a. immediately after the swap contract is signed the interest rate rises 0.5%
time t 0 1 2 3
original annual interest during [0; t] 6% 6:5% 7%
updated annual interest during [0; t] 6:5% 7% 7:5%
xed payment R R R
oating payments 20 21 22
20
1:065
+
21
1:07
2
+
22
1:075
3
=
R
1:065
+
R
1:07
2
+
R
1:075
3
, R = 20: 949 < 20: 952
The xed rate is worth 20: 949, but the xed payer pays 20: 952. His loss is:
20
1:06
+
21
1:065
2
+
22
1:07
3

_
20
1:065
+
21
1:07
2
+
22
1:075
3
_
= 0:510 63
b. immediately after the swap contract is signed the interest rate falls 0.5%
time t 0 1 2 3
original annual interest during [0; t] 6% 6:5% 7%
updated annual interest during [0; t] 5:5% 6% 6:5%
xed payment R R R
oating payments 20 21 22
20
1:055
+
21
1:06
2
+
22
1:065
3
=
R
1:055
+
R
1:06
2
+
R
1:065
3
R = 20: 955 > 20: 952
The xed rate is worth 20: 955, but the xed payer pays only 20: 952
The xed payers gain is:
20
1:055
+
21
1:06
2
+
22
1:065
3

_
20
1:065
+
21
1:07
2
+
22
1:075
3
_
= 1: 029 3
www.actuary88.com c _Yufeng Guo 143
CHAPTER 8. SWAPS
Problem 8.6.
(1) calculate the per-barrel swap price for 4-quarter oil swap
time t (quarter) 0 1 2 3 4
xed payment R R R R
oating payments 21 21:1 20:8 20:5
discounting factor 1:015
1
1:015
2
1:015
3
1:015
4
PV of oating payments = PV of xed payments
21
_
1:015
1
_
+ 21:1
_
1:015
2
_
+ 20:8
_
1:015
3
_
+ 20:5
_
1:015
4
_
= R
_
1:015
1
+ 1:015
2
+ 1:015
3
+ 1:015
4
_
R = 20: 853 3
(2) calculate the per-barrel swap price for 8-quarter oil swap
time t (quarter) 0 1 2 3 4 5 6 7 8
xed payment R R R R R R R R
oating payments 21 21:1 20:8 20:5 20:2 20 19:9 19:8
The discounting factor at t is 1:015
t
(i.e. $1 at t is worth 1:015
t
at t = 0)
PV of oating payments = PV of xed payments
21
_
1:015
1
_
+21:1
_
1:015
2
_
+20:8
_
1:015
3
_
+20:5
_
1:015
4
_
+20:2
_
1:015
5
_
+
20
_
1:015
6
_
+ 19:9
_
1:015
7
_
+ 19:8
_
1:015
8
_
= R
_
1:015
1
+ 1:015
2
+ 1:015
3
+ 1:015
4
+ 1:015
5
+ 1:015
6
+ 1:015
7
+ 1:015
8
_
R = 20: 428 4
(3) calculate the total cost of prepaid 4-quarter and 8-quarter swaps
cost of prepaid 4-quarter swap
21
_
1:015
1
_
+ 21:1
_
1:03
1
_
+ 20:8
_
1:045
1
_
+ 20:5
_
1:06
1
_
= 80: 419 02
cost of prepaid 8-quarter swap
21
_
1:015
1
_
+21:1
_
1:015
2
_
+20:8
_
1:015
3
_
+20:5
_
1:015
4
_
+20:2
_
1:015
5
_
+
20
_
1:015
6
_
+ 19:9
_
1:015
7
_
+ 19:8
_
1:015
8
_
= 152: 925 604
Total cost: 80: 419 02 + 152: 925 6 = 233: 344 62
www.actuary88.com c _Yufeng Guo 144
CHAPTER 8. SWAPS
Problem 8.7.
The calculation is tedious. Ill manually solve the swap rates for the rst 4
quarter but give you all the swaps.
Final result
quarter 1 2 3 4 5 6 7 8
forward 21 21:1 20:8 20:5 20:2 20 19:9 19:8
DiscFactor 0:9852 0:9701 0:9546 0:9388 0:9231 0:9075 0:8919 0:8763
R 21 21:05 20: 97 20: 85 20:73 20:61 20:51 20:43
Ill manually solve for the rst 4 swap rates.
quarter 1 2 3 4
forward price 21 21:1 20:8 20:5
xed payment R R R R
zero coupon bond price
2
0:9852 0:9701 0:9546 0:9388
(1) if theres only 1 swap occurring at t = 1 (quarter)
PV xed=PV oat
21 (0:9852) = R(0:9852)
R = 21
(2) if there are two swaps occurring at t = 1 and t = 2
PV xed=PV oat
21 (0:9852) + 21:1 (0:9701) = R(0:9852 + 0:9701)
R = 21: 049 6 t
21+21:1
2
= 21: 05
(3) if there are 3 swaps occurring at t = 1 ,2,3
PV xed=PV oat
21 (0:9852) + 21:1 (0:9701) + 20:8 (0:9546) = R(0:9852 + 0:9701 + 0:9546)
R = 20: 967 7 t
21+21:1+20:8
3
= 20: 966 67
(4)if there are 4 swaps occurring at t = 1 ,2,3,4
PV xed=PV oat
21 (0:9852)+21:1 (0:9701)+20:8 (0:9546)+20:5 (0:9388) = R(0:9852 + 0:9701 + 0:9546 + 0:9388)
R = 20: 853 636 t
21+21:1+20:8+20:5
4
= 20: 85
3
2
Zero coupon bond price is also the discounting factor.
3
If you run out of time in the exam, just take R as the average oating payments. This is
often very close to the correct answer.
www.actuary88.com c _Yufeng Guo 145
CHAPTER 8. SWAPS
By the way, please note that in the textbook Table 8.9, the gas swap prices
are not in line with the forward price and discounting factors. This is because
the swap prices in Table 8.9 are stand-alone prices made up by the author of
Derivatives Markets so he can set up problems for you to solve:
t
i
(quarter) 1 2 3 4 5 6 7 8
R 2:25 2:4236 2:3503 2:2404 2:2326 2:2753 2:2583 2:2044
To avoid confusion, the author of Derivatives Markets should have used
multiple separate tables instead of combining separate tables into one.
Problem 8.8.
quarter 1 2 3 4 5 6
forward price 20:8 20:5 20:2 20
xed payment R R R R
zero coupon bond price 0:9546 0:9388 0:9231 0:9075
If you run out of time, then R =
20:8+20:5+20:2+20
4
= 20: 375 t 20:38
The precise calculation is:
PV xed=PV oat
20:8 (0:9546)+20:5 (0:9388)+20:2 (0:9231)+20 (0:9075) = R(0:9546 + 0:9388 + 0:9231 + 0:9075)
R = 20: 380 69 t 20:38
www.actuary88.com c _Yufeng Guo 146
CHAPTER 8. SWAPS
Problem 8.9.
If the problem didnt give you R = 20:43, you can quickly estimate it as
(21 + 21:1 + 20:8 + 20:5 + 20:2 + 20 + 19:9 + 19:8) =8 = 20: 412 5
Back to the problem. Please note that this problem implicitly assumes that
the actual interest rates are equal to the expected interest rates implied in the
zero-coupon bonds. If the actual interest rates turn out to be dierent than
the rates implied by the zero-coupon bonds, then youll need to know the actual
interest rates quarter-by-quarter to solve this problem. So for the sake of solving
this problem, we assume that the interest rates implied by the zero-coupon
bonds are the actual interest rates.
quarter 1 2 3 4 5 6 7 8
fwd price 21 21:1 20:8 20:5 20:2 20 19:9 19:8
xed pay 20:43 20:43 20:43 20:43 20:43 20:43 20:43 20:43
xedfwd 0:57 0:67 0:37 0:07 0:23 0:43 0:53 0:63
disct factor 0:9852 0:9701 0:9546 0:9388 0:9231 0:9075 0:8919 0:8763
Loan balance at t = 0 is 0
Loan balance at t = 1 is 0:57.
The implicit interest rate from t = 1 to t = 2 is solved by
0:9852
1+x
= 0:9701: So 1 + x =
0:9852
0:9701
. The 0:57 loan will grow into 0:57
0:9852
0:9701
= 0:579 at t = 2.
The loan balance at t = 2 is 0:579 0:67 = 1: 249:
1: 249 will grow into 1: 249
0:9701
0:9546
= 1: 269 at t = 3.
The loan balance at t = 3 is1: 269 0:37 = 1: 639, which grows into
1: 639
0:9546
0:9388
= 1: 667 at t = 4.
The loan balance at t = 4 is 1: 667 0:07 = 1: 737
1: 737 grows into 1: 737
0:9388
0:9231
= 1: 767 at t = 5.
So the loan balance at t = 5 is 1: 767 + 0:23 = 1: 537
1: 537 grows into 1: 537
0:9231
0:9075
= 1: 563 at t = 6 .
The loan balance at t = 6 is 1: 563 + 0:43 = 1: 133
1: 133 grows into 1: 133
0:9075
0:8919
= 1: 153 at t = 7.
The loan balance at t = 7 is 1: 153 + 0:53 = 0:623
0:623 grows into 0:623
0:8919
0:8763
= 0:634 at t = 8.
So the loan balance at t = 8 is 0:634 + 0:63 = 0:004 t 0
quarter 0 1 2 3 4 5 6 7 8
loan bal 0 0:57 1: 249 1: 639 1: 737 1: 537 1: 133 0:623 0
www.actuary88.com c _Yufeng Guo 147
CHAPTER 8. SWAPS
You can also work backward from t = 8 to t = 0. You know that the loan
balance at t = 8 is zero; overall the xed payer and the oating payer each have
no gain and no loss if the expected yield curve turns out to the real yield curve.
quarter 1 2 3 4 5 6 7 8
fwd price 21 21:1 20:8 20:5 20:2 20 19:9 19:8
xed pay 20:43 20:43 20:43 20:43 20:43 20:43 20:43 20:43
xedfwd 0:57 0:67 0:37 0:07 0:23 0:43 0:53 0:63
disct factor 0:9852 0:9701 0:9546 0:9388 0:9231 0:9075 0:8919 0:8763
Since the loan balance at t = 8 is zero and the xed payer overpays 0:634
4
at t = 8, the loan balance at t = 7 must be 0:634
0:8919
0:8763
= 0:623.
Similarly, the loan balance at t = 7 must be (0:623 0:53)
0:9075
0:8919
= 1:
133 .
And the loan balance at t = 6 must be (1: 133 0:43)
0:9231
0:9075
= 1: 537.
So on and so forth.
This method is less intuitive. However, if the problem asks you to only nd
the loan balance at t = 7, this backward method is lot faster than the forward
method.
4
I use 0.634 instead of 0.63 to show you that the backward method produces the same
correct answer as the forward method. If you use 0.63, you wont be able to reproduce the
answer calculated by the forward method due to rounding (because 0.63 is rounded from
0.634).
www.actuary88.com c _Yufeng Guo 148
CHAPTER 8. SWAPS
Problem 8.10.
The oating payer delivers 2 barrels at even numbered quarters and 1 barrel
at odd quarters.
The cash ow diagram is:
quarter 1 2 3 4 5 6 7 8
fwd price 21 21:1 (2) 20:8 20:5 (2) 20:2 20 (2) 19:9 19:8 (2)
xed pay R 2R R 2R R 2R R 2R
disct factor 0:9852 0:9701 0:9546 0:9388 0:9231 0:9075 0:8919 0:8763
PV oat =PV xed
21 (0:9852) + 21:1 (2) (0:9701) + 20:8 (0:9546) + 20:5 (2) (0:9388)
+20:2 (0:9231) + 20 (2) (0:9075) + 19:9 (0:8919) + 19:8 (2) (0:8763)
= R(0:9852 + 2 0:9701 + 0:9546 + 2 0:9388)
+R(0:9231 + 2 0:9075 + 0:8919 + 2 0:8763)
R = 20: 409 94
Please note that the cash ow diagram is not:
quarter 1 2 3 4 5 6 7 8
fwd price 21 21:1 (2) 20:8 20:5 (2) 20:2 20 (2) 19:9 19:8 (2)
xed pay R R R R R R R R
disct factor 0:9852 0:9701 0:9546 0:9388 0:9231 0:9075 0:8919 0:8763
www.actuary88.com c _Yufeng Guo 149
CHAPTER 8. SWAPS
Problem 8.11.
The key formula is the textbook equation 8.13:
R =

n
i=1
P (0; t
i
) f
0
(t
i
)

n
i=1
P (0; t
i
)
From Table 8.9, we get:
t
i
(quarter) 1 2 3 4 5 6 7 8
R 2:25 2:4236 2:3503 2:2404 2:2326 2:2753 2:2583 2:2044
P (0; t
i
) 0:9852 0:9701 0:9546 0:9388 0:9231 0:9075 0:8919 0:8763
Notation:
P (0; t
i
) is the present value at t = 0 of $1 at the t
i
.
R is the swap rate. For example, for a 4-quarter swap, the swap rate is
2:2404
f
0
(t
i
) is the price of the forward contract signed at t
i1
and expiring at
t
i
The 1-quarter swap rate is
R(1) =
P (0; t
1
) f
0
(t
1
)
P (0; t
i
)
f
0
(t
1
) = R(1) = 2:2500
The 2-quarter swap rate is:
R(2) =
P (0; t
1
) f
0
(t
1
) +P (0; t
2
) f
0
(t
2
)
P (0; t
1
) +P (0; t
2
)
2:4236 =
0:9852 (2:25) + 0:9701f
0
(t
2
)
0:9852 + 0:9701
f
0
(t
2
) = 2:5999
The 3-quarter swap rate is:
R(3) =
P (0; t
1
) f
0
(t
1
) +P (0; t
2
) f
0
(t
2
) +P (0; t
3
) f
0
(t
3
)
P (0; t
1
) +P (0; t
2
) +P (0; t
3
)
0:9546 =
0:9852 (2:25) + 0:9701 (2:60) + 0:9546f
0
(t
3
)
0:9852 + 0:9701 + 0:9546
f
0
(t
3
) = 2:2002
So on and so forth. The result is:
t
i
1 2 3 4 5 6 7 8
R 2:25 2:4236 2:3503 2:2404 2:2326 2:2753 2:2583 2:2044
P (0; t
i
) 0:9852 0:9701 0:9546 0:9388 0:9231 0:9075 0:8919 0:8763
f
0
(t
i
) 2:2500 2:5999 2:2002 1:8998 2:2001 2:4998 2:1501 1:8002
www.actuary88.com c _Yufeng Guo 150
CHAPTER 8. SWAPS
Problem 8.12.
t
i
1 2 3 4
R(8) 2:2044 2:2044 2:2044 2:2044
f
0
(t
i
) 2:2500 2:5999 0:9546 0:9388
P (0; t
i
) 0:9852 0:9701 0:9546 0:9388
i(t
i1
; t
i
) 1:5022% 1:5565% 1:6237% 1:6830%
t
i
5 6 7 8
R(8) 2:2044 2:2044 2:2044 2:2044
f
0
(t
i
) 0:9231 0:9075 0:8919 0:8763
P (0; t
i
) 0:9231 0:9075 0:8919 0:8763
i(t
i1
; t
i
) 1:7008% 1:7190% 1:7491% 1:7802%
First, lets calculate the quarterly forward interest rate i(t
i1
; t
i
), which is
the interest during[t
i1
; t
i
].
By the way, please note the dierence between f
0
(t
i
) and i(t
i1
; t
i
). f
0
(t
i
)
is the price of a forward contract and i(t
i1
; t
i
) is the forward interest rate.
The interest rate during the rst quarter is i(t
0
; t
1
). This is the eective
interest per quarter from t = 0 to t = 0:25 (year). Because P (0; t
1
) represents
the present value of $1 at t = 0:25
P (0; t
1
) =
1
1 +i(t
0
; t
1
)
0:9852 =
1
1 +i(t
0
; t
1
)
i(t
0
; t
1
) =
1
0:9852
1 = 1: 5022%
The 2nd quarter interest rate i(t
1
; t
2
) satises the following equation:
P (0; t
2
) =
1
1 +i(t
0
; t
1
)

1
1 +i(t
1
; t
2
)
=
P (0; t
1
)
1 +i(t
1
; t
2
)
0:9701 =
0:9852
1 +i(t
1
; t
2
)
i(t
1
; t
2
) = 1: 5566%
Similarly,
P (0; t
3
) =
1
1 +i(t
0
; t
1
)

1
1 +i(t
1
; t
2
)

1
1 +i(t
2
; t
3
)
=
P (0; t
2
)
1 +i(t
2
; t
3
)
0:9546 =
0:9701
1 +i(t
2
; t
3
)
i(t
2
; t
3
) = 1:6237%
Keep doing this, you should be able to calculate all the forward interest
rates.
www.actuary88.com c _Yufeng Guo 151
CHAPTER 8. SWAPS
Next, lets calculate the loan balance.
t
i
1 2 3 4
R(8) 2:2044 2:2044 2:2044 2:2044
f
0
(t
i
) 2:2500 2:5999 0:9546 0:9388
f
0
(t
i
) R(8) 0:0456 0:3955 0:0042 0:3046
Loan balance 0:0456 0:4418 0:4444 0:1470
i(t
i1
; t
i
) 1:5022% 1:5565% 1:6237% 1:6830%
t
i
5 6 7 8
R(8) 2:2044 2:2044 2:2044 2:2044
f
0
(t
i
) 0:9231 0:9075 0:8919 0:8763
f
0
(t
i
) R(8) 0:0043 0:2954 0:0543 0:4042
Loan balance 0:1451 0:4430 0:3963 0:0009
i(t
i1
; t
i
) 1:7008% 1:7190% 1:7491% 1:7802%
At t = 0, the loan balance is zero. A swap is a fair deal and no money
changes hands.
At t = 1 (i.e. the end of the rst quarter), the oating payer lends 2:25
2:2044 = 0:045 6 to the xed payer. Had the oating payer signed a forward
contract at t = 0 agreeing to deliver the oil at t = 1, he would have received
2:25 at t = 1. However, by entering into an 8-quarter swap, the oating payer
receives only 2:2044 at the t = 1. So the oating payer lends 2:25 2:2044 =
0:045 6 to the xed payer.
The 0:045 6 at t = 1 grows to 0:045 6 (1 + 0:015022) = 0:046 3
The total loan balance at t = 2 is 0:046 3 + 0:3955 = 0:441 8
The loan balance 0:441 8 at t = 2 grows into 0:441 8 (1 + 0:015565) = 0:448 7
at t = 3
The total loan balance at t = 3 is 0:448 7 0:0042 = 0:444 4
I used Excel to do the calculation. Due to rounding, I got 0:448 70:0042 =
0:444 4 instead of 0:444 5.
So on and so forth. The nal loan balance at t = 8 is:
0:3963 (1 + 0:017491) 0:4042 = 0:000 9 - 0
The loan balance at the end of the swap t = 8 should be zero. We didnt get
zero due to rounding.
www.actuary88.com c _Yufeng Guo 152
CHAPTER 8. SWAPS
Problem 8.13.
t
i
2 3 4 5 6 Total
P (0; t
i
) 0:9701 0:9546 0:9388 0:9231 0:9075 4:6941
i(t
i1
; t
i
) 1:5565% 1:6237% 1:6830% 1:7008% 1:7190%
P (0; t
i
) i(t
i1
; t
i
) 0:0151 0:0155 0:0158 0:0157 0:0156 0:0777
R =

6
i=2
P (0; t
i
) r (t
i1
; t
i
)

6
i=2
P (0; t
i
)
=
0:0777
4:6941
= 1: 655 3%
Problem 8.14.
t
i
P (0; t
i
) i(t
i1
; t
i
) P (0; t
i
) i(t
i1
; t
i
)
1 0:9852 1:5022% 0:0148
2 0:9701 1:5565% 0:0151
3 0:9546 1:6237% 0:0155
4 0:9388 1:6830% 0:0158
Total 3:8487 0:0612
Calculate the 4-quarter swap rate.
R =

4
i=1
P (0; t
i
) r (t
i1
; t
i
)

4
i=1
P (0; t
i
)
=
0:0612
3:8487
= 1: 5901%
Calculate the 8-quarter swap rate.
t
i
P (0; t
i
) i(t
i1
; t
i
) P (0; t
i
) i(t
i1
; t
i
)
1 0:9852 1:5022% 0:0148
2 0:9701 1:5565% 0:0151
3 0:9546 1:6237% 0:0155
4 0:9388 1:6830% 0:0158
5 0:9231 1:7008% 0:0157
6 0:9075 1:7190% 0:0156
7 0:8919 1:7491% 0:0156
8 0:8763 1:7802% 0:0156
Total 7:4475 0:1237
R =

8
i=1
P (0; t
i
) r (t
i1
; t
i
)

5
i=1
P (0; t
i
)
=
0:1237
7:4475
= 1: 6610%
Problem 8.15.
Not on the FM syllabus. Skip.
www.actuary88.com c _Yufeng Guo 153
CHAPTER 8. SWAPS
Problem 8.16.
Not on the FM syllabus. Skip.
Problem 8.17.
Not on the FM syllabus. Skip.
Problem 8.18.
Not on the FM syllabus. Skip.
www.actuary88.com c _Yufeng Guo 154
Guo FM Mock Exam 1
Allotted time: 3 hours
Problem 1
You are given two funds:
Fund ': $100 invested at t = 0 with the annual eective interest rate 6%
Fund : $100 invested at t = 2 (i.e. end of Year 2) with an annual simple
interest j, where j is a constant.
Fund ' and have the same force of interest at the end of Year 10.
Calculate the value of Fund at the end of Year 10.
185 1 195 C 205 1 215 1 225
Problem 2
You are given:
i is the annual eective interest rate
in a years $1 will grow into $10
in / years $2 will grow into $25
in c years $3 will grow into $60
Calculate (1 + i)
+23
0.1 1 0.2 C 0.3 1 0.4 1 0.5
Problem 3
$7 payments are made semiannually for the rst year, the rst payment
due 6 months from today
The payments increase by $4 each year, starting from Year 2 and lasting
forever
The annual eective interest rate is 9%
Calculate the present value of this annuity.
1170 1 1190 C 1210 1 1230 1 1250
Problem 4
A loan of 15, 000 is made at time zero
The annual eective interest rate is 10%
20 annual repayments are made. Each repayment is 1, 400
1
Calculate the loan balance immediately after the 20-th annual repayment is
made.
20, 727 1 20, 777 C 20, 827 1 20, 877 1 20, 927
Problem 5
Date fund balance before deposit and withdrawal deposit withdrawal
1,1,2000 0 100
7,1,2000 110 15
9,1,2000 80 60
1,1,2001 A
The time weighted return during the year is 0.15 more than the dollar
weighted return during the year.
Calculate A
12 1 17 C 22 1 27 1 32
Problem 6
Joe buys an S&P 500 index futures contract for 1500. The initial margins
is 10%. The maintenance margin is 80%. The contract is settled weekly. The
continuously compounded interest rate is 6% per year. If the price of the index
drops to 1400 next week, calculate the amount that Joe has to pay.
17, 260 1 17, 360 C 17, 460 1 17, 560 1 17, 660
Problem 7
Liabilities: $100 per year for ve years, the rst payment due one year
from today
Asset #1: $100 per year at the of Year 3,4, and 5 respectively. The asset
price is $100.
Asset #2: $50 at the end of Year 1 and 2 respectively. The asset price is
$80
Asset #3: $50 per year for ve years, the rst cash ow occurring one
year from today. The asset price is $150.
Asset #4: $100 per year for ve years, the rst cash ow occurring one
year from today. The asset price is $250.
Find the cheapest method to exactly match the liabilities.
Buy some units of Asset #1 and some units Asset #2
1 Buy some units of Asset #1
C Buy some units of Asset #3
1 Buy some units of Asset #4
1 Buy some units of Asset #2 and some units Asset #3
Problem 8
2
A bank oers 5% annual eective interest rate to 6-year CD deposit. In
addition, it oers, at the end of Year 6, a 2% bonus interest payment of the
initial deposit .
Calculate the equivalence annual eective yield of this CD.
4.86% 1 5.06% C 5.26% 1 5.46% 1 5.66%
Problem 9
Fund A accumulates $100 at a simple rate of discount 6% per year for 5
years
Fund B accumulates $100 at a constant force of interest of 4% per year
for 5 years
Calculate the sum of the fund value A and B at the end of Year 5.
213 1 226 C 239 1 252 1 265
Problem 10
$100, 000 loan is made
Monthly interest-only repayments are made for 10 years, the rst repay-
ment occurring one month from today
Level annual repayments of $9, 456 are made for as long as needed to pay
o the remaining balance of the loan, where the rst annual repayment is
made at the end of Year 11.
The annual eective interest rate is 9.2%
Calculate the total number of repayments necessary to pay o the loan.
133 1 140 C 147 1 154 1 161
Problem 11
A loan amount A is repaid by level annual repayments, the rst repayment
due in one year. You are given:
The interest portion of the 5-th repayment is 10251.9
The principal paid in the ( 8)th repayment is 7361.83
The interest paid in the ( 16)th repayment is 7469.15
Calculate A
184, 500 1 184, 750 C 185, 000 1 185, 250 1 185, 500
Problem 12
3
The Macaulay duration of a 2-year annuity due of $50 per year is 0.4884.
Calculate the Macaulay duration of a 3-year annuity due of $100 per year.
0.72 1 0.97 C 1.22 1 1.47 1 1.72
Problem 13
The current stock price is $110
The annualized continuously compounded risk-free interest rate is 8.7%
You buy a one-year to expiration call on the stock with strike price $120
You simultaneously sell a one-year to expiration put on the same stock
with the same strike price
Calculate the net premium you have to pay.
2 1 0 C 2 1 4 1 6
Problem 14
The current stock price is 100
The annual continuously compounded risk free interest rate is 6%
The cost of carry is 2%
Calculate the annual continuously compounded dividend yield of the stock
2.0% 1 2.5% C 3.0% 1 3.5% 1 4%
Problem 15
A company has liabilities of 500 in t = 1, 2, and 3 years. It have the option
of purchasing either 1 year or 3 year zero coupon bonds, with prices of 760 and
650, respectively. Assume a at interest rate of 10%. What mix of 1 year and
3 year zero coupon bonds must it purchase now to exactly match the duration
of assets to duration of liabilities?
$540 of the 1-year bond and $460 of the 3-year bond
1 $570 of the 1-year bond and $490 of the 3-year bond
C $600 of the 1-year bond and $520 of the 3-year bond
1 $630 of the 1-year bond and $550 of the 3-year bond
1 $660 of the 1-year bond and $580 of the 3-year bond
Problem 16
Stock and 1 both pay perpetual annual dividends, the rst dividend due
one year from today.
The dividends of Stock increase by 50q%
4
The dividend of Stock 1 increase by 50q%
The rst dividend of stock is half of the rst dividend of Stock 1
The price of Stock is twice the price of Stock 1
The annual eective interest rate is 10%
Calculate q.
0.04 1 0.06 C 0.08 1 0.10 1 0.12
Problem 17
You are given the following data about a special annuity:
Quarterly payments of 10, 20, 30, and 40 are made per year for 20 years,
the 1st payment due one quarter from today
The interest rate is 8% nominal per year compounded quarterly
Calculate the PV of this special annuity.
A 10 (1a)
4|2%

a
20|8%

B 10 (1a)
4|2%

a
20|824%

C 10

4|2%

a
20|8%

D 10 (1a)
4|2%

a
20|8%

E 10

4|2%

a
20|824%

Problem 18
A loan is repaid through level monthly payments for 10 years, the rst pay-
ment due one month from the loan date.
The annual eective interest rate is 8%. The principal for the 6th payment
is 60.
Calculate the principal for the 24th payment.
49 1 55 C 61 1 67 1 73
Problem 19
For the rst two years the annual eective interest rate is 6%. For t 2
force of interest is
1
1 + t
, where t is the number of years from today.
Calculate the eective annual discount rate over the 5 years.
0.12 1 0.15 C 0.18 1 0.21 1 0.24
Problem 20
A company wants to buy a mixture of bonds that will satisfy its liability
and cost the least.
Companys Liability
5
time t (year) 0 1 2
liability cash ow 100, 000 200, 000
Assets:
Bond #1: 1-year bond with 7.5% annual coupon
Bond #2: 2-year bond with 5.5% annual coupon
Bond #3: 2-year zero coupon bond at 6%
The company also has the option of buying a 7% one year zero coupon bond
starting at time 1. In order to obtain this bond, however, the company needs to
pay 2% of the purchase price at time 0. Calculate the total cost of the mixture
of the bonds that will satisfy its liability and cost the least.
269, 980 1 270, 080 C 270, 180 1 270, 280 1 270, 380
Problem 21
Information about the asset and liability of a retirement fund:
Asset Liability
amount 250, 000 250, 000
duration 16 12
convexity 360 360
Assume a parallel shift of the interest rate. Which of the following statements
is correct?
The retirement fund loses no matter the interest goes up slightly or
down slightly
1 The retirement fund neither gains nor loses no matter the interest
goes up slightly or down slightly
C The retirement fund gains if the interest goes up slightly but loses if
the interest goes down slightly
1 The retirement fund gains if the interest goes down slightly but loses
if the interest goes up slightly
1 The retirement fund gains no matter the interest goes up slightly or
down slightly
Problem 22
John wants a 5000 loan immediately. He can get the loan by one of the two
options:
Option 1. A 5-year loan with interest and principal paid at the end of
Year 5
Option 2. A 3-year loan with interest and principal paid by a 2-year loan
issued at the end of Year 3.
6
Calculate the eective interest rate of the 2-year loan such that John is
indierent with either option.
You are also given the following structure of spot interest rates:
Term (Year) 1 2 3 4 5
spot rate 3.5% 4.5% 5.5% 6.5% 7.5%
9.37% 1 9.77% C 10.17% 1 10.57% 1 10.97%
Problem 23
Which one of these positions will NOT benet from an increase of the stock
price?
A long forward
B short stock
C short put
D long call
Problem 24
Which of the followings is a correct statement about the dierence between
a futures contract and a forward contract
A. Futures contracts are settled at expiration but forward contracts are
settled daily
B. Futures contracts are traded over the counter but forward contracts are
exchange-traded
C. Futures contracts are less liquid than forward contracts
D. Futures contracts are more rigid in terms and conditions, but forward
contracts are more exible
Problem 25
Which of the followings is correct about Redington immunization?
A. The asset cash ows should match the liability cash ows.
B. It immunizes against all the interest rate changes
C. The duration of the asset should be greater than the duration of the
liability
D. The convexity of the liability should be greater than the convexity of the
asset
E. None of the above
Problem 26 You are given two equivalent options of buying a car:
Option 1. Buy the car outright with cash for 30, 000
Option 2. Lease the car for three years with down payment 1000 and
monthly payments of A for 3 years. At the end of the Year 3, the car can
be returned for 2, 000.
7
The annual eective interest rate is 8%. Calculate A.
905 1 930 C 955 1 980 1 1005
Problem 27
You borrow a 1000 loan at an annual eective rate 10%. You accumulate a
sinking fund at the end of Year 10. The value of the sinking fund is 600. The
8-th payment is 140. How much of the 8-th payment goes to interest?
100 1 110 C 120 1 130 1 140
Problem 28
A loan is paid through annual repayments at the end of the year for 30 years.
The annual payments are $5 for each of the rst 10 years, $4 for each of the next
10 years, and $3 for each of the last 10 years. The interest portion of the 11st
payment is one and half the interest portion of the 21-th payment. Calculate
the interest portion of the 21-th payment.
2.4 1 2.5 C 2.6 1 2.7 1 2.8
Problem 29
Company FastGrow Incorporated can have 500M prot (M=million) or 350M
loss with equal probability. If FastGrow uses hedging, its prot is 100M for sure
after hedging.
FastGrow has tax rate 40%. However, it pays no tax and receives no tax
credit if it incurs losses.
Calculate the dierence (in millions) between FastGrows hedged after-tax
prot and unhedged after-tax prot.
75 1 80 C 85 1 90 1 95
Problem 30
The nominal interest rate is 6% per year
The ination rate is 2% per year
Calculate the real interest rate per year.
4% 1 6% C 8% 1 10% 1 12%
8
Guo FM Mock Exam 1
Allotted time: 3 hours
Problem Answer
1 A
2 B
3 A
4 A
5 C
6 C
7 D
8 C
9 E
10 E
11 C
12 B
13 B
14 E
15 E
16 E
17 B
18 D
19 B
20 E
21 D
22 D
23 B
24 D
25 E
26 C
27 A
28 B
29 C
30 A
If you correctly answered 22+ problems, you most likely passed this exam.
If you correctly answered 17-21 problems, you are on the borderline.
If you correctly answered fewer than 17 problems, you most likely failed this
exam.
1
Problem 1
You are given two funds:
Fund ': $100 invested at t = 0 with the annual eective interest rate 6%
Fund : $100 invested at t = 2 (i.e. end of Year 2) with an annual simple
interest j, where j is a constant.
Fund ' and have the same force of interest at the end of Year 10.
Calculate the value of Fund at the end of Year 10.
185 1 195 C 205 1 215 1 225
Solution
Let
1
(t) and

(t) represent the value of Fund ' and at time t


respectively.

1
(t) = 100 1.06
|

(t) = 0 if t < 2 and

(t) = 100 (1 +j (t 2)) if t 2


Force of interest at time t for Fund ':
c
1
(t) =
d ln
1
(t)
dt
=
d ln(100 1.06
|
)
dt
=
d ln100
dt
+
d ln1.06
|
dt
=
1
1.06
|

d1.06
|
dt
=
1
1.06
|
1.06
|
ln1.06 = ln1.06
Notice that ln100 is a constant. Hence
d ln100
dt
= 0.
Force of interest at time t for Fund where t 2
c

(t) =
d ln

(t)
dt
=
d ln[100 (1 +j (t 2))]
dt
=
d ln(1 +j (t 2))
dt
=
j
1 +j (t 2)
c
1
(10) = c

(10)
= ln1.06 =
j
1 + 8j
j = 0.109 15

(10) = 100 (1 + 0.109 15 (10 2)) = 187. 32


Problem 2
You are given:
i is the annual eective interest rate
in a years $1 will grow into $10
in / years $2 will grow into $25
in c years $3 will grow into $60
2
Calculate (1 +i)
o+2b3c
0.1 1 0.2 C 0.3 1 0.4 1 0.5
Solution
1 (1 +i)
o
= 10
2 (1 +i)
b
= 20
3 (1 +i)
c
= 30
(1 +i)
o+2b3c
= (1 +i)
o
(1 +i)
2b
(1 +i)
3c
= (1 +i)
o
h
(1 +i)
b
i
2
[(1 +i)
c
]
3
=
10

25
2

2

60
3

3
= 0.195
Problem 3
$7 payments are made semiannually for the rst year, the rst payment
due 6 months from today
The payments increase by $4 each year, starting from Year 2 and lasting
forever
The annual eective interest rate is 9%
Calculate the present value of this annuity.
1170 1 1190 C 1210 1 1230 1 1250
Solution
The cash ows can be broken down as 3 streams: , 1, and C.
t (year) 0 0.5 1 1.5 2 2.5 3 3.5 4 4.5 5 ...
7 7 7 7 7 7 7 7 7 7 ... 7
1 4 8 12 16 ... ...
C 4 8 12 16 ... ...
+1 +C 7 7 11 11 15 15 19 19 23 23 ... ...
1\ of at t = 0: 7a
|
=
7
,
The 6-month interest rate is , =

1 +i 1 =

1.09 1
PV of B at t = 0.5 is 4 (1a)
|I
= 4
..
a
n|
:
n
i

n
where i = 9%
as : , :
n
0 because
n
approaches zero much faster than :
approaches innity
(Alternatively, you can use IHospitals rule to prove that :
n
0).
..
a
n|
=
1
n
d

1
d
3
Hence (1a)
|I
=
1
d
i
=
1
di
=
1

1
1
1 +i

i
=
1
i
+
1
i
2
PV of B at t = 0 is
4 (1a)
|I
1 +,
Similarly, PV of C at t = 1 is 4 (1a)
|I
and at t = 0 is
4 (1a)
|I
1 +i
Hence the PV of the total annuity at t = 0 is:
7a
|
+
4 (1a)
|I
1 +,
+
4 (1a)
|I
1 +i
=
7

1 + 0.09 1
+ 4
1
0.09
+
1
0.09
2

1 + 0.09
+ 4
1
0.09
+
1
0.09
2
1 + 0.09
= 1168. 38
Problem 4
A loan of 15, 000 is made at time zero
The annual eective interest rate is 10%
20 annual repayments are made. Each repayment is 1, 400
Calculate the loan balance immediately after the 20-th annual repayment is
made.
20, 727 1 20, 777 C 20, 827 1 20, 877 1 20, 927
Solution
First, lets calculate the number of repayments needed to pay o the loan.
Using BA II Plus, we enter:
1\ = 15000 1,1 = 10 1'T = 1400 1\ = 0
If we try to calculate , well get an error message. Whats going on?
It turned out that the annual repayment is less than the interest accrued
during the year. The annual interest accrued on the loan is 15000 0.1 =
1500; the annual repayment is only 1400. If the annual repayment is less
than the interest accrued during the year, then the loan balance will go up
instead of going down; the future value of the loan is negative (i.e. the bor-
rower needs to a nal lump sum repayment). In other words, if you have
1\ = 15000 1,1 = 10 1'T = 1400, then 1\ < 0; theres no
solution for 1\ = 15000 1,1 = 10 1'T = 1400 1\ = 0.
Here is how to solve the problem conceptually:
The loan balance immediately after the 20th repayment is
15000

1.1
20

1400o
20|
= 15000

1.1
20

1400
1.1
20
1
0.1
= 20727
4
Here is how to solve the problem using BA II Plus:
1\ = 15000 1,1 = 10 1'T = 1400 = 20
You should get: 1\ = 20727
Problem 5
Date fund balance before deposit and withdrawal deposit withdrawal
1,1,2000 0 100
7,1,2000 110 15
9,1,2000 80 60
1,1,2001 A
The time weighted return during the year is 0.15 more than the dollar
weighted return during the year.
Calculate A
12 1 17 C 22 1 27 1 32
Solution
Time weighted return:
110
100

80
110 + 15

A
80 60
= 1 +r
|Int
Dollar weighted return:
100

1 +r
Jollo:

+ 15

1 + 0.5r
Jollo:

60

1 +
r
Jollo:
3

= A
r
Jollo:
=
A 100 15 + 60
100 + 15 0.5 60
1
3
=
A 55
87. 5
110
100

80
110 + 15

A
80 60
1 =
A 55
87. 5
+ 0.15
A = 21. 935
Problem 6
Joe buys an S&P 500 index futures contract for 1500. The initial margins
is 10%. The maintenance margin is 80%. The contract is settled weekly. The
continuously compounded interest rate is 6% per year. If the price of the index
drops to 1400 next week, calculate the amount that Joe has to pay.
17, 260 1 17, 360 C 17, 460 1 17, 560 1 17, 660
Solution
At time zero, Joe deposits 0.11500250 = 37500 into his margin account.
Please note theres a scaling factor 250 (see Derivatives Markets Section 5.4).
At the end of Week 1, Joes loss 250 (1500 1400) = 25 000 is marked to
market. Now Joes margin account is 37500c
0.06/52
25 000. = 12543, which is
less than the maintenance margin requirement 37500 0.8 = 30000. Hence Joe
needs to deposit 30000 12 543 = 17 457 into the margin account.
Problem 7
5
Liabilities: $100 per year for ve years, the rst payment due one year
from today
Asset #1: $100 per year at the of Year 3,4, and 5 respectively. The asset
price is $100.
Asset #2: $50 at the end of Year 1 and 2 respectively. The asset price is
$80
Asset #3: $50 per year for ve years, the rst cash ow occurring one
year from today. The asset price is $150.
Asset #4: $100 per year for ve years, the rst cash ow occurring one
year from today. The asset price is $250.
Find the cheapest method to exactly match the liabilities.
Buy some units of Asset #1 and some units Asset #2
1 Buy some units of Asset #1
C Buy some units of Asset #3
1 Buy some units of Asset #4
1 Buy some units of Asset #2 and some units Asset #3
Solution
Cost at t = 0 time t (Year) 0 1 2 3 4 5
Liability 100 100 100 100 100
100 Asset #1 100 100 100
80 Asset #2 50 50
150 Asset #3 50 50 50 50 50
250 Asset #4 100 100 100 100 100
Liability can be matched by
Asset #1 + 2Asset #2. Cost: 100 + 2 (80) = 260
2Asset #3. Cost: 2 (150) = 300
Asset #4. Cost 250.
The least expensive way to exactly match the liability cash ow is to buy
Asset #4.
Problem 8
A bank oers 5% annual eective interest rate to 6-year CD deposit. In
addition, it oers, at the end of Year 6, a 2% bonus interest payment of the
initial deposit .
Calculate the equivalence annual eective yield of this CD.
6
4.86% 1 5.06% C 5.26% 1 5.46% 1 5.66%
Solution
(1 +i)
6
= (1 + 0.05)
6
+ 0.02 = 1. 360 1
i = 5.26%
Problem 9
Fund A accumulates $100 at a simple rate of discount 6% per year for 5
years
Fund B accumulates $100 at a constant force of interest of 4% per year
for 5 years
Calculate the sum of the fund value A and B at the end of Year 5.
213 1 226 C 239 1 252 1 265
Solution
Value of Fund A at t = 5: 100 (1 0.06 5)
1
= 142. 86
Value of Fund B at t = 5: 100c
0.045
= 122. 14
The total fund value at t = 5: 142. 86 + 122. 14 = 265.0
Problem 10
$100, 000 loan is made
Monthly interest-only repayments are made for 10 years, the rst repay-
ment occurring one month from today
Level annual repayments of $9, 456 are made for as long as needed to pay
o the remaining balance of the loan, where the rst annual repayment is
made at the end of Year 11.
The annual eective interest rate is 9.2%
Calculate the total number of repayments necessary to pay o the loan.
133 1 140 C 147 1 154 1 161
Solution
100, 000 = 9456a
n|9.2%
= 9456
1 1.092
n
0.092
: = 41
So the total number of repayments is 12 10 + 41 = 161
Problem 11
A loan amount A is repaid by level annual repayments, the rst repayment
due in one year. You are given:
The interest portion of the 5-th repayment is 10251.9
7
The principal paid in the ( 8)th repayment is 7361.83
The interest paid in the ( 16)th repayment is 7469.15
Calculate A
184, 500 1 184, 750 C 185, 000 1 185, 250 1 185, 500
Solution
1 is the level annual repayment.
1
+1(8)
= 7361.83 1
9
= 7361.83
1

1
+1(16)

= 7469.15 1

1
17

= 7469.15

9
1
17
=
7361.83
7469.15
= 0.985 63

9
= 0.985 63 0.985 63
17

9
+ 0.985 63
17
0.985 63 = 0
The above equation corresponds to the following diagram:
time t 0 1 2 ... 8 9 10 ... 16 17
cash ow $0.985 63 0 0 ... 0 1 0 ... 0 0.985 63
(If you set the PV of the above cash ow to zero, youll get
9
+0.985 63
17

0.985 63 = 0)
Use BA II Plus CF worksheet to solve for the interest rate. Enter
C10 = 0.985 63
C01 = 0 101 = 8 (to account for zero cash ow at t = 1, 2, ..., 8)
C02 = 1 102 = 1 (to account for $1 at t = 9)
C03 = 0 103 = 7 (to account for zero cash ow at t = 10, 11, ..., 16)
C04 = 1 104 = 1 (to account for 0.985 63 at t = 17)
You should get: IRR=5.75 (so the interest rate is 5.75%)
1 1.0575
9
= 7361.83 1 = 12176
The interest portion of the 5-th repayment is:
1

1
+15

= 10251.9
12176

1 1.0575
(+15)

= 10251.9
1.0575
(4)
= 1
10251.9
12176
= 0.158 02
4 =
ln0.158 02
ln1.0575
= 33. 001 = 37
A = 12176a
37|5.75%
= 12176
1 1.0575
37
0.0575
= 185, 000
Problem 12
The Macaulay duration of a 2-year annuity due of $50 per year is 0.4884.
Calculate the Macaulay duration of a 3-year annuity due of $100 per year.
8
0.72 1 0.97 C 1.22 1 1.47 1 1.72
Solution
Macaulay duration is 1
1.c
=
P
tC1 (t)
|
P
C1 (t)
|
2-year annuity due of $50 per year
Time t (year) 0 1
cash ow $50 $50
50 ()
50 (1 +)
= 0.4884 = 0.954 65
3-year annuity due of $100 per year
Time t (year) 0 1 2
cash ow $100 $100 $100
1
1.c
=
100

+ 2
2

100 (1 + +
2
)
=
0.954 65 + 2

0.954 65
2

1 + 0.954 65 + 0.954 65
2
= 0.969 07
Problem 13
The current stock price is $110
The annualized continuously compounded risk-free interest rate is 8.7%
You buy a one-year to expiration call on the stock with strike price $120
You simultaneously sell a one-year to expiration put on the same stock
with the same strike price
Calculate the net premium you have to pay.
2 1 0 C 2 1 4 1 6
Solution
C +1\ (1) = 1 +o
0
The net premium is C 1 = o
0
1\ (1) = 110 120c
0.087
= 0
Problem 14
The current stock price is 100
The annual continuously compounded risk free interest rate is 6%
The cost of carry is 2%
Calculate the annual continuously compounded dividend yield of the stock
2.0% 1 2.5% C 3.0% 1 3.5% 1 4%
Solution
9
The cost of carry is the dierence between the risk-free rate and the dividend
yield (refer to Derivatives Market Chapter 5)
6%r = 2% r = 4%
The annual continuously compounded dividend yield of the stock is 4%
Problem 15
A company has liabilities of 500 in t = 1, 2, and 3 years. It have the option
of purchasing either 1 year or 3 year zero coupon bonds, with prices of 760 and
650, respectively. Assume a at interest rate of 10%. What mix of 1 year and
3 year zero coupon bonds must it purchase now to exactly match the duration
of assets to duration of liabilities?
$540 of the 1-year bond and $460 of the 3-year bond
1 $570 of the 1-year bond and $490 of the 3-year bond
C $600 of the 1-year bond and $520 of the 3-year bond
1 $630 of the 1-year bond and $550 of the 3-year bond
1 $660 of the 1-year bond and $580 of the 3-year bond
Solution
Duration of liability:
500

+ 2
2
+ 3
3

500 ( +
2
+
3
)
=
1.1
1
+ 2 1.1
2
+ 3 1.1
3
1.1
1
+ 1.1
2
+ 1.1
3
=
1. 936 6
The company invests r portion in the 1-year bond and (1 r) portion in
the 3-year bond. We set the PV of the mixture equal to the PV of the liability;
we set the duration of the mixture equal to the duration of the liability.
Since the 1-year bond and the 3-year bond have duration of 1 and 3 respec-
tively, the duration of the mixture is r + (1 r) 3.
r + (1 r) 3 = 1.936 r = 0.532
The PV of the mixture is equal to PV of the liabilities: 500

1.1
1
+ 1.1
2
+ 1.1
3

=
1243. 4
Hence we spend 1243. 4 0.532 = 661. 49 on the 1-year bond (i.e. we buy $
661. 49 worth of the 1-year bond, which represents
661. 49
760
= 0.870 38 fraction
of the 1-year bond).
We spend 1243. 4 (1 0.532) = 581. 91 on the 3-year bond (i.e. we buy $
581. 91 worth of the 3-year bond, which represents
581. 91
650
= 0.895 25 fraction
of the 3-year bond).
Problem 16
Stock and 1 both pay perpetual annual dividends, the rst dividend due
one year from today.
The dividends of Stock increase by 50q%
The dividend of Stock 1 increase by 50q%
The rst dividend of stock is half of the rst dividend of Stock 1
10
The price of Stock is twice the price of Stock 1
The annual eective interest rate is 10%
Calculate q.
0.04 1 0.06 C 0.08 1 0.10 1 0.12
Solution
t (year) 0 1 2 3 4 ...
As dividend a a (1 + 0.5q) a (1 + 0.5q)
2
a (1 + 0.5q)
3
... ...
Bs dividend 2a 2a (1 0.5q) 2a (1 0.5q)
2
2a (1 0.5q)
3
... ...
PV of A:
a+a (1 + 0.5q)
2
+a (1 + 0.5q)
2

3
+... =
a
1 (1 + 0.5q)
=
a
(1 +i) (1 + 0.5q)
=
a
i 0.5q
PV of B:
2a+2a (1 q)
2
+2a (1 q)
2

3
+... =
2a
1 (1 0.5q)
=
2a
(1 +i) (1 0.5q)
=
2a
i + 0.5q
a
i 0.5q
=
4a
i + 0.5q
i + 0.5q
i 0.5q
= 4 q =
6
5
i =
6
5
0.1 = 0.12
Problem 17
You are given the following data about a special annuity:
Quarterly payments of 10, 20, 30, and 40 are made per year for 20 years,
the 1st payment due one quarter from today
The interest rate is 8% nominal per year compounded quarterly
Calculate the PV of this special annuity.
A 10 (1a)
4|2%

..
a
20|8%

B 10 (1a)
4|2%

..
a
20|8.24%

C 10

1
..
a

4|2%

a
20|8%

D 10 (1a)
4|2%

..
a
20|8%

E 10

1
..
a

4|2%

a
20|8.24%

11
Solution
The quarterly eective interest rate is
8%
4
= 2%
The annual eective interest rate is 1.02
4
1 = 8. 24%
t (quarter) 0 1 2 3 4 5 6 7 8 ... 77 78 79 80
cash ow 10 20 30 40 10 20 30 40 ... 10 20 30 40
The above cash ow diagram can be simplied as:
t (quarter) 0 4 ... 8 76
t (Yr) 0 1 ... 2 19
cash ow 10 (1a)
4|2%
10 (1a)
4|2%
10 (1a)
4|2%
10 (1a)
4|2%
10 (1a)
4|2%
The PV of this cash ow is 10 (1a)
4|2%

..
a
20|8.24%

Problem 18
A loan is repaid through level monthly payments for 10 years, the rst pay-
ment due one month from the loan date.
The annual eective interest rate is 8%. The principal for the 6th payment
is 60.
Calculate the principal for the 24th payment.
49 1 55 C 61 1 67 1 73
Solution
The monthly discounting factor is = 1.08
1/12
Let A represent the level monthly payment.
principal for the 6th payment is:
A
1216
= 60
principal for the 24th payment:
A
12124
= A
1216

18
= 60

1.08
1/12

18
= 67.34
Problem 19
For the rst two years the annual eective interest rate is 6%. For t 2
force of interest is
1
1 +t
, where t is the number of years from today.
Calculate the eective annual discount rate over the 5 years.
0.12 1 0.15 C 0.18 1 0.21 1 0.24
Solution
(1 d)
5
= 1.06
2
c

5
2
1
1 +t
J|
R
5
2
1
1 +t
dt = [ln(1 +t)]
5
2
= ln6 ln3 = ln2
(1 d)
5
= 1.06
2
c
ln 2
= 2 1.06
2
12
d = 1

2 1.06
2

1/5
= 0.149 51
Problem 20
A company wants to buy a mixture of bonds that will satisfy its liability
and cost the least.
Companys Liability
time t (year) 0 1 2
liability cash ow 100, 000 200, 000
Assets:
Bond #1: 1-year bond with 7.5% annual coupon
Bond #2: 2-year bond with 5.5% annual coupon
Bond #3: 2-year zero coupon bond at 6%
The company also has the option of buying a 7% one year zero coupon bond
starting at time 1. In order to obtain this bond, however, the company needs to
pay 2% of the purchase price at time 0. Calculate the total cost of the mixture
of the bonds that will satisfy its liability and cost the least.
269, 980 1 270, 080 C 270, 180 1 270, 280 1 270, 380
Solution
1-year bond with 7.5% annual coupon means that if you buy $100 worth of
the bond at t = 0, the bond will pay you 107.5 at t = 1.
2-year bond with 5.5% annual coupon means that if you buy $100 worth of
the bond at t = 0, the bond will pay you 5.5 at t = 1 and 105.5 at t = 2.
2-year zero coupon bond at 6% means that the bond doesnt pay any coupon
and that it is discounted at 6%. If you want the bond to pay you $100 at t=2,
you need to buy 100 1.06
2
= 89 worth of the bond at t = 0.
To ensure that it has 100, 000 at t = 1, the company can buy 100000
1.075
1
= 93023 worth of Bond #1 at t = 0.
To ensure that it has 200, 000 at t = 2, the company can do one the following
two things:
(1) buy 200000 1.06
2
= 177, 999 worth of Bond #3.
(2) buy the special bond so it will pay 200000 at t = 2. The cost at t=1 is
200000 1.07
1
.
The cost at t=0 is 200000 1.07
1
1.075
1
1.02 = 177, 353
Option 2 is cheaper. So the least expensive way to match the liability cash
ow is to buy a mixture of Bond #1 and the special bond. The total cost is
13
93023 + 177353 = 270 376
Please note that Bond #2 is not needed to solve the problem; Bond #2 can
be replicated with some mixture of Bond #1 and Bond #3.
Problem 21
Information about the asset and liability of a retirement fund:
Asset Liability
amount 250, 000 250, 000
duration 16 12
convexity 360 360
Assume a parallel shift of the interest rate. Which of the following statements
is correct?
The retirement fund loses no matter the interest goes up slightly or
down slightly
1 The retirement fund neither gains nor loses no matter the interest
goes up slightly or down slightly
C The retirement fund gains if the interest goes up slightly but loses if
the interest goes down slightly
1 The retirement fund gains if the interest goes down slightly but loses
if the interest goes up slightly
1 The retirement fund gains no matter the interest goes up slightly or
down slightly
Solution
1 (r) the PV of an asset (or liability), which is a function of the interest
rate r
Let
1
1.c
represent the Macaulay duration
C represent the convexity
represent the asset
1 represent the liability
o represent the surplus. o = 1
If o increases when r changes, the retirement fund benets from the change
in the interest rate r.
If o decreases when r changes, the retirement fund loses from the change in
the interest rate r.
If o is the same when r changes, the retirement fund neither gains nor loses
from the change in the interest rate r.
1
1

1
1
d1
dr
r +
1
2
1
1
d
2
1
dr
2
(r)
2
=
1
1 +r
1
1.c
r +
1
2
C (r)
2
14
1
1
1 +r
1
1.c
r +
1
2
C (r)
2
1
.

1
.
1 +r
1
.
1.c
r +
1
.
2
C
.
(r)
2
1
J

1
J
1 +r
1
J
1.c
r +
1
J
2
C
J
(r)
2
Since 1
.
= 1
J
and C
.
= C
J
o = 1
.
1
J

1
.
1 +r
r

1
.
1.c
1
J
1.c

=
1
.
1 +r
r
(16 12) =
1
.
1 +r
r 4
If r 0, then o < 0.
If r < 0, then o 0.
The retirement fund gains if the interest goes down slightly but loses if the
interest goes up slightly
Problem 22
John wants a 5000 loan immediately. He can get the loan by one of the two
options:
Option 1. A 5-year loan with interest and principal paid at the end of
Year 5
Option 2. A 3-year loan with interest and principal paid by a 2-year loan
issued at the end of Year 3.
Calculate the eective interest rate of the 2-year loan such that John is
indierent with either option.
You are also given the following structure of spot interest rates:
Term (Year) 1 2 3 4 5
spot rate 3.5% 4.5% 5.5% 6.5% 7.5%
9.37% 1 9.77% C 10.17% 1 10.57% 1 10.97%
Solution
Johns repayment at t = 5 under Option 1:
5000 1.075
5
Johns repayment at t = 5 under Option 2:
5000 1.055
3
(1 +r)
2
5000 1.075
5
= 5000 1.055
3
(1 +r)
2
r = 10.5 7%
Alternative solution:
15
To avoid arbitrage, the eective interest rate of the 2-year loan must be the
forward rate from t = 3 to t = 5 implied by the term structure of the spot rates.
1.055
3
(1 +r)
2
= 1.075
5
r = 10.5 7%
Problem 23
Which one of these positions will NOT benet from an increase of the stock
price?
A long forward
B short stock
C short put
D long call
Solution
If you sell a stock short and the stock price goes up, you have to buy it back
later at a higher price. So you wont benet from the increase of the stock price.
Problem 24
Which of the followings is a correct statement about the dierence between
a futures contract and a forward contract
A. Futures contracts are settled at expiration but forward contracts are
settled daily
B. Futures contracts are traded over the counter but forward contracts are
exchange-traded
C. Futures contracts are less liquid than forward contracts
D. Futures contracts are more rigid in terms and conditions, but forward
contracts are more exible
Solution
D. Forward contracts are private agreements between two parties and can
have exible terms and conditions to meet the two parties needs.
Problem 25
Which of the followings is correct about Redington immunization?
A. The asset cash ows should match the liability cash ows.
B. It immunizes against all the interest rate changes
C. The duration of the asset should be greater than the duration of the
liability
D. The convexity of the liability should be greater than the convexity of the
asset
E. None of the above
Solution 1
Problem 26 You are given two equivalent options of buying a car:
Option 1. Buy the car outright with cash for 30, 000
16
Option 2. Lease the car for three years with down payment 1000 and
monthly payments of A for 3 years. At the end of the Year 3, the car can
be returned for 2, 000.
The annual eective interest rate is 8%. Calculate A.
905 1 930 C 955 1 980 1 1005
Solution
The PV of option 1 is 30000.
The PV of option 2 is 1000 +ra
36|
2000 1.08
3
, , = 1.08
1/12
1 = 0.6
434%
(, is the monthly eective interest rate)
30000 = 1000 +ra
36|
+ 2000 1.08
3
1000 +
1 1.08
3
0.006 434
r 2000 1.08
3
= 30000
r = 954. 57
Problem 27
You borrow a 1000 loan at an annual eective rate 10%. You accumulate a
sinking fund at the end of Year 10. The value of the sinking fund is 600. The
8-th payment is 140. How much of the 8-th payment goes to interest?
100 1 110 C 120 1 130 1 140
Solution
The information is less than perfect, but you need to make most of it.
In a typical sinking fund, the borrower pays the interest year by year. He
then separately sets up a sinking fund to repay the principal.
The 8-th payment 140 is greater than the interest accrued 1000 0.1 = 100.
So we assume that the 100 goes toward the interest and the remaining 40 is used
to reduce the principal. Please note that the sinking fund balance 600 is less
than the principal 1000. This conrms that the annual sinking fund payment is
greater than the accrued interest; the excess of the annual sinking fund payment
over the accrued interest is used to pay down the principal.
Problem 28
A loan is paid through annual repayments at the end of the year for 30 years.
The annual payments are $5 for each of the rst 10 years, $4 for each of the next
10 years, and $3 for each of the last 10 years. The interest portion of the 11st
payment is one and half the interest portion of the 21-th payment. Calculate
the interest portion of the 21-th payment.
2.4 1 2.5 C 2.6 1 2.7 1 2.8
17
Solution
t (yr) 0 1 2 ... 10 11 12 ... 20 21 22 ... 30
pymt 5 5 ... 5 4 4 ... 4 3 3 ... 3
The interest portion of the 11th payment is:
1
11
= i O1
10
= i

4a
10|
+
10
3a
10|

O1
10
is the outstanding balance immediately after the 10th payment is
made.
the interest portion of the 21-th payment
1
21
= i O1
20
= i 3a
10|
O1
20
is the outstanding balance immediately after the 20-th payment is
made.
i

4a
10|
+
10
3a
10|

= 1.5 i 3a
10|
4 + 3
10
= 4.5

10
=
0.5
3
1
21
= i 3a
10|
= i 3
1
10
i
= 3

1
10

= 3

1
0.5
3

= 2. 5
Problem 29
Company FastGrow Incorporated can have 500M prot (M=million) or 350M
loss with equal probability. If FastGrow uses hedging, its prot is 100M for sure
after hedging.
FastGrow has tax rate 40%. However, it pays no tax and receives no tax
credit if it incurs losses.
Calculate the dierence (in millions) between FastGrows hedged after-tax
prot and unhedged after-tax prot.
75 1 80 C 85 1 90 1 95
Solution
The tax rate is 40% if positive prot and 0% if zero or negative prot
After tax prot if hedged (100% probability of getting 75 before tax):
100 (1 0.4) = 60
After tax prot if unhedged:
0.5 (500 (1 0.4) + (350) (1 0)) = 25
The dierence between hedged prot and unhedged prot is: 60 (25) =
85
(Please refer to Derivatives Markets Section 4.3 for more examples of how
to calculate the after tax prot.)
Problem 30
The nominal interest rate is 6% per year
18
The ination rate is 2% per year
Calculate the real interest rate per year.
4% 1 6% C 8% 1 10% 1 12%
Solution
1 +r
Re ol
=
1 +r
o min ol
1 +r
inf lo|Ion
=
1.06
1.02
= 1. 039 2
r
Re ol
= 3.92%
19

You might also like